Unified Theory of Climate

Note: This was a poster, and adopted into a blog post by the author, Ned Nikolov, specifically for WUWT. My thanks to him for the extra effort in converting the poster to a more blog friendly format. – Anthony

Expanding the Concept of Atmospheric Greenhouse Effect Using Thermodynamic Principles: Implications for Predicting Future Climate Change

Ned Nikolov, Ph.D. & Karl Zeller, Ph.D.

USFS Rocky Mountain Research Station, Fort Collins CO, USA

Emails: ntconsulting@comcast.net kzeller@colostate.edu

Poster presented at the Open Science Conference of the World Climate Research Program,

24 October 2011, Denver CO, USA

http://www.wcrp-climate.org/conference2011/posters/C7/C7_Nikolov_M15A.pdf

Abstract

We present results from a new critical review of the atmospheric Greenhouse (GH) concept. Three main problems are identified with the current GH theory. It is demonstrated that thermodynamic principles based on the Gas Law need be invoked to fully explain the Natural Greenhouse Effect. We show via a novel analysis of planetary climates in the solar system that the physical nature of the so-called GH effect is a Pressure-induced Thermal Enhancement (PTE), which is independent of the atmospheric chemical composition. This finding leads to a new and very different paradigm of climate controls. Results from our research are combined with those from other studies to propose a new Unified Theory of Climate, which explains a number of phenomena that the current theory fails to explain. Implications of the new paradigm for predicting future climate trends are briefly discussed.

1. Introduction

Recent studies revealed that Global Climate Models (GCMs) have significantly overestimated the Planet’s warming since 1979 failing to predict the observed halt of global temperature rise over the past 13 years. (e.g. McKitrick et al. 2010). No consensus currently exists as to why the warming trend ceased in 1998 despite a continued increase in atmospheric CO2 concentration. Moreover, the CO2-temperature relationship shows large inconsistencies across time scales. In addition, GCM projections heavily depend on positive feedbacks, while satellite observations indicate that the climate system is likely governed by strong negative feedbacks (Lindzen & Choi 2009; Spencer & Braswell 2010). At the same time, there is a mounting political pressure for Cap-and-Trade legislation and a global carbon tax, while scientists and entrepreneurs propose geo-engineering solutions to cool the Planet that involve large-scale physical manipulation of the upper atmosphere. This unsettling situation calls for a thorough reexamination of the present climate-change paradigm; hence the reason for this study.

2.  The Greenhouse Effect: Reexamining the Basics

image

Figure 1. The Atmospheric Greenhouse Effect as taught at universities around the World (diagram from the website of the Penn State University Department of Meteorology).

According to the current theory, the Greenhouse Effect (GHE) is a radiative phenomenon caused by heat-trapping gases in the atmosphere such as CO2 and water vapor that are assumed to reduce the rate of surface infrared cooling to Space by absorbing the outgoing long-wave (LW) emission and re-radiating part of it back, thus increasing the total energy flux toward the surface. This is thought to boost the Earth’s temperature by 18K – 33K compared to a gray body with no absorbent atmosphere such as the Moon; hence making our Planet habitable. Figure 1 illustrates this concept using a simple two-layer system known as the Idealized Greenhouse Model (IGM). In this popular example, S is the top-of-the atmosphere (TOA) solar irradiance (W m-2), A is the Earth shortwave albedo, Ts is the surface temperature (K), Te is the Earth’s effective emission temperature (K) often equated with the mean temperature of middle troposphere, ϵ is emissivity, and σ is the Stefan-Boltzmann (S-B) constant.

2.1. Main Issues with the Current GHE Concept:

A) Magnitude of the Natural Greenhouse Effect. GHE is often quantified as a difference between the actual mean global surface temperature (Ts = 287.6K) and the planet’s average gray-body (no-atmosphere) temperature (Tgb), i.e. GHE = Ts Tgb. In the current theory, Tgb is equated with the effective emission temperature (Te) calculated straight from the S-B Law using Eq. (1):

image

where αp is the planetary albedo of Earth (≈0.3). However, this is conceptually incorrect! Due to Hölder’s inequality between non-linear integrals (Kuptsov 2001), Te is not physically compatible with a measurable true mean temperature of an airless planet. To be correct, Tgb must be computed via proper spherical integration of the planetary temperature field. This means calculating the temperature at every point on the Earth sphere first by taking the 4th root from the S-B relationship and then averaging the resulting temperature field across the planet surface, i.e.

image

where αgb is the Earth’s albedo without atmosphere (≈0.125), μ is the cosine of incident solar angle at any point, and cs= 13.25e-5 is a small constant ensuring that Tgb = 2.72K (the temperature of deep Space) when So = 0. Equation (2) assumes a spatially constant albedo (αgb), which is a reasonable approximation when trying to estimate an average planetary temperature.

Since in accordance with Hölder’s inequality TgbTe (Tgb =154.3K ), GHE becomes much larger than presently estimated.

According to Eq. (2), our atmosphere boosts Earth’s surface temperature not by 18K—33K as currently assumed, but by 133K! This raises the question: Can a handful of trace gases which amount to less than 0.5% of atmospheric mass trap enough radiant heat to cause such a huge thermal enhancement at the surface? Thermodynamics tells us that this not possible.

B) Role of Convection. The conceptual model in Fig. 1 can be mathematically described by the following simultaneous Equations (3),

image

where νa is the atmospheric fraction of the total shortwave radiation absorption. Figure 2 depicts the solution to Eq. (3) for temperatures over a range of atmospheric emissivities (ϵ) assuming So = 1366 W m-2 and νa =0.326 (Trenberth et al. 2009). An increase in atmospheric emissivity does indeed cause a warming at the surface as stated by the current theory. However, Eq. (3) is physically incomplete, because it does not account for convection, which occurs simultaneously with radiative transfer. Adding a convective term to Eq. (3) (such as a sensible heat flux) yields the system:

image

where gbH is the aerodynamic conductance to turbulent heat exchange. Equation (4) dramatically alters the solution to Eq. (3) by collapsing the difference between Ts, Ta and Te and virtually erasing the GHE (Fig. 3). This is because convective cooling is many orders of magnitude more efficient that radiative cooling. These results do not change when using multi-layer models. In radiative transfer models, Ts increases with ϵ not as a result of heat trapping by greenhouse gases, but due to the lack of convective cooling, thus requiring a larger thermal gradient to export the necessary amount of heat. Modern GCMs do not solve simultaneously radiative transfer and convection. This decoupling of heat transports is the core reason for the projected surface warming by GCMs in response to rising atmospheric greenhouse-gas concentrations. Hence, the predicted CO2-driven global temperature change is a model artifact!

image

Figure 2. Solution to the two-layer model in Eq. (3) for Ts and Ta as a function of atmospheric emissivity assuming a non-convective atmosphere. Also shown is the predicted down-welling LW flux(Ld). Note that Ld ≤ 239 W m-2.

image

Figure 3. Solution to the two-layer model in Eq. (4) for Ts and Ta as a function of atmospheric emissivity assuming a convective atmosphere (gbH = 0.075 m/s). Also shown is the predicted down-welling LW flux (Ld). Note that Ld ≤ 239 W m-2.

image

Figure 4. According to observations, the Earth-Atmosphere System absorbs on average a net solar flux of 239 W m-2, while the lower troposphere alone emits 343 W m-2 thermal radiation toward the surface.

C) Extra Kinetic Energy in the Troposphere.

Observations show that the lower troposphere emits 44% more radiation toward the surface than the total solar flux absorbed by the entire Earth-Atmosphere System (Pavlakis et al. 2003) (Fig. 4). Radiative transfer alone cannot explain this effect (e.g. Figs. 2 & 3) given the negligible heat storage capacity of air, no matter how detailed the model is. Thus, empirical evidence indicates that the lower atmosphere contains more kinetic energy than provided by the Sun. Understanding the origin of this extra energy is a key to the GHE.

3. The Atmospheric Thermal Enhancement

Previous studies have noted that the term Greenhouse Effect is a misnomer when applied to the atmosphere, since real greenhouses retain heat through an entirely different mechanism compared to the free atmosphere, i.e. by physically trapping air mass and restricting convective heat exchange. Hence, we propose a new term instead, Near-surface Atmospheric Thermal Enhancement (ATE) defined as a non-dimensional ratio (NTE) of the planet actual mean surface air temperature (Ts, K) to the average temperature of a Standard Planetary Gray Body (SPGB) with no atmosphere (Tgb, K) receiving the same solar irradiance, i.e. NTE = Ts /Tgb. This new definition emphasizes the essence of GHE, which is the temperature boost at the surface due to the presence of an atmosphere. We employ Eq. (2) to estimate Tgb assuming an albedo αgb = 0.12 and a surface emissivity ϵ = 0.955 for the SPGB based on data for Moon, Mercury, and the Earth surface. Using So = 1362 W m-2 (Kopp & Lean 2011) in Eq. (2) yields Tgb = 154.3K and NTE = 287.6/154.3 = 1.863 for Earth. This prompts the question: What mechanism enables our atmosphere to boost the planet surface temperature some 86% above that of a SPGB? To answer it we turn on to the classical Thermodynamics.

3.1. Climate Implications of the Ideal Gas Law

The average thermodynamic state of a planet’s atmosphere can be accurately described by the Ideal Gas Law (IGL):

PV = nRT (5)

where P is pressure (Pa), V is the gas volume (m3), n is the gas amount (mole), R = 8.314 J K-1 mol-1is the universal gas constant, and T is the gas temperature (K). Equation (5) has three features that are chiefly important to our discussion: a) the product P×V defines the internal kinetic energy of a gas (measured in Jules) that produces its temperature; b) the linear relationship in Eq. (5) guarantees that a mean global temperature can be accurately estimated from planetary averages of surface pressure and air volume (or density). This is in stark contrast to the non-linear relationship between temperature and radiant fluxes (Eq. 1) governed by Hölder’s inequality of integrals; c) on a planetary scale, pressure in the lower troposphere is effectively independent of other variables in Eq. (5) and is only a function of gravity (g), total atmospheric mass (Mat), and the planet surface area (As), i.e. Ps = g Mat/As. Hence, the near-surface atmospheric dynamics can safely be assumed to be governed (over non-geological time scales) by nearly isobaric processes on average, i.e. operating under constant pressure. This isobaric nature of tropospheric thermodynamics implies that the average atmospheric volume varies in a fixed proportion to changes in the mean surface air temperature following the Charles/Gay-Lussac Law, i.e. Ts/V = const. This can be written in terms of the average air density ρ (kg m-3) as

ρTs = const. = Ps M / R (6)

where Ps is the mean surface air pressure (Pa) and M is the molecular mass of air (kg mol-1). Eq. (6) reveals an important characteristic of the average thermodynamic process at the surface, namely that a variation of global pressure due to either increase or decrease of total atmospheric mass will alter both temperature and atmospheric density. What is presently unknown is the differential effect of a global pressure change on each variable. We offer a solution to this in & 3.3. Equations (5) and (6) imply that pressure directly controls the kinetic energy and temperature of the atmosphere. Under equal solar insolation, a higher surface pressure (due to a larger atmospheric mass) would produce a warmer troposphere, while a lower pressure would result in a cooler troposphere. At the limit, a zero pressure (due to the complete absence of an atmosphere) would yield the planet’s gray-body temperature.

The thermal effect of pressure is vividly demonstrated on a cosmic scale by the process of star formation, where gravity-induced rise of gas pressure boosts the temperature of an interstellar cloud to the threshold of nuclear fusion. At a planetary level, the effect is manifest in Chinook winds, where adiabatically heated downslope airflow raises the local temperature by 20C-30C in a matter of hours. This leads to a logical question: Could air pressure be responsible for the observed thermal enhancement at the Earth surface presently known as a ‘Natural Greenhouse Effect’? To answer this we must analyze the relationship between NTEfactor and key atmospheric variables including pressure over a wide range of planetary climates. Fortunately, our solar system offers a suitable spectrum of celestial bodies for such analysis.

3.2. Interplanetary Data Set

We based our selection of celestial bodies for the ATE analysis on three criteria: 1) presence of a solid planetary surface with at least traces of atmosphere; 2) availability of reliable data on surface temperature, total pressure, atmospheric composition etc. preferably from direct measurements; and 3) representation of a wide range of atmospheric masses and compositions. This approach resulted in choosing of four planets – Mercury, Venus, Earth, and Mars, and four natural satellites – Moon of Earth, Europa of Jupiter, Titan of Saturn, and Triton of Neptune. Each celestial body was described by 14 parameters listed in Table 1.

For planets with tangible atmospheres, i.e. Venus, Earth and Mars, the temperatures calculated from IGL agreed rather well with observations. Note that, for extremely low pressures such as on Mercury and Moon, the Gas Law produces Ts ≈ 0.0. The SPGB temperatures for each celestial body were estimated from Eq. (2) using published data on solar irradiance and assuming αgb = 0.12 and ϵ = 0.955. For Mars, global means of surface temperature and air pressure were calculated from remote sensing data retrieved via the method of radio occultation by the Radio Science Team (RST) at Stanford University using observations by the Mars Global Surveyor (MGS) spacecraft from 1999 to 2005. Since the MGS RST analysis has a wide spatial coverage, the new means represent current average conditions on the Red Planet much more accurately than older data based on Viking’s spot observations from 1970s.

Table 1. Planetary data used to analyze the physical nature of the Atmospheric Near-Surface Thermal Enhancement (NTE). Information was gathered from multiple sources using cross-referencing. The bottom three rows of data were estimated in this study using equations discussed in the text.

3.3. Physical Nature of ATE / GHE

Our analysis of interplanetary data in Table 1 found no meaningful relationships between ATE (NTE) and variables such as total absorbed solar radiation by planets or the amount of greenhouse gases in their atmospheres. However, we discovered that NTE was strongly related to total surface pressure through a nearly perfect regression fit via the following nonlinear function:

image

where Ps is in Pa. Figure 5 displays Eq. (7) graphically. The tight relationship signals a causal effect of pressure on NTE, which is theoretically supported by the IGL (see & 3.1). Also, the PsNTE curve in Fig. 5 strikingly resembles the response of the temperature/potential temp. (T/θ) ratio to altitudinal changes of pressure described by the well-known Poisson formula derived from IGL (Fig. 6). Such a similarity in responses suggests that both NTE and θ embody the effect of pressure-controlled adiabatic heating on air, even though the two mechanisms are not identical. This leads to a fundamental conclusion that the ‘Natural Greenhouse Effect’ is in fact a Pressure-induced Thermal Enhancement (PTE) in nature.

NTE should not be confused with an actual energy, however, since it only defines the relative (fractional) increase of a planet’s surface temperature above that of a SPGB. Pressure by itself is not a source of energy! Instead, it enhances (amplifies) the energy supplied by an external source such as the Sun through density-dependent rates of molecular collision. This relative enhancement only manifests as an actual energy in the presence of external heating. Thus, Earth and Titan have similar NTE values, yet their absolute surface temperatures are very different due to vastly dissimilar solar insolation. While pressure (P) controls the magnitude of the enhancement factor, solar heating determines the average atmospheric volume (V), and the product P×V defines the total kinetic energy and temperature of the atmosphere. Therefore, for particular solar insolation, the NTE factor gives rise to extra kinetic energy in the lower atmosphere beyond the amount supplied by the Sun. This additional energy is responsible for keeping the Earth surface 133K warmer than it would be in the absence of atmosphere, and is the source for the observed 44% extra down-welling LW flux in the lower troposphere (see &2.1 C). Hence, the atmosphere does not act as a ‘blanket’ reducing the surface infrared cooling to space as maintained by the current GH theory, but is in and of itself a source of extra energy through pressure. This makes the GH effect a thermodynamic phenomenon, not a radiative one as presently assumed!

Equation (7) allows us to derive a simple yet robust formula for predicting a planet’s mean surface temperature as a function of only two variables – TOA solar irradiance and mean atmospheric surface pressure, i.e.

image

image

Figure 5. Atmospheric near-surface Thermal Enhancement (NTE) as a function of mean total surface pressure (Ps) for 8 celestial bodies listed in Table 1. See Eq. (7) for the exact mathematical formula.

image

Figure 6. Temperature/potential temperature ratio as a function of atmospheric pressure according to the Poisson formula based on the Gas Law (Po = 100 kPa.). Note the striking similarity in shape with the curve in Fig. 5.

where NTE(Ps) is defined by Eq. (7). Equation (8) almost completely explains the variation of Ts among analyzed celestial bodies, thus providing a needed function to parse the effect of a global pressure change on the dependent variables ρ and Tsin Eq. (6). Together Equations (6) and (8) imply that the chemical composition of an atmosphere affects average air density through the molecular mass of air, but has no impact on the mean surface temperature.

4. Implications of the new ATE Concept

The implications of the above findings are numerous and paradigm-altering. These are but a few examples:

image

Figure 7. Dynamics of global temperature and 12-month forward shifted cloud cover types from satellite observations. Cloud changes precede temperature variations by 6 to 24 months and appear to have been controlling the latter during the past 30 years (Nikolov & Zeller, manuscript).

A) Global surface temperature is independent of the down-welling LW flux known as greenhouse or back radiation, because both quantities derive from the same pool of atmospheric kinetic energy maintained by solar heating and air pressure. Variations in the downward LW flux (caused by an increase of tropospheric emissivity, for example) are completely counterbalanced (offset) by changes in the rate of surface convective cooling, for this is how the system conserves its internal energy.

B) Modifying chemical composition of the atmosphere cannot alter the system’s total kinetic energy, hence the size of ATE (GHE). This is supported by IGL and the fact that planets of vastly different atmospheric composition follow the same PsNTE relationship in Fig. 5. The lack of impact by the atmospheric composition on surface temperature is explained via the compensating effect of convective cooling on back-radiation discussed above.

C) Equation (8) suggests that the planet’s albedo is largely a product of climate rather than a driver of it. This is because the bulk of the albedo is a function of the kinetic energy supplied by the Sun and the atmospheric pressure. However, independent small changes in albedo are possible and do occur owning to 1%-3% secular variations in cloud cover, which are most likely driven by solar magnetic activity. These cloud-cover changes cause ±0.7C semi-periodic fluctuations in global temperature on a decadal to centennial time scale as indicated by recent satellite observations (see Fig. 7) and climate reconstructions for the past 10,000 years.

image

Figure 8. Dynamics of global surface temperature during the Cenozoic Era reconstructed from 18O proxies in marine sediments (Hansen et al. 2008).

image

Figure 9. Dynamics of mean surface atmospheric pressure during the Cenozoic Era reconstructed from the temperature record in Fig. 8 by inverting Eq. (8).

D) Large climatic shifts evident in the paleo-record such as the 16C directional cooling of the Globe during the past 51 million years (Fig. 8) can now be explained via changes in atmospheric mass and surface pressure caused by geologic variations in Earth’s tectonic activity. Thus, we hypothesize that the observed mega-cooling of Earth since the early Eocene was due to a 53% net loss of atmosphere to Space brought about by a reduction in mantle degasing as a result of a slowdown in continental drifts and ocean floor spreading. Figure 9 depicts reconstructed dynamics of the mean surface pressure for the past 65.5M years based on Eq. (8) and the temperature record in Fig. 8.

5. Unified Theory of Climate

The above findings can help rectify physical inconsistencies in the current GH concept and assist in the development of a Unified Theory of Climate (UTC) based on a deeper and more robust understanding of various climate forcings and the time scales of their operation. Figure 10 outlines a hierarchy of climate forcings as part of a proposed UTC that is consistent with results from our research as well as other studies published over the past 15 years. A proposed key new driver of climate is the variation of total atmospheric mass and surface pressure over geological time scales (i.e. tens of thousands to hundreds of millions of years). According to our new theory, the climate change over the past 100-300 years is due to variations of global cloud albedo that are not related to GHE/ATE. This is principally different from the present GH concept, which attempts to explain climate changes over a broad range of time scales (i.e. from decades to tens of millions of years) with the same forcing attributed to variations in atmospheric CO2 and other heat-absorbing trace gases (e.g. Lacis et al. 2010).

Earth’s climate is currently in one of the warmest periods of the Holocene (past 10K years). It is unlikely that the Planet will become any warmer over the next 100 years, because the cloud cover appears to have reached a minimum for the present levels of solar irradiance and atmospheric pressure, and the solar magnetic activity began declining, which may lead to more clouds and a higher planetary albedo. At this point, only a sizable increase of the total atmospheric mass can bring about a significant and sustained warming. However, human-induced gaseous emissions are extremely unlikely to produce such a mass increase.

image

Figure 10. Global climate forcings and their time scales of operation according to the hereto proposed Unified Theory of Climate (UTC). Arrows indicate process interactions.

6. References

Kopp, G. and J. L. Lean (2011). A new, lower value of total solar irradiance: Evidence and climate significance, Geophys. Res. Lett., 38, L01706, doi:10.1029/2010GL045777.

Kuptsov, L. P. (2001) Hölder inequality, in Hazewinkel, Michiel, Encyclopedia of Mathematics, Springer, ISBN 978-1556080104.

Lacis, A. A., G. A. Schmidt, D. Rind, and R. A. Ruedy (2010). Atmospheric CO2: Principal control knob governing earth’s temperature. Science 330:356-359.

Lindzen, R. S. and Y.-S. Choi (2009). On the determination of climate feedbacks from ERBE data. Geophys. Res. Lett., 36, L16705, doi:10.1029/2009GL039628.

McKitrick, R. R. et al. (2010). Panel and Multivariate Methods for Tests of Trend Equivalence in Climate Data Series. Atmospheric Science Letters, Vol. 11, Issue 4, pages 270–277.

Nikolov, N and K. F. Zeller (manuscript). Observational evidence for the role of planetary cloud-cover dynamics as the dominant forcing of global temperature changes since 1982.

Pavlakis, K. G., D. Hatzidimitriou, C. Matsoukas, E. Drakakis, N. Hatzianastassiou, and I. Vardavas (2003). Ten-year global distribution of down-welling long-wave radiation. Atmos. Chem. Phys. Discuss., 3, 5099-5137.

Spencer, R. W. and W. D. Braswell (2010). On the diagnosis of radiative feedback in the presence of unknown radiative forcing, J. Geophys. Res., 115, D16109, doi:10.1029/2009JD013371

Trenberth, K.E., J.T. Fasullo, and J. Kiehl (2009). Earth’s global energy budget. BAMS, March:311-323

===============================================================

This post is also available as a PDF document here:

Unified_Theory_Of_Climate_Poster_Nikolov_Zeller

UPDATE: This thread is closed – see the newest one “A matter of some Gravity” where the discussion continues.

0 0 votes
Article Rating
684 Comments
Oldest
Newest Most Voted
Inline Feedbacks
View all comments
Editor
December 29, 2011 1:14 am

An excellent submission for peer 2 peer review….. i’ll be digesting this one for a while…

Brian H
December 29, 2011 1:19 am

Magnificent.
“only a sizable increase of total atmospheric mass can bring about a significant and sustained warming. However, human-induced gaseous emissions are extremely unlikely to produce such a mass increase. Hence, there is no anthropogenic forcing to global climate.”
Into the dumpster with Warmism and Luke-Warmism!

December 29, 2011 1:31 am

Absolutely fabulous. I take my hat off to you Sir.

David Jones
December 29, 2011 1:42 am

I like the touch of including the graph from Pen State University Department of Meteorology as Figure 1.
Shows chutzpah!

kim
December 29, 2011 1:49 am

I think I’ve never heard so loud
The quiet message in a cloud.
=====================

Fitzcarraldo
December 29, 2011 1:58 am

[snip .. OT . . kbmod]

John Marshall
December 29, 2011 2:07 am

At Last!
I think that I have said all this several times, without all the complicated double integral math.
Its the pressure wot does it.

Petter Tuvnes
December 29, 2011 2:18 am

Very convincing, and in agreement with recent observations by Dr. R. W. Spencer (global temperatures and clouds relationship) and Dr. Svensgard (cosmic rays, the sun and clouds relationship).

December 29, 2011 2:19 am

They write,”Equation (8) suggests that the planet’s albedo is largely a product of climate rather than a driver of it.”
Could this be the key feature of the stablity of Earth’s climate? The warmists claim that climate is subject to unstable equilibrium, with a “tipping point” just around the corner. This UTC asserts that we are instead subject to stable equilibrium; negative feedback.
The warmists’ case can be demolished if their twin fallacies – positive feedback and high CO2 sensitivity – are shown to be hogwash. Nikolov and Mockton seem to have ’em in a pincer movement!

December 29, 2011 2:25 am

Uh-oh.. physics at last. Congratulations.

Oscar Bajner
December 29, 2011 2:31 am

I think I hear bells tolling, or death knelling,
or perhaps it’s the sound of Warmista colliding with reality,
or being slapped with a wet fish.
Anyway, The little Dog laughed to see such sport…

December 29, 2011 2:38 am

“According to our new theory, the climate change over the past 100-300 years is due to variations of global cloud albedo that are not related to GHE/ATE”
Been there, done that:
http://climaterealists.com/attachments/ftp/TheUnifyingTheoryofEarthsClimate.pdf
as regards the title and some of the conclusions
and
http://climaterealists.com/index.php?id=6645
“How The Sun Coul;d Control Earth’s Temperature”
as regards the effect of cloudiness changes.
and
“The fundamental point is that the total atmospheric warming arising as a result of the density of the atmosphere is a once and for all netting out of all the truly astronomic number of radiant energy/molecule encounters throughout the atmosphere. The only things that can change that resultant point of temperature equilibrium are changes in solar radiance coming in or changes in overall atmospheric density which affect the radiant energy going out. In the real world the most obvious and most common reason for a change in atmospheric density occurs naturally when the oceans are in warming mode and solar irradiation is high as during the period 1975 to 1998. The increased warmth allows the atmosphere to hold more water vapour so that total atmospheric density increases and the atmospheric greenhouse effect strengthens. This effect is far greater than any CO2 effect. When the atmosphere cools again water vapour content declines and the atmospheric greenhouse effect weakens. CO2 and other trace gases are far too small a proportion of the atmosphere to have any significant effect in comparison to the water vapour effect. Even the water vapour effect has never provoked any tipping point in the face of the primary solar/oceanic driver so CO2 could never do so.”
from here:
http://climaterealists.com/index.php?id=1562&linkbox=true&position=7
“Greenhouse Confusion Resoived”.
as regards the density issue.

H.R.
December 29, 2011 2:39 am

So the total mass of the atmosphere is more important than its composition? Pressure controls climate and albedo? The current models don’t work and here’s why?
That’s a lot to chew on.

December 29, 2011 2:41 am

Anthony,
What a wonderful end-of-year present. Dr Nikolov has neatly and convincingly explained what others (e.g. ‘The Slayers’) have been broadly asserting for some time but without, in my opinion, providing an intelligible or convincing argument.
Unfortunately, too many climate skeptics have hitherto been uncomfortable with the idea that a body (a layer of the atmosphere in this case) that is forced by the laws of physics (gravity in this case) to have (on average, of course) a fixed volume and pressure must, according to the gas laws, attain a fixed temperature. All that is required is a flow of energy (radiant, convective or conductive) through that body that is at least sufficient to replace the energy loss from it. This rate of flow is amply provided by the solar energy flux irrespective of gas composition or planetary albedo.
Yes, it looks like game, set and match for strong climate skepticism.
What a wonderful way to begin the New Year.

December 29, 2011 2:49 am

This is gonna tic off some…:)
I applaud the efforts to help rectify physical inconsistencies in the current GH concept.

December 29, 2011 2:53 am

A number of years ago here at WUWT I used the adiabatic lapse rates of air and CO2 to back-of-the-envelope calculate that rising in the Venusian atmosphere to 1 bar (Earth surface) pressure, the temperature would be within 10 degrees of the Earth’s, despite Venus being so much closer to the Sun. I reasoned then only that there was no “runaway” greenhouse tipping point and Dr Hansen was full of baloney.
I didn’t make Nikolov’s connection that the pressure itself was responsible for the effect, tho that seems seems to be what I found. Missing the obvious.
As I recall, adiabatically increasing our air atmosphere to Venus’ 90 bar brought the Earth temperature slightly higher than Venus’, tho I’m not that certain I did the calculation right.

December 29, 2011 2:53 am

If this is right then indeed it is paradigm-altering. It needs to be reviewed — proper peer review that aims to break it — and then we can see whether it fits the observed facts more closely than other theories. For example, can it explain the temperature record of the last 50 years? Or is the proposition that recent change is random? Whatever, it’s good to see some real innovative thinking brought to the subject, with the prospect of radically changing our understanding. Thank you!

Philip Mulholland
December 29, 2011 2:53 am

Comparative Planetology at its best!
Well Done.

Mydogsgotnonose
December 29, 2011 2:56 am

These people have independently invented lapse rate heating! When will modern scientists learn the basics before committing themselves to research and publication.
The good thing is that they show the IPCC climate models are bunkum. However, that is because ‘back radiation which they accept is really Prevost Exchange Energy so cannot do any thermodynamic work.
3/10 for effort.

December 29, 2011 2:57 am

“Global surface temperature is independent of the down-welling LW flux known as greenhouse or back radiation, because both quantities derive from the same pool of atmospheric kinetic energy maintained by solar heating and air pressure. Variations in the downward LW flux (caused by an increase of tropospheric emissivity, for example) are completely counterbalanced (offset) by changes in the rate of surface convective cooling, for this is how the system conserves its internal energy.”
Quite so:
http://climaterealists.com/index.php?id=7798
“The Setting And Maintaining of Earth’s Equilibrium Temperature”
“ANYTHING that adds energy to or takes energy from the air just above the ocean
surface merely adds to or subtracts from the rate of evaporation (not affecting the background
energy flow from water to air at all) and is converted to or from latent heat in the air in the
process. Of course conduction from water to air and upward radiation are also involved but
the energy taken up by them simply reduces the energy available for evaporation.
The equilibrium temperature of the oceans is in fact determined by the combination of
atmospheric pressure and the physical properties of the molecular bonds between liquid water
molecules and water vapour molecules. Critically it is dependent on the energy cost or gain of
the switch between liquid to vapour and back again. I need to explain that in some detail.”

Bruce
December 29, 2011 2:58 am

Nice update to the science debate. An additional (very small ~ 60 milliwatts per square meter Earth avg) correction to the Standard Planetary Grey Body surface temperature could be made taking into account the heat from internal radioactive decay of isotopes like K-40 and U and Th isotopes. I believe the general conclusions won’t be changed. I was pleased to see the comment comparing prior computer models and the core idea of the poster ” … Modern GCMs do not solve simultaneously radiative transfer and convection. This decoupling of heat transports is the core reason for the projected surface warming by GCMs in response to rising atmospheric greenhouse-gas concentrations. Hence, the predicted CO -driven global temperature change is a model artifact! “

Paul Maynard
December 29, 2011 3:14 am

Interesting.
Although I am not clever enough to agree or disagreee with the the maths/physics some observations.
1 This analysis shows that convection is the key which is the real reason behind the so called greenhouse effect – the stopping of convection.
2 Although I’ve not read Monckton’s second post yet, I’ve always assumed that his main goal was to use the IPCC’s theories and calculations to show that they were damned by their own work but were able to gloss over the essential weakenesses by bluster, assertion and the gullibility of the MSM and politicians needing to “save the planet”.
3 The famous Trenberth diagram seemed to be based upon the absence of the continued convection of heat away from the equator by the sea and by wind
Can’t wait for the WUWT audience response.
Cheers Paul

Allanj
December 29, 2011 3:15 am

Very interesting. It reminds me of an engineer I used to work with that, from time to time, would say, “This is an N dimension problem with N being a very large number.”
It seems that climate is an N dimension problem and scientists not part of the IPCC are increasingly identifying and examining new dimensions.
Exciting times to live in and WUWT is in the lead in providing a sounding board for alternative ideas. Congratulations.

December 29, 2011 3:16 am

My recall was a few degrees off. Here’s the post.
http://wattsupwiththat.com/2010/05/06/hyperventilating-on-venus/#comment-384746

Tony McGough
December 29, 2011 3:19 am

This will reward serious study. How refreshing to see basic Physics (Gas Laws, Black Body Radiation) along with common sense (greenhouses work by reducing convection) ; and then astronomical data brought in to study cases where there are no man-made effects.
I must try to follow this in detail, if my ageing brain can manage it. At least I am assured, from a first reading, that there are no “sunshine from cucumbers” fairy tales to swallow along the way. (But the integrations may be beyond my limited mathematics).
Did you, like me, not always feel that the effect of humans on the planet is puny? Feelings are not science, of course, but …

openside50
December 29, 2011 3:21 am

I look forward to the BBC’s Richard Black pushing this bigtime, after all when it comes to ‘climate change’ no story is too big or too small for it to escape him

Roger Clague
December 29, 2011 3:23 am

Increase of pressure causes increase of temperature of a gas, without increase in energy. Published over 200 years ago by Joseph Gay-Lussac. and descibed 100 years before that.
Thermodynamics of an ideal gas. The greenhouse effect is INDEPENDENT of the chemical composition of the atmosphere.
Great.

Martin Mason
December 29, 2011 3:29 am

Very good and understandable reasoning. How to get this where it will be seen though?

R. de Haan
December 29, 2011 3:34 am

This is what I call a “killer” publication.
As the “Unified Theory of Climate” provides us with a great explanation how atmosphhic pressure determines our climate the principal AGW doctrine is completely destroyed by these conclusions:
“Can a handful of trace gases which amount to less than 0.5% of atmospheric mass trap enough radiant heat to cause such a huge thermal enhancement at the surface? Thermodynamics tells us that this not possible.”
“In radiative transfer models, Ts increases with ϵ not as a result of heat trapping by greenhouse gases, but due to the lack of convective cooling, thus requiring a larger thermal gradient to export the necessary amount of heat. Modern GCMs do not solve simultaneously radiative transfer and convection. This decoupling of heat transports is the core reason for the projected surface warming by GCMs in response to rising atmospheric greenhouse-gas concentrations. Hence, the predicted CO2-driven global temperature change is a model artifact!”
This is a really, really great publication.

December 29, 2011 3:37 am

“….However, human-induced gaseous emissions are extremely unlikely to produce such a mass increase……”
Heretics!!, burn them, burn them all!!

December 29, 2011 3:38 am

Looks both complete and elegant. I’ll be rereading this for a while, but it’s so well-written that it will reward repeated reads.
Of course you don’t really need satellite observations to know that negative feedback is in full control; the fact that life has flourished for a billion years, through huge fluctuations of all the ‘forcing’ elements, is sufficient. If positive feedback had been significant, the whole mess would have gone critical in one direction or the other a long time ago.
Possibly the model doesn’t give enough credit to the dynamics of living things as part of the biosphere?

R Barker
December 29, 2011 3:44 am

Very interesting. Are there geologic or paleo evidence that confirm the atmospheric mass variations shown in fighure 9?

Brian Johnson uk
December 29, 2011 3:48 am

A breath of pure, fresh scientific observation that cannot be swept under the Warmist carpet [now threadbare] and I expect the unbiased Richard Black [BBC] to give this Unified Theory of Climate the full exposure it deserves [and climate realists expect].

Sparks
December 29, 2011 3:49 am

I’ve read it twice, It’s actually not bad.

Bill Illis
December 29, 2011 3:50 am

I’ve been waiting for someone to solve the gas law temperature equations – Perfect fit.
I think the paleo-climate temperatures used by the authors are over-stated. The Eocene Thermal Maximum temperatures should be reduced to about +6.0C rather than +16.0C (they might be using a version of Zachos 2005 which was built around polar temperatures rather than global temperatures and then someone else exaggerated it even further).

Keith Gordon
December 29, 2011 3:54 am

Could this be Game Set and Match?.
This is the most sensible theory I have heard to explain how the total atmosphere works. The laws of nature work the same throughout the Solar System only subject to different parameters, If it works for Mars Venus etc. why not the Earth. I shall look forward to reading further comments from our illustrious contributors.
Happy New Year to Anthony and all, keep up the enquiring process, the truth will out.
Keith Gordon

simpleseekeraftertruth
December 29, 2011 4:00 am

If this pans out then Nobel Prizes (at least) are in order.

Peter Miller
December 29, 2011 4:00 am

Yet another inconvenient paper which will not be considered for inclusion in the next IPCC fantasy report.

Chuck L
December 29, 2011 4:08 am

Although much of the math is beyond me, this looks to me like an entirely new climate paradigm and is a major game-changer!

Stephen Richards
December 29, 2011 4:09 am

Ok, it’s been many years since I did 3 dimensional intergration ( Debye’s theorem of specific heat) and differential calculus so I haven’t checked the maths but the premise is something that has been proposed for many years the changes on mars for example) and on which futher research may have been blocked by the team and NGO sources. I like immensely the thought processes behind the work and the way in which they have used ‘publicy available data’. From the basics the maths look ok and the equations fit the words. So, overall, a good piece of work and, indeed, groundbreaking. Now let’s see if any other ‘scientists’ come out from under their ‘comfort blanquets’ to confirm / deny this work through a correctly managed scientific process.
Well done the authors and by the way, it’s worth noting, that if not for WUWT this work may never have been made available to the general public. So well to Anthony and the crew.

Rob L
December 29, 2011 4:25 am

Interesting. How does this explain ice ages then?
Ice ages have global temps about 6°C below current which by this theory would require something like the bottom 600m of atmosphere to disappear (about 70-80kg/m²) and then reappear at the end of the ice age. Is air sufficiently soluble in cold water or in ice caps for that to happen? Seems very unlikely. Also what then is the mechanism for falling into or coming out of an Ice Age?
I think that planetary albedo must be the primary effect and this is driven principally by clouds, and snow/ice cover though I do like your theory of atmospheric mass loss to describe the gradual drop in temp over hundreds of millions of years.

Bloke down the pub
December 29, 2011 4:29 am

If only James Hansen had studied the atmosphere of the other planets. Oh, er…
I know that if I knew what I didn’t know back then, what I now know that I don’t know now, I would have tried to understand maths a bit more when I was at school.

December 29, 2011 4:29 am

Many things wrong with this paper. I’ll pick out just one – Fig 8 on surface temperature history, for which the source is given as Hansen 2008. But that’s not in the list of references. So I did some searching, and the only paper that seems to qualify is this one.
Well, it has two graphs which indeed seem to wiggle match. The problem is what they mean, and scaling. Fig 3 is indeed a temperature, but it is global deep ocean temperature. And the scale is different – less total range. The other is Fig 4, where both the number scale and wiggles match reasonably. However, the quantity plotted is total forcing, and the units are W/m2. It matches Fig 3 because it was inferred from the deep ocean temperature.
I cannot see how such an extraordinary surface temperature history could be derived from that paper.

Hoser
December 29, 2011 4:33 am

First principles and experimental evidence (solar system planetary body comparisons) lead to some very interesting conclusions. Ha! They say, “the science is settled.” Not exactly. Let the squealing begin.

Curiousgeorge
December 29, 2011 4:36 am

It’s a very pretty poster. Visually balanced between the left and right sides, and colors are well thought out.

wayne
December 29, 2011 4:42 am

It’s all in the per-point absorption of the incoming radiation on a sphere as so eloquently put in this paper. Everyone here has looked at agw climate ‘science’ and particularly Trenberth-Kiehl diagrams of a flat disk type of absorption of averaged radiation and have all come to the same conclusion… this all seems so physically wrong!! Wrong, wrong, wrong!!
There being a world of difference between the energy found 1 cm below the surface and 1 cm above the surface at every various time and location on this globe and finally someone has tackled this fourth power non-linear aspect across a sphere in respect to Earth’s state temperatures. I’m elated (and finally relieved)!!
Thank you Drs Nikolov & Zeller for this real science. Well done.

AusieDan
December 29, 2011 4:46 am

At first read, this is what I have been looking for.
I’ll read it again and again until I completely understand it.
I will be particuarly interested in the reactions of serious minded luke warmers.
Jeff Id, Steve Mosher and Professor Judith Curry for starters.
This may well be the game changer.

December 29, 2011 4:50 am

I’d like to see more about the effect of ocean temperature on atmospheric volume.

December 29, 2011 4:52 am

atmospheric mass, not atmospheric mix ? who’d have thunk it

Espen
December 29, 2011 4:54 am

Very intriguing paper. If this is correct, it could be really paradigm-changing.
I’m way too much of a layman to judge the quality of this paper, but one thing which immediately occurred to me as a “test question” is if mantle degassing during mega volcanic events like the deccan or siberian traps eruptions really was substantial enough to generate a significant change in atmospheric pressure? “AGW-friendly” geological theories suggest that the (obviously huge) CO2 emissions during these events caused the rise in temperatures.

John-X
December 29, 2011 5:05 am

Great. So now all that’s left to figure out is how big a tax increase we need to combat this Antrhopogenic Excess Pressure.
And we can reincarnate the Chicago Climate Exchange (CCX) as the Chicago Air Pressure Exchange (CAPE).
I will start selling Pressure Offsets. You send me $10,000, and I will fill up a scuba tank and bury it in the ground.
We’ll get people we know and trust as editors at the journals and in the media, and condemn deniers of Man-made Pressure Increase.
The IPCC will function just as it always has – keeping the money flowing to those on The Team.
The annual party in Doha in November will go on just as scheduled.
Everything is going to be just fine.

Andre
December 29, 2011 5:06 am

Nick says “Many things wrong with this paper. I’ll pick out just one”
Well to substantiate the many things, you should pick out many things. So far I only see the omission of one reference.
Anyway Hansen et al’s paleotemperature reconstruction depends on a lot of suppostions, for instance that the isotope ratios of Benthic Foraminifera adequately represent the local sea water isotope ratios. However, the chemical composition and pH play an important role. Moreover different species have different reactions and species die out and new appear all the time.
Then there is the problem of remagnetising of minerals that basically calls the whole reconstruction of plate tectonics into question. Paleo-north is not north anymore and the reconstruction of the earth temperature may simply be only a reconstruction of how much land mass was passing poles in the tectonic plate movements

Darkinbad the Brightdayler
December 29, 2011 5:09 am

No disrespect intended but it should be titled: “Towards a unified theory of climate”.
You have yet to demonstrate completeness or internal consistency across all factors.

December 29, 2011 5:12 am

Given our planet’s surface is mostly water, this seems so intuitively obvious.

December 29, 2011 5:16 am

I fully appreciate the fact you properly capture the geometry of the radiative inbound flux to be absorbed (something Lord Monckton has not addressed). The solar flux to be absorbed is much smaller than most people understand given the fact there is only a small surface that is being radiated full-on at any point in time.
When you include the incident of radiation angle on the surface to absorb, you find the watts/meter will go way down (from the maximum point of radiative flux the amount of radiation drops as you move towards the Day-Night terminator, where it reaches zero).
This means at any instance the emission surface area (the full global surface at your preferred emissions altitude) is much larger than the area absorbing energy from the Sun.
But on these global scales these are massive differences and have an effect – as you note.
Now to the other large effects.
(1) The Earth’s Oceans are also an absorber and transport mechanism – a massive heat sink. Energy can be captured and retained for decades. To my mind this is the great energy balancing mechanism – not the atmosphere. The atmosphere is a transport layer between space and the mass of water. A proper model would probably look more like that, where the atmosphere attenuates absorption and emission.
(2) Emissivity is a function of wavelength and the matter emitting and the matter’s temperature. If the ocean is the true absorber and emitter, its emissivity is the driver. This means it is not constant (because of the temperature factor). Add in the cloud/atmosphere attenuation (which returns emitted/transported heat back to the ocean and land) you have a very complex model. What you require is some reasonable factor averaged over lat and long that represents the attenuation in both directions.
(3) Core heat: the Earth is not warmed solely by the Sun. We have a hot ball of molten iron at the core which has estimated temperature and heat transport flow. Much of the difference between a warm and cold planet is the molten core. Look at the Moon Io versus Mars. Io is much farther distant, gets much less solar radiation, yet is much warmer than Mars (whose molten core cooled off long ago). Our atmosphere remains in place due to the molten core and subsequent magnetic field that acts like a protective shield. Without that you can forget about any Green House effect. We do not know if our planet experiences changes in gravitational forces over millennium as our solar system travels through the galaxy. Increased overall gravitational forces may increase the temperature of our core – who knows. Definitely not the tree ring folks.
(4) Solar radiation spectrum: We get lots of radiation in different forms. Some come right through and others are stopped, deflected or attenuated (e.g., magnetic field). The solar flux coming in is also a function of wave length.
Anyway, You definitely have addressed the correct geometry, so are much closer. I would suggest you add in the hot core as steady heat source that will probably close your gap with Monckton a bit.
Cheers, AJStrata

Myrrh
December 29, 2011 5:16 am

We’ve always had a unified theory of climate. The general figure as given as I’ve seen it is that the whole of the atmosphere is Earth’s greenhouse and therefore all the gases in it are greenhouse gases, predominantly nitrogen, oxygen and water.
Without the atmosphere the Earth would be -18°C, but, it only gets the +33°C warming to 15°C via the dynamics of all the greenhouse gases –
without any atmosphere -18%deg;C
with atmosphere but minus the Water Cycle – 67°C
The water cycle cools the Earth by 52°C
– to bring it down to the 15°C from the 67%deg;C it would be with the main greenhouse gaseous ocean of nitrogen and oxygen above us, pressing down on us a ton/square foot.
Carbon Dioxide is utterly insignificant in that. Heavier than air anyway it will always sink displacing air without any work being done on it, and anyway spontaneously joining with water vapour in the atmosphere to form carbonic acid, all pure rainwater is carbonic acid, as is dew, fog, and so on, so is part and parcel of the ‘greenhouse gas’ cooling as water vapour releases its heat higher in the atmosphere and condenses out as water, falls to Earth.
What needs to re-written is the AGWScience Ficition physics written out.
As here:
Re your Figure 4: whatever happened to the direct from the Sun thermal infrared radiation reaching Earth?
Some of us remember being taught that the heat reaching Earth direct from the Sun is the invisible thermal infrared and that visible light is not thermal energy, it can’t heat land and oceans. Water is transparent to visible light, therefore, visible light does not heat the oceans, for example.
Who originally created this ‘energy budget’? Why are you all paid scientists taking it seriously at all since this ‘basic physics’ is of a fantasy world?

Richard111
December 29, 2011 5:17 am

I think I see a tipping point here. If only I could follow the math more easily. I think Tallbloke had this on his blog first. 🙂

December 29, 2011 5:19 am

Update to last post:
Ugh. Forgot one item. On any given day or season, the height of the atmosphere changes. Warmer periods it expands, cooler periods it contracts. So much so we have to adjust the models for satellite orbits that are in the Low Earth Orbits (LEOs). In addition, you have the radiation belts – by definition these are acting like another attenuation layer as does the atmosphere. There composition also changes over time and would seem to be a second radiative surface and absorption point. Much higher up – much larger surface, much lower density of matter and mostly plasma. But another layer all the same.
Cheers, AJStrata

Barefoot boy from Brooklyn
December 29, 2011 5:24 am

Well, I don’t completely get it. Nor would one expect me to, given my ignorance of the math involved. Nikolev & Zeller say that the total mass of the atmosphere conditions the response of surface T to insolation and other active factors. But over the medium term, say the past 500-2,000 years, if the mass of the atmosphere is constant, this still leaves open the question of atmospheric sensitivity to other presumed “forcings,” such as GHG. I’m wondering that even if the thermodynamics of this thesis prove to be sound, the concept is vulnerable to warmist attacks on the sensitivity assumptions. At least, they still seem like assumptions to me. Just asking so as to learn.

ShrNfr
December 29, 2011 5:25 am

Oddly, all these papers seem to ignore the contribution to the surface temperature from the nuclear reactions in the earth’s core. I grant it is probably not large and if somebody could point me at a paper that guesstimates it, I would be most obliged. There is a reason that the core and mantle are a bit molten after 4 billion years and it is not CO2.

December 29, 2011 5:25 am

Andre says: December 29, 2011 at 5:06 am
“So far I only see the omission of one reference.”

I don’t mind (much) having to look for the reference. But a lot is deduced from Fig 8, and if it is from that paper, it looks like they have just plotted something that isn’t global surface temperature anomaly.
As to other things, we could start with the 133K greenhouse effect. That is based on an airless planet where the temperature goes to absolute zero at the poles. That’s not what the 33K GH effect calc is based on. And then they say

“This raises the question: Can a handful of trace gases which amount to less than 0.5% of atmospheric mass trap enough radiant heat to cause such a huge thermal enhancement at the surface? Thermodynamics tells us that this not possible.”

The figure is their own invention. But thermodynamics does not tell us that it is not possible.

commieBob
December 29, 2011 5:26 am

According to Eq. (2), our atmosphere boosts Earth’s surface temperature not by 18K—33K as currently assumed, but by 133K!

That’s a big discrepancy. Is there evidence to support it? What evidence is there to say what the Earth’s temperature would be without an atmosphere? We could ask about the temperature of the moon.

Temperatures on the Lunar surface vary widely on location. Although beyond the first few centimeters of the regolith the temperature is a nearly constant -35 C (at a depth of 1 meter), the surface is influenced widely by the day-night cycle. The average temperature on the surface is about 40-45 C lower than it is just below the surface. (http://www.asi.org/adb/m/03/05/average-temperatures.html)

The above quote has the average surface temperature of the moon as -75 to -80 deg. C. So, what is the average temperature of the Earth?

The average temperature of Earth according to NASA figures is 15°C. (http://www.universetoday.com/14516/temperature-of-earth/)

That gives a difference in average temperature between the Earth and Moon of about 95 deg. C. (or K if you prefer) That gives some reason to believe the 133 degree figure.

chuck nolan
December 29, 2011 5:29 am

David Jones says:
December 29, 2011 at 1:42 am
I like the touch of including the graph from Pen State University Department of Meteorology as Figure 1.
Shows chutzpah!
———————————————————
I concur plus using Hansen for Figure 8. Dynamics of global surface temperature during the Cenozoic Era reconstructed from 18O proxies in marine sediments (Hansen et al. 2008).
Using their data Nikolov & Zeller show there is a better answer than CO2.
Are you listening Lisa?

cal
December 29, 2011 5:40 am

I think there are a lot of interesting points made but I am not convinced that the overall case is made. For example if the downwelling radiation is not dependent on GHGs why is the fequency of this radiation characteristic of water vapour and carbon dioxide. If these molecules were not there where would the radiation at these frequencies come from? There would be no downwelling radiation if the atmosphere was just oxygen and nitrogen.
The reason I do not find the logic completely convincing is as follows.
The downwelling radiation is dependent on the number of molecules capable of radiating ( and thus absorbing ) infrared photons. These photons must be within an energy band characteristically emitted by a body at a temperature of about 300C because this is where the energy originally comes from. The number of those molecules is dependent on the percentage of those molecules in the atmosphere and the total number of molecules in a given volume of the atmosphere which, at a given temperature is proportional to pressure. So unless the atmosphere contains no greenhouse gases of any sort the greenhouse effect will always be proportional to total pressure. Furthermore, as the pressure increases, the effective concentration of greenhouse gases needed before the effect saturates will drop to a few parts per billion in the case of Venus.
So I cannot see how the correlations presented disprove the GHG effect. I do not believe that GHGs are responsible for the CHANGES in our climate because I think the effect is near saturation and now too small to be significant, but it would take more than this to convince me that they do not contribute to the steady state system.

tallbloke
December 29, 2011 5:44 am

Great to see this getting the mighty WUWT megaphone after we scooped it at the Talkshop yesterday:
http://tallbloke.wordpress.com/2011/12/28/unified-theory-of-climate-nikolov-and-zeller/
I’ll be following this discussion intently and collating salient comments on our post for additional consideration.

JPeden
December 29, 2011 5:55 am

It’s nice to see Gravity finally brought back into atmospheric science.

wayne
December 29, 2011 5:58 am

NK2011: “Pressure by itself is not a source of energy! Instead, it enhances (amplifies) the energy supplied by an external source such as the Sun through density-dependent rates of molecular collision. This relative enhancement only manifests as an actual energy in the presence of external heating.”
True, this is not the same discussion as if it was pressure in a tire. The air in a tire does not have radiation passing through it. The air in our atmosphere does.
Also, there is another aspect I have always had with agw climate ‘science’. This is so radiative logic, so to speak. As the pressure increases as you descend into our atmosphere the molecules get closer and closer together per the increase in density, mass per volume. Now interplay that with a factor termed “mass extinction coefficient” that increases markedly and non-linearly the deeper and deeper toward the surface you go and is solely dependant on the mass (density). The same effect work in reverse for outgoing LW radiation. In essence this supplies the energy (P•V in joules) needed to physically support the atmospheric mass above each point at various altitudes. Otherwise the atmosphere would simply collapse.
This seems to mean that the often ignored ~78 wm-2 absorbed incoming SW and the net ~23 wm-2 of LW outgoing radiation (that is 396-333=63-40=23 per K-T) plays as one of the largest factors throttling the mean temperatures at a given latitude and altitude. Cold increases the density, pressure does not necessarily change, from the surface upwards placing more absorption lower. The opposite for increase temperatures, force these absorptions higher in the atmosphere and it all depends only on the mass this radiation is passing through.
Forget the bicycle tire analogies.

richard verney
December 29, 2011 5:58 am

I remember as a young boy back in the 60s/70s when there was great interest in space that my Dad told me that the reason why Venus was hot was due to the pressure of its atmosphere and he iullustrated it with a bicycle pump. I guess that different science was being taught back in that day and age.

MattE
December 29, 2011 6:00 am

That is the most compelling introduction I have ever read. Trillions of dollars at stake and lots of reasons to think the ‘consensus’ hasn’t got it right.

December 29, 2011 6:04 am

“Hence, the atmosphere does not act as a ‘blanket’ reducing the surface infrared cooling to space as maintained by the current GH theory, but is in and of itself a source of extra energy through pressure.”
It cannot be overemphasized that ALL of the energy comes from the Sun, contrary to what the authors seem to be saying in that quote. The governing hydrostatic vertical pressure distribution only determines the vertical temperature distribution, not the actual temperatures, because the atmosphere’s ability to hold heat energy from the Sun increases with the pressure. Filling the atmosphere with heat energy is like filling a glass with water, and the glass is wider, and so can hold more water, the farther down you go, that’s all. The actual temperature at any given pressure is basically fixed by the incident solar energy, with local and/or transient modifications by clouds or other particulates, and weather phenomena in general. I would offer two warnings about this article: 1) As in their claiming that the atmosphere is a source of extra energy, they are loose in their statements of physical cause and effect, to the point of being fundamentally misleading about the real physics; and 2) Their relationship (Eq. 7 and Fig. 5) basically shows the ATE does not explain the surface temperatures of the bodies with very thin atmospheres at all, and quickly becomes relatively insensitive to changes in surface pressure beyond the “knee” in the curve. Only Earth and Titan fall in the sensitive range of the equation, and the Earth data point is slightly above the mathematical prediction line while Titan is slightly below it. My detailed Venus/Earth temperature comparison, at pressures over the range of Earth tropospheric pressures, shows there is no albedo effect upon the Venus/Earth temperature ratio, though Venus reflects 70% of the solar radiation while Earth reflects only 30%. I look upon this article as an attempt to harmonize the radiation transfer theory with the real thermodynamics of the atmosphere, basically through application of the ideal gas law, but from my Venus/Earth findings (that the atmospheres are warmed by direct absorption of solar infrared radiation, and that there is neither the consensus-defined greenhouse effect nor an albedo effect) I know this is in the end a vain attempt. This article is just a demonstration of the predominance of a governing hydrostatic vertical temperature lapse rate in sufficiently massive planetary atmospheres, in my opinion. That is something my Venus/Earth comparison already well indicates, and it boils down, in climate science, to saying “The Standard Atmosphere model of Earth’s atmosphere is THE equilibrium state of the atmosphere, and any and all deviations from it constitute the weather as we know it.”

December 29, 2011 6:08 am

Very interesting. Ned and Karl, you gave me enough material to study and think about for the next days!

D. W. Schnare
December 29, 2011 6:11 am

I would like to see the predictive power of this theory, both through backcasting and forecasting over a thirty year period. Can this theory predict temperature (pressure) variations observed from 1980 thru 2010?

Leonard Weinstein
December 29, 2011 6:14 am

The analysis is elegant and has many good points. Unfortunately, It is not quite fully correct. If an atmosphere had only gases with no long wave IR absorption, there would be no radiation absorption up and no back radiation at all. An atmosphere of say Argon (with no water present) is such an example. For that case, surface albedo and solar insolation would be mainly important. There would be an atmospheric effect, to transport some of the conducted and convected energy from lower latitudes to higher latitudes, where the atmosphere could conduct some energy to the local ground (there still would be heating from the surface to the gas and back), and the integrated area 4th power radiation out would be modified compared to no convection. However, if the gas did not transport much of the surface energy compared to the direct radiation out, the pressure and volume of the gas would not affect the average surface temperature. There has to be radiation absorbing gases (or aerosols) for a greenhouse effect. There would still be an adiabatic temperature profile as long as mixing from convection were strong enough, but the profile would be locked to match the surface value. With a greenhouse gas (or aerosols), the profile is locked to the temperature at the location of average outgoing radiation, and the adiabatic effect results in the atmosphere below this height warming. For Venus, the outgoing radiation comes from near the top of the atmosphere, so the adiabatic effect is dominate to determine surface temperature.
In the case of all real planets, there is always present greenhouse gases (CO2, water vapor, methane, etc.) and solid or liquid aerosols (dust, water drops, etc.). However, the present writeup is correct that the pressure and volume of the atmosphere determine the surface temperature. It is how the volume is determined. It is changes in pressure and volume due to greenhouse gases that was not considered. Much of the mass change for Earth’s atmosphere (and thus pressure) is controlled by water vapor content. However, the increased CO2, while not changing mass much, does change the temperature slightly by absorbing outgoing radiation (as does water vapor), and the slightly higher temperature does change the volume slightly. The whole concept of back radiation is misplaced. It is not back radiation that causes the extra heating, it is the movement of the average location of the outgoing radiation due to absorption. Feedback is another issue, and I agree there seems to be negative feedback for CO2 due to change in albedo (due to dominance of water vapor). That is a separate issue.

wayne
December 29, 2011 6:17 am

Barefoot boy from Brooklyn says:
December 29, 2011 at 5:24 am
… But over the medium term, say the past 500-2,000 years, if the mass of the atmosphere is constant, this still leaves open the question of atmospheric sensitivity to other presumed “forcings,” such as GHG.
>>>
Read within the text again. Not variations in any trace GHGs but short living variations in the albedo… variations in the anount of cloud cover, plant life cover and color, differences in the color of the oceans (plankton), ice/snow cover, aerosols, even solar insolation itself, etc.

Jimmy Haigh
December 29, 2011 6:17 am

Phil Jones will be checking all these formulae out on Excel right now I should imagine.

Chris B
December 29, 2011 6:18 am

Mike McMillan says:
December 29, 2011 at 3:16 am
My recall was a few degrees off. Here’s the post.
http://wattsupwiththat.com/2010/05/06/hyperventilating-on-venus/#comment-384746
__________________________________________
I remember the post, and thinking that pressure was a better explanation for Venus’s high atmospheric temperature than CO2 content.
Your mid 2010 post also got me thinking that there must be a calculable increase in Earth’s atmospheric temperature due to the small increase in atmospheric volume/pressure due to the burning of fossil fuels ( basically converting them from an upper crust liquid/solid to an atmospheric gas ), and simply due to the release of heat in the combustion process. Both increases are probably relatively small but most likely contribute to an increase rather than a decrease in atmospheric temperature, notwithstanding possible negative feedbacks.
Perhaps someone has already done the back of an envelope calculation.

December 29, 2011 6:20 am

Harry Dale Huffman says:
December 29, 2011 at 6:04 am
Extremely well put Harry, thank you.

December 29, 2011 6:31 am

I know a lot of readers would love to jump on this bandwagon & call the hypothesis good at this point, but this is a radical departure from current thinking & as it has been said, “extraordinary claims require extraordinary proof “. I think this paper is a good start by using the lines of evidence presented, but I would call it a start , not a completion
I see the general merits of this hypothesis, but would like to see many more studies of specifics , both by these authors & by other independent authors supporting this idea before I buy off on it. At this point, I would like some one to cleverly & independently develop an estimate of Earth’s atmospheric mass with time, calculate the associated surface pressures & compare it figure 9 – that would be a good start.
I see the need for a much more extensive investigation of implication 4c (albedo & cloud cover), as that is critical to explaining short period variations, such as the lack of warming over the last decade or so. The authors state this is a result of their equation 8. There probably needs to be multiple papers using modern datasets to further test this hypothesis & equation 8.
The one red flag I see is the long term predicted pressure profile in figure 8. Eocene pressures max out at ~ 185 kPa – that’s approaching double today’s standard pressure of ~ 101 kPa !! I would think at those kind of pressures there would be some biological effects which might be manifested in the fossil record – how life adapted to such high pressures.
The authors suggest mantle outgassing rates as a source of atmospheric mass variation. That would only be on the “input” side of the equation. What about the “output” side of the equation – ie mass being taken out of the atmosphere (if all we did is add to the atmosphere, the it would get continually heavier / hotter with time, only the rate of increase would change). So, the big factor there is taking CO2 out of the atmosphere via carbonate rock production. As such, there ought to be some sort of correlation of temps to carbonate rock production (ie in periods of increased carbonate rock production, we ought to see falling temps, all else being equal).
Similarly, we are all familiar with the Vostok ice core – temp data offset & correlation :
http://wattsupwiththat.com/2009/01/30/co2-temperatures-and-ice-ages/
So, it would be good to see what these changes in CO2 translate to in terms of changes in atm pressure & see what the implied temp change would be – another good way to investigate the hypothesis.
Sp, nice start – I will be interested to see where the research goes from here.

December 29, 2011 6:33 am

Why the theory of man-made climate change is fishier than a carp’s armpit. In one page. Brilliant.

Kelvin Vaughan
December 29, 2011 6:33 am

Bloke down the pub says:
December 29, 2011 at 4:29 am
If only James Hansen had studied the atmosphere of the other planets. Oh, er…
I know that if I knew what I didn’t know back then, what I now know that I don’t know now, I would have tried to understand maths a bit more when I was at school.
It’s never too late to learn. just harder work when your older!

Pamela Gray
December 29, 2011 6:34 am

Very interesting. And it seems, could be modeled rather quickly and tested against observations. I know this will cause a collective groan but I think this deserves further serious work and funds.

richard verney
December 29, 2011 6:36 am

commieBob says:
December 29, 2011 at 5:26 am
////////////////////////////////////////////
This is not the sort of paper that you can glance at. It demands to be read slowly and pondering on propositions as you go. I have not yet had the time to properly read it.
As regards the comparision with the Moon’s temperature aren’t the authors calculating that both the Moon and the no atmospheric Earth have the same temperature of 154.3K? Is this not what the SPBG Mean Surface Temperature calculation suggests?
Having calculated the SPBG Mean Surface Temperature of 154.3K they use this assessment to then assert that GHE is 133K (ie., about 287.3K).
Thus perhaps your question goes to the assessment of the APBG Mean Surface Temperature and whether the principles applied to that assessment are correct.

Chris B
December 29, 2011 6:37 am

Martin Mason says:
December 29, 2011 at 3:29 am
Very good and understandable reasoning. How to get this where it will be seen though?
_____________________________________
Maybe a title change to something more acceptable to the CAGW sympathizers. You know, pull them in with the title to get them to read it. It would be interesting to see how far into it they got before saying, “Hey, wait a minute.”
How about: Unified theory of Climate Change saves Polar Bears.

James of the West
December 29, 2011 6:42 am

Rob L – glacation requires lots of water – if some of that water came from the atmosphere due to falling atmospheric temperature unable to hold the H2O as a gas could it result in pressure changes of that magnitude? What % of the atmosphere by mass is H2O and if we lowered the average temp of the troposphere by 10 degrees how much of that water would condense out as precipitation (snow and ice) over land?

Keith
December 29, 2011 6:49 am

Will and Harry:
How would you modify their equations?

Jimmy Haigh
December 29, 2011 6:52 am

Does this mean that “mountain top removal” mining will reduce the global average temperature?…

December 29, 2011 6:53 am

guys, I’m thinking this paper is a bit slick, maybe a knee-jerk reaction, totally a gut thing on my part.
I am reminded yet again of the utility of science, the subservience to greater ‘gods’ i.e. bosses, status quo, et al.
there’s a book, ‘the sleepwalkers’. (our discoveries were accidental and unseen) there might be unseen discoveries in this paper.
pressure is a ‘new’ view to me, and seems to fit in with theories of critical mass, and the fate of astronomical bodies, to be a neutrino or a black hole.
the real deal, from my point of view, is litigating against IPCC contributers for fraud, and IPCC benefactors for lack of due diligence.
a great blog, the best in my world !

richard verney
December 29, 2011 6:55 am

wayne says:
December 29, 2011 at 5:58 am
,,,,,,Forget the bicycle tire analogies.
//////////////////////////////////////////////////////
Wayne
I do not think that the bicycle tyre analogy is a bad analogy. What people tend to overlook is that side wall flexing of a tyre will maintain the air temperature within the tyre. This is well known in motor racing and why it is necessary to run a racing car at speed if good mechanical grip is to be maintained and why if a car is stationary for any period tyres lose heat and pressure leading to a loss of grip.
Earth has the equivalent of this side wall flexing, it is called the diurnal/atmosheric bulge. I would suggest that the constant flexing of this bulge is an important factor in maintaining the heat in the atmosphere which heat in the first instance is derived by the pressure of the atmoshere (gravitational forces compressing the atmosphere). This atmosheric bulge is driven by the rotation of the Earth, the Sun/Earth gravity interaction and no doubt also by solar irradiance heating the atmosphere/surface as Earth spins on its axis and presents an ever changing face to the sun.
As the authors of this paper note, one needs only the amount of energy required to replace the heat loss for the atmosphere to sustain a broadly stable temperature.

Pamela Gray
December 29, 2011 6:55 am

For those of us who have been around here for awhile the thought of atmospheric pressure came up frequently when discussing the seemingly different responses to CO2 concentrations among the planets. Those different temperature responses seemed to be explained by atmospheric pressure differences. So well in fact that CO2 could be ignored. Any kind of gas mix would work if pressure were great enough. And that changes in pressure was the direct cause of temperature changes on such a scale.
Fertile ground for examination it would seem.

ScuzzaMan
December 29, 2011 7:04 am

That seems to make sense …
What **really** bugs me about the whole “precautionary-principle-climate-change-will-kill-us-all” crowd, is why the precautionary principle **doesn’t** apply to the single most demonstrably savage and widespread cause of calamity known to human history: giving more power to fewer people?
Things that make you go ‘Hmmmmm….’ …

DirkH
December 29, 2011 7:12 am

Oh look. O2 has some emission/absorption lines in the IR range. Paywalled, 1970. I’ve only read the abstract, I don’t have access.
http://www.opticsinfobase.org/abstract.cfm?uri=ao-9-6-1419
(I was wondering how, in the theoretical absence of the usual GHG’s, CO2 and H2O, the atmosphere could radiate to space. These emission lines would provide a means to that, and given the high partial pressure of O2, an efficient one. FUNNY how the IPCC scientists have never mentioned this. At least I don’t remember.)

Mydogsgotnonose
December 29, 2011 7:16 am

Having seen the comments pile up, it’s worth remembering a few important facts about this paper.
1. They re-invent lapse rate heating. Why didn’t they read up about the subject?
2. The real present GHG warming of the Earth is ~9 K; the IPCC’s claim of 33K is a not very clever deception..
3, The authors fall into the trap of imagining that ‘back radiation’ can do thermodynamic work. Asahenius was wrong as any professional engineer experienced in modelling and designing heat transfer will confirm,
4. It’s nicely presented but seriously flawed because of 3. However, it if destabilises the IPCC fraud, there may be some good out of it!

DirkH
December 29, 2011 7:16 am

DirkH says:
December 29, 2011 at 7:12 am
“Oh look. O2 has some emission/absorption lines in the IR range.”
Oh sorry – they don’t mention O2 but OI so that probably means oxygen ions (?), not molecular oxygen.

Pamela Gray
December 29, 2011 7:18 am

I agree with others in that this is a theory ripe with many testable hypothesises. The beauty of the theory is that gas mixes appear not to matter. Therefor planetary studies outside of Earth could be used to determine the robustness of this theory. If it is just a special case here on Earth, I give it less reliability as well as validity. A paradigm shift by nature must prove to be robust across varying conditions.

Bob
December 29, 2011 7:21 am

My scalp tingled as I read this. I still have to go back and read it more closely, but this should become a a better fundamental physical description of the dynamics of global temperature.

December 29, 2011 7:22 am

After appearing on Fox a couple of times on this matter, it is gratifying to see that the very things got a polemic launched against me by leftist environmentalists are being brought up here.
I opined that the amount of co2 launched into the atmosphere could have no effect on the earths energy budget, and so there could be no true change in temperature, as energy can neither be created nor destroyed. Co2 being natural to the system, could therefore not have an effect on the system that could remain permanent. Of course that got people saying that I did not understand the GHE, which to me is a misnomer anyway as the “trapping blanket” so to speak is not a greenhouse pane, but more or less something that is not even a blanket but something you might find in Victorias Secret, thin, flimsy etc. ( that is supposed to be a joke to make my point, okay) So this statement jumped out at me:
Can a handful of trace gases which amount to less than 0.5% of atmospheric mass trap enough radiant heat to cause such a huge thermal enhancement at the surface? Thermodynamics tells us that this not possible.
I believe my quote on FOX referred to it as a trace gas needed for life on the planet that could have no effect on the energy budget of the earth, but in the same mode
Also gratifying to read:
B) Modifying chemical composition of the atmosphere cannot alter the system’s total kinetic energy, hence the size of ATE (GHE).
The other part was simply reducing the matter to La Chateliers principle, which simply put states that systems in distress fight to return toward normalcy
“Any change in status quo prompts an opposing reaction in the responding system.”
It is my belief that while research into the complex may reveal that there is more to all this than the simplistic views that I have become convinced rule the day ( one of them being the big natural drivers and basic principles far outweigh the effects of a gas natural to the system), it is highly unlikely that it will produce anything beyond people that wish to shackle the progress of society because of their desire for a) control b) worship ( they are looked upon as smart) or c) to make money off the whole thing. In the private sector, one is paid for the results of the research, ie I study the weather ( not paid) make a forecast and am paid if I am correct enough for the client to find value. The whole AGW industry is based on research and non verifiable results, leading to the idea that any result verifies the research. This is a God send for anyone who does not wish to be judged on actual results or being forced to compete!
One more thing… While I have my simplistic sectarian reasons listed above for what I believe, and I am gratified to see some of the ideas at least show up here in what looks to be a an exhaustive, well researched and as the authors will soon find, a paper to be attacked and ridiculed, it may all come down to what I have concluded after watching the majesty of the creation all these years:
Eccl 1:9
What has been will be again, what has been done will be done again; there is nothing new under the sun.
That really gets them mad!

Viv Evans
December 29, 2011 7:23 am

Very interesting paper – I definitely must read this agin, very s-l-o-w-l-y!
And I’m looking forward to the excellent comments here, from which I hope to learn even more.

John Wilkes Booth
December 29, 2011 7:25 am

“a) the product P×V defines the internal kinetic energy of a gas (measured in Jules) that produces its temperature; ”
This is wrong and anyone with an air compressor can prove it. Pressurize a tank of air. It will initially be warmer according to the idea gas law. But wait a few days. Its temperature will equalize with the ambient air outside the tank. Yet the pressure and volume of the compressed has not changed.
What the authors propose would in effect be a power source for a perpetual motion machine.
FAIL
Big time. FAIL

John G
December 29, 2011 7:27 am

I can’t judge the math or the physics but assuming this holds up we’re in for a paradigm shift that will blow the current crop of climate alarmists (for whom an implicit argument has always been “there isn’t any other explanation than enhanced greenhouse effect”) out of the water. A theory of climate that depends on planetary mechanics, atmospheric mechanics, planetary motions, solar insolation and cosmic radiation is far more satisfying than one that depends solely on the amount of a trace gas in the atmosphere. In fact when you put it like that one wonders how serious scientists ever came to that conclusion.

GabrielHBay
December 29, 2011 7:28 am

Before reading any of the comments above, and while my maths is far too rusty to follow the detail, the clear and concise and comprehensive verbal explanation is the first that I have seen from climate science (in this context) that hits (for me) that warm spot that says: “YES!” Must confess that I have been, from day one, part of that really BAD group of extreme sceptics who do not accept at all the concept of a nett greenhouse effect in a real, ‘live’, atmosphere… and this paper makes perfect sense to me. What a rush.. Now to wade through all the other comments…. there goes my evening 🙂

John Wilkes Booth
December 29, 2011 7:32 am

“3, The authors fall into the trap of imagining that ‘back radiation’ can do thermodynamic work. Asahenius was wrong as any professional engineer experienced in modelling and designing heat transfer will confirm,”
Asahenius did not imagine that back radiation could accomplish any work any more than wearing an overcoat accomplishes any work. What Asahenius imagined is the greenhouse gases can act as insulation. Back radiation is simply the mechanism by which the insulation works and it does indeed work as insulation.

December 29, 2011 7:32 am

So what is the expected heating effect of the mass of CO2 we have added and will add over time? I would like to see that added as a “Summary For Policymakers.”

JustaMom
December 29, 2011 7:33 am

Could someone please translate this for those of us who have say a BA?….and, unfortunately for me, even that is not in math, physics or engineering.

December 29, 2011 7:43 am

“This is wrong and anyone with an air compressor can prove it. Pressurize a tank of air. It will initially be warmer according to the idea gas law. But wait a few days. Its temperature will equalize with the ambient air outside the tank”
Not a good analogy because the continuing insolation reaching Earth is more akin to continuing pressurisation of a leaky tank of air. Obviously, if insolation stopped then the analogy would fit and the temperature of Earth would equalise with that of space but it does not as long as insolation continues.

December 29, 2011 7:47 am

Makes a lot of sense to me. I think this is a more sensible approach then the nonsense we have all been attacking for all these years. We will still have the better part of a decade ahead listening to “but the model, but the model” to put up with.

Pamela Gray
December 29, 2011 7:49 am

Mr.ReallyBadChoiceInScreenName, please read the PDF. Pressure is always changing on Earth. There is no waiting. So stand by that tire and adjust the pressure up and down during those few days. Under that condition, you never give the temp a chance to equalize now do you.

wayne
December 29, 2011 7:54 am

@ richard verney:
December 29, 2011 at 6:55 am
Hi Richard. I see and have considered your point.
But look at the physics units. Pressure is mass × acceleration per area. That is why you can calculate the mean surface pressure with just the mass of the atmosphere, gravitational acceleration, and, the area of this world, and it only depends on those parameters.
The mean pressure at the surface will not change just because temperature rises, if evenly everywhere, not causing any local effects. It is the drop in density, mass per volume, that fluffs up the atmosphere to higher altitudes in the daytime and collapses at night, pressure does not change. My only point was it is not strictly pressure but the ratio of pressure to density that needs to be brought into focus.
Another way of putting it is that you seems to be viewing the atmosphere’s pressure going up and down with the temperature each day. That is not strictly correct. Just temperature will increase pressure if tightly constrained, as in a bottle and constant volume, but in the gravity held atmosphere the volume, as seen in a higher top of the atmosphere, will increase to compensate so the pressure does not actually increase, volume goes up, density goes down.
I see this interplay between density, pressure and temperature as one of the most impressive points within this N-K paper ruling all atmospheres. I’ve been waiting for such a paper for literally years now.

John Wilkes Booth
December 29, 2011 7:56 am

JustaMom says:
December 29, 2011 at 7:33 am
“Could someone please translate this for those of us who have say a BA?….and, unfortunately for me, even that is not in math, physics or engineering.”
The physics are flawed. This wouldn’t pass peer review in any reputable journal which is why it appears here instead of there.

Mark K
December 29, 2011 7:56 am

“John Wilkes Booth says:
December 29, 2011 at 7:25 am
“a) the product P×V defines the internal kinetic energy of a gas (measured in Jules) that produces its temperature; ”
This is wrong and anyone with an air compressor can prove it. Pressurize a tank of air. It will initially be warmer according to the idea gas law. But wait a few days. Its temperature will equalize with the ambient air outside the tank. Yet the pressure and volume of the compressed has not changed.
What the authors propose would in effect be a power source for a perpetual motion machine.
FAIL
Big time. FAIL”
From what I remember of physics, the pressure in your tank will decrease as the temperature decreases – just like what happens to the air pressure in your tires during winter.

Mydogsgotnonose
December 29, 2011 8:00 am

John Wiles booth: Sorry about the wrong spelling of Aarhenius [new pair of glasses].
Aarhenius’ mistake was to imagine that the S-B equation for a single body predicts the net radiated energy. This is not true as any professional engineer or scientist should know. At constant temperature there is no net energy interchange, the oldest of the Radiation laws,
Climate science fondly imagines that the signal measured by a radiometer pointing upwards is real. It’s not. That signal in the absence of the radiometer is offset exactly by upward radiation. A Dutch student proved this experimentally recently by shinning up an 800 foot radio mast and measuring the up-down signal at night: it fell exponentially to zero, Beer-Lambert.
So ‘back radiation’ is a myth as is cloud albedo effect cooling supposed to offset it. The latter is because the aerosol optical physics in the models used to predict it is plain wrong. In 2004 NASA claimed that small droplets reflected more sunlight: wrong – it’s a large droplet effect as can be seen with any rain cloud.
Because of the many elementary [to professional physicists but clearly not to climate science] mistakes, the IPCC version is plain wrong. Even Spencer and Curry have got it wrong in places and are being re-educated via their blogs. God help the students taught incorrect physics, e.g that IR energy is thermalised in an insulating layer as you claim What really happens is that there is an increase of optical path length by scattering of IR with thermalisation at second phases, especially cloud droplets which have gettered local CO2..
Until climate science admits its serious scientific errors, there can be no credence to the subject or the models.

Austin
December 29, 2011 8:00 am

Finally, a treatment using gas and thermodynamic fundamentals.
I suffered through to epiphany in two engineering thermo classes and two grad thermo classes and what has always bothered me is that I never saw a treatment on the topic based on what I learned in those classes.
This is the treatment. I enjoyed reading it. I predict this paper will become a classic and will be in most thermo textbooks and will show up on a lot of finals. And will open up a new field of study for exoplanets. And find its way into terraforming studies.
Excellent work gentlemen.

December 29, 2011 8:01 am

The problem I have is the lack of quantitative consideration of the hydrological cycle. This cycle explains the atmospheric energy transfer.
http://www.miltonconservative.blogspot.com/2011/02/simple-chemistry-and-real-greenhouse.html

michael hart
December 29, 2011 8:03 am

It’s good to see the [area-weighted] fourth-power of absolute temperature getting some more attention. It almost always just seems to be “temperature anomalies”. Perhaps this is already addressed in the models, but I rarely see it discussed. It might have saved us all a lot of time if the issue was addressed more directly, more often.
That, and the question how can a change in the height of the radiative-surface [the “top of the atmosphere”?], by an increment in a GHG, further raise the temperature underneath the radiative surface [which is itself already quoted as being above the black-body “effective temperature”] with respective to the net radiative flux of the earth?
Apologies for the long sentences.
Thank you mydogsgotnonose, I find your short posts [Prevost Exchange Energy] illuminating.

Stephen Richards
December 29, 2011 8:03 am

Will says:
December 29, 2011 at 7:24 am
Pamela Gray says:
December 29, 2011 at 6:55 am
“And that changes in pressure was the direct cause of temperature changes on such a scale.”
Why do so many seem to confuse cause and effect.
More energy = more pressure which manifest as an increase in temperature.
Learn the basic cause and effect relationships and rid yourself of ignorance and confusion.
Will, Sorry, but you have failed the test. Pam is actually as correct as you. The problem you perceive stems from not starting at the ‘source’. Start with a sealed container of gaseous material. Input some energy. Energy causes the gas moles to move quicker and impinge more frequently on the inside of the container and pressure is perceived to rise. So Pam is also correct as you. Well done to you both.

pochas
December 29, 2011 8:04 am

This would constitute a paradigm shift, a rational basis for climate science. The proposition that there has been a major variation in atmospheric mass over geologic time is a stunner! IMHO equivalent to plate tectonics in import. We need to see direct evidence for this. Something for the now-disenfranchised AGW syncophants to work on, but the rock hounds will probably get into the action first.

Warren in Minnesota
December 29, 2011 8:06 am

I agree with @Espen regarding the degassing.
Also I remember that the earth’s atmosphere continues to lose mass to space which would mean that the earth will continue to cool as we lose mass and diminish our atmospheric pressure.
But I also don’t agree (or disagree) with Figure 7 statement that the temperature “…appear(s) to have been controlling” the clouds. That conclusion from the graph doesn’t jump out at me.

JDN
December 29, 2011 8:07 am

@Ned: I have a problem with the “kinetic energy” argument for lower troposphere anomalous warming. The heat capacity of air may be small, but the heat capacity of water vapor or clouds in air is large. You made a hand-waving argument. I suspect that if you flesh out your argument you’ll find that the conversion of kinetic energy to thermal energy for the anomalous warming would require ridiculous amounts of kinetic energy.

michael hart
December 29, 2011 8:07 am

In my last post I should have said more specifically that the “surface” temperature seems to be quoted as being above the black-body “effective temperature”

John Wilkes Booth
December 29, 2011 8:10 am

FYI
Ned Nikolov’s PhD is in “Forest Ecology”. I’m unsure of what all that entails but it most certainly doesn’t require understanding the ideal gas law.
http://www.fs.fed.us/rm/analytics/staff/nikolov.html
Karl Zeller’s excuse is less clear:
PhD, Colorado State University, Fluid Mechanics & Wind Engineering (micrometeorological emphasis) 1990
http://www.fs.fed.us/rm/analytics/staff/zeller.html
Colorado State University has some explaining to do since they graduated both of these guys without imparting a basic understanding of the relationship between temperature, pressure, and volume of an ideal gas. Of course they both ARE federal government employees like James Hansen and that in itself is a usally a good indicator of merit (or lack thereof).
REPLY: Nice try Dave Springer aka “John Wilkes Booth”, note our policy page prohibits switching handles – Anthony

December 29, 2011 8:12 am

A major point of this paper is the observation that the warmth at the surface is a thermodynamic effect and not a radiative effect.
It fits nicely with the description that I published back in May 2008
“The fundamental point is that the total atmospheric warming arising as a result of the density of the atmosphere is a once and for all netting out of all the truly astronomic number of radiant energy/molecule encounters throughout the atmosphere. The only things that can change that resultant temperature equilibrium are changes in solar radiance coming in or changes in overall atmospheric density which affect the radiant energy going out”
from here:
http://climaterealists.com/index.php?id=1562&linkbox=true&position=7
” Greenhouse Confusion Resolved”
Obviously the final outcome of the thermodynamic process is radiative but the process itself is not radiative.
The denser an atmosphere the more tightly packed the molecules of whatever gas it is comprised of. The tighter the packing the more resistance there is to the transmission of radiative energy because the individual molecules are more easily able to pass kinetic energy between themselves multiple times before it is finally released back to space in radiative form.
It doesn’t matter whether the constituents of the atmosphere are generally transparent to light or not. The fact is that whatever the radiative characteristics of individual molecules they all act together thermodynamically when under pressure.
To illustrate that we can all see that the Oxygen and Nitrogen molecules near the surface are at much the same temperature as the GHG molecules near the surface DESPITE their different thermal characteristics in terms of radiative ability.
Thus the effect of atmospheric density and pressure is to dictate the surface temperature regardless of composition.
This paper neatly deals with the ‘problem’ of explaining why composition is less important than density and validates the assertions I made in my article back in 2009.
That said, GHGs do have a thermal effect but it is not one that increases the system energy content as a whole as I have explained elsewhere. Instead all they achieve on our watery world is to provke a negative system response that precisely cancels out their warming effect in exchange for a miniscule adjustment in the speed of the water cycle and a miniscule adjustment in the surface air pressure distribution.
The authors clearly agree with me on that point too because they say:
“Variations in the downward LW flux (caused by an increase of tropospheric emissivity, for example) are completely counterbalanced (offset) by changes in the rate of surface convective cooling, for this is how the system conserves its internal energy.”
That accords with my contention that although GHGs slow down the rate of energy loss to space the redistribution of surface air pressure acting via a faster water cycle cancels it out again for a zero net effect on total system energy content.
My work then goes on to link all that to solar activity from above and oceanic variability from below for a more complete Unified Theory than that presented here (IMHO).
In particular I don’t think it is magnetic variability that achieves the observed effects but rather wavelength changes acting on ozone differentially at different levels in the atmospheric column.

December 29, 2011 8:13 am

John Wilkes Booth, have a closer look at that pressure gauge as the temperature drops.

ferd berple
December 29, 2011 8:14 am

Different authors have different interpretations on this theme, but the underlying point held in common is this:
If earth’s moon had a thick N2 atmosphere, would the surface temperature of the moon change?
The radiative transfer model (AGW) says that since N2 is not a GHG, the surface temperature of the moon would not be changed by a dense N2 atmosphere.
The alternative models of a gravity bound atmosphere say otherwise. That even an N2 atmosphere will warm the surface as compared to a planet with no atmosphere.
It would seem that at present the way forward to resolve this is to compare planets based on atmospheric density and GHG concentrations, to see which theory is correct. Generating artificial gravity in a lab to test this does not appear to be a practical alternative.
Computer models alone cannot provide this answer, because the earth is only a single point of reference. An infinite number of trend lines will satisfy. By adding data from other planets and moons, this limits the trend-lines that will fit the data, revealing which theory is more accurate at predicting surface temperatures.

December 29, 2011 8:17 am

Wonderful job! Drs Nikolov and Zeller are to be commended for such a great effort. WUWT has come a long way. Pamela Gray is right, (as usual), the application of the Ideal Gas Law has been brought up here a few times in the past. The reception to it then wasn’t as nearly pleasant.
Rather than getting into a debate with some of the critics here, I’m going to digest what these guys are stating. I would implore Drs. Nikolov and Zeller to respond and perhaps modify this work in response to some of the criticism.
My thoughts are they pretty much nailed it. However, it needs cleaned up. I don’t believe figures 8 and 9 are required. The reconstructions that far in the past are controversial. And, I don’t believe 65 million years ago is relevant to discussions of today. I would leave that part out. In my mind it is sufficient to simply state, “things were different back then.” Make the posit as simple as possible. Leave the distracting stuff out.

Chris B
December 29, 2011 8:25 am

Harry Dale Huffman says:
December 29, 2011 at 6:04 am
“Hence, the atmosphere does not act as a ‘blanket’ reducing the surface infrared cooling to space as maintained by the current GH theory, but is in and of itself a source of extra energy through pressure.”
It cannot be overemphasized that ALL of the energy comes from the Sun, contrary to what the authors seem to be saying in that quote.
__________________________________
Actually some of the energy comes from the core of the earth which is hot due to gravitational pressure and radioactive decay of the ancient “stardust” that made up our planet. Even Al Gore agrees, although the eath is only a few thousands of degrees in temperature on the underside of the crust, not millions of degrees.
The earth’s surface would probably be a lot colder if not for this “stored” heat, that continues to slowly cool as our solar system ages.

commieBob
December 29, 2011 8:26 am

Reply to Richard Verney:
I was taken by the 133 number. It is dramatically different than the figure that we were used to seeing.
What I was doing was a kind of back-of-the-envelope calculation to see if the number was close to reasonable. I suspect that it is.

December 29, 2011 8:27 am

Asahenius = Arrhenius

ferd berple
December 29, 2011 8:29 am

If it can be shown that the earth’s atmosphere is not as dense as previous, and that there has been a corresponding change in surface temperature, that would give us a second data point to confirm the gravity bound atmospheric model.

richard verney
December 29, 2011 8:29 am

John Wilkes Booth says:
December 29, 2011 at 7:25 am
//////////////////////////////
I have not read this paper properly yet, but I believe that you misunderstand their point when you consider the paper fails as a matter of principle for the reason stated by you.
As you note, there is a relationship between pressure and temperature. As you further note, when ever there are temperature differences, heat will flow from warm to cool so that a warm body will lose heat and thereby tend to assimulate the temperature of its cooler environs. Ie., there is heat loss.
My understanding of what the authors are suggesting is that if there is a source of energy that is sufficient to replenish the daly heat loss, then the heat brought about by the pressure is effectively forever maintained (within broad limits).
The authors are suggesting that solar irradiance is in itself sufficient to replenish the heat loss and therefore the atmoshperic heat brought about by the pressure of the atmospher does not (within broad limits) cool down. It is not irrevocably lost since the heat loss is being constantly replaced.
An analogy would be to consider placing a pot of cold water on a stove. You may need X joules of energy to raise the temperature from 15 degC to 50 deg C. However, if at 50 degC the heat loss to the environs is B joules then you only need to apply B joules of energy to maintain the the 50 deg C heat indefinitely.

richcar1225
December 29, 2011 8:30 am

As I sit in my livingroom in Denver Colorao I am experiencing the heat released from the down flowing sixty mile an hour Chinook winds. Temperatures wil be in the sixties today with clear skies and low relative humidity. Albedo will decrease. There will be little downwelling infrared. Kinectic energy rules.

December 29, 2011 8:36 am

John Wilkes Booth says:
December 29, 2011 at 7:32 am
“…What A[r]ahenius imagined is the greenhouse gases can act as insulation. Back radiation is simply the mechanism by which the insulation works and it does indeed work as insulation.”
Please let me and others know the R-factor of back radiation.

Kevin Kilty
December 29, 2011 8:37 am

I have to agree first with Will says:
December 29, 2011 at 2:43 am
that the disparity between the temperature of the moon and this calculation jumps right out at a person. However, a few days ago I suggested that one can achieve equilibrium with a variety of temperature distributions, and these will lead to various average temperatures, making mean surface temperature not useful for much of anything…oh, I suppose it is useful as a sort of box-score entry for whether or not the Earth is warming, but not for identifying cause. So, here I agree with Messrs Nikolov and Zeller. The calculation of a mean surface temperature through the Stefan-Boltzmann law is not informative.
However, there are a slug of problems with this paper.
John Wilkes Booth rightly points out that PV is not internal kinetic energy. Engineers refer to this term as work-flow and it is a part of enthalpy of a fluid. He also identifies the fallacy that A[r]ahenius did not imagine that back radiation could accomplish any work any more than wearing an overcoat accomplishes any work. This is my nomination for quote of the week!
Mike McMillan once again emphasizes that the high surface temperature on Venus is not related to CO2 per se, but to adiabatic work. See: Mike McMillan says:
December 29, 2011 at 3:16 am
My recall was a few degrees off. Here’s the post.
http://wattsupwiththat.com/2010/05/06/hyperventilating-on-venus/#comment-384746
. At the time of this post I recall calculating the lapse rate on Venus and comparing it to the observed temperature versus height graph. Nearly perfect; and most unlike what one would achieve at radiative equilibrium. In effect, radiation is absorbed high in the atmosphere, raising the temperature there, and the high surface temperature results from work done on parcels of atmosphere through convection down to the surface. Again Messrs Nikolov and Zeller point to something important, though the details look wrong to me.
I think the misunderstanding of the gas law, that temperature is not determined by pressure, is this contribution’s glaring flaw. Equality of energy loss and gain is what determines temperature ultimately. Pressure results from gas at some level having to support the weight of overlying gas. Density then becomes the dependent variable. You’ll note that Nikolov and Zeller leave out any discussion of gravity at all. So Figure 5 is meaningless, and the whole discussion surrounding it is a mess.

Jimmy Haigh
December 29, 2011 8:38 am

John Wilkes Booth says:
December 29, 2011 at 8:10 am
What are your credentials? Who are you to criticise this work? I would look you up but somehow I don’t think it’s your real name…

December 29, 2011 8:38 am

To R Barker:
Indirect evidence points to much higher gas concentrations in the past with giant fossil dragonflys that could not live in today’s thinner atmosphere. Since insects breath through their surface rather than with lungs.
It would be interesting to see some reconstructions combined with insect size in the fossil record as a possible proxy of atmospheric pressure and thus temps.
There may be detail problems with what this paper says, but the overall thrust that agw as currently theorized is very well crushed in so many ways. A green house effect in a convective atmosphere? That’s always been a core problem.
Going to be fun to see this adjusted and corrected though peer review. Looks like they are looking at cloud variance as the main driver combined with air pressure which means really a simple solar connection but with a complete picture of the system to show why this is so.
Very interesting stuff.

DirkH
December 29, 2011 8:39 am

John Wilkes Booth says:
December 29, 2011 at 7:25 am
““a) the product P×V defines the internal kinetic energy of a gas (measured in Jules) that produces its temperature; ”
This is wrong and anyone with an air compressor can prove it. Pressurize a tank of air. It will initially be warmer according to the idea gas law. But wait a few days. Its temperature will equalize with the ambient air outside the tank. Yet the pressure and volume of the compressed has not changed.
What the authors propose would in effect be a power source for a perpetual motion machine.
FAIL”
John, the Ideal Gas Law assumes that the volume of the container holding the gas is infinitely larger than the volume taken up by the gas molecules themselves, so a compressor might not be the right device to test the Ideal Gas Law.

iya
December 29, 2011 8:43 am

Is the “observed mean surface temperature”, in the table, observed or calculated? In the pdf it also says the last 3 rows are calculated, but it has an additional row.
Especially the moon temperature of 154.3K = -119 °C and mercury seem to cold.

commieBob
December 29, 2011 8:45 am

John Wilkes Booth says:
December 29, 2011 at 7:25 am
“a) the product P×V defines the internal kinetic energy of a gas (measured in Jules) that produces its temperature; ”
This is wrong and anyone with an air compressor can prove it. Pressurize a tank of air. It will initially be warmer according to the idea gas law. But wait a few days. Its temperature will equalize with the ambient air outside the tank. Yet the pressure and volume of the compressed has not changed.

You should pay better attention to your air compressor. It works this way:
1 – I use some air and decrease the tank pressure below the switch lower limit.
2 – The motor runs the compressor.
3 – The pressure rises above the switch upper limit.
4 – The motor turns off.
5 – The tank gives off some heat.
6 – If the pressure drops below the switch lower limit, the motor comes on again.
7 – The process repeats until the pressure stays above the switch lower limit.

Kevin Kilty
December 29, 2011 8:45 am

John Wilkes Booth says:
December 29, 2011 at 8:10 am
FYI …
Karl Zeller’s excuse is less clear:
PhD, Colorado State University, Fluid Mechanics & Wind Engineering (micrometeorological emphasis) 1990 …

Booth, it is the wind engineering that identifies the problem here. As a specialty it is not rigorous enough to give someone a solid foundation in anything like thermodynamics. You and I agree that the discussion and application of the gas law in this contribution is a mess.

Brian H
December 29, 2011 8:47 am

Jeff L says:
December 29, 2011 at 6:31 am

The one red flag I see is the long term predicted pressure profile in figure 8. Eocene pressures max out at ~ 185 kPa – that’s approaching double today’s standard pressure of ~ 101 kPa !! I would think at those kind of pressures there would be some biological effects which might be manifested in the fossil record – how life adapted to such high pressures.

That barely scratches the surface:
http://levenspiel.com/octave/dinosaurs.htm
and
http://pubs.acs.org/subscribe/archive/ci/30/i12/html/12learn.html
Thicker air allowed >100lb. pteranodons to fly. They couldn’t in a 1 bar atmosphere.

Tilo Reber
December 29, 2011 8:49 am

Very impressive work gentlemen. The math is a little over my head; but the physical explanations definitely make sense. Plus, your theory explains the observations much better than current GHG theory. I’m bookmarking this one. You’ve done the work, now it’s up to the rest of us to make sure that these findings are not ignored.

Kevin Kilty
December 29, 2011 8:52 am

Jimmy Haigh says:
December 29, 2011 at 8:38 am
John Wilkes Booth says:
December 29, 2011 at 8:10 am
What are your credentials? Who are you to criticise this work? I would look you up but somehow I don’t think it’s your real name…

Real name or not, credentials or none, Booth is dead right with regard to this paper. The application of the gas law here is horrid. The authors seem to not realize that the only proper application of the gas law in this context is that density is the dependent variable–temperature and pressure obtain from other considerations. Nikolov and Zeller point to the issue right here
The thermal effect of pressure is vividly demonstrated on a cosmic scale by the process of star formation, where gravity-induced rise of gas pressure boosts the temperature of an interstellar cloud to the threshold of nuclear fusion.
It isn’t the gravity induced rise of gas pressure, but rather the work done by gravity versus heat lost to radiation that increases temperature in star formation.

Joel Shore
December 29, 2011 8:53 am

This leads to a logical question: Could air pressure be responsible for the observed thermal enhancement at the Earth surface presently known as a ‘Natural Greenhouse Effect’?

Logical answer: No, not unless you want to repeal Conservation of Energy, a bedrock principle of physics. The Earth’s surface is emitting 390 W/m^2 whereas the entire Earth system is receiving 240 W/m^2 from the sun. If this entire 390 W/m^2 were escaping to space, then the Earth would RAPIDLY cool down. However, as seen from space, the Earth is actually only emitting 240 W/m^2, i.e. (within about a W/m^2) exactly what it is absorbing from the sun.
The only way that this can be explained is by the fact that the Earth’s atmosphere is not transparent to the radiation emitted by the Earth’s surface…i.e., there are elements in the atmosphere that absorb that radiation (and subsequently emit it…but at a lesser intensity). This is what we call “the greenhouse effect” and it doesn’t magically disappear if you correctly understand the implications of the ideal gas law and the lapse rate.
In summary, this post is completely nonsense. And the praise that it is receiving from many commenters is evidence of how easy it is to fool people who want to believe something.

Joe
December 29, 2011 8:54 am

Magnificent little paper by Ned Nikolov, Ph.D. & Karl Zeller, Ph.D.
If this holds up to testing, and so far it looks really good, this could be the Climate Sciences version of E=MC^2, or the discovery of DNA. A brilliant, elegant and easily digested explanation that truly furthers the science, rather than an agenda.

Kevin Kilty
December 29, 2011 8:54 am

Cripes! Blockquote gets me again. The last paprgraph in my last posting is mine not Nikolov’s and Zeller’s. The authors seem unaware of this fact about temperature in their contribution.

REPLY:
Fixed, Anthony

GeologyJim
December 29, 2011 8:54 am

This. Could. Change. Everything. (in your best Howard Cosell voice)
The statement is made that directional cooling over the last 55 million years is due (in large part) to decline in mantle degassing, which should be a function of sea-floor spreading rates. This needs to be tested against the geologic record
Numerous geologists have tried to explain climatic events by attributing warming to CO2 production from vast basalt fields (Deccan traps at K-T time, for example). Nikolov and Zeller’s idea would explain similar correlations, but attribute the thermal effect to increased atmospheric density.
Question: If atmospheric-mass loss to space is due to solar wind and cosmic-ray flux, that brings the Nir Shaviv ideas of Earth’s path through the Milky Way density field into play as well
Question: How has Venus managed to hold its thick atmosphere against the solar-wind flux? Is it just the greater molecular weight of CO2 compared to N2, O2, and such?
Verrrrry interrrresting.

Brian H
December 29, 2011 8:54 am

Peter Ward says:
December 29, 2011 at 2:53 am
If this is right then indeed it is paradigm-altering. It needs to be reviewed — proper peer review that aims to break it — and then we can see whether it fits the observed facts more closely than other theories. For example, can it explain the temperature record of the last 50 years? Or is the proposition that recent change is random? Whatever, it’s good to see some real innovative thinking brought to the subject, with the prospect of radically changing our understanding. Thank you!

Over at Tallbloke’s The Talkshop, there is the additional comment:
There is more on the way. “This write-up is only a summary of our research results detailed in 4 papers which we are currently preparing for submission to a peer-reviewed journal.”

ChE
December 29, 2011 8:56 am

Sorry, I’d really like to buy some of this, but there’s way too much handwavium. For example, you learn in undergrad thermo that for a diatomic gas like air, P1V1^1.3 = P2V2^1.3. The fact that this wasn’t mentioned suggests a weakness in thermo.
Conceptually, they may have gotten some things right, such as convection dominating in the troposphere. But let’s see some real serious thermo, ok?

Frumious Bandersnatch
December 29, 2011 9:00 am

>John Wilkes Booth says:
>December 29, 2011 at 7:32 am
>
>“a) the product P×V defines the internal kinetic energy of a gas (measured in Jules) that >produces its temperature; ”
>
>This is wrong and anyone with an air compressor can prove it. Pressurize a tank of air. It will >initially be warmer according to the idea gas law. But wait a few days. Its temperature will >equalize with the ambient air outside the tank. Yet the pressure and volume of the compressed >has not changed.
>
>What the authors propose would in effect be a power source for a perpetual motion machine.
>
>FAIL
>
>Big time. FAIL
I don’t claim to be an expert, by any means, but it seems to me that you are confusing kinetic energy with temperature. Apples and oranges. The author explicitly says several times that pressure, by itself, does *NOT* raise temperature. Rather he claims that it magnifies the temperature change caused by an outside source (such as solar radiation).
Since the author obviously agrees with your point, I don’t see where he failed in this instance…

Brian H
December 29, 2011 9:01 am

Stephen Richards commented on Unified Theory of Climate.

it’s worth noting, that if not for WUWT this work may never have been made available to the general public. So well to Anthony and the crew.

Be fair. It was Tallbloke’s “scoop”, see link above.

RockyRoad
December 29, 2011 9:03 am

This goes along with a paper in Forbes entitled “Even the Warmists Don’t Believe in Global Warming”:
http://www.forbes.com/sites/louiswoodhill/2011/12/28/even-the-warmists-dont-believe-in-global-warming/
And now we know why.

James Reid
December 29, 2011 9:03 am

Do I hear deathly silence from the Lazy Teenager and R Gates?

December 29, 2011 9:04 am

Joel Shore said:
“The only way that this can be explained is by the fact that the Earth’s atmosphere is not transparent to the radiation emitted by the Earth’s surface…i.e., there are elements in the atmosphere that absorb that radiation (and subsequently emit it…but at a lesser intensity).”
But that is what they do say and what I said back in May 2008.
The density of the atmosphere results in exchanges of kinetic energy beteween ALL molecules of the atmosphere which delays the exit of radiative energy to space.
The radiative characteristics of the molecules are irrelevant because they ALL share in the kinetic activity and the level of that kinetic activity is density/ pressure dependent NOT composition dependent.

GabrielHBay
December 29, 2011 9:05 am

From Mark K.
“John Wilkes Booth says:
December 29, 2011 at 7:25 am
“a) the product P×V defines the internal kinetic energy of a gas (measured in Jules) that produces its temperature; ”
This is wrong and anyone with an air compressor can prove it. Pressurize a tank of air. It will initially be warmer according to the idea gas law. But wait a few days. Its temperature will equalize with the ambient air outside the tank. Yet the pressure and volume of the compressed has not changed.
———————————————————————————-
As I understand it, you are leaving out the fact that that in the atmosphere, the energy load is continuously replenished by the sun, balancing (with the help of other mechanisms) the energy loss with energy gain. Thus the formula works. In your example you have only nett loss to the cooler ambient environment. To summarise in my own simplistic way: The energy load of the atmosphere per molecule determines the potential for temperature. The actual temperature achieved is then, for a given energy load, dependant on the pressure (hence compression) experienced by the atmosphere, bringing the molecules closer together. The higher rate of impingement of molecules on each other due to the closer proximity registers as temperature.
Ok, all you very smart persons out there… do have that right in laymans’ terms? 🙂 Makes perfect sense to me. but what do I know.. (TIC)

DR
December 29, 2011 9:06 am

http://hockeyschtick.blogspot.com/2010/09/shattering-greenhouse-effect.html
Where JAE states

The “backradiation” does TRY to keep the atmosphere warmer, but convection spoils it all. Just like opening the windows in a real greenhouse.
Otherwise, it would not be hotter in a desert than it is in a humid area at the same elevation and latitude (both day AND night), since the GHG concentration in the humid area is about 3 times higher than in the desert. (I wonder how many times I’ve offered this comment and received no logical response….).

That too has also been a thorn in my side concerning the ‘GHE’ explanation. After giving convection, conduction and gravity more thought, the idea of ‘back radiation’ being the dominant feature just doesn’t make much sense. Surely convection is not properly accounted for.

Joe
December 29, 2011 9:06 am

DirkH says:
John, the Ideal Gas Law assumes that the volume of the container holding the gas is infinitely larger than the volume taken up by the gas molecules themselves, so a compressor might not be the right device to test the Ideal Gas Law.

Exactly. John’s example assumes that the Earth’s atmosphere operates like a compressor which is laughable. This is no different than the now debunked “simple experiment” in which a jar with CO2 is shown to warm faster than a jar of air… it only works that way if you limit the expansion of the gas (like John’s compressor). When you take the lid off the jars and allow the gas to expand then the jar with CO2 warms at the same rate as the jar of air.

Kevin Kilty
December 29, 2011 9:07 am

Nikolov and Zeller point here to several issues that are important and either ignored completely or examined insufficiently. Specifically these are: 1) the importance of actual temperature distribution on radiative equilibrium and mean surface temperature; and 2) the impact of convection on radiative equilibrium. Unfortunately the glaring errors in application of the gas law will detract from their work, and people will miss the value of the several good suggestions they make.
This will be a busy thread today I predict.

Michael D Smith
December 29, 2011 9:08 am

I was right! You’ve just captured what I’ve been working on. I’m a former lukewarmer, but I’ve been pretty convinced lately that GHG’s do nothing more than change the behavior and timing of convective events, nothing more. Net effect of GHG’s to temperature is zero, zip, nada, zilch.
Brilliant work. I thought I was alone on this one. Groundbreaking stuff. Thanks.
Now let’s try to tear it apart… gently.

TexUte
December 29, 2011 9:10 am

Wilkes Booth: Your statement about pressure in a gas cylinder being independent of temperature is just wrong. The mechanical work of compressing a gas transfers heat, so a newly filled cylinder will cool over time if the ambient temperature is lower than the T of the gas. Pressure will also drop in proportion to the change in T, since the volume is fixed. Heating the cylinder will increase the pressure; relieving the pressure will cool the cylinder. This is all very well understood and easily demonstrated.

DirkH
December 29, 2011 9:11 am

ChE says:
December 29, 2011 at 8:56 am
“Sorry, I’d really like to buy some of this, but there’s way too much handwavium. For example, you learn in undergrad thermo that for a diatomic gas like air, ”
Air is a diatomic gas?

Bill Illis
December 29, 2011 9:11 am

We also have the Faint Young Sun paradox to resolve – solar irradiance was as much as 27% lower when the Earth formed (increasing in close to a straight line in the time since).
The Earth should have been a frozen snowball until about 500 million years. It was actually very cold once Oxygen became prevalent about 2.4 billion years ago to about 580 million years ago, but the earlier periods seem to have been warm enough. Perhaps Oxygen thinned out the early atmosphere. Perhaps the early water vapour content (the oceans formed as water vapour rained out of the atmosphere) provides another part of the picture.

Tilo Reber
December 29, 2011 9:11 am

John Wilkes Booth:
“This is wrong and anyone with an air compressor can prove it. Pressurize a tank of air. It will initially be warmer according to the idea gas law. But wait a few days. Its temperature will equalize with the ambient air outside the tank. Yet the pressure and volume of the compressed has not changed.”
First, I submit that in your experiment the pressure will in fact have lowered after you have waited those few days. Second, your experiment does not include the inflow of external energy that Nickolov & Zeller discuss below. And your comment suggests that you do not understand that they are already aware of your objection.
Nickolov & Zeller: “NTE should not be confused with an actual energy, however, since it only defines the relative (fractional) increase of a planet’s surface temperature above that of a SPGB. Pressure by itself is not a source of energy! Instead, it enhances (amplifies) the energy supplied by an external source such as the Sun through density-dependent rates of molecular collision. This relative enhancement only manifests as an actual energy in the presence of external heating.”

Alex
December 29, 2011 9:12 am

come on people when you pump a tire it gets hot from the work not the preasure, I don’t buy this.

G. Karst
December 29, 2011 9:17 am

Great, but who… besides shunned skeptics, will read this paper, and give it appropriate consideration?! The IPCC will certainly ignore it, and MSM will minimize it, if acknowledged. No amount of logic and physical science seems capable of piercing the agendized armor of the AGW industry and politics. The gatekeeper team have not fallen asleep. Unlikely academic revolution, would be required. GK

December 29, 2011 9:21 am

Kevin Kilty says:
December 29, 2011 at 8:37 am
“I think the misunderstanding of the gas law, that temperature is not determined by pressure, is this contribution’s glaring flaw.”
quote from Ned and Karl excellent (adventure) post: “The thermal effect of pressure is vividly demonstrated on a cosmic scale by the process of star formation, where gravity-induced rise of gas pressure boosts the temperature of an interstellar cloud to the threshold of nuclear fusion.”
So according to you stars get hot, then the pressure increases. HMMMM. Weary interesting. By the way would you please call the folks at Cummins Diesel and let them know how wrong they are.
Further, comparing a solid (overcoat) to a gas I don’t think qualifies as a quote of the weak.

Pamela Gray
December 29, 2011 9:21 am

Great discussion. BadChoiceOfScreenName is the one who deserves the epic fail for bringing up credentials. State your case for or against the content of the poster. Your comment about “credentials” is utter nonsense unless we are discussing research on the relationship between credentials and scientific discovery. If that were our focus here, you would fail on your position thoroughly.

Theo Goodwin
December 29, 2011 9:22 am

Refreshing! Bracing! This theory schematic might very well lead to the well confirmed physical hypotheses that climate science needs to break out of its infancy. This account contains many interesting new ideas, especially its treatment of convection, and gives proper place to natural processes that make up our atmosphere. By contrast, Warmists accounts use a radiation only model that studiously ignores all contributions from natural processes other than radiation and leaves us with all kinds of puzzles, especially the grand one as to how back radiation actually increases temperatures at the surface. Well done.

Don Monfort
December 29, 2011 9:22 am

Slow down. Read Leornard Weinstein’s post. Jeff L, and even Joel Shore. The celebration is premature, again.

Tilo Reber
December 29, 2011 9:27 am

Alex: “come on people when you pump a tire it gets hot from the work not the preasure, I don’t buy this.”
What part of the N&Z explanation don’t you understand:
“Pressure by itself is not a source of energy! Instead, it enhances (amplifies) the energy supplied by an external source such as the Sun through density-dependent rates of molecular collision. “

APACHEWHOKNOWS
December 29, 2011 9:29 am

Try pumping a tire out here in West Texas or Eastern New Mexico in August and see who gets warm fastest. Ya and it last until you jump in the first water tank at the first wind mill.

December 29, 2011 9:29 am

Alex:
Adiabatic compression….pressure goes up and so does temperature.
Sorry, basic thermo..
Suggest this…
http://galileo.phys.virginia.edu/classes/109N/more_stuff/flashlets/carnot.htm

December 29, 2011 9:30 am

lol @ the people wishing to refute the Idea Gas Law. People, there is a reason why this is called a law and not a theory or postulate. Look up the difference. There are plenty of things to pick at about this submission. The IGL isn’t one of them. You can assign reasoning and factors which go into it, but you’re not going to be able to get around, PV = nRT. ………. goobers.

Joel Shore
December 29, 2011 9:30 am

Just to comment on a few other things:

Figure 1. The Atmospheric Greenhouse Effect as taught at universities around the World (diagram from the website of the Penn State University Department of Meteorology).

Yes…This is taught throughout the world, but not as the most complete understanding of the greenhouse effect but rather as the simplest picture of the greenhouse effect. Hence, to criticize it as incomplete is silly…Everyone knows that it is incomplete. It is not meant to be the most complete or quantitatively-correct model. It is meant to be the simplest picture illustrating the basic effect.

Since in accordance with Hölder’s inequality Tgb ≪ Te (Tgb =154.3K ), GHE becomes much larger than presently estimated.

Hölder’s inequality only has a significant effect if the temperature range on the planet is very large on an absolute temperature scale. For the Earth, the temperature range is moderate enough on an absolute scale that the difference between averaging the temperature and taking the fourth root of the average of T^4 is quite small.

However, Eq. (3) is physically incomplete, because it does not account for convection, which occurs simultaneously with radiative transfer.

What convection does is to basically cause the atmosphere to be unstable when the lapse rate is greater than the appropriate (dry or saturated) adiabatic lapse rate. I.e., convection causes any temperature profile steeper than the adiabatic lapse rate to go back to the adiabatic lapse rate. It is correct that the inclusion of convection reduces the greenhouse effect over what it calculated in the absence of convection; however, it does not reduce it to the extent that these authors claim (because the authors incorrectly assume convection try to relax the atmosphere to a completely isothermal temperature profile with altitude)… And, of course, all quantitative calculations of the greenhouse effect are made using models that include such convection.

Theo Goodwin
December 29, 2011 9:30 am

Andre says:
December 29, 2011 at 5:06 am
“Anyway Hansen et al’s paleotemperature reconstruction depends on a lot of suppostions, for instance that the isotope ratios of Benthic Foraminifera adequately represent the local sea water isotope ratios. However, the chemical composition and pH play an important role. Moreover different species have different reactions and species die out and new appear all the time.”
Very well said. Consider doing a post on this topic.

December 29, 2011 9:30 am

What always seems to get lost is the idea that Earth is in a steady-state condition: energy is always coming in, energy is always going out. Such systems tend toward stability.
There are minor variations, of course. Solar activity has a marked effect on incident cosmic rays; cosmic rays, in turn, have a marked effect on cloud formation, thus on albedo. And the Sun is a variable star, on a bunch of different cycles. And the orbit of Earth changes over time as well.
What has long bothered me has been the question of latency: suppose carbon dioxide stored outgoing radiation. How long would that energy be held? It seems to me that, regardless of the latency time (milliseconds or centuries) that stored heat energy would be released, and thus the long-term stability of the atmospheric PVT would not change.
Besides, without a good understanding of the carbon dioxide budget (absorption/release from the oceans, absorption/release from plants, absorption/release from animal life), all of these strongly dependent on temperature, we have little to go on as yet.
Combine our ignorance of the details of the whole C02 budget with the enormous bugger factors used in producing the false hockey stick, and you get exactly what we have now: bogus science.
My money is on Nikolov and Zeller. There have never been SUV’s on Titan or Triton.

December 29, 2011 9:31 am

Bravo – this will put paid to the alarmists
I thought as much some time ago, but not so elegantly – see http://climate-facts.com

Joel Shore
December 29, 2011 9:33 am

G. Karst says:

Great, but who… besides shunned skeptics, will read this paper, and give it appropriate consideration?! The IPCC will certainly ignore it, and MSM will minimize it, if acknowledged. No amount of logic and physical science seems capable of piercing the agendized armor of the AGW industry and politics.

This paper will be ignored because it is pseudoscientific nonsense that would never pass peer review in the scientific community. Its only purpose is to fool those who do not have the scientific background to recognize its glaring errors.

Archonix
December 29, 2011 9:34 am

To counter Booth and Kilty’s simplistic views I offer my own simplistic reply: The part of the paper that has apparently been missed by these two was not that the pressure itself maintains the temperature, but that the pressure reduces the amount of energy required to maintain that temperature.
They also neglect surface area. A small cylinder filled with high pressure gas has an enormous surface area compared to the globe and is taking in very little energy.
It would also, I’d expect, be a lot easier to keep a pressurised container at a specific temperature with an external heater (surely if a gas bottle is the same as the earth, a two bar electric fire is the same as the sun?) than an equivalently sized empty container; the pressurised container would store a lot more heat within a given volume and would radiate for a much longer period.

Gary Pearse
December 29, 2011 9:44 am

Nick Stokes on Hansen’s paper and the graph.
I can see some exasperation re missing reference, but look, the derivative graph is done accepting the temperatures (they are not theirs and they may even be wrongly composed) but the point here is that they can do this calculation – somebody else’s graph would just give different wiggles but the calculation based on it could be made just the same. I’d rather you show us why we should reject the IGL as a pre-eminent explanation. Explain, for example, that their calculations, although almost exactly equal to the observed temps using IGL are wrong because …… Boy if IPCC contributors could calculate the temps on Mars, Venus, and even Earth using CO2 concentrations that came halfway close to IGL results, I would have been sold long ago. If you want to shoot these guys down, you are going to have to do it with the IGL. Figures used in the prevalent GHG theory haven’t yet been so convincing.

Theo Goodwin
December 29, 2011 9:45 am

Kevin Kilty says:
December 29, 2011 at 8:45 am
“John Wilkes Booth says:
December 29, 2011 at 8:10 am
FYI …
Karl Zeller’s excuse is less clear:
PhD, Colorado State University, Fluid Mechanics & Wind Engineering (micrometeorological emphasis) 1990 …”
Gawd! Are you people dependent on authority? Stop with the fallacious arguments from authority and ad hominems.

December 29, 2011 9:45 am

Extremely interesting paper, which certainly appears to hold up on first reading!
It will be very interesting to follow the reaction of those who have hung their hat on other mechanisms for heating.
Thinking for a moment about the implications of this approach, and how they relate to significant global temperature shifts such as ice ages.
Can enough CO2 and other gases be absorbed in a cold sea to materially change the planetary atmospheric mass? Could an ice age be triggered simply by a periodic shutdown of volcanic out gassing, or perhaps by some external mechanism that strips off significant atmospheric mass such as changes in solar wind, UV ionization changing the size (volume) of the atmosphere and decomposing water vapor into oxygen and hydrogen with eventual loss of hydrogen to space.
Would a major asteroid or cometary impactor, “blow off” enough atmosphere to temporarily cool the earth by changing the atmospheric mass to a new lower value until volcanic activity had time (100,000 years or so) to replenish the blown off atmosphere?
Would the atmospheric mass periodically vary due to local changes in the interstellar environment as the earth moves through the galactic disk? Perhaps as it enters or leaves areas where higher or lower flux of small dust and micrometeorites changes the constant external contribution to atmospheric gasses as these particles vaporize as they enter the atmosphere?
All of the above?
Lots of secondary questions generated by this new approach.
Larry

Joel Shore
December 29, 2011 9:48 am

[SNIP: Joel, that comment is not helpful. You can do better. -REP]

Theo Goodwin
December 29, 2011 9:49 am

Joel Shore says:
December 29, 2011 at 9:30 am
“Just to comment on a few other things:
Figure 1. The Atmospheric Greenhouse Effect as taught at universities around the World (diagram from the website of the Penn State University Department of Meteorology).
Yes…This is taught throughout the world, but not as the most complete understanding of the greenhouse effect but rather as the simplest picture of the greenhouse effect. Hence, to criticize it as incomplete is silly…Everyone knows that it is incomplete. It is not meant to be the most complete or quantitatively-correct model. It is meant to be the simplest picture illustrating the basic effect.”
Ah, yet another who does not understand the differences between theories and models. I will give you the very basic difference: Theories describe reality but models reproduce reality. Are you talking about theories or models?

ChE
December 29, 2011 9:50 am

Air is a diatomic gas?

For thermodynamic purposes, yes. What’s it made out of?

Richard M
December 29, 2011 9:51 am

I’ll mention once again that the “cooling effect” of GHGs continually gets ignored. It was ignored by Monckton as well in the previous article. About 1/3 of the energy in our atmosphere gets there by means other than surface radiation. For that energy the GHGs are the only means to radiate it to space. More CO2 will only enhance the “cooling effect”.
In addition, it works spatially in 3 dimensions rather than the 2 dimensions for radiation. It could very well be that this “cooling effect” completely cancels the “warming effect” known more commonly as the GHE. It must reduce it to some degree.
Add in convection, albedo, etc and I think we can get a better picture of the entire process.

Chris B
December 29, 2011 9:51 am

For those interested in what temperature is here’s a primer from Wikipedia.
http://en.wikipedia.org/wiki/Temperature
Pretty good for Wiki.

Theo Goodwin
December 29, 2011 9:59 am

For those who are complaining that this article could not be published in a peer reviewed journal, your claim is trivially true. What this article contains is a brief introduction to a new theory. No journal publishes brief introductions to a new theory.

Tilo Reber
December 29, 2011 10:03 am

Joel Shore: “Yes…This is taught throughout the world, but not as the most complete understanding of the greenhouse effect but rather as the simplest picture of the greenhouse effect.”
This is a nonsense objection on your part. N&Z are not critizising it’s simplicity. They are critizising the fact that it is wrong because what is left out is more important than what is included.
“It is correct that the inclusion of convection reduces the greenhouse effect over what it calculated in the absence of convection; however, it does not reduce it to the extent that these authors claim (because the authors incorrectly assume convection try to relax the atmosphere to a completely isothermal temperature profile with altitude)…”
This is dumb. Go out and feel the wind on your face. And look at the strength of the jet stream. Convection is highly active all the time. And then remember that a doubling of CO2 in a test tube only produced 1C with zero convection.

Theo Goodwin
December 29, 2011 10:09 am

Don Monfort says:
December 29, 2011 at 9:22 am
“Slow down. Read Leornard Weinstein’s post. Jeff L, and even Joel Shore. The celebration is premature, again.”
Yeah. It is a bit early to declare the article “true” and a bit early to declare it “false.” What might be more interesting and useful at this point is a look at its new approach to the problem. Drawing out differences between this approach and the “radiation only” approach of the Warmists could be very useful. The approach in this article describes some natural processes in the atmosphere other than processes of radiation. Warmists studiously ignore all such natural processes by treating them as epiphenomena of radiation changes. (By the way, any article on climate that studiously ignores all natural processes other than radiation changes should never have been published in a peer reviewed journal of science. And all of them should be withdrawn at this time.)

Kevin Kilty
December 29, 2011 10:10 am

Theo Goodwin says:
December 29, 2011 at 9:45 am
Kevin Kilty says:
December 29, 2011 at 8:45 am
“John Wilkes Booth says:
December 29, 2011 at 8:10 am
FYI …
Karl Zeller’s excuse is less clear:
PhD, Colorado State University, Fluid Mechanics & Wind Engineering (micrometeorological emphasis) 1990 …”
Gawd! Are you people dependent on authority? Stop with the fallacious arguments from authority and ad hominems.

Theo, you mis-understand the point JWB and I were making. In fact we are arguing against the authority of credentials, just as you suggest we should. The fact that Zeller and Nikolov both have Ph.D.s should have no bearing on how anyone should view this paper, but it should have imparted them with some caution about this publication. That they made a mess of their application of the gas law stands on its own. I have a number of advanced degrees, but I rarely weigh in on most subjects because I don’t know enough to make a reasonable contribution. I know well, from decades of university teaching, that engineering students can wiggle through to a Ph.D. being very deficient in understanding some fundamental subjects. As long as they remain in a very narrow discipline this deficiency causes no trouble and isn’t even visible. Wind Engineering is a pretty narrow specialty. Take as an example a Ph.D. in Transportation. It is a civil engineering degree, but it focusses narrowly and doesn’t put much if any emphasis on thermodynamics, or fluids, or any of a range of other engineering and scientific topics, even at the introductory level. The letters behind a person’s name have meaning only when you look at specialty, research background, specific education, informal education, and a lot of other information. Neither JWB nor I were making any fallacious argument–please point out specifics if you disagree.

Albert Einstein
December 29, 2011 10:13 am

Kevin Kilty says:
December 29, 2011 at 8:52 am
Jimmy Haigh says:
December 29, 2011 at 8:38 am
John Wilkes Booth says:
December 29, 2011 at 8:10 am
What are your credentials? Who are you to criticise this work? I would look you up but somehow I don’t think it’s your real name…
“Real name or not, credentials or none, Booth is dead right with regard to this paper. The application of the gas law here is horrid.”
Booth got sent back to his grave but now I know why I was spinning in mine.
You don’t have to be theoetical physicist like me to understand the ideal gas law but evidently you have to be more than a US federal government forestry scientist. LOL

Jose Mayo
December 29, 2011 10:14 am

Perdón, pero…
Cómo puede variar la “densidad” de la atmósfera, y la “presión”, si no hay variación en la gravedad?
[TRANSLATION: How can you change the “density” of the atmosphere, and “pressure” if there is no variation in gravity? -REP]

DirkH
December 29, 2011 10:15 am

ChE says:
December 29, 2011 at 9:50 am
“[DirkH:]Air is a diatomic gas?
For thermodynamic purposes, yes. What’s it made out of?”
Up to 4 % water?

Gary Pearse
December 29, 2011 10:19 am

Joel Shore says:
December 29, 2011 at 9:30 am
Just to comment on a few other things: simplistic Penn State graph, Holders inequality, affect of convection… pulling a few barnacles off a humpback whale doesn’t hurt the whale that much. How about chopping up the blubber of the calculations based on IGL for the planetary bodies and their comparison with the measured temps? When something like this work comes along, you can sure tell who have serious dogs in the fight (those with the biggest dogs didn’t show up). Nick Stokes also attacked a few bumps on the beast, too.

Kevin Kilty
December 29, 2011 10:20 am

Theo Goodwin says:
December 29, 2011 at 9:59 am
For those who are complaining that this article could not be published in a peer reviewed journal, your claim is trivially true. What this article contains is a brief introduction to a new theory. No journal publishes brief introductions to a new theory.

Please go back up this thread and read the detailed objections, along with a few compliments, that I made regarding this contribution. It could not be published in the peer-reviewed literature because of the serious mistakes it makes. Now, could these authors publish this in book form on their own? Of course they could, and then their work would have to stand the test of time. My guess is it would be a short test.

Phil's Dad
December 29, 2011 10:24 am

So far, 130 odd comments in (putting aside JWB’s pointless ad hominems and those who say it is wrong without explaining why), the criticisms of this paper appear to be two.
One person is in a flap because he can not find the source of a graph referred to (granted the reference should be there) and then sets up the classic red-herring that it does not appear to match a graph that depicts something different from some another paper. So far, so trivial.
The second criticism is one I would very much like to hear the authors’ remarks on. There has been some concern expressed in these comments over the -133°K atmospheric boost shown by Eq. (2) in this presentation. As far as I can tell this is entirely reasonable given the difference between Earth and Moon surface temperatures if you also consider the cooling effect of the Earthly water cycle. How would the authors address the concerns expressed?
PS just to put in my penny’s worth – if we are looking for a Unified Theory I am prompted to wonder about the effect of adiabatic magnetism.

ChE
December 29, 2011 10:26 am

Up to 4 % water?

Which leaves 96+% made out of what?

Dan in Nevada
December 29, 2011 10:28 am

This is really fascinating to me as it appears to make a lot of sense, if the author’s claims can be verified. I haven’t retained enough of the math I used to somewhat understand to know if the authors’ equations make any sense and would love for some of the more knowledgeable WUWT readers to verify the math.
Assuming (I know) that the math is correct, I don’t understand some of the objections raised here. Nick Stokes claims that a lot is deduced from figure 8. As far as I can tell, the only thing deduced is figure 9 where I believe they are saying that their theory says that historical temperatures would imply specific pressures and proceed to give an example. Objecting to the inputs doesn’t change anything.
Also, a lot of grumbling about whether the GHE adds 33 or 133 degrees C. The authors claim that their way of looking at GB temperatures (I’m looking at their Table 1) would give both the earth and moon a mean surface temperature of 154.3 degrees, which makes sense given the same location relative to the sun. They also say that the airless moon has a measured temperature of 154.3 degrees. If this is all true (which I don’t claim to know), then that’s seems to say their equations are pretty good. It also means that the earth’s measured temperature is 133 degrees higher than their claimed GB temperature and that they can account for that. They then proceed to do just that.
This is really one of the few things I’ve seen that actually makes sense. That doesn’t mean it’s correct, of course. Any insights are welcome.

beng
December 29, 2011 10:30 am

****
GeologyJim says:
December 29, 2011 at 8:54 am
Question: How has Venus managed to hold its thick atmosphere against the solar-wind flux? Is it just the greater molecular weight of CO2 compared to N2, O2, and such?
****
Not sure, but that would be my guess. H2O is easy to dissociate by UV (& then lost to space) compared to CO2, so water is lost first. O2 & N2 are next easiest, so Venus (& Mars), without magnetic fields, end up w/mostly CO2, which is heavier & relatively resistant to dissociation & loss to space. A planetary scientist (or Leif Sv) would know.
AFA this post is concerned, I’m reading & thinking. Time must pass to properly grok. Some flaws are obvious. Certainly Joel Shore is agitated.

Kevin Kilty
December 29, 2011 10:32 am

mkelly says:
December 29, 2011 at 9:21 am
Kevin Kilty says:
December 29, 2011 at 8:37 am
“I think the misunderstanding of the gas law, that temperature is not determined by pressure, is this contribution’s glaring flaw.”
quote from Ned and Karl excellent (adventure) post: “The thermal effect of pressure is vividly demonstrated on a cosmic scale by the process of star formation, where gravity-induced rise of gas pressure boosts the temperature of an interstellar cloud to the threshold of nuclear fusion.”
So according to you stars get hot, then the pressure increases. HMMMM. Weary interesting. By the way would you please call the folks at Cummins Diesel and let them know how wrong they are.
Further, comparing a solid (overcoat) to a gas I don’t think qualifies as a quote of the weak.

You are arguing exactly the reverse of what I said. Please read back up this thread to where I state that pressure results from gas at some level having to support the weight of gas above. It is basic statics, mkelly. Thus temperature has no bearing on pressure in the case of a star or on a planetary atmosphere–gravity is the principal agent. The temperature rise is from gravitational work. Once that work stops; once the star or atmosphere reaches an equilibrium size, then the temperature is determined entirely by energy in versus out. It is basic thermodynamics, mkelly. In the case of the Cummins diesel, the work done on the cylinder charge by the piston is what increases pressure and temperature–work is the causative agent. I never stated that temperature increases then pressure increases. Those are your words, do not try to put them in my mouth.

Arfur Bryant
December 29, 2011 10:38 am

What I welcome most about this paper, notwithstanding that it may or may not be proved to be correct, is the fact that, at last, Science is attempting to go back to Square One and take a refreshing look at the ‘Greenhouse Effect’, its cause and its internal workings.
Refreshing and welcome indeed.
ps, Does this mean that the science is more or less settled? 🙂

edbarbar
December 29, 2011 10:42 am

Why not do the following test. Put a satellite with a big surface with IR sensors in orbit. Create a tight beam of IR, shoot it at the IR sensor, and measure how the IR is affected.
Wait for C02 content to increase in the earth, and measure it again. Keep the satellite in low earth orbit so you can test lots of different locations, with different conditions.

DirkH
December 29, 2011 10:44 am

ChE says:
December 29, 2011 at 10:26 am
“Up to 4 % water?
Which leaves 96+% made out of what?”
So you’re saying 4% more or less is of no interest to you, fine.

John Tyndall
December 29, 2011 10:48 am

Kevin Kilty says:
December 29, 2011 at 10:32 am
Frustrating, isn’t it?

DirkH
December 29, 2011 10:51 am

Kevin Kilty says:
December 29, 2011 at 10:32 am
“The temperature rise is from gravitational work. Once that work stops; once the star or atmosphere reaches an equilibrium size, then the temperature is determined entirely by energy in versus out.”
So far so good, and when the temperature then rises due to more energy input, what does the pressure do? It rises. In fact, temperature and pressure must rise synchronously when the Ideal Gas Law holds. So, you are saying, the energy makes the temperature rise and that makes the pressure rise? Are you sure it’s not the energy making the pressure rise, which in turn makes the temperature rise?
Ah, how can we solve that? Maybe we can ask Booth, and he can then deride one of us?

Martin Mason
December 29, 2011 10:53 am

I’ve read before that the density of the atmosphere was once much higher than today’s and that this was why there were so many large flying dinosaurs.

AnonyMoose
December 29, 2011 10:56 am

In addition to the recalculation of the surface temperature, the viewpoint that it’s the number of gas atoms which is more significant than the types of atoms is an interesting contribution.

Kevin Kilty
December 29, 2011 10:58 am

James Sexton says:
December 29, 2011 at 9:30 am
lol @ the people wishing to refute the Idea Gas Law. People, there is a reason why this is called a law and not a theory or postulate. Look up the difference. There are plenty of things to pick at about this submission. The IGL isn’t one of them. You can assign reasoning and factors which go into it, but you’re not going to be able to get around, PV = nRT. ………. goobers.

No one is trying to refute the ideal gas law, but its application here is a mess. Pressure does not determine temperature in this instance. Gravity in effect determines pressure, energy balance determines temperature, then once those two variables are set all that the ideal gas law can possibly do is determine density.
People on this thread are citing air compressors, diesel engines, and all sorts of other contraptions to defend N&Z here, but the one thing no one has bothered with, including N&Z, is gravity, which is the most important factor determining pressure in a star or a planetary atmosphere. Gravity has no impact at all on pressure in compressors or diesel engines–it has everything to do with pressure and lapse rate in atmospheres.
Folks, I love the ideal gas law as much as the next person, but you cannot misapply it. Period.

crosspatch
December 29, 2011 10:58 am

I absolutely agree that it is likely the loss of atmospheric pressure over time that has caused this reduction in temperature over time. We have direct physical evidence of that fact.
http://levenspiel.com/octave/dinosaurs.htm
http://pubs.acs.org/subscribe/archive/ci/30/i12/html/12learn.html
It is estimated that Earth probably has about 125 to 250 million years left for sustaining life. At that point its atmosphere will be too void of CO2 to support plant life as we know it. The CO2 will have been scrubbed from the atmosphere to such a low level that it can no longer support plants which will then result in the loss of all the animal life. Going forward as the planet continues to cool and plate tectonics cease, even the oceans will outgas into space leaving a rather mars-like planet devoid of even water. It is plate tectonics that replenishes our atmosphere. That process is slowing. And we even have crazy people calling for extraction of geothermal heat on a global industrial scale which will slow that process even faster. Most of Earth’s U-235 has decayed. There is about 3% of it in the Earth as there was when Earth first formed. We’re doomed but it isn’t from anthropogenic climate change, it is due to geological climate change.

George Castanza
December 29, 2011 10:59 am

Kevin Kilty says:
December 29, 2011 at 10:32 am
Frustrating, isn’t it?

December 29, 2011 11:02 am

“The application of the gas law here is horrid.”
==============================================
What part of law do you not understand? It’s a LAW!!!! It always applies. lol, I’m wondering how many argued against the law of gravity when considering flight. It’s like saying “it doesn’t count because that’s not the way I’m thinking about it!”

DR
December 29, 2011 11:03 am

Earth Energy Budgets without ‘Greenhouse Gases’ or ‘Back Radiation’
It is a bit amusing how the missing hot spot has evolved into “it doesn’t matter” by AGW proponents. The topic is avoided like the plague.

Scott Covert
December 29, 2011 11:05 am

This paper looks testable and falsifiable. IT’S NOT “Climate Science”. /sarc
How about correlating radiometer readings for cloudless days vs barometric pressure? There should be loads of data available in the public domain. Possibly the expansion and compression of the atmosphere due to winds and turbulance could mask the effect but with enough data a signal might emerge. Expansion and compression might just cause overshoot in the temperature, downwelling radiation might oscillate in synch with pressure.
Worth looking at I think.

DR
December 29, 2011 11:07 am

Climate Models Without a ‘Greenhouse Effect’
Nick Stokes said such propositions are “pseudoscience” and would never be accepted in peer review journals.
Tell us then Nick Stokes, how did so many hockey stick papers get published?

DaveS
December 29, 2011 11:08 am

Joel Shore says:
December 29, 2011 at 9:30 am
“Just to comment on a few other things:
Figure 1. The Atmospheric Greenhouse Effect as taught at universities around the World (diagram from the website of the Penn State University Department of Meteorology).
Yes…This is taught throughout the world, but not as the most complete understanding of the greenhouse effect but rather as the simplest picture of the greenhouse effect. Hence, to criticize it as incomplete is silly…Everyone knows that it is incomplete. It is not meant to be the most complete or quantitatively-correct model. It is meant to be the simplest picture illustrating the basic effect.”
——————————————
I’ve not had time to read beyond the first couple of paragraphs. But that’s sufficient to see that the authors state “Figure 1 illustrates this concept using a simple two-layer system known as the Idealized Greenhouse Model (IGM).” Where do you get the idea from that the authors regard that illustration any differently to you?
On your second point I entirely agree that if the difference between the two methods of calculation isn’t quantified in the text, it should have been. We could then see more easily how the claimed difference between 133K and 18-33K is actually derived. But you haven’t quantified it either – ‘quite small’ is a meaningless phrase – so it’s impossible to judge.

Bengt Abelsson
December 29, 2011 11:12 am

JWBs home compressor tank is probably following the gas laws, a 15 K drop in temperature gives a 5 % drop in pressure that You won´t notice. (and a very very small drop in volume).
JWB should really know that.
DirkH, air is mostly N2 and O2 -hence diatomic. But the basic (ideal) gas laws applies also for the monoatomic gasas He and Ar

Pamela Gray
December 29, 2011 11:13 am

Funny Joel. I have read something similar about other paradigm shifts. To bad you have no idea who these paradigm smashing scientitsts were or else you would steer clear of such broad strokes in your paint over of this poster. At least this poster doesn’t lament about missing heat. The explanation for that missing heat is also missing in your settled science. Go complain about that on a warmist web site.

George Castanza
December 29, 2011 11:22 am

crosspatch says:
December 29, 2011 at 10:58 am
“It is estimated that Earth probably has about 125 to 250 million years left for sustaining life.”
Life as we know it, perhaps, but there are plenty of extremophiles that will live on.
But perhaps the way it works out is that intelligent technological species like ourselves come along and instead of being helpless in the face of geological changes they do some proactive terraforming to keep the status quo alive. In the last gasp however the sun is going to turn into a red giant and no amount of terraforming will save the planet. At that point it’s up to the technological species to pack up samples of the biosphere and rocket off to more hospitable location. You ever wonder why humans waste so much time and money on telescopes and space exploration? It’s probably an important long term survival instinct crafted by a long history of planet hopping…

Theo Goodwin
December 29, 2011 11:22 am

Kevin Kilty says:
December 29, 2011 at 10:20 am
Your specific criticisms of the article will be judged on their merits. I made no mention of them. You said that the article could not be published in a peer reviewed journal. Why did you say that? Wasn’t it a cheap shot. The article was clearly not designed to be published in a peer reviewed journal. Let’s not confuse the people who have no experience with peer reviewed journals.

ChE
December 29, 2011 11:24 am

So you’re saying 4% more or less is of no interest to you, fine.

I’m saying that it won’t affect the exponent. Which is all that changes when you go from monatomic to diatomic to polyatomic. It’ll be 1.3 within the accuracy of the equation.

George Castanza
December 29, 2011 11:28 am

Bengt Abelsson says:
December 29, 2011 at 11:12 am
“JWBs home compressor tank is probably following the gas laws, a 15 K drop in temperature gives a 5 % drop in pressure that You won´t notice.”
Yes, he probably wouldn’t notice. But I bet someone would notice a 5% drop in atmospheric pressure when the temperature goes down 15K. I’m pretty sure my ears would be popping like mofo when a cold front blows through if that was the case.
Pressure DOES NOT maintain temperature. As a banned poster here use to say… Write that down!

Pamela Gray
December 29, 2011 11:32 am

Kevin, are u saying that Earth’s pressure is thus in equilibrium and that any temp anomaly must be driven by some other mechanism? I question whether or not our gravity controlled pressure is in equilibrium. All the time? Please enlighten me on your thinking related to this point.

Ed_B
December 29, 2011 11:34 am

Kevin Kilty
Here is what the authors state:
“Equation (7) allows us to derive a simple yet robust formula for predicting a planet’s mean surface temperature as a function of only two variables – TOA solar irradiance and mean atmospheric surface pressure..”
I have no problem with this at all. It fits my understanding of the ideal gas law. I don’t understand what your complaint is. The effects of gravity are directly evidenced in the form of pressure (and density)

December 29, 2011 11:36 am

Finally, a writing that puts the whole picture together with the math/physics to back it up. We all had bits and pieces of this floating about our cranium; now they come together.
Finally, a redefinition of our open non-greenhouse atmosphere: Near-surface Atmospheric Thermal Enhancement (ATE)
I will be emailing this to many others who do not take the time to investigate on their own.
Thanks for a complete and fine work!

Chris B
December 29, 2011 11:43 am

Kevin Kilty says:
December 29, 2011 at 10:58 am
………….People on this thread are citing air compressors, diesel engines, and all sorts of other contraptions to defend N&Z here, but the one thing no one has bothered with, including N&Z, is gravity, which is the most important factor determining pressure in a star or a planetary atmosphere. Gravity has no impact at all on pressure in compressors or diesel engines–it has everything to do with pressure and lapse rate in atmospheres.
Folks, I love the ideal gas law as much as the next person, but you cannot misapply it. Period.
___________________________________-
Ya, I did, here……….
Chris B says:
December 29, 2011 at 8:25 am
Harry Dale Huffman says:
December 29, 2011 at 6:04 am
“Hence, the atmosphere does not act as a ‘blanket’ reducing the surface infrared cooling to space as maintained by the current GH theory, but is in and of itself a source of extra energy through pressure.”
It cannot be overemphasized that ALL of the energy comes from the Sun, contrary to what the authors seem to be saying in that quote.
__________________________________
Actually some of the energy comes from the core of the earth which is hot due to gravitational pressure and radioactive decay of the ancient “stardust” that made up our planet. Even Al Gore agrees, although the eath is only a few thousands of degrees in temperature on the underside of the crust, not millions of degrees.
The earth’s surface would probably be a lot colder if not for this “stored” heat, that continues to slowly cool as our solar system ages.

Kevin Kilty
December 29, 2011 11:43 am

DirkH says:
December 29, 2011 at 10:51 am
Kevin Kilty says:
December 29, 2011 at 10:32 am
“The temperature rise is from gravitational work. Once that work stops; once the star or atmosphere reaches an equilibrium size, then the temperature is determined entirely by energy in versus out.”
So far so good, and when the temperature then rises due to more energy input, what does the pressure do? It rises. In fact, temperature and pressure must rise synchronously when the Ideal Gas Law holds. So, you are saying, the energy makes the temperature rise and that makes the pressure rise? Are you sure it’s not the energy making the pressure rise, which in turn makes the temperature rise?
Ah, how can we solve that? Maybe we can ask Booth, and he can then deride one of us?

First, let’s leave JWB out of this. He’s been banished, but being unpopular or even rude, doesn’t make someone wrong.
Second, you ask “… when the temperature then rises due to more energy input, what does the pressure do? It rises….” No it does not. What occurs is that the atmosphere expands. in fact, let’s look at what happens on Earth. In summer, when the temperature goes up because of increased energy input, the atmosphere in the summer hemisphere expands, surfaces of equal pressure rise in height, and some of its mass goes into the winter hemisphere. Pressure then at some elevations actually goes down, while at others it goes up. You see, blind application of the ideal gas law leads to wrong conclusions. In a large system gravity has an impact that is not apparent in a small system and the ideal gas law includes no explicit provision for gravity.
Third, and last, you say “…So, you are saying, the energy makes the temperature rise and that makes the pressure rise? Are you sure it’s not the energy making the pressure rise, which in turn makes the temperature rise? ” In compressors and engines, where work is adiabatic (i.e. work without heat transfer) it is work that makes pressure and temperature rise together (in all cases though the ideal gas law pertains). But when heat transfer dominates the situation, as it always does in slow processes, or in the long run, then balance of energy determines temperature. Static mechanical equilibrium is what determines pressure (even in a compressor receiver tank this is so). If pressure and temperature are set by other considerations, the ideal gas law then simply specifies one state variable that is left — specific volume or its inverse, density.

Bengt Abelsson
December 29, 2011 11:54 am

G Castanza – the volume is not constant in your example.

Philip Bradley
December 29, 2011 11:54 am

I wouldn’t assume cloud variation is due to GCRs. Some proportion (and perhaps a large proportion) is due to variations in anthropogenic and natural aerosols (which seed clouds).
This affect will be exaggerated by surface temperatures being predominantly measured around urban areas where the anthropogenic aerosol effect is largest.
A study measuring cloud changes over urban areas versus non-urban areas would be interesting.

Kevin Kilty
December 29, 2011 11:57 am

Pamela Gray says:
December 29, 2011 at 11:32 am
Kevin, are u saying that Earth’s pressure is thus in equilibrium and that any temp anomaly must be driven by some other mechanism? I question whether or not our gravity controlled pressure is in equilibrium. All the time? Please enlighten me on your thinking related to this point.

Basically that is what I am saying. The predominant effect on pressure is gravity. If you measure the actual air pressure up here in Wyoming it is 200mb lower than sea level on average simply because of “hydrostatic” equilibrium. There are small departures from this equilibrium pressure, and this leads to wind of course, but these are small departures. A few mb of difference results in the winds of a mid-latitude cyclone but a few mb is also the difference you experience going up in elevation by a few tens of meters.
At any rate, saying that atmospheric pressure determines temperature is wrong, just as saying that temperature declines with elevation because of pressure drop is wrong. Temperature declines with elevation because of a change in energy balance, and this balance also includes the mechanical work done when air is raised or lowered in elevation. The ideal gas law applies to all of this only indirectly.
Did I clarify my thinking?

Urederra
December 29, 2011 12:02 pm

Ugh. From wiki and “Mercury Fact Sheet”. NASA Goddard Space Flight Center. November 30, 2007. Retrieved 2008-05-28.

The mean surface temperature of Mercury is 442.5 K,[3]

In Table 1 it says that the “observed” mean temperature is 248.2 K. I could not find why the authors are using 248.2 instead of 442.5 K as NASA Goddard Space Flight Center suggests. http://nssdc.gsfc.nasa.gov/planetary/factsheet/mercuryfact.html

December 29, 2011 12:10 pm

Kevin Kilty says:
December 29, 2011 at 10:32 am
mkelly says:
December 29, 2011 at 9:21 am
Kevin Kilty says:
December 29, 2011 at 8:37 am
KKilty says: “…, that temperature is not determined by pressure, ….”
Thus temperature has no bearing on pressure in the case of a star or on a planetary atmosphere–gravity is the principal agent. The temperature rise is from gravitational work.
Sir, you change the discussion from pressure/temperature to work. In a star the gravitional force increases the pressure while the volume is shrinking thus T must rise. In regard PV =nRT if P is increased and V held steady then T must rise. If you ascribe the rise in P to work done by gravity I agree. Also with the diesel work. But we were just talking about the gas law and not about work.

wayne
December 29, 2011 12:14 pm

Many commenters here are already getting pressure and density miss applied within their words resulting in non-science dribble. Don’t give the local agw zombies a hammer. Once more time…. pressure has only to do with mass (the mass of the atmosphere), acceleration (gravitational acceleration), and area (the area of Earth’s surface). That’s all. Look at the units:
Density: mass/volume = kg/m3
Pressure: Pa = force/area = N/m2 = kg (m/s2) / m2 = (finally, true but no meaning) kg/m/s2.
kg = mass of atmosphere
m/s2 = gravitational acceleration
m2 = area of Earth’s surface
In a mostly stable gravitationally held atmosphere:
Changes in pressure (the cause) reflects in changes in temperature (the effect), constant volume but an atmosphere is not constant volume.
Changes in temperature (the cause) reflects in changes in density (the effect), constant pressure, an atmosphere has a constant mean pressure.
Temperature cannot cause first order changes in pressure in atmospheres but it does change density.
Density cannot cause first order changes in either temperature or pressure, it is an end effect. But density does affect radiation passing through that mass.
Everyone seems to have P•V=n•R•T burned in their mind but in relation to a gravitation held atmospheres these equivalents are much more relevant and helpful:
P•V•M = m•R•T and
P•M = ρ•R•T or P/ ρ = T•R/M
M being molar mass, m being mass, ρ being density
Rearrange as needed.
Seemed to be needed here. Hope that might help some here and hopfully no mistakes.

December 29, 2011 12:16 pm

Kevin Kilty says:
December 29, 2011 at 10:58 am
you’re not going to be able to get around, PV = nRT. ………. goobers.
“““““““““““““““““““““““““““““““““““““““““““““““““““““
No one is trying to refute the ideal gas law, but its application here is a mess. Pressure does not determine temperature in this instance. Gravity in effect determines pressure, energy balance determines temperature, then once those two variables are set all that the ideal gas law can possibly do is determine density.
……….. Gravity has no impact at all on pressure in compressors or diesel engines–it has everything to do with pressure and lapse rate in atmospheres.
Folks, I love the ideal gas law as much as the next person, but you cannot misapply it. Period.
=================================================
And, yet, PV still equals nRT . So, instead of focusing on the pressure part of the equation, I’d look elsewhere. Of course, we should also note, our pressure isn’t constant, and the delta in temps that we’re talking about is about 0.5 Kelvin. (55° to 56° for instance)
I think what most people are getting hung up on is scale and sensitivity. Everyone likes to think our temp change has been dramatic. It hasn’t been. Everyone is used to seeing the temp graphs with the accelerating increase. But, if one scales it properly and puts it on a Kelvin scale…. it wouldn’t even be a noticeable bump on the line.

J Martin
December 29, 2011 12:19 pm

@ GeologyJim says “Question: How has Venus managed to hold its thick atmosphere against the solar-wind flux? Is it just the greater molecular weight of CO2 compared to N2, O2, and such?”
Perhaps it hasn’t and it was once even greater than it is now. Or / and as Venus is that much nearer the sun it’s innards must be subject to far greater tidal churn and so is or has been much more volcanically active than earth.
Dale Huffman should be feeling happy that this paper vindicates him;
http://theendofthemystery.blogspot.com/2010/11/venus-no-greenhouse-effect.html
@ Stephen Wilde, you said “It fits nicely with the description that I published back in May 2008”
you also said “My work then goes on to link all that to solar activity from above and oceanic variability from below for a more complete Unified Theory than that presented here (IMHO).”
Please post a link for this or better still ask Anthony to put it on WUWT as an article / paper, it sounds more than interesting enough and I like the idea of extending it to solar and sst.

JPeden
December 29, 2011 12:20 pm

So I guess the CO2 molecules are to blame for about 0.0392% of that “unprecedented” “catastrophic” warming we are suffering under now?

Kevin Kilty
December 29, 2011 12:23 pm

Ed_B says:
December 29, 2011 at 11:34 am
Kevin Kilty
Here is what the authors state:
“Equation (7) allows us to derive a simple yet robust formula for predicting a planet’s mean surface temperature as a function of only two variables – TOA solar irradiance and mean atmospheric surface pressure..”
I have no problem with this at all. It fits my understanding of the ideal gas law. I don’t understand what your complaint is. The effects of gravity are directly evidenced in the form of pressure (and density)

Here is my complaint. My engineering students constantly misapply the ideal gas law by confusing the relationship it suggests among P, V, n, and T with causation. P, V, and n do not cause temperature, and that is what the authors either do maintain or are implying here. Temperature in the long run is always the result of energy balance. So, I have no problem with N&Z saying that irrandiance at TOA is an important factor–it is. Period. However, to the extent that pressure has anything to do with this, it is just acting as a proxy for the true important factors. On earth you could think of pressure as a proxy for optical depth which is important; but it is only a proxy.
Let’s take Venus as an example. We know the surface temperature is very high. We also know the surface pressure is very high. But the surface pressure does not cause the high temperature. The surface temperature results from 1) irradiance that is absorbed predominantly high in the atmosphere leading to a high temperature there, and 2) then a lot of work input by gravity as convection takes parcels from up high to the surface. There is also a little irradiance absorbed at the surface which is what drives the convection. The high surface temperature results from the lapse rate, and we know the lapse rate depends on specific heat at constant pressure and the gravitational constant–pressure does not enter the problem explicitly.
Now you could object by saying, well the atmosphere absorbs irradiance so high because there is so much mass in Venus’ atmosphere and pressure is related to the mass in the atmosphere, and you could also say that the work involved in taking parcels of atmosphere to the surface is just pressure-volume work, and so involves pressure. But really irradiance and gravity are the factors, and pressure is but a proxy for the latter. Think of a planet like Venus, but with an argon atmosphere, that is far more transparent (less optical depth), yet has the same surface pressure. The difference in transparency would lead to a very different surface temperature, yet irradiance TOA and surface pressure would be the same.
I hope I am making myself clear.

Bengt Abelsson
December 29, 2011 12:23 pm

Urederra
Yes, but Wikipedia says that the range is 90 – 700 K. It takes a bit of math to estimate the equivalent blackbody temperature from that.
Some explanation should be nice.

Pamela Gray
December 29, 2011 12:26 pm

I get the colder planet equation. It makes sense and allows a global temperature comparison with a global temperature mechanism with and without an atmosphere. Otherwise u are only measuring a point on the Earth against a global average. This poster looks at the gray body as a sphere on an axis and explains the temp of the gray body at all points, those facing the sun and its angles, as well as those points facing away from the Sun. The authors then went looking for a mechanism with enough energy to get us to room temperature so to speak.
So, if the first equation holds true, greenhouse effects cannot alone explain why we are as warm as we are. So warmists, what are your thoughts on the temperature equation without an atmosphere?

December 29, 2011 12:26 pm

Kevin, are you saying that high altitude areas are colder than low altitude areas because of differences in radiative balance, not the difference in pressure?

Phil's Dad
December 29, 2011 12:28 pm

@ Mr Kilty
I am not sure that you are arguing the same case. As far as I can tell what the authors are saying is that for a given gravity and a given energy balance if you increase the total amount of gas present the pressure at the surface will increase. Given (as stated) the unchanged energy balance, the surface temperature will increase. How is this a miss-application of the law?

J Martin
December 29, 2011 12:32 pm

Oops, I missed the fact that Dale Huffman had commented above on this paper. Having scanned through Dale’s comment, I remain of the view that the two papers are in fundamental agreement.

Urederra
December 29, 2011 12:32 pm

@Gengt
It is not only wikipedia, but also the NASA Goddard Space Flight Center that says that the average temperature on Mercury is 440 K

Mercury Atmosphere (Exosphere)
Surface pressure: ~10-15 bar (0.001 picobar)
Average temperature: 440 K (167 C) (590-725 K, sunward side)

http://nssdc.gsfc.nasa.gov/planetary/factsheet/mercuryfact.html

Kevin Kilty
December 29, 2011 12:33 pm

mkelly says:
December 29, 2011 at 12:10 pm
Kevin Kilty says:
December 29, 2011 at 10:32 am
mkelly says:
December 29, 2011 at 9:21 am
Kevin Kilty says:
December 29, 2011 at 8:37 am
KKilty says: “…, that temperature is not determined by pressure, ….”
Thus temperature has no bearing on pressure in the case of a star or on a planetary atmosphere–gravity is the principal agent. The temperature rise is from gravitational work.
Sir, you change the discussion from pressure/temperature to work. In a star the gravitional force increases the pressure while the volume is shrinking thus T must rise. In regard PV =nRT if P is increased and V held steady then T must rise. If you ascribe the rise in P to work done by gravity I agree. Also with the diesel work. But we were just talking about the gas law and not about work.

Mkelly, I’m going to try to explain this better. The original temperature rise that got the star going in the first place was from gravitational work. Once that work is done, however, the star temperature results from energy balance, not from pressure. Oh, you do have to maintain a pretty high temperature and pressure to keep the nuclear reactions going, but pressure does not cause temperature.
I am not changing the topic from one thing to another. They are interrelated.
The ideal gas law is just an equation of state that relates one state variable (T say) to others (P,V, and n). Thermodynamics is the application of energy concepts to the system these state variables describe. I bring work into all of this because work+heat is the essence of energy balance. You cannot explain what goes on in an atmosphere or a diesel engine without considering heat+work. I am not changing the topic, I’m simply adding what has to be added to make sense.

crosspatch
December 29, 2011 12:34 pm

“Life as we know it, perhaps, but there are plenty of extremophiles that will live on.”
For a while, but then even those will die. The point being that the planet Earth will be uninhabitable for human beings long before the Sun gets too hot because our atmosphere will blow away in the solar wind and volcanism will drop below the level required to sustain it.

December 29, 2011 12:44 pm

Kevin, when Carl Sagan first estimated the surface temperature of Venus, he based it on radar depth to the surface, the known cloud top altitude and temperature, and two guesses at the atmosphereic composition, one entirely nitrogen and one CO2. His estimate for a nitrogen atmosphere was hotter than for a CO2 atmophere, purely due to the different lapse rates between N2 and CO2.
I think what the authors are arguing (or should be) is that the lapse rate determines the delta T’s, and then some aspect of the atmosphere system or planet will set the radiative equilibrium with the external environment, putting absolute temperatures to all the points in the system. The radiative equilibrium could be set at the surface (if the atmosphere was completely transparent to radiation), in which case the surface temperature would the same as with no atmosphere at all (and the atmosphere itself would of course get colder and colder than that temperature as you go up). Or the radiative equilibrium could be set much higher up, such as the cloud tops of Venus, in which case the atmosphere at deeper levels will remain much hotter than the cloud tops.
Let me try an analogy somewhat better than a diesel engine and switch us to a Brayton Cycle, using a multi-stage axial flow compressor followed by an expansion turbine. As air moves vertically through an atmosphere, the pressure changes result in temperature changes, just as the air gets hotter through each compressor stage and cooler through each expansion turbine stage. As long as the engine runs (and gravity runs forever), the high pressure air will be hotter than the low pressure air. If we connect the high pressure air to a heatsink at room temperature, it will equilibrate to room temperature and the low pressure air will be very cold. If we hook the low pressure air to our heatsink then the high pressure air will stay very hot. So the planetary question is what level of the atmosphere is locked in thermal equilibrium with the external environment.

Kevin Kilty
December 29, 2011 12:45 pm

George Turner says:
December 29, 2011 at 12:26 pm
Kevin, are you saying that high altitude areas are colder than low altitude areas because of differences in radiative balance, not the difference in pressure?

Partially, let me elaborate a bit. High altitude areas are colder because the energy balance is different, and radiation is only a part of the energy balance. Work is a big factor. As air moves upward to higher elevation, either because of convection or general flow of the air, it has to do work against gravity and this leads to a cooling of about 6F per thousand feet or almost 10C per kilometer. That is a lot of cooling to be made up with only radiation. Also, while the surface heats well at high elevation from solar irradiance, the atmosphere is much more transparent to IR radiation loss back to space. So the result is that a difference in energy balance leads to lower air temperature at high elevation. Pressure is not the cause.

Bengt Abelsson
December 29, 2011 12:46 pm

Uerderra
I agree that average temp is stated as 440 K.
My point is that arithmetic mean (T1 + T2)/2 may give a wrong answer to the question of the equivalent black (grey ?) body which depend on T^4.

December 29, 2011 12:48 pm

J Martin says:
December 29, 2011 at 12:19 pm
“Please post a link for this or better still ask Anthony to put it on WUWT as an article / paper, it sounds more than interesting enough and I like the idea of extending it to solar and sst.”
I’ve posted it several times before but no real takers. However the Nikolov findings add relevance so here goes again:
For the top down solar influence see here:
http://climaterealists.com/attachments/ftp/How%20The%20Sun%20Could%20Control%20Earths%20Temperature.pdf
For the bottom up oceanic effect see here:
http://climaterealists.com/attachments/ftp/TheSettingAndMaintainingOfEarth.pdf
And the Unifying system response see here:
http://climaterealists.com/attachments/ftp/TheUnifyingTheoryofEarthsClimate.pdf
As far as I can see at present all is consistent with the Nikolov paper and with observations but inconsistent with established climate theory.
I see Nikolov as adding quantitative science to my qualitative description.

J Martin
December 29, 2011 12:50 pm

crosspatch said “…because our atmosphere will blow away in the solar wind and volcanism will drop below the level required to sustain it.”
If mankind faces extinction, then I imagine that future generations will not go down without a fight. Including trying to open up or create volcanos. Or import atmosphere from other planets. Use fusion somehow, pity there’s no nitrogen in H2o, but maybe we can liberate nitrogen from somewhere.
A fight for survival we thankfully won’t have to face.

Phil's Dad
December 29, 2011 12:50 pm

@ Will (says: December 29, 2011 at 12:19 pm )
This is one of those cases where you must take more than a single factor into account. You are right that if we had no atmosphere (and thus no water cycle) the temperature would be similar to that of the Moon. However, in order to isolate the atmospheric effect on temperature on an Earth which has both atmosphere and a water cycle, you must also eliminate the water cycle as a factor – which is currently keeping us cooler than we would otherwise be. (There will be other factors too). Factoring in the knowns, the figure of 133K is entirely reasonable and need not detract from the rest of the paper.

LazyTeenager
December 29, 2011 12:52 pm

richard verney says:
December 29, 2011 at 5:58 am
I remember as a young boy back in the 60s/70s when there was great interest in space that my Dad told me that the reason why Venus was hot was due to the pressure of its atmosphere and he iullustrated it with a bicycle pump. I guess that different science was being taught back in that day and age.
——-
Nup. Same science then as now.
The whole idea is a common misconception. It results from confusing the process of compressing the gas with the state of being compressed.
The heating of the gas in the bicycle pump arises because the pressure is being CHANGED.
The heating does not arise because the gas is at a higher pressure.
A simple counter-example to your idea is a cylinder of compressed gas. That gas might be at 2000bar pressure. But you won’t burn your hand if you touch it. It will be at room temperature.
The atmospheric pressure on Venus is not changing so there is no pressure heating. Instead it’s temperature is maintained by solar heating.

Kevin Kilty
December 29, 2011 12:55 pm

Phil’s Dad says:
December 29, 2011 at 12:28 pm
@ Mr Kilty

You know, I have now typed so much on this topic, with a 2 year-old and a baby distracting me, that my memory of exactly what the authors said is getting foggy in my mind, and perhaps I should go back and re-read. But there is a pervasive that the ideal gas law is causative, in that a given P, V, and n will cause a particular temperature. You can see this in the comments here, and I find it to be common among my second and third year engineering students. I sensed it to be part of what the authors were implying. That is the idea I am trying to squash. Temperature results from energy balance. Pressure and density then follow according to the ideal gas law and the total material (n) in a system.

Joe
December 29, 2011 12:59 pm

“Gravity in effect determines pressure, energy balance determines temperature”
Ummmm.. what?
Second, you ask “… when the temperature then rises due to more energy input, what does the pressure do? It rises….” No it does not. What occurs is that the atmosphere expands. in fact, let’s look at what happens on Earth.
As you say, let’s actually look at what happens on Earth. When energy is introduced to the atmosphere the atmosphere expands, true… but to say it only expands is to ignore gravity. Since Earth actually has a gravitational field, that gravity creates pressure and denies fully free expansion of the gas, so that added energy will in part expand the atmosphere, and it part heat the atmosphere. As such, the gravitational pressure that heat.

David, UK
December 29, 2011 1:03 pm

James Reid says:
December 29, 2011 at 9:03 am
Do I hear deathly silence from the Lazy Teenager and R Gates?

Haha! You read my mind!

Theo Goodwin
December 29, 2011 1:07 pm

The word ‘energy’ is ambiguous in multiple ways. For the purposes of this discussion, an important use of the word ‘energy’ has to do with the Earth-Sun system of radiation exchange so familiar from Warmist models. With regard to that system, Earth does not create energy.
But there are other uses of the word ‘energy’. Windmills are created to transform the energy of the winds into electricity. In that use of the word ‘energy’, the reference is not to the Earth-Sun system and the statement is perfectly meaningful.
Because the article under discussion departs from the “radiation only” model of the Earth-Sun system used by the Warmists, when the authors write or imply that energy is being created they are referring to a set of physical hypotheses that is independent of the Earth-Sun system. Give them a chance. Try to understand their use of the word ‘energy’ by reference to the physical hypotheses that they are attempting to elucidate.

Joe
December 29, 2011 1:14 pm

There was a video distributed here earlier (in the debunking of Al Gore’s “24 hours of realism” video) that touched on this theory as well. It took a slightly different tack in proving it’s point by arguing that, when you account for the difference is solar irradiance, Venusian atmosphere is actually roughly the same temperature as Earth’s atmosphere at equal atmospheric pressure.
I think that video “Greenhouse in a Bottle, Reconsidered” has been taken down now, but it espouses the same theory. It would be nice if someone had a copy of it and could post it. I can’t seem to find it.

Chad Jessup
December 29, 2011 1:14 pm

Stephen Wilde – thank you for your continuing climate educational efforts.

December 29, 2011 1:19 pm

Kevin Kilty says:
December 29, 2011 at 12:33 pm
Sir, I don’t disagree with what you say. As I said last time I agree. I was being more narrowly focused as to what the equation says. However, work is seldom mentioned (which is a shame) in discussions of climate.

Martin Mason
December 29, 2011 1:23 pm

Surely the average temperature of the earth without an atmosphere wouldn’t be the same as the moon because the rate of rotation is different?

Stephen Richards
December 29, 2011 1:24 pm

Joe says:
December 29, 2011 at 12:59 pm
“Gravity in effect determines pressure, energy balance determines temperature”
Ummmm.. what?
Second, you ask “… when the temperature then rises due to more energy input, what does the pressure do? It rises….” No it does not. What occurs is that the atmosphere expands. in fact, let’s look at what happens on Earth.
As you say, let’s actually look at what happens on Earth. When energy is introduced to the atmosphere the atmosphere expands, true… but to say it only expands is to ignore gravity. Since Earth actually has a gravitational field, that gravity creates pressure and denies fully free expansion of the gas, so that added energy will in part expand the atmosphere, and it part heat the atmosphere. As such, the gravitational pressure that heat.
The closest (simple) analogy to the atmosphere as Joe describes would be a balloon. As the air heats the pressure inside increases and the outside expands. If you think of gravity as the skin of the balloon then you will see that the pressure inside the balloon increases but not at the rate one would expect if the balloon was rigid. It’s far more complex than I have explained here but I hope you get the gist.
So , “Gravity in effect determines pressure, energy balance determines temperature” If you look deeply into this phrase you will note that the author might have a point but may not have expressed in a way that makes it easy to understand what he/she actually meant to say.

Nullius in Verba
December 29, 2011 1:28 pm

I’m not going to bother rehashing the greenhouse effect physics. I gave an explanation of why pressure is indeed more important than backradiation here: http://judithcurry.com/2010/12/02/best-of-the-greenhouse/ Professor Curry splits my explanation into two sections, so scroll down.
However, it’s not true to say that it is purely due to pressure, or that greenhouse warming would occur with a totally transparent atmosphere.
Somebody asked for a more extensive list of issues with the paper. Here are just a few I picked out going through.
“According to the current theory,…”
While this theory is commonly found in explanations to the general public, it is not in fact the one used by climate scientists for actual calculations. Those calculations take convection and pressure effects fully into account.
“To be correct, Tgb must be computed via proper spherical integration of the planetary temperature field…”
It’s still not correct. The actual temperature with an atmosphere will also involve horizontal heat transfer via convection cycles, and the loss of heat from the surface is not just by radiation but also latent and sensible heat. Heat storage, and surface thermal conductivity also have an effect because of the day/night cycle. The temperature is proportional to the fourth power of the heat radiated, so to get the ‘radiative average’ temperature, you have to take the fourth root of temperature to get power, average, and then raise to the fourth power again to get a single equivalent temperature. The difference is not so large.
“Can a handful of trace gases which amount to less than 0.5% of atmospheric mass trap enough radiant heat to cause such a huge thermal enhancement at the surface?”
Yes. If it wasn’t for convection, the average surface temperature on Earth would be about 65-70 C. And CO2 is not the only GHG – water vapour constitutes more than 0.5%.
“Modern GCMs do not solve simultaneously radiative transfer and convection.”
As I understand it, they do – although they sometimes fudge the details somewhat.
“the term Greenhouse Effect is a misnomer”
True, but I think that’s well-accepted, even by advocates.
“…guarantees that a mean global temperature can be accurately estimated from planetary averages of surface pressure and air volume…”
No, because it also depends on their vertical distribution, and because the volume is a function of the temperature distribution (which in turn is due to radiation/convection effects), not vice versa. Cause and effect are reversed.
“…related to total surface pressure through a nearly perfect regression fit…”
You have a number of planets with no atmosphere, to which the regression is insensitive, and only three planets are used to define the curve. Since you have four free parameters, a perfect fit is not surprising. It’s not significant.

P Walker
December 29, 2011 1:30 pm

Folks ,
Somewhere in the midst of this melee , one of the authors ( I forget which and don’t hace the time to look ) commented that this was just a preview of what are to become four papers that they will submit for peer reviewed publication . No point in getting too worked up over this . BTW the harshest critics here are probably doing the authors a tremendous service , whether they mean to or not .

Editor
December 29, 2011 1:30 pm

Brian H says:
December 29, 2011 at 8:47 am

Jeff L says:
December 29, 2011 at 6:31 am

The one red flag I see is the long term predicted pressure profile in figure 8. Eocene pressures max out at ~ 185 kPa – that’s approaching double today’s standard pressure of ~ 101 kPa !! I would think at those kind of pressures there would be some biological effects which might be manifested in the fossil record – how life adapted to such high pressures.
That barely scratches the surface:
http://levenspiel.com/octave/dinosaurs.htm
and
http://pubs.acs.org/subscribe/archive/ci/30/i12/html/12learn.html
Thicker air allowed >100lb. pteranodons to fly. They couldn’t in a 1 bar atmosphere.

Hmm. I don’t know beans about air pressure differences in the geologic past. I’ll call my geologist brother to fill me in.
Brian’s links are far more fascinating than the vitriol over gas laws. I thought we settled some of that stuff in the Venus threads. Everyone, climb a mountain and chill out from the adiabtic expansion! Then read those links. BTW, they argue that big flying things needed “at least 3.7–5.0 bar” when they were around 100-65 Mya. That’s a lot more than the 1.6 bar from figure 9 at 65 Mya.
I really, really like to see multiple lines of research support some outrageous claim. Clearly more study is needed.
I think the dinosaur blood pumping question is not nearly so good. The real question is what was the venous blood pressure, and what kept the veins from collapsing or developing a vacuum.
GeologyJim says:
December 29, 2011 at 8:54 am

Question: How has Venus managed to hold its thick atmosphere against the solar-wind flux? Is it just the greater molecular weight of CO2 compared to N2, O2, and such?

A claim in the second link is that the Earth’s carbonates hold the equivalent of 55 bar of CO2. So the answer may be that Earth did not lose its CO2, but by keeping its H2O, pathways opened to take CO2 out of the atmosphere and put it underfoot.

crosspatch
December 29, 2011 1:33 pm

I imagine that future generations will not go down without a fight. Including trying to open up or create volcanos.

We can attempt to enrich the CO2 of volcanoes maybe by dumping limestone into them or something. Maybe we could inject nuclear waste on the subducting side of thrust faults, I don’t know. Nuclear power is pretty much the only answer, though, long term.

Stephen Richards
December 29, 2011 1:35 pm

Kevin Kilty says:
December 29, 2011 at 12:45 pm
George Turner says:
December 29, 2011 at 12:26 pm
Kevin, are you saying that high altitude areas are colder than low altitude areas because of differences in radiative balance, not the difference in pressure?
This is a cause and effect problem. Kevin, I had to read your comment several times to get the gist.
The feeling of cold at altitude is due to the lack of pressure. The reason there is a lack of pressure is because there is a lack of gas. The reason there is a lack of gas is down to gravity. Work is done when the state of a mass is changed from steady to accelerating. In the lower atmosphere where an anticyclone (high pressure) is present for a long period what is the profile of temperature through the lower atmosphere and where a low pressure is present how does that change the temperature profile.?

Theodore
December 29, 2011 1:39 pm

simpleseekeraftertruth says: “If this pans out then Nobel Prizes (at least) are in order.”
And Nobel Prizes in SCIENCE no less.

wayne
December 29, 2011 1:40 pm

I am going to use one of those equations just posted above to form of a conclusion to this Ideal Gas Law controlling the temperature.
P•M = ρ•R•T can rewrite as T = P/ ρ • M/R. Now that is curious, P/ ρ is just pressure per density ratio and M/R is just the molar mass ratio to the gas constant. What can possibly change?
The mean averaged pressure over all of the earth is ~101325 Pa and except for local weather effects has not changed for a many, many years. It depends on the gravitational acceleration which depends on the Earth’s mass and it has not changed significantly. It also depends on the surface area of the Earth and it has not changed. And it is dependant on the mass of the Earth’s mass, well, covered that above.
And the mean density (ρ) has not changed either since the mass of the atmosphere is basically constant as well as the surface area of the Earth.
How about R, the gas constant, well, it IS a constant.
The only term left is M, the mean molar mass of the atmosphere’s components and per the Engineering Toolbox and Wikipedia at 28.97 g/mol and 28.966 according to the Handbook of Chemistry and Physics (1968). If these are correct then this is the only parameter in the Ideal Gas Law above and so T may have changed by 0.004/28.966 or 0.0138% from 288.15 listed as the mean temperature in 1968 and that would be 288.15 • 0.000138 or 0.039 K.
We know the atmosphere is no an ideal gas per se but I can’t find any feasible deviations from an ideal gas that would affect the equation above when speaking of anomalies, as the +0.039°C anomaly calculated since 1968.
So am I wrong, I see no way by the above calculations using the Ideal Gas Law that the Earth’s mean temperature could EVER change without something very catastrophic as a meteor hit, ejecting a sizeable amount of our atmosphere’s mass into space. I don’t see it.
If my calculations above are wrong, please tell me.

December 29, 2011 1:42 pm

Expected geomagnetic storm in progress now
http://flux.phys.uit.no/cgi-bin/plotgeodata.cgi?Last24&site=tro2a&

Cherry Pick
December 29, 2011 1:49 pm

This is interesting and I am still reading it. Fast comments.
Equations 7 and 8 are just curve fittings, regression equations that are said to resemble the natural gas laws which is demonstrated in figures 5 and 6. Please, rewrite that and show that observations match the theoretical calculations.
How does this theory explain Earth’s Thermosphere?

wayne
December 29, 2011 1:54 pm

opps… mistake above at December 29, 2011 at 1:40 pm :
“And it is dependant on the mass of the Earth’s mass, well, covered that above.”
should have been more like…
“And it also dependant on the mass of the Earth’s atmosphere and it has not been known to have changed significantly.”

December 29, 2011 1:55 pm

Interesting.
Will reward a second reading, and probably a third.
Not sure my maths is up to a full comprehension – of the paper or of the various comments.
However, taken together, there appears to be a very strong feeling [feelings are NOT science, I know] that there are many [very many] factors at work in weather and climate.
Some of these are new to this dilettante observer.
One or two, shall we sy, seem to be unfamiliar to the Team.
Certainly a thought-provoking paper, and the thoughts provoked, on this thread alone [Tallbloke, your comments noted!], will make the science stronger [if a little less-settled, hey, Team?].
Looks like good work all round – science that is being reviewed by [near]-peers, challenged and – please – talked about.
Where are the comments from George Monbiot, say? Or Geoffrey Lean, even?
Might this paper be t h e game-changer?
I don’t know, but it does ask questons that I, for one, would like to see answered.
Finally, happy New year to all reading this – pro/anti/neutral whatever it is!

Joel Shore
December 29, 2011 1:59 pm

Gary Pearse says:

How about chopping up the blubber of the calculations based on IGL for the planetary bodies and their comparison with the measured temps

Okay…fine…Let’s deal with the two aspects of their post that haven’t yet been shredded to pieces.
(1) The ideal gas law: Who would think that if you calculate the surface pressure and number density then you can use the ideal gas law to determine the temperature? Wow…That’s quite a shock! It certainly won’t surprise my intro physics students who I already tell that the ideal gas law works very well for real gases as long as you stay away from high (close to liquid) densities. But just knowing that there is this relationship between pressure, temperature, and density doesn’t get you very far in uniquely determining the surface temperature of a planet.
(2) Their Equation (7): They’ve written down an expression relating temperature and pressure with 4 free parameters that they have fit to data for various bodies in our solar system and now they are shocked that it does a pretty good job for 3 or 4 of them and (and not for hot for others). And we’re supposed to impressed by this?

Mydogsgotnonose
December 29, 2011 2:00 pm

Nullus in Verba: 1.28 pm.
I read the Curry blog piece and I must disagree with the ‘back radiation ‘ part, also 33K present GHG warming.. This is where climate science has gone horribly wrong.
1. ’33 K’. Take out the atmosphere and albedo falls from 0.3 to 0.07 [no clouds or ice] so equilibrium radiant temperature to space =~0°C meaning maximum GHG warming is 15 K. Do a proper calculation including the aerosols and it falls to ~9 K with 24 K lapse rate warming at the surface.
2. ‘Back Radiation’:Imagine two parallel plates at the same temperature. Put a radiometer in the gap and it measures ‘back radiation’ Turn it 180° and it measures the same in the opposite direction. So, ‘back radiation’ is cancelled out at equilibrium. A Dutch researcher showed over a height of 800 feet at night, up-down fell exponentially to zero, Beer’s law absorption of IR from the Earth’s surface with zero net IR from the atmosphere towards the Earth’s surface.
So, back radiation can do no thermodynamic work. It’s a measure of temperature and emissivity. It is the means by which the density of IR states in the emitter communicate with those in the absorber and vice versa, hence Kirchhoff’s law of Radiation. It is ‘Prevost Exchange Energy.’.
As I said, in my view climate science has gone horribly wrong because of serious misunderstanding of the heat transfer and the real GHG scaling. It gets out of the trap by incorrect aerosol optical physics and the imaginary negative net AIE There’s a second optical effect which makes this slightly positive now but much more in ice ages or polar regions. I am publishing this, wjich is another GW/AGW mechanism. GHG-GW/AGW is very much smaller than claimed [maximum of 15% of the IPCC median 3 K climate sensitivity].

ntesdorf
December 29, 2011 2:01 pm

Great Paper! It is very good to see physics and inter-planetary studies at last making a come-back in the arguments. What we need now are similar papers on the effects of the Oceans and the hot core in the earth and we will be drawing aside the last of the veils.
Congratulations on a good start..

Dave Wendt
December 29, 2011 2:02 pm

Joel Shore says:
December 29, 2011 at 9:30 am
“Hölder’s inequality only has a significant effect if the temperature range on the planet is very large on an absolute temperature scale. For the Earth, the temperature range is moderate enough on an absolute scale that the difference between averaging the temperature and taking the fourth root of the average of T^4 is quite small.”
The daily range is routinely over 100 degrees and at maximum approaches 150 degrees, that doesn’t strike me as all that moderate relative to an average of 288.

December 29, 2011 2:06 pm

“So am I wrong, I see no way by the above calculations using the Ideal Gas Law that the Earth’s mean temperature could EVER change without something very catastrophic as a meteor hit, ejecting a sizeable amount of our atmosphere’s mass into space. I don’t see it.”
Depends what you mean by ‘mean’ temperature. To get at that you need to somehow obtain a n average temperature for the entire atmosphere and oceans combined. Not easy.
I prefer the concept of system energy content defined as solar energy that has arrived in the system and not yet departed.
That isn’t easy to measure either but we can work out a few general principles and they go way beyond the Ideal Gas Law.
As regards the total system energy content I don’t think that does change much at all, just as you say.That is because total system energy content is a function only of solar energy input, atmospheric pressure and the power of the bond between water molecules (which is itself affected by pressure).
If anything seeks to change that balance all one gets is a negative system response which alters the rate of energy flow through the system so as to maintain the background equilibrium.
The Ideal Gas Law is involved but only from surface upward. It has no relevance to the background rate of energy flow from ocean to air.
Since the ocean energy content (which is released at variable rates) controls the temperature of the air above, the Ideal Gas Law does not give any accurate guide as to the temperature of the system as a whole or even the actual temperature of the air above the oceans.
The temperature nominally derived from the Ideal Gas Law would be different from the actuality if the release of energy from the oceans were to vary and apparently it does due to internal oceanic oscillations.
The item missing from your calculation is the rate of energy release from the oceans.

December 29, 2011 2:09 pm

I’m not a scientist: I’m a science fiction writer. However, science fiction MUST be based on known science, or it becomes fantasy (some exceptions are allowed, such as FTL travel, etc.). This article provides much for me to ponder, but at the same time, answers a bunch of questions I’ve had in my mind about a lot of things. If Nobel Prizes were awarded for ideas, these two scientists deserve one. The only flaw I find in the article is the failure to include the discussion of gravity as a factor in gas pressure. I look forward to the continuing discussion.
This article gives us much to ponder in looking for exosolar planets that may contain life as we know it. We need a planet in the right location, one with an atmosphere and water, and one thing missing from the above equations: a moon. We can see what happens when a planet doesn’t have a moon of the proper dimension by looking at Venus. Venus could be terraformed simply by moving Ceres into a stable orbit about 185,000 miles above its surface, and adding a few thousand icy comets to supply water. Mars would be harder: you would have to add a decent-sized moon, move the smaller ones further out from the surface, and add planetary mass (both water and minerals) until the mass was sufficient to hold a decent atmosphere. Both are feasible, at least in an engineering sense.
Lots of silliness here about star formation: stars are formed when the density of the gasses in a given location reach sufficient MASS to condense enough to become hot enough to begin stellar fusion, and also great enough to counter the outward pressure such fusion would cause. Gas heat and pressure, caused by gravitational compression, explain star formation. It also accounts for the fact that Jupiter and Saturn are warmer than they would otherwise be. I’ve read somewhere, can’t remember where now, that if Jupiter were about six times larger, it would be a brown dwarf instead of a planet. I can’t vouch for the veracity of that statement, but it’s plausible.
For all its supposed flaws, I find this theory far superior to the current hypothesis of catastrophic anthropogenic global warming. I also feel this will be the final nail in the coffin of that ridiculous hypothesis.

Tim Ball
December 29, 2011 2:23 pm

I commended Nikolov and Zeller when I first read their paper. I also commend Anthony for posting it for wider dissemination and discussion.
I don’t want to detract from these efforts, however, I did draw Nikolov’s attention to the work of Hans Jelbring:
http://ruby.fgcu.edu/courses/twimberley/EnviroPhilo/FunctionOfMass.pdf
I don’t know Hans’ situation as he disappeared off the blogs a little while ago. Nikilov was going to contact him.
I also suggest people look at Jelbring’s 1998 doctoral thesis at Stockholm University titled “Wind Controlled Climate”. Wind is the most overlooked variable in climate studies yet critical in rates of evaporation, transpiration and heat transfer vertically (convection) and horizontally (advection). Consider the change in global dynamics created by a wind speed increase or decrease of 1 kph.

Ryan
December 29, 2011 2:27 pm

This analysis seems to assume the volume of the atmosphere stays constant and I’m pretty sure it doesn’t.

Bill Illis
December 29, 2011 2:51 pm

The back radiation measured at 2 metres height, originates millimetres to a few metres above the instrument.
The molecules at this level are colliding with each other 6.7 billion times per second, many times faster than the average relaxation/emission timeline for GHGs.
The backradiation is coming from all the gaseous molecules since the emission spectrum for backradiation is very close to a blackbody spectrum (at the temperature of the 2 metre air) – Not the signature of CO2 or H2O high up in the troposphere as so many people assume.
When it is not cloudy, (35% of the time), there is small loss in the atmospheric window region, but when clouds are present (65% of the time), it is a perfect blackbody spectrum.

gnomish
December 29, 2011 2:51 pm

gas laws don’t deal with phase change, either.
water gas will rise without any convection, too, until it stops being a gas. (while carrying the family joules that no other molecules can manage)

Ryan
December 29, 2011 2:53 pm

@ Wayne
Some random ideas:
First, remember that an average temperature, in the thermodynamic sense, is calculated by integrating T(t) from t1 to t2 and dividing by t2-t1. Adding the daily high and low temperature together and dividing by two will only give you the same answer on accident.
Now, as to how one would observe a trend in the second kind of average. The oceans go through hot and cold surface cycles, and big changes to humidity actually affect the molar density of surface air in a substantial way. Second, cloud cover causes vertical winds which decrease surface pressure and increase it at higher altitudes.
Let’s assume we’ve taken enough data such that ocean trends and albedo trends have smoothed out to no net effect. If we had a series of several billion identical working thermometers evenly spread accross the globe and they took constant measurements of T and computed the average with calculus then I don’t know if one could observe much of a trend in surface temperature. But if you randomly placed a much smaller number of non-identical (and to some extent non-functioning) thermometers and calculated the average temperature incorrectly then why wouldn’t one observe some random pattern?

Tilo Reber
December 29, 2011 2:53 pm

Kevin Kilty: “It isn’t the gravity induced rise of gas pressure, but rather the work done by gravity versus heat lost to radiation that increases temperature in star formation.”
Looks to me like you are simply handwaving here. What is the work that is done by gravity? It is an increase in pressure. The increase in pressure results in an increase in temperature.
Kevin: “John Wilkes Booth rightly points out that PV is not internal kinetic energy. ”
N&Z recognize this and address it. If you want to agree with Booth you need to understand that he is not speaking to anything that N&Z are not aware of and have not addressed.
Kevin: “You and I agree that the discussion and application of the gas law in this contribution is a mess.”
It looks to me like the only mess is your understanding of what they are talking about. It also looks to me like they are perfectly aware of your objection and have dealt with it. An increase in gas pressure means that the molecules are more closely packed together. When they are more closely packed together their temperature is more effected by solar radiation passing through. Simply think of a transparent cube in space that is empty and one that has gas in it. Now pass solar radiation through each cube and guess what the results will be. The cube with gas in it is able to absorb much more energy from the sun than the one with no gas in it. And it will have a higher internal temperature than the one with no gas in it by the time it reaches radiative equilibrium. Booth’s experiment of pumping up a cylinder has no relevance to the issue that K&Z are talking about.

Chesty Puller
December 29, 2011 2:59 pm

Bengt Abelsson says:
December 29, 2011 at 11:12 am
“JWBs home compressor tank is probably following the gas laws, a 15 K drop in temperature gives a 5 % drop in pressure that You won´t notice. (and a very very small drop in volume).”
PV = rNT
Sure. (V)olume is constant if we pretend the tank is made of something that doesn’t exhibit shrinkage when it’s cold. (N)umber of moles of air in the tank is also constant if we pretend it’s completely air tight. (T)emperature is changing becaue there is no internal source of heat to maintain the tank temperature above ambient air temperature. (P)ressure will change to follow externally driven temperature change.
Any questions so far?
On a planet (P)ressure is determined by composition of the air, (N)umber of moles in the vertical column, and a new term g for acceleration of (g)ravity. Note that pressure does not vary with (T)emperature in the planetary atomosphere.
Still with me?
If pressure doesn’t vary with temperature then temperature doesn’t vary with pressure.
OMG!
QED.

December 29, 2011 3:00 pm

wayne says:
December 29, 2011 at 1:40 pm

And the mean density (ρ) has not changed either since the mass of the atmosphere is basically constant as well as the surface area of the Earth.

So am I wrong, I see no way by the above calculations using the Ideal Gas Law that the Earth’s mean temperature could EVER change without something very catastrophic as a meteor hit, ejecting a sizeable amount of our atmosphere’s mass into space. I don’t see it.
If my calculations above are wrong, please tell me.

You are basically re-stating my first comment in this thread, — we assume that atmospheric mass is more or less constant over geological time. But is that is a valid assumption?
You don’t necessarily need to blow off a large amount of the atmosphere to change atmospheric mass, although that would certainly do it.
You can also remove atmospheric mass (and incidentally increase the solid earth mass) through either condensation, or freezing out a component of the atmosphere as on Mars where CO2 is pulled out of the gaseous atmosphere and frozen out in the polar caps.
You could also chemically bond and tie up portions of the atmosphere through chemical reactions and conversion to solid or liquid state. For example O2 oxidizing iron in the early earth sucked huge amounts of free oxygen out of the atmosphere. Plants converting CO2 and water to biomass (coal, oil plant residue like peat)
Or you can dissolve portions of the gaseous atmosphere in the oceans as happens with CO2.
You can also increase the atmospheric mass if the earth were to pass through a zone of interstellar space where it ingested huge quantities of dust that contained volatile compounds that would be gasified to a non-condensable gas form, during re-entry. Such as sulfur that was converted to sulfur dioxide (which eventually would be stripped out of the atmosphere by reaction to water), or ammonia and methane ices etc.
Is there a reasonable physical proxy that could be used to build an ice core record of ancient barometric pressures or some chemical marker in some biological process that changes at pressures near normal atmospheric pressure?
Larry

Mydogsgotnonose
December 29, 2011 3:00 pm

Bill Iillis: what is the view angle of the IR detector?

Tilo Reber
December 29, 2011 3:01 pm

Mike: “The only flaw I find in the article is the failure to include the discussion of gravity as a factor in gas pressure.”
Mike, you can compress gas with gravity or you can compress it in a container. How it got to be compressed makes no difference regarding how the gas reacts to solar radiation passing through it. All that matters is that it has a certain pressure, not how it got that pressure. In looking at conditions on other planets, both the amount of gas and the gravity pulling on that gas effect the pressure. But again, how it got the pressure it has doesn’t matter. It only matters what the pressure is.

richard verney
December 29, 2011 3:05 pm

wayne says:
December 29, 2011 at 7:54 am
///////////////////////
I accept that Earth’s atmosphere is not in a confined cylinder and that there can be volumerric changes. That siad, gravity is a containment field.
However, we see pressure changes on a daily basis. Low pressure areas are places where there is less mass of atmosphere above and high pressure areas are places where there is a greater mass of atmosphere above. The atmosphere is in constant flux. for example, preceding a chinook wind there are pressure changes ditto huricanes, tornados and the like.
Even where there is only a volumetric change, work is being done and an inevitable by product of this work is heat.
I do not know whether the net effect of all these changes (including the passage of the diurnal bulge) is enough to maintain (in broad terms) the temperature of the atmosphere, but certainly some heat is being put into the atmosphere as a consequence of the work done, That must be an inescapable fact but its sufficiency is entirely a different matter.

Kevin Kilty
December 29, 2011 3:11 pm

Cherry Pick says:
December 29, 2011 at 1:49 pm
This is interesting and I am still reading it. Fast comments.
Equations 7 and 8 are just curve fittings, regression equations that are said to resemble the natural gas laws which is demonstrated in figures 5 and 6. Please, rewrite that and show that observations match the theoretical calculations.
How does this theory explain Earth’s Thermosphere?

Yes, I think this is an interesting thread, and has been very useful. This is what WUWT is all about, isn’t it? Letting people comment and ask questions and argue and learn.
This theory cannot explain the thermosphere at all. The thermosphere has higher temperature than anything on earth but volcanic eruptions. The sun’s corona could find any explanation in here at all either. And I’m glad you brought this up. It helps reinforce the truth of this matter. Temperature is the result of energy balance, We need work+heat considerations to arrive at an energy balance. The gas laws provide nothing regarding energy balance. They are equations of state. This is why figures 5,6 have little to do with 7,8.

December 29, 2011 3:12 pm

Bengt Abelsson, Urederra.
Yes, but Wikipedia says that the range is 90 – 700 K.
So says NASA’s fact sheet (590-725 K, sun-ward side), but this theory says the mean temp has to be 248K at Mercury, and it is not.
Same thing happens to the Moon (253K, and not 154K). It’s easy. They have a new way to calculate the effective temperature of a planet (everything is based on that). And for planets without atmosphere, they choose the calculated temp as the “observed” one, and everything fits. That’s where the problem with the -133K comes. Looks funny.

Tilo Reber
December 29, 2011 3:14 pm

Bill: “The Earth should have been a frozen snowball until about 500 million years. ”
Bill, it seems to me that the earth, as it gathered mass and compressed, was it’s own heat source.
If I remember right, it’s crust was, at one time, molten. It has been loosing it’s own energy of formation for most of its life.

December 29, 2011 3:20 pm

Ryan says:
December 29, 2011 at 2:27 pm
This analysis seems to assume the volume of the atmosphere stays constant and I’m pretty sure it doesn’t.
============================================
There’s a winner!! And, because the volume doesn’t, neither does the mole mass. Many people here get so caught up in the P vs T thing they miss the other variables. The fact is, the P and the T don’t vary very much. At least they haven’t in the last few thousand years. We’re talking about 0.5K here. Another thing to note, the IGL doesn’t ascribe causation. It is simply an equation. For some reason, this causes people to miss the forest because of all the trees infront of it. Pressure isn’t a determination on its own, and neither is temperature. They are functions of other things such as work. But, work isn’t a thing in and of itself either.
How I would use the IGL would be as a check to see if my other formulas and thoughts were correct. I don’t believe we have to precision to know the exact volume or mole mass (or temps for that matter) of our entire atmosphere. So, we’re not going to be able to detect a change of a few hundredths of a degree C or F using the IGL. But, its a law, so other thoughts concerning our atmospheric temps must conform to the IGL.

December 29, 2011 3:31 pm

Joel Shore (December 29, 2011 at 8:53 am):
I believe you’ve misread the article. The authors do not deny the existence of the back radiation. However they assert that, at Earth’s surface, a change in the intensity of the back radiation is matched by a change of the free convective heat transfer. Consequently, changes in the intensity of the back radiation at Earth’s surface lack effects upon the transfer of heat from Earth’s surface.

Mydogsgotnonose
December 29, 2011 3:34 pm

Bill illis: I have thought on your experimental observations. There is now no doubt in my mind that the Trenberthian view of back radiation is plain wrong. No climate model can predict climate. CO2-AGW has been vastly over-estimated. basically, The modelling work has to be restarted with proper physics.

Chesty Puller
December 29, 2011 3:45 pm

Tilo Reber says:
December 29, 2011 at 2:53 pm
Kevin Kilty: “It isn’t the gravity induced rise of gas pressure, but rather the work done by gravity versus heat lost to radiation that increases temperature in star formation.”
Looks to me like you are simply handwaving here. What is the work that is done by gravity? It is an increase in pressure. The increase in pressure results in an increase in temperature.
——————————————————————————————
Not really. The increase in temperature happens because the same amount of gas was forced into a smaller volume against its will. That takes work to accomplish just like it takes work to force a cat into a pet carrier against its will – both you and the cat will get warmer as the work is performed. Whether it’s a compressor or gravity or cat stuffing it is this work being accomplished which heats up the [ahem] working fluid. When the work is finished so is the rise in temperature and the temperature of the gas is then determined by something else. That something else in a star is nuclear fusion. If it fails to reach fusion temperature/pressure it will get colder and colder once gravity and gas pressure reaches equilibrium and volume stops shrinking – unless something external changes like more mass falling into it.

Robert of Ottawa
December 29, 2011 3:46 pm

This explication goes in the direction I have been thinking, but I don’t have the tools or time to pursue. It just makes sense. In 100 years, we may be able to add a few extra-solar planets to that graph.

Kevin Kilty
December 29, 2011 3:49 pm

Stephen Richards says:
December 29, 2011 at 1:35 pm
Kevin Kilty says:
December 29, 2011 at 12:45 pm
George Turner says:
December 29, 2011 at 12:26 pm
Kevin, are you saying that high altitude areas are colder than low altitude areas because of differences in radiative balance, not the difference in pressure?
This is a cause and effect problem. Kevin, I had to read your comment several times to get the gist….

Let me respond to a few of your comments.
The feeling of cold at altitude is due to the lack of pressure. The feeling of cold is because the temperature is low, but also human beings have skin sensors for heat flow, and so they tend to feel cold when heat transfer out of the body is high. Heat transfer from the human body is largely through radiation, convection, and evaporation. At high elevation radiation and evaporation transfer out of the body are rapid.
The reason there is a lack of pressure is because there is a lack of gas. The reason for low pressure is that there is less and less gas above us for the atmosphere to support the higher we go.
The reason there is a lack of gas is down to gravity. OK.
Work is done when the state of a mass is changed from steady to accelerating. Usually we don’t see high acceleration except in cases like tornadoes, etc. More commonly work is done to raise or lower a mass of air.
In the lower atmosphere where an anticyclone (high pressure) is present for a long period what is the profile of temperature through the lower atmosphere and where a low pressure is present how does that change the temperature profile.? This sounds like a complicated topic that would take us far afield probably, but isn’t it interesting that in mid-latitude cyclones, high surface pressure correlates to cold surface temperatures. Not always mind you, because sometimes descending air in a high pressure system can lead to significant warming of the Chinook form. It is very difficult to come up with general answers isn’t it?

Robert of Ottawa
December 29, 2011 3:50 pm

Jame sSexton, I didn’t see anything here about volume in the equations. Yes, the volume of the atmosphere does appear to change, it appears to match the activity of the Sun. But the mass doesn’t change a lot. The volume change would appear to alter the pressure gradient above the troposphere.

Phil's Dad
December 29, 2011 3:55 pm

Will says:
December 29, 2011 at 1:00 pm
Sssh, no shouting (post Christmas hangover).
As I have said, I agree with your assertion regarding -18C and indeed that the atmosphere free Earth would exhibit its characteristic-emission temperature of 254.6K, 33.4 K cooler than today, at its characteristic-emission surface (which would of course change, as it is itself atmosphere dependant). That is still entirely consistent with the atmosphere producing 133k of warming if other factors (such as the water cycle) are producing a compensating cooling effect. You need to factor out everything else before ariving at the contribution of the atmosphere by itself.

Levi
December 29, 2011 3:59 pm

I just have one comment for now before I do some more digesting. I am having a hard time believing that all other scientists currently working on the theory of AGW simply forgot to consider adiabatic heating due to atmospheric pressure. It doesn’t seem possible that such an obvious thing could be ignored. I kind of assumed that this effect had already been considered. I’d be interested to hear some folks from the IPCC respond to the ideas proposed here.

Joel Shore
December 29, 2011 4:01 pm

Dave Wendt says:

The daily range is routinely over 100 degrees and at maximum approaches 150 degrees, that doesn’t strike me as all that moderate relative to an average of 288.

If you think it is significant, then assume some reasonable temperature range over the Earth and do the calculation. You will find that the difference between averaging the temperature and taking the 4th root of the average of T^4 is pretty small (a few degrees at most) for any reasonable approximation to earth-like temperature ranges.
Terry Oldberg says:

I believe you’ve misread the article. The authors do not deny the existence of the back radiation. However they assert that, at Earth’s surface, a change in the intensity of the back radiation is matched by a change of the free convective heat transfer. Consequently, changes in the intensity of the back radiation at Earth’s surface lack effects upon the transfer of heat from Earth’s surface.

I have not misread it at all and I have explained exactly what they get wrong: Convection relaxes the temperature profile only back to the adiabatic lapse rate, not to a temperature profile that is independent of altitude as they assume.

Chesty Puller
December 29, 2011 4:02 pm

Terry Oldberg says:
December 29, 2011 at 3:31 pm
“I believe you’ve misread the article. The authors do not deny the existence of the back radiation. However they assert that, at Earth’s surface, a change in the intensity of the back radiation is matched by a change of the free convective heat transfer. Consequently, changes in the intensity of the back radiation at Earth’s surface lack effects upon the transfer of heat from Earth’s surface.”
The authors fail to realize that radiative energy leaving the earth’s surface is travelling at approximately 186,000 miles per second or for you European cats that’s about 300 million meters per second. Convective updrafts travel even in the most severe convective cells fail to reach 100 meters per second.
Atmospheric greenhouse warming is a result of impedance in the free radiative path from surface to space. Greenhouse gas molecules intercept the radiation and essentially reflect about half of it back at the source. The authors are correct that this may result in greater rate of convection but that is acheived through a greater surface temperature which condutively heats the air to a greater degree making it that much lighter which in turn makes it rise faster. At the same time the higher surface temperature also increases the motive force of radiative transfer and more of it gets squeezed through the impedance of the greenhouse gases just like when you increase the pressure of water more of it will make through a pipe whose diameter remained constant.
This isn’t rocket science but it appears to be more advanced than forestry science.

wayne
December 29, 2011 4:03 pm

Well, what do you know? It is calculable. Using the alternate Ideal Gas Law equations I just gave above, they give these results for the three major planets with known atmospheres. Now who says that law does not apply to averaged planets. Well for me, it’s better than current “climate science”.
Used: T = P/ ρ • M/R

                    Venus      Earth       Mars
                 --------   --------   --------
P - pressure      9220000     101325        605  N/m2 (Pa)
ρ - density            65      1.217      0.015  kg/m3
M - molar mass     0.0434    0.02897    0.04334  kg/mol
R - gas constant  8.31451    8.31451    8.31451  J/K/mol
                 --------   --------   --------
T - temperature    740.40     290.09     210.24  K

Many of you will recognize these numbers tossed about here over the last few years. If not, see the data in the post or the NASA planet data sheets.

Kevin Kilty
December 29, 2011 4:08 pm

mkelly says:
December 29, 2011 at 1:19 pm
Kevin Kilty says:
December 29, 2011 at 12:33 pm
Sir, I don’t disagree with what you say. As I said last time I agree. I was being more narrowly focused as to what the equation says. However, work is seldom mentioned (which is a shame) in discussions of climate.

Thanks, Mkelly. How true about work!

December 29, 2011 4:08 pm

This theory is very similar to what Prof. Claes Johnson has said in his “Climate Thermodynamics” http://www.nada.kth.se/~cgjoh/atmothermo.pdf
The lapse rate in the atmosphere is caused by pressure which, as I have been saying for months, affects the rate at which warm air rises and thermal energy converts to potential energy. The lapse rate has absolutely nothing to do with any radiative transfer theory, back radiation or whatever.
To the above we can add the fact that Claes has also proven that any back radiation (or even solar radiation in the lowest IR frequencies) cannot warm the surface if its frequency is below kT/h as explained in my review of Claes Johnson’s “Computational Blackbody Radiation.” (See http://www.climate-change-theory.com/RadiationAbsorption.html )
We can further add the fact that Prof Nahle has published an experiment which confirms that back radiation does no warming http://principia-scientific.org/publications/New_Concise_Experiment_on_Backradiation.pdf
We can also add the fact that temperature records for the Arctic island of Jan Mayer show warmer temperatures in the 1930’s and so no warming whatsoever for over 80 years. (And the Arctic is “meant” to rise up to 8 degrees by 2100.) See my home page: http://climate-change-theory.com
Finally it would appear that back radiation “measurements” are very spurious. Infra-red thermometers measure frequency and convert it to temperature using Wien’s Displacement Law. But to then apply SBL to convert this temperature to radiative flux from a colder layer of atmosphere to a warmer layer is very spurious. Normally, when applying SBL in situations where the surrounds are not at absolute zero, you need to subtract the effective radiation in the opposite direction. The SBL cannot give a positive result for any situation involving a cooler body supposedly radiating to a warmer one. Above all, even if we consider the wave motion of radiation to be two-way, the transfer of thermal energy can only be from warmer to cooler. The temperature information is transferred in the cut-off frequency, just as is used in infra-red cameras and thermometers. Radiation below cut-off will have no effect on the surface. (For more detail see the footnote on my page http://www.climate-change-theory.com/RadiationAbsorption.html )
We are now in the post-greenhouse era, everyone. But, as with many medical discoveries like penicillin, we can expect it to take 20 years before there is general acceptance. But level or slightly declining temperatures should continue until 2028, so that will help. We who think are at the forefront of this breakthrough. Unfortunately, however, you can lead a horse to water but you can’t make him think.

December 29, 2011 4:10 pm

James Sexton says: December 29, 2011 at 8:17 am

…WUWT has come a long way. Pamela Gray is right, (as usual), the application of the Ideal Gas Law has been brought up here a few times in the past. The reception to it then wasn’t as nearly pleasant.

A profound reflection – that a hypothesis which has been around for a while, suddenly finds acceptability. And WUWT has earned the right to be the platform for such a tipping-point.
This has been brewing. Tim Channon at Tallbloke’s Talkshop already posted this paper. A couple days earlier, Tim posted his own observations on the extraordinary heat of Jericho which lies some 200+ m. below sea level. Tim’s post shows the straight-line link between temperature and altitude for both Venus and Jupiter.
(This post led me to observe three things so obvious I suspect we simply forget them: the amazingly horizontal line of snow on the hills; the amazingly horizontal lines of most cloud undersides; and the heat of deep underground mines, in contrast to deep undersea temperatures. All direct effects of pressure in the compressible medium of air.)
Huffman is vindicated, as I’ve long felt he deserved. Probably trickles of people have been visiting his pages and thought about his stunning table of correlation of temperatures of Venus and Earth based solely on pressure and distance from the Sun. Plus recent remarks from the inscrutable Paul Vaughan have firmly placed limits on the indomitable Leif.
There are references here and at Talkshop to Levenspiel’s work that is also a brilliant paradigm-shifter and goes hand-in-hand with Nikolov & Zeller: higher past atmospheric pressures were necessary for pteranodon flight and for brontosaurus to raise their heads. Levenspiel, of course, has had multiple refusals of publication.
Innocent question (?) Atmospheric temperature profiles show a minimum (at the mesopause) of some -100°C. How then can anyone claim the Earth’s surface temperature without gases would be a mere 33°C less than 15°C?

Robert of Ottawa
December 29, 2011 4:10 pm

The Temp of the atmosphere is essentially determined by the solar energy input, albedo and the ratio of mass of atmosphere to planet.

Joel Shore
December 29, 2011 4:11 pm

Levi: Indeed, you are correct that any elementary book on climate science will talk about the basic thermodynamics of the atmosphere, such as the ideal gas law, the condition for hydrostatic equilibrium, and the adiabatic lapse rate. The difference between such books and this post is they correctly put it in context rather than using it to make nonsensical conclusions as Nikolov and Zeller do.

Chesty Puller
December 29, 2011 4:14 pm

Kevin Kilty says:
December 29, 2011 at 3:49 pm
“Heat transfer from the human body is largely through radiation, convection, and evaporation.”
Not in water. It’s all conductive then and its far more rapid than in air. This is why jumping into water at 60F feels a lot colder than stepping into air at 60F. A LOT colder. You can survive indefinitely in air at 60F with no more than goosebumps but water at 60F will kill you in about 6 hours if you’ve got a life jacket and about 2 hours without one because you’ll become unconscious after 2 hours and drown.
1

Phil's Dad
December 29, 2011 4:15 pm

Mrss Puller and Kilty,
An appeal to your area of expertise.
Since gravity is a force of acceleration is not its “work” a constant.

Robert of Ottawa
December 29, 2011 4:20 pm

Levi, when I first heard of this CO2 warming theory, back in the 1980s, I considered it; I was clever enough to understand the concept, but not knowledgeable enough to have counter-arguments at hand. I did not have a horse in the race, so could afford to change my opinion, which I have.
Many “clever” people also do not understand, but have committed themselves, and their careers, the AGW theory. They do not have the luxury to change opinion.

Joel Shore
December 29, 2011 4:21 pm

The lapse rate in the atmosphere is caused by pressure which, as I have been saying for months, affects the rate at which warm air rises and thermal energy converts to potential energy. The lapse rate has absolutely nothing to do with any radiative transfer theory, back radiation or whatever.

Well…almost true. The adiabatic lapse rate has nothing to do with what you have mentioned. However, the adiabatic lapse rate is only a stability limit on the actual lapse rate. In an atmosphere heated strongly from below and cooled from above (which is true because of most solar radiation not being absorbed until the surface and also because of the radiative transfer due to greenhouse gases in the atmosphere), the lapse rate will indeed tend to be pegged at the adiabatic lapse rate.
However, the lapse rate alone doesn’t determine the surface temperature, just like the slope of a line doesn’t determine the value of y for any x. You need the temperature at at least one point. In this case, that temperature is the effective blackbody temperature of 255 K and its location in the atmosphere is determined by atmospheric opacity to radiation, i.e., the greenhouse elements of the atmosphere.

To the above we can add the fact that Claes has also proven that any back radiation (or even solar radiation in the lowest IR frequencies) cannot warm the surface if its frequency is below kT/h as explained in my review of Claes Johnson’s “Computational Blackbody Radiation.” (See http://www.climate-change-theory.com/RadiationAbsorption.html )

No he has not. Claes has replaced a century of physics, involving the entire field of statistical physics, by a hokey axiom that basically asserts the result that he wants. He has given no reason to abandon a century of physics in favor of his ideas. Furthermore, all he has succeeded in doing is re-deriving an equation that still has the greenhouse effect in it. You don’t need so-called “back-radiation” (for which there is abundant experimental evidence) to have the greenhouse effect. You just need the fact that the rate at which a warmer object cools depends on the temperature of a colder object surrounding it as well as its own temperature.

Chesty Puller
December 29, 2011 4:23 pm

“Innocent question (?) Atmospheric temperature profiles show a minimum (at the mesopause) of some -100°C. How then can anyone claim the Earth’s surface temperature without gases would be a mere 33°C less than 15°C?”
Because the sun doesn’t heat the air in the mesopause. It heats the surface and the surface heats the air. The farther you get from the source of heat the colder it gets whether it’s a pot of coffee farther from a campfire or a volume of air farther from the earth’s surface. Higher than the mesosphere the atmosphere starts absorbing some of the sun’s radiation and temperatures in the thermosphere rise to thousands of degrees. The air temperature the space station is flying through is thousands of degrees. If it weren’t for the fact that it’s almost a vacuum at that point and thus have almost no heat capacity someone could get a nasty burn from it!
Huffman is not vindicated by the way. He’s just some extra cranks to keep him company. I have no idea why Anthony posts stuff like this here because it just makes his blog look like a crank sanctuary.

December 29, 2011 4:24 pm

Levi says: December 29, 2011 at 3:59 pm

I am having a hard time believing that all other scientists currently working on the theory of AGW simply forgot to consider adiabatic heating due to atmospheric pressure. It doesn’t seem possible that such an obvious thing could be ignored. I kind of assumed that this effect had already been considered. I’d be interested to hear some folks from the IPCC respond to the ideas proposed here.

Welcome to the real world of Climate Science Levi. There is a lot more stupidity, entrenched stupidity, jeering stupidity, and financially-rewarded stupidity than one would dream possible. IPCC was, from inception, on a mission to assert AGW, subverting true science. Most of us here were once starry-eyed believers in science as practised. And hopefully, here at least, still hold fast this ideal, while not blind to the realities in practice.

Robert of Ottawa
December 29, 2011 4:25 pm

Chesty Puller, you have simply thrown dust into the air. The speed of energy transfer – SOL versus convection – has no bearing on steady state. I call Straw Man.

December 29, 2011 4:29 pm

Tim Ball “Wind is the most overlooked variable in climate studies”.
Agreed. Especially so at high latitudes. You demonstrate that point in your work in relation to Canada that I found in “The Year Without a Summer”.
So, I am prompted to put forward a paper that focuses on how and why the wind (and cloud cover) changes in line with geomagnetic activity. See: http://climatechange1.wordpress.com/
And I will be most interested to see further comment from Nikolov and Zeller that expands on their observations on the relationship between cloud cover and temperature and the associated statement from their post, namely:
“However, independent small changes in albedo are possible and do occur owning to 1%-3% secular variations in cloud cover, which are most likely driven by solar magnetic activity. These cloud-cover changes cause ±0.7C semi-periodic fluctuations in global temperature on a decadal to centennial time scale as indicated by recent satellite observations (see Fig. 7) and climate reconstructions for the past 10,000 years.”
But most of all I want to see comment from those who have long maintained that the posited greenhouse effect is real, but exggerated. Come in Monkton, Svalgaard, Eschenback and Spencer. Your support for the notion of a greenhouse effect due to radiative influences needs to be re-assessed.

December 29, 2011 4:29 pm

“Chesty Puller” says:
“Huffman is not vindicated by the way. He’s just some extra cranks to keep him company.”
He makes a good case re: Earth vs Venus. You, OTOH, are apparently a wannabe war hero, and I doubt the real Chesty Puller would condone using his name.

Tom in Florida
December 29, 2011 4:39 pm

Just a peanut gallery question. How does this all tie in with changes in orbital parameters that have caused the fairly regular periods of glaciation and interglacials over the last 3 million years or so?

Kevin Kilty
December 29, 2011 4:42 pm

Phil’s Dad says:
December 29, 2011 at 4:15 pm
Mrss Puller and Kilty,
An appeal to your area of expertise.
Since gravity is a force of acceleration is not its “work” a constant.

I’ll let Puller add whatever he wants, but gravity is a force that we measure through its acceleration on a standard mass. It is more or less constant the world over, yes. And the work done is then related to how much vertical distance one moves a mass and the size of the mass. One kilogram, moved a distance of 1 meter, is about 10J of work. Work=mgh. It is not a constant.

Kevin Kilty
December 29, 2011 4:49 pm

wayne says:
December 29, 2011 at 4:03 pm
Well, what do you know? It is calculable. Using the alternate Ideal Gas Law equations I just gave above, they give these results for the three major planets with known atmospheres. …

OK, then do the following. There is pressure at altitudes of, say, 100km or 500km above the earth. Please use your modified equation to calculate the temperature up there.

December 29, 2011 5:02 pm

Thermodynamics applies only to systems in equilibrium. No place on earth is ever in equilibrium. You cannot use averages and you do not know them anyway. The Greenhouse theory ignores the outside climate which is mainly controlled by convection and evaporation/condensation, but is currently only partly understoood. It also igniores the crucial difference between day and night.
It is sad that intelligent people can be fooled by the absurd greenhouse model.

Theo Goodwin
December 29, 2011 5:06 pm

Levi says: December 29, 2011 at 3:59 pm
“I am having a hard time believing that all other scientists currently working on the theory of AGW simply forgot to consider adiabatic heating due to atmospheric pressure. It doesn’t seem possible that such an obvious thing could be ignored.”
Apparently, you are not aware that mainstream climate science recognizes no natural processes apart from radiation transfer in the Earth-Sun system. If recognized at all, adiabatic heating due to atmospheric pressure would be treated as an epiphenomenon of radiation transfer.

December 29, 2011 5:12 pm

A simple summation:
i) AGW theory states that the greenhouse effect is caused by gases in the air with a high thermal capacity warming the surface by radiating energy downwards.
ii) This paper describes the greenhouse effect in the way I have always understood it i.e. ALL the molecules near the surface (of whatever thermal capacity) jostle more tightly together under the influence of gravity and share kinetic activity (provoked initially by solar irradiation but actually being a consequence of all energy transfer mechanisms combined) amongst one another until that kinetic energy can escape to space by radiative means albeit slightly delayed by all the jostling about.The delay results in a temperature rise.
The beauty of ii) is that it decouples the greenhouse effect from the matter of composition leaving atmospheric density as the controlling factor at any given level of solar irradiation. It is the matter of composition that so distresses AGW proponents but in fact it is irrelevant. ALL molecules at or near the surface are involved whether they be GHGs or not.
There has been some confusion caused by Harry Huffman, Claus Johnson and others by virtue of their contention that there is no greenhouse effect when actually they mean that i) above is untrue whilst they accept ii) to be true (I think).
So there is a greenhouse effect but it is not significantly affected by GHGs. In so far as GHGs do have an effect it is negated by faster removal of energy to space by various means (especially evaporation on a water planet) because pressure places a limit on the amount of kinetic energy that can be retained by gases at the surface.
Or have I missed something?

Chesty Puller
December 29, 2011 5:17 pm

File Under Correlation Is Not Causation:
A legitimate question is posed:
“Why is the atmosphere of Venus approximately the same temperature as the earth at the same pressure level when Venus is so much closer to the sun?”
The correct answer is:
The albedo of Venus is 0.70 while the albedo of the earth is 0.30. The lesser distance from the sun is precisely negated by the higher albedo.
Any questions?

December 29, 2011 5:18 pm

Further to my comment above regarding the spurious nature of back radiation, you would think that someone among all the AGW proponents would have set up a simple experiment such as this to measure the assumed warming effect of back radiation and, at the same time, proving it exists …
Obtain two identical metal plates painted soot black with accurate thermometers attached. Place them on similar ground but at a reasonable distance from each other (a) at night and (b) in the open shade of a large building in daylight hours. Shield one with a larger reflective plate or upward facing mirror and observe if the back radiation received by the other leads to any difference in temperature. Subsequently shield the other and repeat the experiment in case there are unknown variables.
Yet another experiment: Paint one of the plates grey and this time place then vertically and parallel to each other and reasonably close in open air. Heat the black one to, say, 100 deg.C and time how long it takes to return to say 80 deg.C. Now warm the grey one to 80 deg.C and retain that temperature with a thermostatic device. Again measure how long the black one takes to cool from 100 deg. to 80 deg.C in order to determine if it is absorbing any thermal energy from the cooler grey one, such absorption slowing the cooling process.
Anyone taking bets?

Theo Goodwin
December 29, 2011 5:19 pm

http://climate-change-theory.com says:
December 29, 2011 at 4:08 pm
I thank you for your contribution to this discussion. The kind of post that we are discussing should involve some perspectives that are not part of mainstream climate science. Unfortunately, there are those whose comments on this post could be the work of mainstream climate scientists who are doing nothing but enforcing the tautology that “radiation only” theory has become. This blog should be a place for open minds rather than a place where one view of “the truth” is rigorously enforced.

jorgekafkazar
December 29, 2011 5:21 pm

I can always tell when a post here has the Warmistas shaking in their boots: all the trolls and sock puppets show up in high dudgeon. Now we even have JohnWilkesBooth, a sock puppet’s sock puppet. This paper has given the Warmista cause a serious kick in the “Jules.” Or joules.
Yes, Stephen Wilde, I remember well your post along very similar lines from some time back. NIce.
G. Karst: have no fear. the media may ignore it, but most scientists will not, including those who know little about climate. They will be intrigued to see this methodology and will explore it for themselves. Remember, Climatology, as practiced, depends on the fact that there are very few “in the know.” The more who know, now, the greater the Team’s shame.
Lucy: Good answer. AGW was never about science; it’s about the greatest power-grab in the history of mankind. And they almost got away with it.

Phil's Dad
December 29, 2011 5:25 pm

Kevin Kilty:
December 29, 2011 at 4:42 pm
Thank you. I suppose I should have rephrased the term “is not its work a constant?” as “is it not constantly doing work?”
You say; “…the work done is then related to how much vertical distance one moves a mass and the size of the mass.”
As I interpret your explanation gravity will produce a given gas density through moving atmospheric molecules vertically (well towards the centre of Earth’s mass anyway) to occupy a smaller volume than otherwise would be the case.
However in order to maintain surface atmospheric density through the pressure resulting from the work done on the mass of the atmosphere above; it seems to me that it must constantly do work to counter the tendency of those same molecules to push each other away – even though there is no apparent vertical movement. In other words work is being done even though the pressure / density is not changing.
What am I missing?
PS I apologise for taking you away from the kids at tea time. They are far more important than this discussion.

wayne
December 29, 2011 5:28 pm

Kevin Kilty says:
December 29, 2011 at 4:49 pm
wayne says:
December 29, 2011 at 4:03 pm
Well, what do you know? It is calculable. Using the alternate Ideal Gas Law equations I just gave above, they give these results for the three major planets with known atmospheres. …
OK, then do the following. There is pressure at altitudes of, say, 100km or 500km above the earth. Please use your modified equation to calculate the temperature up there.
>>>
Well I don’t think the temperature outside the International Space Station is relevant here. Please, no more herring, I’m full. Besides, I’m celebrating an apparent end of my two year+ trek through the dark “CLIMATE SCIENCE” forest.

kwik
December 29, 2011 5:31 pm

Lucy Skywalker says:
December 29, 2011 at 4:10 pm
“Huffman is vindicated, as I’ve long felt he deserved. Probably trickles of people have been visiting his pages and thought about his stunning table of correlation of temperatures of Venus and Earth based solely on pressure and distance from the Sun.”
I have been there myself several times. And never found a logical fallacy about his Venus argument. I am very interested in the warmist’s response to it.

David A. Evans
December 29, 2011 5:37 pm

I often get shouted down on this one so I’ll approach it from a slightly different angle.
Clue 1. A real greenhouse gets warm by constraining convection!
Clue 2 when confronted with that, “but it cools quickly at night due to radiation!” No, that’s due to conduction through the, (usually 2mm glass,) and thence convection.
Let’s put another form of greenhouse up for examination, the conservatory. Double, triple, or in extreme climes, even quadruple glazing works just fine! (There seems to be a diminishing returns barrier at quadruple.)
Losses now become minimal! In fact, losses via the frame become more critical, which is why modern frames have thermal breaks. This implies surface radiative losses are minor!
DaveE.

December 29, 2011 5:38 pm

Nick Stokes,
isn’t ready to admit that this isn’t a redo of adiabatic heating any more than a number of the posters.
If you don’t understand why this is NOT saying that adiabatic heating sets the temperature, please ask questions of the poster or Stephen Wilde!!

December 29, 2011 5:39 pm

Joel Shore says:
December 29, 2011 at 9:33 am
“This paper will be ignored because it is pseudoscientific nonsense that would never pass peer review in the scientific community. Its only purpose is to fool those who do not have the scientific background to recognize its glaring errors.”
Such an open mind, It takes my breath away :).
J.

Chesty Puller
December 29, 2011 5:39 pm

Theo Goodwin says:
December 29, 2011 at 5:06 pm
“Apparently, you are not aware that mainstream climate science recognizes no natural processes apart from radiation transfer in the Earth-Sun system.”
That would probably be because there is no other process apart from radiation transfer in the Earth-Sun system. Conduction and convection don’t work across a vacuum. All energy entering and leaving the earth does so via radiation.
We have very accurate figures for how much energy enters the system at the top of the atmosphere. The problem is we don’t have accurate figures for how much energy leaves at the top of the atmosphere or its frequency distribution. Approximately 35% is reflected and thus does not raise surface temperature. The keyword here is “approximate”. It’s plus or minus a few percent. A one percent inaccuracy is equivalent to all the modeled anthropogenic forcings combined. The models cannot be tested when the means of measurement have grossly insufficient precision. That’s one source of imprecision. Another source of error is that we cannot accurately measure how much thermal radiation comes from the surface where we experience it and how much comes from the cloud deck where we don’t experience it. So we can’t tell apart evaporative cooling from radiative cooling without an accurate measure of temperature gradient from surface to mid-troposphere. The only way the models can be tested is by prediction of the future and comparison of the future when it arrives. To the climate boffin’s great distress but as of yet little chagrin the predictions have failed most spectacularly in the past decade where atmospheric CO2 has been increasing at a record pace but global average temperature has not budged. In any legitimate scientific endeavour this constitutes falsification of the models but climate science turned into climate dogma decades ago.

kwik
December 29, 2011 5:47 pm

Stephen Wilde says:
December 29, 2011 at 5:12 pm
“There has been some confusion caused by Harry Huffman, Claus Johnson and others by virtue of their contention that there is no greenhouse effect when actually they mean that i) above is untrue whilst they accept ii) to be true (I think).”
Stephen, it is Claes Johnson;
http://www.csc.kth.se/~cgjoh/

jae
December 29, 2011 5:48 pm

HA! I’ve been supporting this general concept for about 4 years, now. However, all I could offer is some empirical evidence and common sense. Congrats to these guys for finally putting all the physics together in a nice package that should make all the know-it-alls (including most “skeptics” reconsider their positions ! And may the warmistas have some great nightmares over this one, LOL!

R. Gates
December 29, 2011 5:49 pm

Very interesting. Some quick comments:
1) It seems quite a stretch to suggest that the Earth is 133K warmer than it would be without a greenhouse atmosphere. Even a simply comparison to the moon, without any atmosphere at all, but similar solar heating and allowing for the slight differences in albedo and internal thermal energy, shows that the “133K warmer” figure seems wildly inaccurate. The moon’s surface temperature averages about -23C, which is of course slightly colder than the -18C the earth would be without greenhouse warming, and this makes sense when accounting for the factors allowed for above, but your assumption that the earth would average about -118C (or -133C lower than the current average) without an atmosphere simply doesn’t seem to compute.
2) Also, the “extra” energy in the atmosphere (more than supplied by solar alone) comes from the earth’s own terrestial LW, which must be added to the total energy budget. When considering this LW, earth’s energy budget balances (almost), without need for the energy from atmospheric pressure.
3) Finally, the diurnal atmospheric temperature changes are quite within all theoretical limits to a high degree of accuracy by measuring standard downwelling solar SW, downwelling LW, and upwelling LW, when considering standard greenhouse theory (i.e. the forcing caused by clouds at night for example) without need for introducing changes in atmospheric pressures – which may or may not follow diurnal patterns as SW and LW radiation patterns do.
Again, an “interesting” theory, but it doesn’t stand up to basic physics.

Chesty Puller
December 29, 2011 5:52 pm

Phil’s Dad says:
December 29, 2011 at 5:25 pm
“However in order to maintain surface atmospheric density through the pressure resulting from the work done on the mass of the atmosphere above; it seems to me that it must constantly do work to counter the tendency of those same molecules to push each other away – even though there is no apparent vertical movement. In other words work is being done even though the pressure / density is not changing. What am I missing?”
You’re missing a way to convert this constant work into a perpetual motion machine. Once gravitationally compressed it takes work to uncompress it. You have to raise the mass of air against the force of gravity to uncompress it or put it in a bottle and expend energy with a vacuum pump. Extracting useful energy to perform work from gravitational potential energy only comes from dropping deeper into the gravity well. It requires the expenditure of energy to raise it out of the gravity well. An sterling example of this is hydroelectric power. Solar energy is absorbed by water which then rises until it condenses. Upon condensation is still has gravitational potential energy. If it evaporates at sea level and ends up as water at a higher elevation then we can get some work out of it as it flows downhill deeper into the gravity well. There’s no free lunch to be had here.

iya
December 29, 2011 5:53 pm

The truth might be somewhere in the middle:
The mass of the atmosphere determines the effective radiation height.
The lapse rate determines the surface temperature.
Between TOA and surface the necessary energy transport via conduction, convection and radiation is established.
This is the only way to explain Venus huge surface temperature. “Closing of the window” will not do it because the window is already very small on earth.
We need a theory to explain all temperature profiles in our solar system, and this gets closer than ever before.

kwik
December 29, 2011 5:54 pm

I forgot to mention; Steven, you might want to click on the “Climate Thermodynamics” link.
That will bring you here;
http://www.csc.kth.se/~cgjoh/climatethermoslayer.pdf

ChE
December 29, 2011 5:54 pm

Lucy: Good answer. AGW was never about science; it’s about the greatest power-grab in the history of mankind. And they almost got away with it.

…if it wasn’t for those meddling Canadians…

December 29, 2011 6:01 pm

Stephen Wilde says: “Have I missed something?”
Yes, Stephen, you have missed something. Radiation with frequency below the cut of frequency for the absolute temperature, T of the surface (such cut off frequency being determined by Wien’s Displacement Law and being equal to kT/h where k=Boltzmann’s constant, h=Planck’s constant) cannot be converted to (additional) thermal energy in that surface.
In other words, even if it exists, back radiation cannot warm the surface.
That is what Prof Claes Johnson has proved with a detailed mathematical derivation, and what Prof Nahle has proved empirically. See links here http://www.climate-change-theory.com/RadiationAbsorption.html
This is the real breakthrough this year which we all need to heed.

wayne
December 29, 2011 6:02 pm

Chesty Puller says:
December 29, 2011 at 5:17 pm
The albedo of Venus is 0.70 while the albedo of the earth is 0.30. The lesser distance from the sun is precisely negated by the higher albedo.
>>>
No, you are confused and mixing visiual geometric albedo with Bond albedo.
Earth’s Bond albedo is ~0.306
Venus’s Bond albedo is ~0.90
(by NASA data anyway)

u.k.(us)
December 29, 2011 6:02 pm

She, in the short term, will make any attempt to understand her moot, just out of spite.
In the long term, She has forces that can be called upon to destroy any theory.

palindrom
December 29, 2011 6:05 pm

The pressure at the bottom of the atmosphere is trivially equal to the weight per unit area of the column of air above it. The temperature depends on entirely different factors, in particular how difficult it is for heat to be transferred upward through the layers, and ultimately radiated away into space. The equilibrium of incoming and outgoing radiation is the key to understanding the problem, and the authors really have nothing interesting to say about this.
I understand quite a bit about atmospheric physics and thermodynamics, and for the life of me I can’t figure out exactly what the authors are trying to say. The “thermal effect of pressure”, for example, is a completely meaningless phrase to anyone who understands statistical mechanics. The paper is also weirdly written, by the common standards of presentation in the mathematical sciences. If I were refereeing it, I would surely reject it for lack of clarity.
Sorry, folks, this is going to go down in history as another Gerlich and Tscheuschner – my prediction is that it will be so radioactive that no one will cite it within a month or so.

Jimmy Haigh
December 29, 2011 6:06 pm

Albert Einstein says:
December 29, 2011 at 10:13 am
Why should anyone take any account of what an assistant examiner in a patent office says?…

Bernie
December 29, 2011 6:10 pm

I believe the poster is on the right track but they mightly confused the situation because they have rediscovered Lapse Rate and given it a new name. I would expect that most atmospheric scientists already take the lapse rate into account when calculating atmospheric temperature profiles.
I want to clarify the issue of work. As a parcel of air moves upwards it does not do work against gravity because the mass of air will be exactly counter-balanced by the same mass moving downwards nearby. Gravity combined with the gas properties determines the average pressure gradient. As the air rises it undergoes isentropic expansion because it is doing work against the rest of the atmosphere. The isentropic expansion, as opposed to adiabatic expansion, means the the air temperature falls significantly. It is this expansion related cooling that determines the average dry lapse rate for the atmosphere. The opposite effect occurs when the air decends and warms as it has work done it by the surrounding atmosphere. These effects would not occur if the atmosphere was not compressible as with water.
It has also been stated by others that if the atmospheric gases didnt not contain any IR absorbing/emitting compenents then atmocpheric convection would not occur because it is driven by the transfer of heat from near the earth’s surface to the TOA where the heat can only be radiated to space. If the air could not cool down and decend the convection would cease.

David L. Hagen
December 29, 2011 6:15 pm

Compliments Ned and Karl on correcting the foundational integrals.
This thermodynamic theory can be complimented by incorporating:
1) A thermodynamically rigorous lapse rate that incorporates the absorptive/radiative properties of the greenhouse gases. See: Robert H. Essenhigh, Energy & Fuels 2006, 20, 1057-1067, “Prediction of the Standard Atmosphere Profiles of Temperature, Pressure, and Density with Height for the Lower Atmosphere by Solution of the (S-S) Integral Equations of Transfer and Evaluation of the Potential for Profile Perturbation by Combustion Emissions”

The solution predicts, . . . a linear decline of the fourth power of the temperature, T4, with pressure, P, and, at a first approximation, a linear decline of T with altitude, h, up to the tropopause at about 10 km (the lower atmosphere). . . .the variations of pressure, P, and density, ρ, with altitude, h, are also then obtained, with the predictions . . .up to 30 km altitude (1% density). . . .

and
Sreekanth Kolan, Study of energy balance between lower and upper atmosphere
2) A quantitative Line By Line radiative model to evaluate atmospheric absorptivity/emissivity as a function of elevation, temperature and composition. e.g. Ferenc Miskolczi’s HARTCODE.
The stable stationary value of the earth’s global average atmospheric Planck-weighted greenhouse-gas optical thickness FM Miskolczi – Energy & Environment, Volume 21, Number 4 / August 2010 Pages243-262
See further pubs at: http://miskolczi.webs.com/
3) A earth heat engine climate model to evaluate temperature variations and winds from equator to the poles. e.g.,
Thermodynamic optimization of global circulation and climate, Adrian Bejan and A. Heitor Reis Int. J. Energy Res. 2005; 29:303–316
4) David Stockwell Accumulation Theory 22 Aug 2011 – relating the Pi/2 (90 deg) lag in ocean temperature from TSI oscillations.
5) Ed Fix Ch.14. The relationship of sunspot cycles to gravitational stresses on the sun
(Search for <Barycenter”) in Evidence Based Climate Science 2011 Don Easterbrook Ed.
See also comments above ref to:
The “GREENHOUSE Effect” As A Function Of Atmospheric Mass
Jelbring, Hans, Energy & Environment · Vol. 14, Nos. 2 & 3, 2003, pp 351-356
Jelbring, H. R. Thesis 1998. Wind Controlled Climate. Paleogeophysics & Geodynamics, Stockholm University. 111pp.
The Thermodynamic Relationship Between Surface Temperature And Water Vapor Concentration In The Troposphere, William C. Gilbert Energy & Environment · Vol. 21, No. 4, 2010 pp 263-276

Al Gored
December 29, 2011 6:21 pm

Yes indeed, very interesting. I have no idea how valid the particulars are but I concluded long ago that the global climate must be ‘driven’ by the kind of variety of factors suggested here. So I applaud this kind of big thinking.
Look forward to seeing how it stands up to real peer review, by real objective scientists.
But it will never pass the kind of rigorous pal review demanded by the IPCC. It appears to be lacking sufficient social engineering levers.

Phil's Dad
December 29, 2011 6:22 pm

Chesty Puller says:
December 29, 2011 at 5:52 pm
“Once gravitationally compressed it takes work to uncompress it.”
I would not argue that point. I suggest that if you removed the force of gravity it would uncompress pretty quickly. The work would be done by the kinetic energy already in action but previously constrained by the work of gravity acting on the atmosphere (and thus manifest as heat).
“Once gravitationally compressed…” rather suggests that gravity stops acting on it when it reaches an equilibrium pressure. I had imagined the stasis was due to equal and opposite forces in action. If you remove one the other becomes immediately apparent.
PS The perpetual motion thing was rather unkind.

Myrrh
December 29, 2011 6:28 pm

The problem I have with the IGL, ideal gas law, is the way AGWScience Fiction uses it. Let me explain.
When I first began investigating AGW claims I noticed a lot of arguments about ‘back-radiation’ and saw that convection and thermodynamics had been thrown out to emphasise radiation, but I could find no arguments about something that puzzled me, the claim that carbon dioxide was well-mixed in the atmosphere and that it could accumulate in the atmosphere for hundreds and even thousands of years. I was questioning an avid promoter of AGW at the time and asked him to explain the reasoning behind these claims, he is a physics teacher (PhD).
He told me that carbon dioxide, nitrogen and oxygen are ideal gases, that they obey the ideal gas law, in particular, he said that, ‘the atmosphere is empty space’, that ‘molecular collisions are elastic’, that ‘these gases are travelling at vast speeds through empty space continually colliding with each other and so becoming thoroughly mixed’.
I said that doesn’t make any sense, carbon dioxide is heavier than air and will therefore always sink displacing air unless work is being done on it. He said ‘not so’, that ‘carbon dioxide becomes very quickly so thoroughly mixed through these ideal gas collisions at immense speeds that it takes a great deal of work to separate it out’.
I gave him real world examples of carbon dioxide heavier than air separating out and sinking to the ground – breweries, volcanic venting – and with his moderator privileges, his post denying this disappeared, but, he still claimed that becoming thoroughly mixed as per ideal gas scenario is what would happen even if ‘an amount of carbon dioxide decended to the ground by taking the whole package of air it was in with it’, still unable to credit carbon dioxide with real world properties.
Leaving that aside for then, I asked him to confirm what he was claiming for CO2 in how it became well-mixed. I suggested a thought experiment: that in a room a volume of carbon dioxide had pooled on the floor, there was no work done to alter any of the conditions in the room from what it was that allowed the CO2 to pool, no windows opened, no fan put on. I said the carbon dioxide being heavier than air would remain pooled on the ground. He said the carbon dioxide would immediately begin to move at great speeds colliding with the other molecules in the air of nitrogen and oxygen and would become very quickly thoroughly mixed.
I known I’m right and he wrong. Real world observation proves it, basic real world physics understands and explains it.
What we have here is, astonishingly, the ideal gas law non-properties actually, physically, being given to real gases.
The Ideal gas is of course entirely imaginary, no real gas obeys the ideal gas law.

“How do we describe an ideal gas? An ideal gas has the following properties:
1. An ideal gas is considered to be a “point mass”. A point mass is a particle so small, its mass is very nearly zero. This means an ideal gas particle has virtually no volume.
2. Collisions between ideal Gases are “elastic”. This means that no attractive or repulsive forces are involved during collisions. Also, the kinetic energy of the gas molecules remains constant since these interparticle forces are lacking.” http://pages.towson.edu/ladon/gases.html

What AGWScience Fiction Inc’s meme producing department has done here is to create an imaginary world, the ideal gas world, and claim it is our real atmosphere.
From this all else follows: this fantasy world doesn’t have convection, it doesn’t have the water cycle cooling the Earth when lighter than air water vapour takes up the heat and condenses out to fall as rain as carbonic acid, because ideal gases don’t have weight or inelastic collisions.. They skip over all these real world scenarios because they don’t fit in with their ideal gas volumeless empty space atmosphere of their imaginary world. They have no sound in their world so they can’t hear this…
Instead they talk about ‘sinks’ and somehow unexplained how half the carbon dioxide in the atmosphere ends up in these sinks while the rest stays up in the air accumulating still thoroughly mixing in elastic collisions.

https://www.e-education.psu.edu/meteo469/node/116
The Carbon Cycle.
Credit: from Dire Predictions (Mann & Kump) We refer to the amount of emitted CO2 that actually stays in the atmosphere as the airborne fraction of CO2. So far, only roughly half of our carbon emissions remain airborne. The other half has been absorbed by carbon sinks. The primary carbon sink is the upper ocean, which has absorbed roughly 25-30% of the CO2, while the terrestrial biosphere has absorbed another 15-20% of the CO2.
These sinks are not constant over time, however. Numerous studies indicate that both the upper ocean and terrestrial biosphere are likely to become less able to absorb and hold additional CO2 as the globe warms. Were this to happen, the airborne fraction of CO2 in the atmosphere would increase, and CO2 would accumulate in the atmosphere more quickly for a given rate of emissions.

And don’t expect any more internal cohesion in their claims then there is between their imaginary world and our real world, like, how can this be if they claim carbon dioxide levels have been constant around 280 ppm for hundreds of thousands of years and only man’s recent imput from burning coal raising the levels? We should be choc a bloc full of carbon dioxide instead of it continuing to be a trace gas.
So, the only way one can use ideal gas law is by bringing in all that it excludes, that is, real world properties of gases such as volume, inelastic collisions (attraction), weight relative to each other (gravity), and so on, and the real world methods of heat transfer including conduction and convection, and weather, follow.
What I would like to see, and not only because the maths language is not one I use with ease, is the real basic properties and processes discussed in English. What this discussion so far shows, is people so busy doing the maths they’re missing out concepts and using imaginary concepts and talking at cross-purposes, which is how all these discussions invariably end up.

palindrom
December 29, 2011 6:39 pm

Looking at the comments above, Joel Shore and Kevin Kilty seem to understant what they’re talking about. There may be some others mixed in, but they’re the ones who jump out as the competent ones.
Many of the other commenters are suffering from one confusion or another, or many at the same time.

palindrom
December 29, 2011 6:44 pm

Myrrh, the solution to your conceptual difficulty is as follows:
1) Even in perfectly still conditions, the scale height of CO2 would not be that much smaller than that of the other components;
2) Diffusion is a powerful effect that tends to keep things mixed equally. It is entropically much more favorable for gases to be mixed evenly than separated. Gravitational settling will occur in very quiescent conditions, and under high gravity (such conditions exist in white dwarf stars,where everything heavier than hydrogen sinks out of sight), but in the earth’s atmosphere it’s just not important.

Snowlover123
December 29, 2011 7:06 pm

Absolutely excellent posting. It’s a good thing to see more climate scientists like Dr. Nikolov and Dr. Zeller speak out against AGW.

OzWizard
December 29, 2011 7:20 pm

Kevin Kilty says:
December 29, 2011 at 12:23 pm

“The surface temperature results from 1) irradiance that is absorbed predominantly high in the atmosphere leading to a high temperature there, and 2) then a lot of work input by gravity as convection takes parcels from up high to the surface. There is also a little irradiance absorbed at the surface which is what drives the convection.”

OK, Kevin. Engineering degrees at 10 paces. Ready?
I may be misreading what you are trying to say but, your proposition that the irradiance on Venus is “absorbed predominantly high in the atmosphere”, flies in the face of the following ‘facts’:
1. The density there is lowest.
2. The atmosphere is transparent to incoming irradiance.
So how does convection get the high, hot, ‘thin’ stuff down to the ‘relatively cool’ surface? Won’t convection tend to keep the hot stuff high up. Hot CO2 rises, cool CO2 falls, n’est-ce pas?
You do mention the surface having a high temperature, but ascribe this to energy absorbed at TOA. I don’t follow your thinking about ‘little irradiance absorbed at the surface’. ( I make no comment about your ‘parcels’ bringing it down.)
So, I’m with Ned and Karl at present and you are not swaying my thinking one little bit, yet.

December 29, 2011 7:27 pm

palindrom says:
December 29, 2011 at 6:44 pm
Myrrh, the solution to your conceptual difficulty is as follows:
1) Even in perfectly still conditions, the scale height of CO2 would not be that much smaller than that of the other components;
2) Diffusion is a powerful effect that tends to keep things mixed equally. It is entropically much more favorable for gases to be mixed evenly than separated. Gravitational settling will occur in very quiescent conditions, and under high gravity (such conditions exist in white dwarf stars,where everything heavier than hydrogen sinks out of sight), but in the earth’s atmosphere it’s just not important.

The only thing wrong with that explanation is it does not match with reality. In the real world CO2 is not well mixed when it is only subject to simple diffusion. A classic example is CO2 pooling in a stable layer in abandon mines or storage tanks where the only force to mix the gasses is diffusion.
In those environments CO2 will form a stable layer along the floor of the mine or tank.
Every year a few people die in such situations as the walk into that stable layer.
The classic scenario in mine rescue is the “mine tourist” walks into the mine a few hundred feet with no problem at all, then as they descend into that stable layer or their motion through it mechanically mixes it with the breathable air above, they suddenly pass out due to anoxia and then suffocate as they lie unconscious on the floor of the mine.
It is mechanical mixing from convection and stirring due to winds and turbulence as it moves past obstacles that keeps the gasses mixed. Without those forces our atmosphere would be a layer cake of gases sorted by density.
Larry

JPeden
December 29, 2011 7:37 pm

@Chesty Puller says:
Atmospheric greenhouse warming is a result of impedance in the free radiative path from surface to space. Greenhouse gas molecules intercept the radiation and essentially reflect about half of it back at the source. The authors are correct that this may result in greater rate of convection but that is acheived through a greater surface temperature which condutively heats the air to a greater degree making it that much lighter which in turn makes it rise faster. At the same time the higher surface temperature also increases the motive force of radiative transfer and more of it gets squeezed through the impedance of the greenhouse gases just like when you increase the pressure of water more of it will make through a pipe whose diameter remained constant.
Then how was/is the ghg, water vapor, able to stop its own ghg effect, and indeed keep it from “spiraling upward” ? Why would CO2 override the hydrological cycle any more than water vapor already couldn’t? And why would water vapor suddenly assist CO2 to do what it, water vapor, couldn’t do before?

December 29, 2011 7:45 pm

Myrrh says “Wjat I would like to see … is the real basic properties and processes discussed in English”
Try my site: http://earth-climate.com and (seriously) I would appreciate feedback on anything you or others question, as this will become the basis of a planned small “simple English” book.

Myrrh
December 29, 2011 7:48 pm

palindrom says:
December 29, 2011 at 6:44 pm
Myrrh, the solution to your conceptual difficulty is as follows:
1) Even in perfectly still conditions, the scale height of CO2 would not be that much smaller than that of the other components;
2) Diffusion is a powerful effect that tends to keep things mixed equally. It is entropically much more favorable for gases to be mixed evenly than separated. Gravitational settling will occur in very quiescent conditions, and under high gravity (such conditions exist in white dwarf stars,where everything heavier than hydrogen sinks out of sight), but in the earth’s atmosphere it’s just not important.
=======
I’m not the one here with a problem conceptualising.
Methane is lighter than air, it rises and rises locally, see Catatumbo. It doesn’t mix thoroughly in the atmosphere around all the Earth by diffusion. Water vapour is lighter than air, it is always evaporating, it evaporates more quickly the hotter it is but still it evaporates locally and doesn’t ‘diffuse into the mythical well mixed background’. Carbon dioxide is one and half times heavier than air – it will always sink displacing air – it does not spontaneously diffuse into the atmosphere but will move in local conditions – it is lumpy because it is local.
Which is where plants expect it to be, they have evolved with stomata on the underside of their leaves to take in carbon dioxide.
And everytime it rains whatever carbon dioxide is around in the atmosphere at the time will ALL come down as carbonic acid, which is what pure rainwater is. Again, local. Winds and rain will move it around, but that is local for the most part. The reality is totally at odds with the AGW claim that carbon dioxide is well-mixed proportionally in all the atmosphere.

palindrom
December 29, 2011 7:57 pm

hotrod —
Ah, I bet you’re right. The key insight would seem to be that the timescale for gravitational settling of the heavier species is much longer than the timescale for the atmosphere to re-mix. I suppose that my point about the scale heights (which would apply if the atmosphere were strictly collisionless, which it is far, far from) might have some relevance to how weak the tendency toward stratification is, but you’ve pretty much got to be right that a highly collisional, quiescent gas would have to stratify. Thanks.

palindrom
December 29, 2011 8:07 pm

Myrrh — hotrod has it right about the mixing. But here’s another point.
The source you quote gives the following properties of an ideal gas:
“1. An ideal gas is considered to be a “point mass”. A point mass is a particle so small, its mass is very nearly zero. This means an ideal gas particle has virtually no volume.
2. Collisions between ideal Gases are “elastic”. This means that no attractive or repulsive forces are involved during collisions. Also, the kinetic energy of the gas molecules remains constant since these interparticle forces are lacking.
(1) confuses mass with volume. An ideal gas has to be diffuse enough that the volume taken up by the molecules is a small fraction of the total. Air satisfies this condition pretty well.
(2) “Elastic” does not mean that there are no attractive or repulsive forces — otherwise there would be no collisions! — it means that the collisions preserve kinetic energy. So the “Also” phrase here is the tail that wags the dog. An example of an inelastic collision would be one in which some of the molecules’ KE were converted into other forms (e.g., excitation of a molecule’s energy levels). In air molecules, translational KE is frequently converted into rotational energy of the diatomic molecules, but this doesn’t affect the ideality of the gas to any great extent, since on average an equal amount of rotational energy is converted into kinetic.
Your source seems to have a rather shaky grasp on physics terminology.

palindrom
December 29, 2011 8:28 pm

Myrrh @7:48 —
You know, of course, that it’s possible to tell empirically whether a gas is well-mixed or not, without regard to theory. CO2 is well-mixed. Methane isn’t as well-mixed in part because its lifetime in the atmosphere is of order a decade.
The stomata of plants, being on the lower side of the leaves, are about a half-millimeter lower in elevation. Surely you aren’t claiming that this is a significant effect. I’d think keeping rainwater out would be a much more important driver.

December 29, 2011 8:52 pm

Congratulations to the authors of this theory. I have only a small commentary: Pressure could be a source of energy if we associate it with gravity. Essentially, gravity field is a sink of thermal energy. The higher the system is placed with respect to the surface, the higher the unusable potential energy., i.e. potential energy that cannot be used to do work or to be transformed into thermal radiation.

December 29, 2011 10:33 pm

Galane says:
December 29, 2011 at 8:55 pm
When is someone going to investigate the climatic effect of Earth’s decreasing axial tilt on the climate? As it tilts towards its minimum, ………
=====================================================
I agree, and I disagree. Assuming the earth is sphere, (it isn’t a perfect sphere) the tilt has nothing to do with the average global temp. If one part of the world tilts away from the sun, the other part will tilt towards the sun. So, everything will remain the same on the globe.

December 29, 2011 10:56 pm

[snip . . you know the rules . . kbmod]

December 29, 2011 11:30 pm

OzWizard says:
December 29, 2011 at 7:20 pm
Kevin Kilty says:
December 29, 2011 at 12:23 pm
“The surface temperature results from 1) irradiance that is absorbed predominantly high in the atmosphere leading to a high temperature there, and 2) then a lot of work input by gravity as convection takes parcels from up high to the surface. There is also a little irradiance absorbed at the surface which is what drives the convection.”
“OK, Kevin. Engineering degrees at 10 paces. Ready?
I may be misreading what you are trying to say but, your proposition that the irradiance on Venus is “absorbed predominantly high in the atmosphere”, flies in the face of the following ‘facts’:
1. The density there is lowest.
2. The atmosphere is transparent to incoming irradiance………..”

Ozwizard, the facts are there is almost no solar irradiance at Venus surface, the atmosphere is essentially opaque.  Full facts and reference can be seen at page 11 et seq of  http://www.climate-facts.com/pdf/challenge.pdf    links to the papers and references cited are at page 19 of that documents.
Notably in the words of A. A. Lacis; “greenhouse effect requires that a substantial fraction of the incident solar radiation must be absorbed at the ground in order to make the indirect greenhouse heating of the ground  surface possible”  and this essential element just does not exist on Venus.   As will be seen A.A. Lacis 2010 paper is falsified by his 1975 paper.  (links to both papers and others are at the above link)
 

E.M.Smith
Editor
December 29, 2011 11:41 pm

Per folks stressing over the temperature graph from Hansen 2008, I found this interesting paper by Hansen from 2011 (near as I can tell):
http://www.columbia.edu/~jeh1/mailings/2011/20110118_MilankovicPaper.pdf
that has substantially the same graph in it, but with the PETM “cooled” to about 12 C and with the start cooled to about 9 C; while the Pleistocene looks to me to have been “warmed” but one or 2 C.
Gee, if that 2008 Hansen paper can be found, it would be an interesting A/B study in how The Team cook the books over time to flatten things they want flattened and warm the present…
The wiki has a graph that looks like the start matches Hansen 2011, but leaves the Pleistocene (‘near now’) at -4 C for the ‘center of mass’ of the color.
http://en.wikipedia.org/wiki/File:65_Myr_Climate_Change.png

This figure shows climate change over the last 65 million years. The data are based on a compilation of oxygen isotope measurements (δ18O) on benthic foraminifera by Zachos et al. (2001) which reflect a combination of local temperature changes in their environment and changes in the isotopic composition of sea water associated with the growth and retreat of continental ice sheets.

So looks to me like, at most, you have a couple of C in the PETM to argue about. Oh, and asking just how Hansen manages to keep cooling off the past and warming the present every time he touches any temperature data sets…
I did find one 2008 paper where he erases some of the recent cooling via “assigning” some of it to ice formation:
http://www.moraymo.us/2008_Hansenetal.pdf

we assign the remaining 􀀁18O change to ice
volume linearly at the rate 60 msl per mil 􀀁18O change (thus
180 msl for 􀀁18O between 1.75 and 4.75). Equal division of
􀀁18O between temperature and sea level yields sea level
change in the late Pleistocene in reasonable accord with
available sea level data (Fig. S8). Subtracting the ice volume
portion of 􀀁18O yields deep ocean temperature Tdo (°C) = -2
(􀀁18O -4.25‰) after 35 My, as in Fig. (3b).

So looks like he is able to make the changes in the record via “assignment”, as needed…
Or maybe those are adjustments… or corrections… or fudge… or whatever the current “term of art” is these days… “Climate Art”…. hmmm catchy, that 😉
So looks to me like Hansen is happy to have the numbers wander around over time from paper to paper as needed. Still, would be nice to have all the assumptions, er, assignments, detailed and just which paper had the version, variation, “artwork” with these numbers that Hansen made in it…

David
December 30, 2011 12:12 am

James Sexton says:
December 29, 2011 at 10:33 pm
Galane says:
December 29, 2011 at 8:55 pm
When is someone going to investigate the climatic effect of Earth’s decreasing axial tilt on the climate? As it tilts towards its minimum, ………
=====================================================
I agree, and I disagree. Assuming the earth is sphere, (it isn’t a perfect sphere) the tilt has nothing to do with the average global temp. If one part of the world tilts away from the sun, the other part will tilt towards the sun. So, everything will remain the same on the globe.
————————–
James, think a little more on this. The SH is about ? 8o% ocean, the NH is about ? 60% land. The earth in January recieves about 7% more insolation, or about 90 watts per sq M more energy, than in July, yet the earth’s atmosphere is, on average, 4 degrees cooler. The NH is reflecting more SWR to space, the SH is absorbing far more energy into the oceans then in July. If this were to persist would the oceans warming, where the residence time of energy is far longer, eventually overcome the atmosphere cooling due to the loss of energy entering the ocean and reflecting back to space?

E.M.Smith
Editor
December 30, 2011 12:15 am

L says: December 29, 2011 at 6:31 am

The one red flag I see is the long term predicted pressure profile in figure 8. Eocene pressures max out at ~ 185 kPa – that’s approaching double today’s standard pressure of ~ 101 kPa !! I would think at those kind of pressures there would be some biological effects which might be manifested in the fossil record – how life adapted to such high pressures.

IIRC, the increased air pressure in past eras shows up in larger insect sizes (they limit on oxygen transport through the joints as they must pump O^3 with mass increase but get only Area^2 increase in air flow with size). That’s why we could have 3 foot wingspan ‘dragonflies’ in the past, but not now. Also Megabat type Bats evolved during the Eocene IIRC too. Finally, there are those Pterodactyls that can not theoretically fly in the present atmosphere. IIRC they need about 2 x the present air density to have the aerodynamics vs muscle kinetics keep them in the air.
There are some other lines of reasoning that exist as well which all point to higher air pressures in the past. (Not the least of which is the volume of carbon in coal et. al. that WAS in the air before…)
Bottom line is that there is significant evidence for air pressure swings in the past, much of it to higher pressures (though at least once the 02 level plunged to near extinction for our proto-mammal ancestors… but a small rodent like critter survived, so you and I exist…)

Duster
December 30, 2011 1:34 am

… IIRC, the increased air pressure in past eras shows up in larger insect sizes (they limit on oxygen transport through the joints as they must pump O^3 with mass increase but get only Area^2 increase in air flow with size). That’s why we could have 3 foot wingspan ‘dragonflies’ in the past, but not now. Also Megabat type Bats evolved during the Eocene IIRC too. Finally, there are those Pterodactyls that can not theoretically fly in the present atmosphere. IIRC they need about 2 x the present air density to have the aerodynamics vs muscle kinetics keep them in the air. . . .
Now that is quite elegant. I’ve corresponded with an engineer and sci-fi author, sadly now deceased, who quite seriously argued that the planetary mass absolutely had to have changed, otherwise dinosaurs could not have walked around nor could the large pterodactyl’s have made it airborn.

Mydogsgotnonose
December 30, 2011 1:41 am

Only last night I received a contribution from an experimentalist who has done the spectral analysis of ‘back radiation’, the concept by Aarhenius that underpins the CAGW scam.
It’s almost pure black body indicating it has little to do with GHGs. There is a contribution in clear skies but under clouds it is simply a measure of temperature.
The concept of trapped energy in a GHG blanket radiating energy spherically is pure, unadulterated bunkum.. Any professional scientist or process engineer knows you cannot use the S-B equation in this way for a single emitter. With no temperature gradient, that spherical radiation is exactly offset by incoming radiation from the inner surface of the circumscribing sphere. In a temperature gradient, net transfer of energy is down the temperature gradient.
This is the most spectacular failure of basic science in modern history. Aarhenius’ excuse was that the radiation laws, particularly Kirchhoff’s had only just been developed. There is no such excuse now those at whose untrained feet lies responsibility for this monumental cock up.

E.M.Smith
Editor
December 30, 2011 2:07 am

Joel Shore says: December 29, 2011 at 9:48 am
[SNIP: Joel, that comment is not helpful. You can do better. -REP]

Not from what I’ve seen… Mostly just repeating AGW Talking Points and calling folks idiots (in various ways), and carping, near as I can tell. So while I encourage your efforts at improving the quality of the contributions, well, there’s this problem with teaching new tricks…
But while I’m on the subject:
John Wilkes Booth is just sooo wrong. To even start in an undergraduate program in biological sciences takes chemistry, and with it the Ideal Gas Law. Heck, I had it in high school as college prep. So a guy with a Ph.D in forestry is going to be up to his eyeballs in chem. Organic. Inorganic. Bio-chem up the wazoo. (Krebs cycle ring any bells? How about transpiration and the chemistry of CO2 absorption / respiration chem / redox happening reversibly day / night?)
The sheer mind-blindness to what it takes to understand chemistry and gas laws in learning how a forest works is just astounding.
I went to an Ag school and sat in on one lecture on soils chemistry. The physical and surface chemistry involved is, er, “not simple”. Dynamics of root dissolution of minerals in a complex mix of organic, inorganic, and biochemical stew is enough to make your head spin (and I’d already had the full series of chem, organic chem, and was starting bio-chem; oh and upper division genetics too).
So tell you what, JWB and similar “argument to authority” folks: Go sit in on some biochem classes, especially those involving stomata and gas transport with enzyme systems, then tell me that PV=nRT is not clear to those folks.
Pressure x Volume = amount-of-stuff x gas constant x Temperature.
As the ‘amount of stuff’ is pretty constant in the short term, and the constant is a constant, that pretty much puts Temperature dependent on Pressure and Volume. As in a 1 G field, pressure is directly proportional to G and ‘amount of stuff’, we’re pretty much left with V and T.
So look at the height of the air column for an isobaric point. During high solar activity, it went up. When the sun shut down the UV dramatically in 1998, it started dropping. T dropped with it. It really IS that simple, and that direct, and CO2 can do NOTHING to change that. At best it can crank the water cycle a trivial fraction of a percent faster and well below the error bars in what we measure.
Want to test these guys paper? Plot the orbit height that was calculated as “OK” over time based on where the drag was found to be high enough to matter. Plot it against sunspot cycles and solar UV output. Plot that against temperatures. I’m willing to bet dollars to doughnuts that it ends up being a 3 way match. V vs SolarHeat vs T. No way around it.
@JustAMom:
I’ve not finished all the comments yet, so maybe someone else gave you the simplified form. I did a posting of the intuitive form of this some time ago, but failed to take the time to work on through to this rather nicely done math exposition. Those very readable, but somewhat math deficient, postings are:
http://chiefio.wordpress.com/2010/12/02/does-convection-dominate/
http://chiefio.wordpress.com/2010/12/28/ignore-the-day-at-your-peril/
The “crib notes” version is pretty direct:
On the daylight side of the planet, heating happens only until convection picks up speed. A few hours later, massive quantities of heat are being dumped at the top of the air column. Water too. In the topics, you get huge storms from all that heat and water going well up in the air, to the base of the stratosphere. THAT is where the heat gets dumped. It just bypasses that whole IR argument with mass transport and heat of vaporization (and fusion for snow). At most a tiny percent increase in CO2 can cause a tiny percent increase in water cycling or height of the convection. The earth basically works like a giant heat pipe (and they can have thermal conductivity greater than solid metals).
http://chiefio.wordpress.com/2011/07/11/spherical-heat-pipe-earth/
Or, for a simple demonstration:
Put a pot of water on the stove. Turn the heat up until it boils. Now adjust the heat up / down. All that happens is the water boils out faster or slower. Put a lid on it (like massive greenhouse gasses) and all that happens is that the water still boils out as steam (escaping past the lid) but with more jiggling of the lid… (i.e. convection moving the CO2 around).
All that matters is heat in (sun) and mass transport (evaporation / convection). The temperature at which this happens will change with pressure, so at the top of a mountain the temperature will be lower, or at sea level higher, so changes in total air mass over time can shift the equilibrium temperature, but not much else…
@All the folks talking about loss of air to space:
One heck of a lot more of it went into ROCKS. Carbonates especially, but all those ‘banded iron deposits’ have bound O2 in them. Gasses react with metals and form solids. Iron Oxide. Calcium Carbonate. Magnesium Carbonate. Etc. (I hesitate to think of the amount of oxygen bound to the semi-metal silicon as silicates… or the Potassium and Aluminum bound oxygen in various mineral including all the feldspars…)
But for vulcanism doing recycle, we’d be hard vacuum now. Oh, and the nuclear decay driven recycle is going to end in a few hundred million years or so…. Either we get off this rock now, or life does not have time to re-evolve and try again…

Espen
December 30, 2011 2:15 am

Maybe it’s both GHG absorption and pressure. I found this paper: http://www.gps.caltech.edu/~jkirschvink/pdfs/Li_Pressure_Broadening_PNAS2009.pdf
They argue that higher pressure broadens the absorption lines of greenhouse gases, and that pressure may have been higher in earlier geological periods.

Graeme M
December 30, 2011 3:46 am

OT, but for those considering air density of the past and its effect on dinosaurs, David Esker’s site is a good read…
http://www.dinosaurtheory.com/

Arthur Rörsch, The Netherlands
December 30, 2011 4:57 am

Is this a new paradigm and/or a unified theory of climate?
I do not think so.
That throughout the troposphere the temperature is a function of pressure is based on the gas law and the first law of thermodynamics. It is explained and calculated in any handbook on climatology that the pressure gradient (which originates from gravity) leads to a temperature lapse rate. By adiabatic expansion. With a specific surface temperature and pressure, the decrease of pressure cools the air at higher altitude. Not the reverse.
Forget about the ice ball metaphor. (See below) The essence of the scientific greenhouse theory is bases on the description of the radiation transfer process through the atmosphere with the Schwarzchild equation which is based on the law for absorption of Lambert-Beer and the law for emission of Planck. Each layer in the atmosphere with IR active molecules will emit upwards and downwards. The downwards radiation is what is named the back radiation. This phenomenon that a layer emits in two directions should not be denied and therewith also not the phenomenon of a radiation flux downwards to the surface. However, this does not ‘warm’ the surface, because the positive net flux to space is determined by the temperature difference between the surface and the effective temperature of the back radiating atmospheric layer.
The theory of the greenhouse effect, caused by the radiation transfer process, says that the effect is essentially that the temperature lapse rate moves over all altitudes to higher temperature, parallel to the one without radiation transfer processes, and also at the surface. This sounds logic at first sight. If one wants to disprove an effect of the radiation transfer processes, it is necessary to prove that the lapse rate does not move parallel. Here comes in the increase in convection and latent heat removal by evaporation if there is a potential rise of surface temperature expected. It does not sounds very logic to deny the existence of the radiation transfer processes as such, ruled by Lambert-Beer and Plank’s law.
Is the Nikolov&Zeller a ‘unified theory of climate’? No. It is a theoretical consideration of processes to occur in a single air column without taking in account the winds, ocean flows and the hydrological cycle in general.
Without an atmosphere, the globe would not be an ice ball. Because of the unequal insolation the temperature near the equator would reach 100 C. Because of the large coverage by oceans, a deep convection with evaporation occurs, and this limits the surface temperature of the tropic oceans to 29 C.. The surplus heat from the surface moves by the Hadley cells up to 18 km altitude. It is partly radiated out to space as IR, partly moved South and North with the winds, were the rest will be radiated out to space. The tropical winds and oceans flows keep the moderate and polar zones warmer, than the local insolation can produce. This is all classical climatology from the time before the CO2 obsession.
Modern measurements sustain the view that climate (temperature) at a global scale originates from the processes near the equator. The tropical ocean surface temperature shows an annual variance which may be as large as 1 C. Then it is observed that that global average temperature, calculated from all latitudes, follows this annual variation remarkably precise.
Then the radiation balance, which nowhere on earth is in an equilibrium state. In the tropical zone, between 40 south and north, it is negative, in the sense that more radiation is received from the Sun than radiated out as IR at the top of the atmosphere. In the moderate and polar zone the situation is the reverse: more IR is radiated out to space than radiation received from the Sun. (It looks here like a ‘negative’ greenhouse’ effect).
At latitudes 40 south and north, two global wind zones meet, warm winds from the tropics moving pole wards, and colder winds from the moderate zones to the tropical region. According to classical climatology, the (small) climate changes observed in historical times can be explained by changes in the borders of these meeting wind zones. Now it is also remarkable that at this latitudes also the global radiation balance changes from negative to positive. This leads to the suggestion that this change in local radiation balance is of major importance to rule the global climate, rather than the radiation transfer processes in a vertical air column in the troposphere. It seems to me that they who are obsessed by a CO2 effect, have forgotten about this (global) unified theory of climate.

gnarf
December 30, 2011 5:10 am

You integrate the cosine of sun rays angle from 0 to 1…this only covers half a sphere…on the other half it goes from 1 to 0.
As a result, you integrate on half a sphere, and divide by surface of the whole sphere-> the result you get 154K should be doubled.
Am I wrong?

Joe Straub
December 30, 2011 5:18 am

Karl,
I do not have the slightest idea what all this means. Could you write it up as if you are giving the article to the, say, the Smithsonian Magazine. See you at a reunion some time! ’65,’65,’65

December 30, 2011 5:58 am

“Arthur Rörsch, The Netherlands says:
December 30, 2011 at 4:57 am”
Pretty much a summary of my recent work.
And I agree that it is based on past knowledge that seems to have been forgotton.
The point where it all went wrong was when someone in a position of power within climatology decided that the composition of the atmosphere was more important for surface temperature than atmospheric pressure.
That seems to have entered the zeitgeist unchallenged and it has been downhill for climate science ever since.

December 30, 2011 6:00 am

“CO2 can do NOTHING to change that. At best it can crank the water cycle a trivial fraction of a percent faster and well below the error bars in what we measure”
At last. I have spent the past four years trying to explain why that is so !!

kwik
December 30, 2011 6:08 am

I must say Figure 5 is intriguing……It should be make or break for this paper?
Claes Johnson is Professor in Applied Physics. At the end of his paper here;
http://www.csc.kth.se/~cgjoh/climatethermoslayer.pdf
he has a very good citation;
“If someone points out to you that your pet theory of the universe is
in disagreement with Maxwell’s equations, then so much the worse
for Maxwell’s equations. If it is found to be contradicted by observation,
well, these experimentalists do bungle things sometimes. But
if your theory is found to be against the second law of thermodynamics,
I can give you no hope; there is nothing for it but to collapse in deepest humiliation”
(Sir Arthur Stanley Eddington in The Nature of the Physical World, 1915)
And another;
“…no one knows what entropy is, so if you in a debate use this concept,
you will always have an advantage.” (von Neumann to Shannon)
I like that one.
And;
“Every mathematician knows it is impossible to understand an elementary
course in thermodynamics.” (Mathematician V. Arnold)
And finally;
“A theory is the more impressive the greater the simplicity of its
premises, the more different kinds of things it relates to, and the
more extended its area of applicability. This was therefore the deep
impression that classical thermodynamics made upon me. It is the
only physical theory of universal content which I am convinced will
never be overthrown, within the framework of applicability of its
basic concepts.” (Einstein)
The last one is a good point; I would say; Clean the paper up, remove all talk that
fuzzy the main argument and that gives opponents the change to discuss this and that.
Focus on the main message?

Joel Shore
December 30, 2011 6:11 am

In other words, even if it exists, back radiation cannot warm the surface.
That is what Prof Claes Johnson has proved with a detailed mathematical derivation

Again, this statement shows profound ignorance at several levels. First of all, Claes has not proven anything. The starting point for his nonsensical derivation is to replace a century of statistical physics with a bizarre assumption that something that every other applied mathematician would tell you is an artifact of doing numerical approximations of differential equations is some fundamental universal law of nature. (Talk about believing models over reality!!!!) In particular, he wants to replace the deep notion of how dissipation and irreversibility in general arises at the macroscopic level from the statistics of large numbers of particles undergoing reversible interactions at the microscopic level with the silly notion that one gets dissipation at the macroscropic level simply because it exists at the microscopic level (because it does in a particular way that numerical methods approximate the equations). This notion is completely without foundation and he has not shown that he can derive a century’s worth of results from statistical physics, let alone that he can go further than statistical physics can.
Second of all, even if you buy Claes’s nonsense, he still ends up with an equation that says that the amount of heat transferred by radiation between a warmer body and a cooler body depends on the temperature of the cooler body as well as the warmer body, which is the basis of the greenhouse effect. You can’t make the greenhouse effect disappear by just putting a different “spin” on the meaning of the terms in an equation. It is the equation itself that gives the greenhouse effect.

exNOAAman
December 30, 2011 6:40 am

Am I misreading things, or is it not significant that the USFS has allowed such questioning of GH theory? Federal employees like Hansen you say?

Jimmy Haigh
December 30, 2011 7:11 am

E.M.Smith says:
December 30, 2011 at 2:07 am
Always a delight to read your posts Sir.

December 30, 2011 7:19 am

Myrrh said something like “Water is transparent to visible light, therefore, visible light does not heat the oceans, for example.” What this indicates is that Myrrh should not participate in discussions with “paid scientists” unless/until he learns some basic physics.
FWIW, let’s correct this. Water is not transparent to visible light. It is simply more transparent than, say, rock. Visible light incident on the air/water surface is partially reflected and partially transmitted. One can easily derive the reflection and transmission coefficient (the fraction of the incident flux that goes either way) as a function of angle, and even do things like predict the polarization of the reflected wave and transmitted wave. The derivations are in e.g. J. D. Jackson’s Classical Electrodynamics or my own online textbook of electrodynamics.
The reflected part is a major contributor to the “mean albedo” of the planet because the atmosphere really is mostly transparent to visible light. I say “mostly”, because visible light actively and immediately heats the atmosphere is as well — not all of the energy that is incident on the upper atmosphere makes it all the way down to the surface, and the difference (absorbed by the atmosphere) contributes to its energy content and hence temperature.
The transmitted light — the part that goes down into the oceans through their “transparent” surfaces is pretty much 100% absorbed by the oceans. Let’s reduce it to simple terms that are easy to understand. Energy cannot be created or destroyed. Just beneath the surface of the see, at midday, one can easily see things just beneath the surface of the water — a significant fraction of the intensity of sunlight above the surface is transmitted down into the water where it can light up a (say) fish. One can easily see the fish swimming beneath the surface — not quite as bright as the fish would be pulled out of the water and directly illuminated, but hardly “dark” and most certainly not as bright.
As one descends in the water column towards the bottom of the sea, it gets dark. Very dark. Even a few hundred meters down only a tiny fraction of the light intensity that was transmitted into the surface remains.
Where does all of the power in that light go (power is energy per unit time)? It was absorbed by the water in between the surface and the depths. This power continuously heats the water. This is one of many reasons that the surface waters of the ocean and lakes in summertime are “warm”. Especially warm compared to the mean temperature of the oceans, ~4C. In the tropics, light incident at close to right angles maximally penetrates the water and delivers a significant amount of incident visible light solar energy into heating the top 100-500 meters of water, with the most heat delivered close to the surface and less and less delivered to the depths. This warmer water is less dense than the colder water beneath and remains at the surface; the ocean’s thermal profile is highly stratified with a thermocline between warm surface waters and the comparatively uniformly cool and dense waters beneath ~1 km. In these cooler waters, things like salinity regulate density and hence convection; near the surface there is still thermally driven convection and turbulence. At the poles sunlight hits the water at a more grazing angle and much more of it is reflected, just as it gets dark underwater faster than it gets dark above the surface of the water in the approaching afternoons. Less light transmitted means less warming.
I agree with several of the previous posters who note that the article above is just a step towards a unified climate model, and that the oceans are an important meso-scale moderator with contributions over non-Markovian time scales on the order of years to as long as 1000 years (estimated to be the thermal “mixing”, or equilibration time, to the extent that a non-equilibrium system can be said to have one). I’m not convinced that greenhouse gases are as irrelevant as they claim — although their demonstration of the scaling of planetary temperatures independent of atmospheric composition is a powerful argument, I’d like to see more of the details and something beyond an empirical fit curve. They hint that perhaps the curve can be derived, but that isn’t the same as deriving it. I agree, however, that the modulation produced by GHGs (including water vapor) is a secondary variation around a base temperature that is very likely stably determined by density as they assert.
The most important contribution of this paper is that it restores some of the missing physics (physics I was just beginning to realize was being omitted myself, as I’m still a bit new to this game). The details of convective transfer up to the top of the troposphere are, I think, of great interest and should probably be the focus of further work. This paper may well have found the basis for (say) 94 to 98% of average global temperature, but the remaining 2-6% — and its distribution — appear sufficient to produce ice ages, interglacial periods, and one a shorter time frame MWPs and LIAs.
I would hold off on asserting a “unified” theory of climate unless and until it can do things like a) predict coarse-grained global temperatures; b) predict the spatial distribution of coarse-grained (averaged over some interval) temperatures — a good model should be able to explain and predict NH heating and SH cooling and tropical whatever, if not variations on at least a continental/oceanic scale. One of the biggest problems with the current CAGW nonsense has been the rush to assert “settled” science when the supersimplified models were laughably unable to predict the observed variations. This is an obviously better model if only because it contains and obviously neglected term (and hence is a more general model). It appears that this terms is significant. It is not yet clear that the model is yet complete or sufficient to explain all of the observations, and in particular it has yet to be shown that GCMs built using this improved model are in better agreement with the observational data on the same time scales as the existing GCMs.
This latter condition is a low bar (given the poor agreement of the latter already) but it is an important bar, if only because it is a great opportunity to “debug” the model and determine what the parameters need to be in order to be quantitatively predictive.
I am generally very cynical of multivariate models, because multivariate models with empirically determined parameters are really nothing but thinly disguised nonlinear function fits, and I’ve been in that game — covariances abound and it is often easy to fit the same base data with two completely different models very nearly equally well — especially in a noisy system with huge error bars. Two things make models “believable” — a sound basis in physics (where the basis of this model is better than that of any model neglecting convection from the beginning, because this model includes radiation-only models as a special parametric case), and the ability to predict new things.
This one has both of these to some extent — better physics and the “prediction” of a correlation of planetary mean temperatures independent of atmospheric mix. This is, truly, a startling and profound observation — scaling laws always are as they can change everything. What it still lacks is any demonstration that it can do a better job of “predicting” global average temperatures over (say) the last thousand years, including the variations!
And yeah, it isn’t going to succeed in that without the solar connection being included, correctly, and quantitatively. One more missing piece of physics (where some of the physics itself may be obscure).
rgb

December 30, 2011 8:01 am

Chesty Puller says: The authors fail to realize that radiative energy leaving the earth’s surface is travelling at approximately 186,000 miles per second or for you European cats that’s about 300 million meters per second. Convective updrafts travel even in the most severe convective cells fail to reach 100 meters per second.
Atmospheric greenhouse warming is a result of impedance in the free radiative path from surface to space. Greenhouse gas molecules intercept the radiation and essentially reflect about half of it back at the source. The authors are correct that this may result in greater rate of convection but that is acheived through a greater surface temperature which condutively heats the air to a greater degree making it that much lighter which in turn makes it rise faster. At the same time the higher surface temperature also increases the motive force of radiative transfer and more of it gets squeezed through the impedance of the greenhouse gases just like when you increase the pressure of water more of it will make through a pipe whose diameter remained constant.

Well, yes, but in nearly any laboratory situation where both radiative cooling and convective cooling are enabled (good thermal contact with a disequilibrated fluid), convection tends to dominate.
To put it another way, much of the thermal energy leaving the earth’s surface travels at 3×10^8 m/sec for a distance of 10^{-10} meters (one Angstrom), where it is picked up by a fluid molecule that has just collided with a molecule of the surface. That molecule is now “warmer”, and — surrounded by many other “warmer” molecules — is less dense and hence displaced upward. As it rises, it transports the heat it absorbed upwards. Much of the earth’s surface is not only in contact with a fluid, it is wet. Those fluid molecules that whack into it often carry away water. The water evaporatively cools the surfaces and yes, carries heat into the surrounding gas where it is rapidly lifted up, carrying heat with it.
As this warmer gas is uplifted, it penetrates through the “blanket”! The model of a “simple blanket” is therefore fundamentally, deeply flawed. One isn’t only looking at a surface and a blanket and a single channel, one is looking at a mix of three different cooling mechanisms at the surface and three different heat transfer and exchange mechanisms that operate all the way through the troposphere, followed by a relatively static (but still apparently long-time-scale variable) “blanket” in the stratosphere. In the end, quite a bit of the heat given off by the surface goes through most of the blanket by non-radiative means.
I actually think that this is a lot harder than rocket science. Rockets are actually rather simple.
rgb

Gary Pearse
December 30, 2011 8:19 am

E.M.Smith says:
December 30, 2011 at 2:07 am
Re loss of all that oxygen to space:
http://bing.search.sympatico.ca/?q=oxygen%20content%20of%20earth%20rocks&mkt=en-ca&setLang=en-CA
The earth’s crust, 5 to 30km or so thick is composed of 47% oxygen. Chiefio, you are right, we needn’t be getting ourselves into a knot about how much atmos must have been lost to space. This is why one can’t just go with rarified physics to explain all. Geological input tends to get overwelmed by physicists’-how-hard-can-it-be-to-do- paleoclimatology, etc…with the results we have been getting from such as the hockey team leading to having to keep moving the goalposts.

G. Karst
December 30, 2011 9:00 am

I find the complicating factor, wrt atmospheric pressure and volume changes to be the ever present water content. It can be regarded as large compression spring, in the atmos. Slight changes in pressure causes a continuous phase change from vapor to liguid and back again. This causes a volume change of 1600 to 1. Far greater than any non condensing gas change volume. Compression of vapor water also causes a large additional release of enthalpy heat energy and vise versa. These are enormous inputs and I am not sure of the value of gas analysis without careful steam table input quantification. A tough nut to crack.

jorgekafkazar says:
December 29, 2011 at 5:21 pm
G. Karst: have no fear. the media may ignore it, but most scientists will not, including those who know little about climate. They will be intrigued to see this methodology and will explore it for themselves. Remember, Climatology, as practiced, depends on the fact that there are very few “in the know.” The more who know, now, the greater the Team’s shame

Agreed. However, scientist do not set policy. Government and the people’s perception of science issues do. This is the frustration I was venting. GK

Joel Shore
December 30, 2011 9:07 am

Robert Brown says:

The most important contribution of this paper is that it restores some of the missing physics (physics I was just beginning to realize was being omitted myself, as I’m still a bit new to this game).

Such physics is not missing from any models that are used for quantitative calculations. It is only missing from the simplest models used to qualitatively demonstrate the greenhouse effect. Furthermore, the authors of this post do not include it in the correct manner. There is no contribution whatsoever of this paper to science.

December 30, 2011 9:09 am

@Arthur Roersch
> Is this a new paradigm and/or a unified theory of climate? I do not think so.
> …
> The essence of the scientific greenhouse theory is based on the
> description of the radiation transfer process
In your own words above you say GHE is based on radiative issues. But N&Z have made the bold claim that GHE does not depend directly on radiative issues. (Other than the implicit enhancement in pressure caused by solar irradiation).
@N&Z
“…our results suggest that the GH effect is a thermodynamic phenomenon, not a radiative one as presently assumed.
Yes, lapse rate is already understood as “decrease of pressure cools the air at higher altitude”, but the GHE is not exclusively based on that priniciple, so I think that qualifies the N&Z claim as ‘new’.
I think they can also claim ‘unified’ (or at least major steps in that direction) because their theory broadens the surface temps discussion to other planets, simplifying everything with plain thermodynamics, and offers plausible explanations for historical anomalies.

Arthur Rörsch, The Netherlands
Reply to  John Day
December 30, 2011 10:35 am

Day
See another blog on WUTW by Ira Glickstein “Unified Climate Theory may Confuse Cause and Effect’ and a comment by Willis Eschenbach, with which I both largely agree.
Arthur Rorsch

BigWaveDave
December 30, 2011 11:06 am

Joel Shore said:
December 30, 2011 at 6:11 am

In other words, even if it exists, back radiation cannot warm the surface.
That is what Prof Claes Johnson has proved with a detailed mathematical derivation
Again, this statement shows profound ignorance at several levels.

Yep, yours!
More to follow.

Ged
December 30, 2011 11:18 am

I am amused by people talking about the atmosphere “expanding” and forgetting the “where to?”. Air can’t actually go anywhere, gravity creates a closed container. If the atmosphere expands above you, it’s still -on top of you- and you are still feeling its full force of pressure from gravity.
Local atmospheric expansion will absorb some of the heat energy as it works against gravity, and contraction will release heat. This is why high pressure systems increase the local temperature, as they are systems of falling air that get compressed as they drop (conversely for low pressure systems, and why high pressure is less cloudy and less raining, absorbing moisture, while low pressure systems are the opposite).
We could increase the atmosphere to 10 times its height above the Earth, decreasing the density, and the pressure at Earth’s surface would still be 1 bar.

December 30, 2011 11:25 am

kwik (Dec. 30, 2011 at 6:08):
The second law is nothing more than an application of the principle of logical reasoning that has been called “entropy maximization.” Under this principle, numerical values are assigned to the probabilities of the ways in which an outcome can occur for a statistical event by maximization of the missing information per event or “entropy,” under constraints expressing the available information. Through applications of entropy maximization, the theorist ensures that probability values express all of the available information but no more.
Thermodynamics is a consequence from an application of entropy maximization in which the ways in which an outcome can occur are the microstates of the referenced physical system and the constraint on entropy maximization is energy conservation. The entropy is the missing information per event about the microstate of the system. The continuing mystification of university students regarding the origins of thermodynamics can be traced to a lack of training in logic in the curriculum of higher education.

Arthur Rörsch, The Netherlands
December 30, 2011 11:27 am

Shore,
May I add that in climate modelling much attention is given to thermodynamics. See e.g. the 800 page handbook General Circulation Model Development; past present and future” ed. David A. Randall (AP 2000) and in this chapter 22 by R. Johnson “Entropy, the Lorenz cycle and climate” and there in section IV “The classical concept of the Carnot Cycle and the Driftless Climate state”.
Sorry to say so, I get a bit annoyed by people inventing the wheel again with ‘new’ models, whereas there is background information available from the serious scientific literature.
In my opinion, the problems with which atmospheric scientists still wrestle is with the working of the thermodynamic laws, in dynamic processes which proceed far from the thermodynamic equilibrium, which are located in the top of the troposphere.
Arthur

December 30, 2011 11:31 am

“We could increase the atmosphere to 10 times its height above the Earth, decreasing the density, and the pressure at Earth’s surface would still be 1 bar”
Correct.
But expanding the atmosphere upwards or contracting it downwards alters the height of the tropopause and thus effects a surface pressure redistribution involving the shifting of the permanent climate zones, changes in cloudiness and global albedo, changes in the speed and size of the water cycle and changes in the rate of energy flow through the system.
That is how one can move from this paper to a truly Unified Climate Theory:
http://climaterealists.com/attachments/ftp/TheUnifyingTheoryofEarthsClimate.pdf
from 8th January 2009. Revised 4th September 2011

December 30, 2011 12:02 pm

Arthur Rörsch, The Netherlands says:
December 30, 2011 at 10:35 am
Day
See another blog on WUTW by Ira Glickstein “Unified Climate Theory may Confuse Cause and Effect’ and a comment by Willis Eschenbach, with which I both largely agree.
Arthur Rorsch

Ok, I saw those comments => (paraphrasing Willis) “I don’t undestand the article, therefore it must be nonsense.”
Perhaps the problem that you and Willis share is that you’re trying to understand it top-down, from the beginning. I invite you to start near the bottom, at the following sentence, which is the crux of the argument, and work your way back to the top:
@N&Z
> Equations (5) and (6) imply that pressure directly controls
> the kinetic energy and [thus the] temperature of the atmosphere.

December 30, 2011 12:48 pm

Chesty Puller says – December 29, 2011 at 4:02 pm
The authors fail to realize that radiative energy leaving the earth’s surface is travelling at approximately 186,000 miles per second or for you European cats that’s about 300 million meters per second. Convective updrafts travel even in the most severe convective cells fail to reach 100 meters per second.
Is that really all you can do to contribute to a fascinating scientific debate here that has already attracted hundreds of intelligent comments?
When I search for my cat in the garden after dark I use a device (a torch) that fires photons at 186,000 miles per second. I don’t think of that as anything special (because it isn’t) and the physics involved is certainly no more technologically significant than our outside brazier that burns logs and keeps us warm on a UK summer evening after a barbecue. 
What tosh you do talk, mate. And, by the way, those of us who are from the UK are quite comfortable with Imperial measures. We invented them. So there’s no need to be so smart and condescending.
Could we now please now get back to the scientific debate that this blog has generated and to which you have contributed zilch.

Richard S Courtney
December 30, 2011 3:34 pm

Anthony:
In the abstract to their paper Nikolov & Zeller wrongly claim;
“We show via a novel analysis of planetary climates in the solar system that the physical nature of the so-called GH effect is a Pressure-induced Thermal Enhancement (PTE), which is independent of the atmospheric chemical composition.”
Their analysis is NOT novel.
It is a repeat of the Jelbring Hypothesis
(ref. Jelbring H, ‘The Greenhouse Effect as a function of atmospheric Mass’, Energy & Environment,• Vol. 14, Nos. 2 & 3, (2003)).
Jelbring’s 2003 paper can be read at
http://ruby.fgcu.edu/courses/twimberley/EnviroPhilo/FunctionOfMass.pdf
In my opinion, the fact that the paper by Nikolov & Zeller is a repeat of much earlier work deserves mention at the top of this thread.
Happy New Year to you and your family.
Richard

December 30, 2011 4:59 pm

Wow — the Jelbring hypthosis certainly does appear to be essentially the same work. The current Unified Climate Theory discussion being an elaboration to extend and independently validate the concept.
It seems to me that the easiest way to quantify the radiation driven GE (green house effect) is to use the Jelbring hypthosis to compute the gravity field lapse rate GE and subtract that from the total observed GE. What is left must include other special case energy transfer processes such as the presence of water vapor and clouds and the green house gas GE.
Such a calculation would put an absolute upper limit on how much of the GE can be ascribed to the radiative processes (which I suspect will be trivial).
Thanks for bringing that to our attention Richard!
PS. how did the original paper get ignored or pushed under the rug? It would be interesting to dig through the climate gate emails to see if any of them mention the Jelbring paper!
Larry

December 30, 2011 5:06 pm

@RichardSCourtney
> Their analysis is NOT novel. It is a repeat of the Jelbring Hypothesis
Similar hypothesis => “… the atmospheric mass exposed to a gravity field is the cause of the substantial part of GW”
But I think the N&Z regression analysis of planetary surface temps (Eq 8 and Fig 5) still stands as ‘novel’.
😐

Phil's Dad
December 30, 2011 5:32 pm

G. Karst says:
December 30, 2011 at 9:00 am
jorgekafkazar says:
December 29, 2011 at 5:21 pm
“G. Karst: have no fear. the media may ignore it, but most scientists will not,…”
Agreed. However, scientist do not set policy. Government and the people’s perception of science issues do. This is the frustration I was venting. GK

G. Karst: have no fear. the media may ignore it, but policy makers watch this site very carefully, including those who know they know little about climate.

Erinome
December 30, 2011 5:33 pm

You know, this article has a major error in the very first line:
Recent studies revealed that Global Climate Models (GCMs) have significantly overestimated the Planet’s warming since 1979 failing to predict the observed halt of global temperature rise over the past 13 years.
In fact, UAH lower temperatures have increased at a rate of 0.18 C/decade over the last 13 years (Jan-1999 to Oct-2011).
And its doubtful that that is even a climatologically relevant interval.

Richard S Courtney
December 30, 2011 6:04 pm

hotrod (Larry L) and John Day:
Larry, at December 30, 2011 at 4:59 pm you say;
“Wow — the Jelbring hypthosis certainly does appear to be essentially the same work. The current Unified Climate Theory discussion being an elaboration to extend and independently validate the concept.”
Indeed, that is what I pointed out in my post at December 30, 2011 at 3:34 pm.
And I stand by my request to Anthony for an update at the top of this thread to state what you and I agree.
Also, you ask me;
“how did the original paper get ignored or pushed under the rug? It would be interesting to dig through the climate gate emails to see if any of them mention the Jelbring paper!”
The Team ignored it and, therefore, it is not mentioned in the Climategate emails.
John:
You say at December 30, 2011 at 5:06 pm
[snip]
“But I think the N&Z regression analysis of planetary surface temps (Eq 8 and Fig 5) still stands as ‘novel’.”
Perhaps, but Jelbring assessed his hypothesis by comparison to planetary temperatures. A novel way to do the same comparison is NOT “a novel analysis of planetary climates in the solar system”: it is merely a novel method to validate Jelbring’s work that Nikolov & Zeller fail to mention and/or reference.
Happy New Year
Richard

Ned Nikolov
December 30, 2011 6:10 pm

Hello Everyone
As an author of the paper discussed on this blog, I have been watching in amazement the diverse views and perspectives expressed over the past 2 days, some of them quite predictable while other totally ‘out of the blue’. Overall, I find this exchange quite useful for it helps us understand the type of challenges people face when presented with this new paradigm. Your comments gave me several ideas on how to better address and explain key aspects of our theory.
Instead of responding to individual comments, I thought it would be more beneficial for the whole group, if we (the authors) prepare a brief ‘reply’ article that clarifies the main issues/questions raised on this blog such as the magnitude of the GH effect, the meaning of the terms ‘extra energy’ and ‘pressure thermal effect’, the role of GH gases and their relationship to convection, the physical meaning of the ‘effective emission height’ in radiative transfer, and a few others.
We will try to post our reply/clarification by the end of next week (Jan 6). In the mean time, I urge everyone seriously interested in the subject to read our paper in full at least twice while taking time to contemplate on different aspects of it. We know from experience that digesting a new paradigm takes time since it requires a SHIFT in perception (hence, the term ‘new paradigm’!). The way to reach that mental shift is by trying to think about the issue (in this case the GH effect) from a different (new) vantage point … It took us close to 12 months to fully realize all implications of Equations 7 and 8 with their amazing accuracy in predicting the mean temperature of planets over such a broad range of conditions. We then spent another 8 months to figure out how this relationship fits in with the climate forcings proposed by other studies before we were able to craft Figure 10.
As far as we understand it now, all pieces of the new paradigm fit together very nicely, but this has to be conveyed to others in a way they can see it too. And that’s where our current challenge and commitment is … One should always remember, though, that this type of situation is not new to the history of science. Think about Copernicus and the 250 years it took for his idea of the Sun-centered solar system to become a mainstream science concept. The history repeats itself! We just hope that this time the paradigm shift will happen much quicker … -:)
Happy New Year!

Richard S Courtney
December 30, 2011 6:20 pm

Erinome:
Even by troll standards your post at December 30, 2011 at 5:33 pm is weak. The statement you dispute is correct.
The statement is;
“Recent studies revealed that Global Climate Models (GCMs) have significantly overestimated the Planet’s warming since 1979 failing to predict the observed halt of global temperature rise over the past 13 years.”
And you dispute it by saying;
“In fact, UAH lower temperatures have increased at a rate of 0.18 C/decade over the last 13 years (Jan-1999 to Oct-2011).”
But the IPCC AR4 said “committed warming” (i.e. warming certain to occur because it was already ‘in the pipeline’) would be 0.2 C per decade averaged over the first two decades of this century:
see http://www.ipcc.ch/publications_and_data/ar4/wg1/en/tssts-5-1.html
So, the warming you cite as having happened is less than a tenth of that which the IPCC said was certain to happen over that period (unless you think global temperature is going to jump 0.4 C now and stay that high for the next decade).
And nobody – not even a troll – is going to claim the 0.4 C jump is possible, so the GCMs cited by the IPCC “have significantly overestimated the Planet’s warming since 1979 failing to predict the observed halt of global temperature rise over the past 13 years”.
Richard

Richard S Courtney
December 30, 2011 6:29 pm

Ned Nikolov:
Thankyou for your post at December 30, 2011 at 6:10 pm. I especially thankyou for your promise to respond to comments.
I ask that your response acknowledges Jelbring’s prior work. I feel sure that you were not aware of it which is why you dfid not reference it. But I am cognisant with the objections to the Jelbring Hypothesis that arose in 2003/4 so I may be able to help address some points of dissent that you had not anticipated but which have already been discussed.
Please note that I do not know if the Jelbring Hypotghesis is right or wrong, but I have seen some very flawed objections to it.
Happy New Year
Richard

Joel Shore
December 30, 2011 7:19 pm

Ned Nikolov says:
Think about Copernicus and the 250 years it took for his idea of the Sun-centered solar system to become a mainstream science concept. The history repeats itself! We just hope that this time the paradigm shift will happen much quicker … -:)
Dr. Nikolov: Unfortunately, for every Copernicus, there are probably at least 1000 people who think they are Copernicus but are just plain wrong. This included many people such as yourself who are very intelligent and have strong scientific backgrounds.
The scientific community is not rejecting your notions because it involves a paradigm shift; it is rejecting them because upon perusal they are seen to be wrong (as I have discussed here: http://wattsupwiththat.com/2011/12/29/unified-theory-of-climate/#comment-846787 , here http://wattsupwiththat.com/2011/12/29/unified-theory-of-climate/#comment-846836 , and here http://wattsupwiththat.com/2011/12/29/unified-theory-of-climate/#comment-847131 )
Your work is flawed from the outset: It doesn’t even obey the First Law of Thermodynamics (i.e., energy conservation)! It ignores the fact that lapse rates below the adiabatic lapse rate are not unstable to convection when considering the effect of convection! And, it ignores the work of others who have considered the thermodynamics principles correctly. You are not the first people to realize that one can apply the ideal gas law to the atmosphere or that convection is important in doing quantitative calculations… In fact, I challenge you to find anybody in the field who believes otherwise. I would strongly suggest that you read a textbook on climate science, one particularly good one being Ray Pierrehumbert’s “Principles of Planetary Climate”. I think you will be surprised to see what is already considered and understood.
[Moderator’s Note: Joel, thankyou for your courtesy and understanding. Happy New Year to you. -REP]

George E. Smith;
December 30, 2011 10:46 pm

Phil says that (many) Astro-Physicists see everything as a star, so their views on simple planetary climate issues, may not be believable.
Perhaps that is the problem that Dr Willie Wei Hock Soon, and Dr Sally Baliunas have with their associations with the Harvard Smithsonian Center for Astro-Physics. I must say, that I have found Dr Soon’s book on the Maunder Minimum, and the variable sun-earth connection to be quite illuminating and informative.
It turns out, that the Director of that institution; Professor Charles Alcock, is a graduate of the University of Auckland (BSc (hons)), and PhD Cal Tech.
Professor Alcock is the recipient of the 2012 UofA Distinguished Alumni Award in the Science and Engineering category; facetiously described as the University “knighthoods”, and thery are a big deal to the University, and its community. The annual dinner at which Professor Alcock will receive his award will be in March 2012.
I am planning to attend the dinner and award ceremony, since they invite me, and give me and a guest a free ticket every year (since 2000), so maybe I will get a chance to ask the good professor about his view of the shenanigans relating to the Baliunas/Soon Paper and donnybrook.
Dr Chris de Freitas is also on the staff at UofA, so I am hoping I can meet with him again at the same time, and find out what is happening there.
The UofA Society may award up to five of these awards in any year; but only one in Science and Engineering. It turns out that for 2012, one of the other recipients will be Dr Mark Sagar, who is some sort of engineering software guru. His PhD research at UofA lead to the systems for rendering anatomically correct simulations of parts of critters for use in digital movies. Moviegoers have and can see the results of his work in the movies King Kong, Spiderman 2, and that blue green propaganda film Avatar. I have no idea of his association with the politics of Avatar; but I guess his animations are pretty cool. I hav;t seen either of the other two movies.
So maybe I can get a word with him too. small world it is.

dr.bill
December 30, 2011 10:58 pm

When radiation is absorbed by the molecules of a gas, the energy can go into one of four places:
(1) Translational Energy,
(2) Rotational Energy,
(3) Vibrational Energy,
(4) Electron Transitions.
Only (1) is involved in determining temperature.
While (2) and (3) can “suck up” energy, and thus contribute to the heat capacity of the gas, the energy absorbed in this way does not make the molecules move faster, it just makes them spin or vibrate to a greater extent, and thus does not raise their temperature, which (if I may repeat myself) only depends on their Translational Kinetic Energy. Case (4) will also generally not result in a permanent increase in the speed of the molecules, apart from a “jolt” on absorption and a “counter-jolt” upon emission of the radiation that is produced when the electrons drop back to their normal states.
To the extent that any of these routes depend on pressure and density, it is apparent that the amount of temperature change obtained for a given energy input will be a variable quantity. It may well be that the authors have identified the extent to which this is true, thus possibly explaining their “enhancement” effect, at least in phenomenological terms.
/dr.bill

Richard S Courtney
December 31, 2011 1:34 am

dr.bill:
Thankyou for your helpful post at December 30, 2011 at 10:58 pm.
As an addition to that I copy here something I wrote on the other thread to avoid it being missed by those who only read this thread.
The Jelbring Hypothesis (now also presented by Nikolov & Zeller) amounts to the following.
‘All the radiative, convective and evaporative effects in a planet’s atmosphere adjust such that the atmosphere obtains a temperature lapse rate approximated by –g/cp, and this lapse rate defines the planet’s average surface temperature. The average surface temperature is observed to agree with the Jelbring Hypothesis on each planet with a substantial atmosphere that has a mass which varies little through the year.’
Clearly, some effects (e.g. convection) do adjust. At issue is whether the interaction of all the radiative, convective and evaporative effects provides the suggested adjustment.
Happy New Year
Richard

December 31, 2011 6:28 am

@ dr. bill
> … radiation is absorbed by the molecules of a gas …
> (1) Translational Energy,
> (2) Rotational Energy,
> (3) Vibrational Energy,
> (4) Electron Transitions.
>
> Only (1) is involved in determining temperature.
That is incorrect. At the molecular all three degrees (translation, rotation and vibration) of kinetic freedom contribute to heating. Proof: microwave ovens exploit the vibrational degree of water dipoles to heat your food and beverages.
http://en.wikipedia.org/wiki/Heat_capacity#Degrees_of_freedom
So how is the kinetic energy distributed among these degrees of freedom? Good question. For starters read up on the Equipartition Theorem, which holds, up to quantum effects, for most substances:
http://en.wikipedia.org/wiki/Equipartition_theorem
😐

Joel Shore
December 31, 2011 6:41 am

By the way, I have realized that Nikolov and Zeller’s calculation of the “natural greenhouse effect” (and, in particular, the temperature in the absence of a greenhouse effect) done here is basically the same as that performed by Gerlich and Tscheuschner in their paper and it is wrong for the same reason: It assumes no heat storage or transport, i.e., that the local temperature is determined purely by the local insolation. While an Earth without greenhouse gases might have a larger temperature range than the current Earth, it is hard to imagine it being particularly close to the assumption that is made. A better way to look at things is what actually is done by the climate science community: To calculate what the average temperature would have to be if the temperature were uniform across the Earth’s surface and then to note that to the extent that the temperature deviates significantly from this, Holder’s Inequality tells us that the average temperature would be lower than this value.

December 31, 2011 8:04 am

At some point someone seems to have decided that atmospheric composition involving radiative processes makes a significant difference to the temperature set by thermodynamic and gravitational influences.
I think one can deal with the resulting confusion by accepting BOTH scenarios but putting them in proper proportions.
As I see it the GHG aspect is in the air only and the gravitational pressure aspect is in air and ocean but mostly in ocean.
Gravity is blind to anything other than mass so the thermal characteristics of GHGs are an irrelevance to that portion of the story.
Since downwelling IR from GHGs cannot get into the oceans it is limited in its effects to the air but the oceans control air temperaure.
The only way the system could deal with the GHG portion of the effect is to alter the rate of energy flow from surface to space.
In other words the GHGs fractionally alter the balance between sea surface and surface air temperatures by increasing the energy content of the air (mostly in the form of latent heat) and reducing the energy content of the oceans by converting incoming solar energy to longwave before it can get into the oceans.
The system then has to correct that GHG induced imbalance between sea surface and surface air temperatures and must do so by shifting the surface air pressure distribution and the positions of the permanent climate zones.
I think that tops and tails it very effectively.
But the GHG effect remains miniscule compared to what sun and oceans achieve on multicentennial timescales.

Richard S Courtney
December 31, 2011 9:39 am

Joel Shore:
At December 31, 2011 at 6:41 am you present a ‘straw man’ when you write:
“By the way, I have realized that Nikolov and Zeller’s calculation of the “natural greenhouse effect” (and, in particular, the temperature in the absence of a greenhouse effect) done here is basically the same as that performed by Gerlich and Tscheuschner in their paper and it is wrong for the same reason: It assumes no heat storage or transport”
NO!! It assumes no such thing. Have you never heard of convection?
There are some good arguments against the hypothesis. Please provide a good argument instead of imagining one that does not exist. In other words, please try to think before making a post.
Richard

D. Patterson
December 31, 2011 10:01 am

GeologyJim says:
December 29, 2011 at 8:54 am
This. Could. Change. Everything. (in your best Howard Cosell voice)
[….]
Question: How has Venus managed to hold its thick atmosphere against the solar-wind flux? Is it just the greater molecular weight of CO2 compared to N2, O2, and such?

The principal reason why “Venus managed to hold its thick atmosphere” is because of its lack of aerobic lifeforms which convert atmospheric carbon dioxide into biosphere mass and deposits of carbon compounds and oxygen compounds into the lithosphere and atmosphere, and oxygen into the atmosphere.
The Solar planets began with atmospheres dominated by the high proportion of Hydrogen and Helium and some other components such as methane present in the primordial Solar nebula from which they coalesced by gravitational accretion.. These first atmospheres underwent immediate change as the Solar wind of the awakening Sun stripped the inner planets of much of their lightest atmospheric gases, Hydrogen and Helium, creating the planets’ second atmospheres.
The Earth’s second atmosphere has been described as being about 100 times greater in mass than Earth’s present atmosphere, and it was composed of nearly all carbon dioxide. Nitrogen, Oxygen, and other gases were present in only trace amounts. Oxygen could not exist in the Earth’s second atmosphere in any but very minor trace amounts, because there was too much iron and other reactants present which combined with what little oxygen was released to produce iron oxide or rust and other compounds. When aerobic life consumed nearly all of the atmospheric carbon dioxide, the exhaled oxygen combined with the available iron in the lithosphere to create the great beds of iron oxide minerals mined for iron today, and Oxygen began to accumulate in the atmosphere in large percentages once the available reactants had been saturated with Oxygen.
Surface air pressures in the massive carbon dioxide dominated second atmosphere were staggeringly heavy, and the effects upon optics, geochemistry, geomorphology, weather, and hydrodynamics were somewhat alien to current understandings of the Earth’s current environment. Venus and Mars retained remnants of their massive carbon dioxide dominated second atmospheres because they didn’t have Earth’s kind of experience with aerobic lifeforms eating all but trace amounts of the carbon dioxide out of the atmosphere.

cba
December 31, 2011 10:04 am

There are a number of fatal problems with this stuff. First off, they use a very simplistic model and then start to overcomplicate it and try to do a rather advanced analysis on it that appears to be unrelated to the foundational physics. They then come up with wild answers that should be a warning of fatal errors yet they make the claim that there is a problem with the conventional result, that of 133 C rise due to ghgs and the presence of the atmosphere.
It appears the problem concerning this 133 deg C value rather than the conventional 33 deg C occurs because of eqn 2. It would seem they are integrating over a sphere assuming a unit radius (or surface area) and only integrating over half the sphere. That is the integration range of 0 to 1 for du, a cosine function. That’s an integration of 0 to pi/2 (or 0 to 90 deg) which would be half the sphere. Double the value and one finds a result that is about 3 deg C higher than the 287 or 288 K.
Another obvious ‘problem’ is their inclusions of the background radiation of cs = 13.025×10^-5 and since that adds to the solar incoming power and referenced with the microwave background radiation caused by 2.72K temperature of space radiation, one can assume this number must be the incoming power due to that source. The give away is that the accuracy of the incoming solar is assumed only to 1 W/m^2 and the value they provide is 0.00013025 W/m^2. This is far too many significant figures tosssed in for an accuracy that can be no better than +/- 0.5 W/m^2 and quite frankly, that accuracy value is dreaming because we don’t know the average solar flux to that accuracy nor do we know the albedo to that level. This is a freshman physics class error. Also, if we try to use S-B to find the incoming power from the microwave background radiation of the universe, we have a number that comes out to be 0.3 x 10^-5 W/m^2 and that suggests that either the constant is improperly described or calculated incorrectly.
At present, I don’t see any reason to continue going through their presentation as it appears to be totally FUBAR.

dr.bill
December 31, 2011 10:07 am

John Day, December 31, 2011 at 6:28 am :
Gas molecules, John, gas molecules…
Liquids and solids have their own rules for degrees of freedom.
/dr.bill

D. Patterson
December 31, 2011 10:22 am

REPLY

Martin Mason says:
December 29, 2011 at 10:53 am
I’ve read before that the density of the atmosphere was once much higher than today’s and that this was why there were so many large flying dinosaurs.

The Earth’s second atmosphere has been described as being about 100 times more massive than the present atmosphere, and it was composed of almost all carbon dioxide. Methane, nitrogen, and the other gases were trace amounts by percentage of the second atmosphere. This second atmosphere lasted a couple of billion years from not long after the Earth was formed to about 2.2 billion years ago. Aerobic lifeforms developed, and they ate the atmospheric carbon dioxide until its percentage remaining in the composition of the atmosphere was reduced to only trace amounts about 2.2 billion years ago. Where the atmospheric carbon dioxide concentration was once something like 966,000 parts per million in the second atmosphere 100 times more massive some 4 billions of years ago or less, it was reduced to the current 300-400 parts per million in the recent centuries. During the last 550 million years, the carbond dioxide concentrations have varied fro around 6,000 or 8,000 parts per million to less than 200 parts per million during the latest ice ages.

D. Patterson
December 31, 2011 10:39 am

Jose Mayo says:
December 29, 2011 at 10:14 am
Perdón, pero…
Cómo puede variar la “densidad” de la atmósfera, y la “presión”, si no hay variación en la gravedad?
[TRANSLATION: How can you change the “density” of the atmosphere, and “pressure” if there is no variation in gravity? -REP]

That’s simple. You change the atmospheric mass.
The Earth’s second atmosphere was something like 100 times more massive and composed of almost all carbon dioxide. Aerobic life ate the atmospheric carbon dioxide, removing its mass from the atmosphere.
On the temporal scale of the past century or centuries, atmospheric mass is constantly being added and subtracted by a number of natural events. One of the most important examples if the water cycle, which is constantly exchanging mass between the atmosphere, hydrosphere, cryosphere, lithosphere, and biosphere. These exchanges affect orbital mechanics by tidal influences on the hydrosphere and atmosphere, atmospheric pressure distributions by wind, oceanic thermal distribution by currents, and much much more.

D. Patterson
December 31, 2011 11:03 am

Bill Illis says:
December 29, 2011 at 9:11 am
We also have the Faint Young Sun paradox to resolve – solar irradiance was as much as 27% lower when the Earth formed (increasing in close to a straight line in the time since).
The Earth should have been a frozen snowball until about 500 million years. It was actually very cold once Oxygen became prevalent about 2.4 billion years ago to about 580 million years ago, but the earlier periods seem to have been warm enough. Perhaps Oxygen thinned out the early atmosphere. Perhaps the early water vapour content (the oceans formed as water vapour rained out of the atmosphere) provides another part of the picture.

No, the atmosphere was about 100 times more massive, and it was composed of almost all carbon dioxide. Aerobic life ate the carbon dioxide, removing it from the atmosphere. The one percent of the atmosphere remaining to day was enriched with oxygen from the biosphere converting the carbon dioxide and returning oxygen to the atmosphere. Oxygen could not persist in the second atmosphere, because the rocks and other minerals reactive to oxygen rusted or reacted to remove the oxygen as fast as it was released into the atmosphere. It wasn’t until the biosphere released enough oxygen into the atmosphere from the conversions of carbon dioxide to rust all of the iron into the geological iron band formatons that free oxygen was able to accumulate in the atmosphere. During the great extinctions, however, the biospheric production of oxygen was interrupted and oxygen concentrations fell to dangerously low levels. Likewise, the biosphere keeps consumes carbon dioxide until the trace amounts become too low to support the life forms dependent upon carbon dioxide.

D. Patterson
December 31, 2011 11:13 am

Pamela Gray says:
December 29, 2011 at 11:32 am
Kevin, are u saying that Earth’s pressure is thus in equilibrium and that any temp anomaly must be driven by some other mechanism? I question whether or not our gravity controlled pressure is in equilibrium. All the time? Please enlighten me on your thinking related to this point.

Atmospheric pressure can never reach equilibrium so long as the Earth has an atmosphere. Equilibrium of the atmosphere requires a static atmosphere. The atmosphere can never become static due to orbital mechanics, Coriolis force, tidal forces, mass exchanges of water in and out of the atmosphere, biosphere variability, and more. The atmosphere will always seek equilibrium and never attain it.

D. Patterson
December 31, 2011 12:28 pm

E.M.Smith says:
December 30, 2011 at 2:07 am
[….]
Either we get off this rock now, or life does not have time to re-evolve and try again…

A little historical note you have touched upon. In 1970-1972, a handful of us representing futurist groups lobbied Congress to get the appropriations to build the Space Shuttle Transport System. We made our case before the U.S. House of Representatives Space and Aeronautics Sub-Committee with publications and presentations. They were impressed enough to invite take us over to NASA and speak with Dr. Fletcher and onwards to dinner, (where a grilled cheese sandwich cost $30.00), for further discussion of the space program goals. Present at the dinner were Senators, Congressmen, a NASA Deputy Director, and President Nixon’s White House representatives. We were more than a little surprised by all of this attention, having expected to be gently patted on the head and then studiously ignored after the committee meeting. On the contrary, it was a very unexpected event.
The political background going before the Congressional subcommittee was very very discouraging. The Nixon Administration and the Republican Party were not really wanting to take on a major budgetary increase for a new NASA program, especially with the budgetary problems being faced at the time. They could see merit in the ideals, but the budget and political hurdles and obstructions the Democrats were threatening to employ diminished the Republican enthusiasm for what they saw as a non-essential program. The Democrats were generally opposed to using the budget to fund a NASA program giving a Republican Administration good public relations for years to come and reducing the funds available for the Democrat’s social welfare projects. So, we were warned the idea of continuing the space program, especially a manned spaceflight program, with major funding was an unwelcome idea with both of the major political powers.
At the dinner, however, we were met by leaders from both parties and houses in Congress, and they let us know they wanted to hear our ideas about the future of manned spaceflight and the role of one of the three proposed space shuttle designs. When my turn came to speak, I explained why it was vital for humanity to expeditiously develop a permanent presence and communities beyond the Earth before one of many catastrophes I named could destroy the current civilization and humanity’s opportunity to ever again develop the technological capacity to colonize new homes beyond the Earth and insure the survival of humans. It may please you to know that this argument was well received and enthusiastically agreed upon by many of the government leaders at that dinner. They shared your own sentiments.
While we were mildly disappointed by their subsequent choice of the lowest cost design proposal at the insistence of President Nixon, we were more than pleased to have contributed at least some small effort towards seeing to it that the two political parties did not kill manned spaceflight or at least some form of the space shuttle. After the 1971 cancellation of NASA’s manned spaceflight to Mars in 1987, the future of manned spaceflight was very much in debate and doubt at the time.

December 31, 2011 12:56 pm

Regarding the moon, 255K:
http://www.asi.org/adb/02/05/01/surface-temperature.html
Otherwise, I think the water is key. We don’t have another example with water to compare to.

George E. Smith;
December 31, 2011 1:44 pm

“”””” Bill Illis says:
December 29, 2011 at 2:51 pm
The back radiation measured at 2 metres height, originates millimetres to a few metres above the instrument.
The molecules at this level are colliding with each other 6.7 billion times per second, many times faster than the average relaxation/emission timeline for GHGs.
The backradiation is coming from all the gaseous molecules since the emission spectrum for backradiation is very close to a blackbody spectrum (at the temperature of the 2 metre air) – Not the signature of CO2 or H2O high up in the troposphere as so many people assume.
When it is not cloudy, (35% of the time), there is small loss in the atmospheric window region, but when clouds are present (65% of the time), it is a perfect blackbody spectrum. “””””
Bill,
I take it from your wording, that YOU have actually observed/measured the “back radiation” or “down radiation” from the atmosphere, and in your final sentence you say it is a “perfect blackbody spectrum”.
Can I assume that you mean by that, that the wavelength or frequency dependent spectrum of such observed back radiation actually matches the calculated Planckian spectrum corresponding as you say, to the Temperature of the atmosphere, in the immediate vicinity of the instrument ?? Of course, one would not expect the spectral radiant intensity or irradiance, to match that computed for a true black body at that Temperature; but the frequency spectrum should.
Now why is it, that YOU and I seem to be the only people around these parts, who seem to believe that the ordinary atmospheric gases, N2, O2, and Ar, DO in fact radiate a continuum thermal spectrum, that is characteristic of the Temperature of those molecules; as distinct from the resonance molecular spectra of the GHGs that have or can adopt a sizable non zero electric dipole moment. H2O of course has a permanent non zero moment, and CO2 which is symmetrical in the ground state, can easily adopt a non zero moment, by bending; as in the 15 micron band, or assymmetrically stretching, in the 4 micron band.
I have pointed out now several times here at WUWT, that the ordinary neutral gases, even Argon, that are charge symmetrical in the free flight state, become highly assymmetrical during collisions between molecules; which of course are the characteristic of TEMPERATURE, since the molecular nuclear mass is about 3675 times as massive as the electron cloud, so all the momentum is in the nuclei, not the electron cloud, so the nuclei simply continue on during a collision, while the electron clouds repel, and decelerate, and eventually reverse in a head on collision; so the colliding molecules develop a varying and decidedly non zero electric dipole moment during a collision, and that is all that is required to radiate EM waves according to Maxwell’s equations. What the particle physicist sees as an accelerated electric charge; which MUST radiate, the Radio-Physicist, sees as a time varying electric current flowing for a non zero distance, aka a radio antenna; which also must radiate.
The exact same mechanism, also allows those colliding molecules to absorb EM radiation, which also can have a frequency or wavelength continuum spectrum, as distinct from the molecular line spectra of polar molecules.
Can you describe the type of spectrometer equipment that is used to make such measurements as you apparently have observed. I’m almost ready to go and buy my own damn spectrometer, and go make the observations myself.
I’m encouraged to hear that you seem to have actually observed, what I have only theoretically presumed MUST happen.
One final point; you say when there is cloud cover, that the spectrum is true BB shape, but absent clouds, there is some line specral leakage. Does that mean that the clouds are not the source of the observed down radiation spectrum, but simply block the energy escape via absorption and then thermalize it to establish an atmospheric Temperature ?
I must say that this post of yours, is probably the most valuable single post of this whole double thread about this alleged “unified theory” of climate.
If you would care to comment off line about these observations, you have my permission to ask Anthony for my working e-mail, and I hereby give him permission to give you that.
Thanks
George

Myrrh
December 31, 2011 2:22 pm

http://climate-change-theory.com says:
December 29, 2011 at 7:45 pm
Myrrh says “Wjat I would like to see … is the real basic properties and processes discussed in English”
Try my site: http://earth-climate.com and (seriously) I would appreciate feedback on anything you or others question, as this will become the basis of a planned small “simple English” book.
Thanks, I’ll take a closer look at it. But what concerns me is that the ‘basic physics’ as presented by the AGW crowd is being taken seriously by non AGW crowd and I think the discussion needs to go back to this in basic English… For example, as shown in this discussion, y’all (generic) seem to have bought into the nonsense that visible light heats land and oceans.., a physical impossibility in the real world, and the water cycle is missing completely which cools the Earth by 52°C, and this mythical background well mixed CO2 which magically defies gravity to accumulate for hundreds and thousands of years and never joins in the the cooling of of the Earth because it never joins the water cycle in evaporation and rain and fog and dew (all pure clean rainwater is carbonic acid) but again somehow magically gets taken into ‘carbon sinks’ in this static fanatasy world without convection, etc. etc.
The problem is not so much the maths, but maths which describes and has nuanced arguments about nonsense concepts, it’s the concepts that have to be cleared up first because unless it can be shown to make sense in common language the maths just makes it appear that something intelligent is being said when no such reality in place..
Lucy Skywalker had a project going to get a web site up with counter arguments, don’t know if that is still happening, but you could add your book’s info to it in some form perhaps.
.

December 31, 2011 2:26 pm

dr.bill says:
December 31, 2011 at 10:07 am
John Day, December 31, 2011 at 6:28 am :
Gas molecules, John, gas molecules…
Liquids and solids have their own rules for degrees of freedom.
/dr.bill

Ok, at low density/temperature, molecular gases may behave like monatomic gases. But in general, in gases where the molecules collide, vibration and rotation also contribute to the internal energy of a gas, 1/2 kT per degree of freedom.
So I would still quibble with your assertion that ‘only translational energy is involved in determining temperature’. The gas molecular energy is transferred among the different degrees of free according the Equipartition Theorem, such that the energy is equally distributed. If I could somehow make the molecules vibrate faster, that raises the internal energy and is shared with the other degrees via collisions, and the temperature goes up accordingly.
I invite you to read Tom Vonk’s excellent post on LTE (local thermal equilibrium) for more info on this:
http://wattsupwiththat.com/2010/08/05/co2-heats-the-atmosphere-a-counter-view/
Vonk’s post should also be of interest to D. Patterson et al. because it shows how the atmosphere can be in equilibrium locally.

Joel Shore
December 31, 2011 2:32 pm

Richard S Courtney says:

NO!! It assumes no such thing. Have you never heard of convection?
There are some good arguments against the hypothesis. Please provide a good argument instead of imagining one that does not exist. In other words, please try to think before making a post.

Perhaps you misunderstood what I was talking about: I was talking specifically about their calculation of the average surface temperature in the absence of a greenhouse effect that they gave in the section entitled “Magnitude of the Natural Greenhouse Effect.”
But, since you asked the question seemingly about their larger hypothesis: Convection does not explain how the Earth and its atmosphere could be absorbing 240 W/m^2 but emitting 390 W/m^2, unless you are suggesting that convection transfers heat from the colder atmosphere to the warmer Earth surface, which would of course really violate the Second Law of Thermodynamics.
cba says:

It appears the problem concerning this 133 deg C value rather than the conventional 33 deg C occurs because of eqn 2. It would seem they are integrating over a sphere assuming a unit radius (or surface area) and only integrating over half the sphere. That is the integration range of 0 to 1 for du, a cosine function. That’s an integration of 0 to pi/2 (or 0 to 90 deg) which would be half the sphere. Double the value and one finds a result that is about 3 deg C higher than the 287 or 288 K.

I hate to be defending this silly paper…but they did not make the factor of two error that you seem to think they did. Their integration is correct for what they are trying to do, but what they are trying to do is silly: I.e., they are calculating what the average surface temperature of the Earth would have to be if radiative balance had to be satisfied locally, i.e., if the local temperature were determined by the local (and instantaneous) insolation. It is a pretty silly approximation but they have done the calculation correctly as near as I can tell and have obtained about the same answer that Gerlich and Tscheuschner obtained for the same calculation (except neglecting the 3 K background temperature). And, they have gotten an answer that obeys Holder’s Inequality, whereas your proposed result does not.
Their result is wrong because it is too naive conceptually, not because they made any calculation error.

dr.bill
December 31, 2011 3:11 pm

John Day, December 31, 2011 at 2:26 pm :
John, try this (or any intro textbook on gas kinetics):
http://hyperphysics.phy-astr.gsu.edu/hbase/kinetic/kintem.html
/dr.bill

Myrrh
December 31, 2011 3:28 pm

palindrom says:
December 29, 2011 at 8:07 pm
Myrrh — hotrod has it right about the mixing. But here’s another point.
The source you quote gives the following properties of an ideal gas:
“1. An ideal gas is considered to be a “point mass”. A point mass is a particle so small, its mass is very nearly zero. This means an ideal gas particle has virtually no volume.
2. Collisions between ideal Gases are “elastic”. This means that no attractive or repulsive forces are involved during collisions. Also, the kinetic energy of the gas molecules remains constant since these interparticle forces are lacking.
(1) confuses mass with volume. An ideal gas has to be diffuse enough that the volume taken up by the molecules is a small fraction of the total. Air satisfies this condition pretty well.
(2) “Elastic” does not mean that there are no attractive or repulsive forces — otherwise there would be no collisions! — it means that the collisions preserve kinetic energy. So the “Also” phrase here is the tail that wags the dog. An example of an inelastic collision would be one in which some of the molecules’ KE were converted into other forms (e.g., excitation of a molecule’s energy levels). In air molecules, translational KE is frequently converted into rotational energy of the diatomic molecules, but this doesn’t affect the ideality of the gas to any great extent, since on average an equal amount of rotational energy is converted into kinetic.
Your source seems to have a rather shaky grasp on physics terminology.

far as I can see this is on par with the general basic ideal gas law, but there’s no need to get too picky for accuracy here because there’s no actual real consistency even when they refer back to ‘laws’.. 🙂 Ideal gas diffusion and Brownian motion both used to ‘prove’ the point with no awareness that these come from completely different set of circumstances… So my point was only that this is where their unrealistic picture of the atmosphere comes from – direct from ideal gas non-properties.
So, they really have a picture of the atmosphere above us as empty space of the basic ideal gas description, which is how they account for the volumeless molecules moving at great speeds because there’s nothing to stop them, and so thoroughly mixing and not being subject to gravity, so they have no weight, and so on – whereas, our real atmosphere is an actual fluid ocean of gas above us, it has volume, it has weight, molecules do not move at ideal speeds through this as if empty space – we have sound.
Sound is only possible because this gaseous ocean above us has real volume which means the molecules are not moving at high speeds but hang around on the spot as it were. When a sound travels through this it moves the molecules to hit the molecules next to it which then relaxes back to where it was, it is the energy of sound being passed along, and just like the ocean, the air stays where it is, it’s the wave that moves. Think mexican wave.
Air is real volume, it is volumes of air which travel as wind which arise from the differences between volumes of air. Within that real molecules are subject to gravity, they have real weight relative to each other, ideal gas doesn’t have this because it’s a maths thing like ‘average’, it doesn’t really exist.
It is really peculiar how this has been created to present a world totally imaginary, but argued for as if real.. So they think carbon dioxide diffuses into the atmosphere like an ideal gas and can’t separate out because ideal gas doesn’t have weight or rather that weight doesn’t matter because it acts like an ideal gas.
palindrom says:
December 29, 2011 at 8:28 pm
Myrrh @7:48 —
You know, of course, that it’s possible to tell empirically whether a gas is well-mixed or not, without regard to theory. CO2 is well-mixed. Methane isn’t as well-mixed in part because its lifetime in the atmosphere is of order a decade.
Look, I don’t have the reference to hand, but even the IPCC reports have 3-5 years residence time for carbon dioxide.
And, it still bugs me that the AIRS data is being fudged. They still haven’t released upper and lower troposphere – their conclusion however, is that ‘contrary to beliefs that CO2 is well-mixed it is lumpy’ Just don’t expect them to elaborate on that, because like the 3-5 years residence time it’s not needed for the propaganda.
The stomata of plants, being on the lower side of the leaves, are about a half-millimeter lower in elevation. Surely you aren’t claiming that this is a significant effect. I’d think keeping rainwater out would be a much more important driver.
Water lily leaves have their stomata on the top.. 🙂
Carbon dioxide is heavier than air, one and a half times heavier, it will always sink through air displacing oxygen and nitrogen to come to the ground, it also comes down in rain which is carbonic acid, but what is important here is that because heavier than air it does not readily rise in air. Local winds, movement of animals, heat and so on will move it around and a plant that wants to catch it will evolve adapting to this, so the underside of the leaves the best chance of getting carbon dioxide for photosynthesis.

Myrrh
December 31, 2011 3:51 pm

Robert Brown says:
December 30, 2011 at 7:19 am
Myrrh said something like “Water is transparent to visible light, therefore, visible light does not heat the oceans, for example.” What this indicates is that Myrrh should not participate in discussions with “paid scientists” unless/until he learns some basic physics.
FWIW, let’s correct this. Water is not transparent to visible light. It is simply more transparent than, say, rock.

Hmm, I prefer to go with the paid physicists who taught me..
Water is transparent to visible light. Visible light direct from the Sun is tiny, it works on the electronic transition level which means that it doesn’t move the whole molecule into vibration, which is what heat is, it works by hitting or not hitting electrons, but being absorbed or not being absorbed by electrons.
In the atmosphere (contrary to the claim that the atmosphere is transparent to visible light), visible light hits and bounces off the electrons of the molecules of oxygen and nitrogen, this means that briefly the electrons absorb visible light’s energy before chucking it out – think pinball machine because that’s how we get our blue sky, blue light is even more nervily highly energetic than red and more apt to get bounced around the sky.
Water is transparent to visible light, which means the electrons of water don’t absorb the energy,and so visible light doesn’t even get to play with them as it does in the atmosphere, it is transmitted through.
What is erroneously claimed for visible light in the atmosphere is actually true for visible light in the oceans. It cannot heat water because it is transmitted through water without being absorbed on any level.
In the real world I live in this is very well known, for example in producing visible light for photosynthesis to extend growing conditions for plants without burning them up – the object of the exercise being to take out of lamps that which heats the plants…
Thermal infrared, heat energy, thermal energy on the move direct from the Sun by radiation does have the oomph to move whole molecules into vibration, to raise the temperature , to heat them up.
This is basic real world physics.
I’m sorry that this has become practically impossible for so many to understand because the actual basic physics has been perverted in the education system to facilitate confusion necessary to sell the AGW meme.
Go on, heat a cup of water with a blue led light, let us know when it’s hot enough for you to make a coffee..

JimF
December 31, 2011 6:02 pm

Wilkes Booth says:
December 29, 2011 at 8:10 am
BZZT! Sorry, but you win The Big Loser in Debate Award for 2011. Not only do you not know anything about these people, nor about the content of their respective specializations, but you even impugn at least one university. Attack the paper, Loser, not the people. As a result, I’ll just put you on my “ignore” list as a contemptible troll.
And your buddy (or just another screen name?) Kevin Kilty (@December 29, 2011 at 10:10 am) backs you up with this hilarious entry: “…Take as an example a Ph.D. in Transportation. It is a civil engineering degree, but it focusses narrowly and doesn’t put much if any emphasis on thermodynamics, or fluids, or any of a range of other engineering and scientific topics, even at the introductory level….” Yeah, well, with a PhD of that nature, one could aspire to be the Chairman of the IPCC, the world’s most important body deliberating on the advancing horrors of Global Whatever. Oh, wait, a Railway Engineer from a formidable university in India already got the job! Darn it! Anyway, I’ll ignore him too, as Living Proof of the Left Hand Side of the Bell Curve.
[Moderator’s Note: You can safely ignore John Wilkes Booth. Really. Don’t let him disturb your New Year. -REP]

wayne
December 31, 2011 6:40 pm

@ John Day: December 31, 2011 at 2:26 pm
@ dr.bill
John, have to agree with you, sorry dr.bill. Dr.bill, you seem to speaking of a gas out of LTE. You should really read that link to Dr. Vonk’s posts John Day gave but you’ll have to ignore the formatting defects. It seems #4’s exact energy level that is excluded and that is also why there are no purely vibrational OR electronic transfers, they always include a small rotational energy level component also. Upon re-emission these rotational levels are not included, just the exact jump between the vibrational or electronic levels. That is key in equipartition across all degrees of freedom the way I have read, but it hard to find sites on the web delving deep enough in these picky aspects to get a proper viewpoint.

dr.bill
December 31, 2011 7:30 pm

wayne, December 31, 2011 at 6:40 pm :
Hi Wayne,
It’s been a while. Hope all’s well with you. I’ll surmise that John’s livelihood doesn’t depend on the accuracy of his knowledge of such things, but mine does, so I’ll stick with my original statement. It’s just basic Physics, and it’s pointless to get into a “this guy said, the other guy said” thing about it. The derivation is at the link I gave earlier, and it’s in every Physics book on the subject, going back forever. Click on the “Show” button to get the details. I’m done with it…
/dr.bill

JimF
December 31, 2011 7:57 pm

Weatherford says:
December 29, 2011 at 2:09 pm “…The only flaw I find in the article is the failure to include the discussion of gravity as a factor in gas pressure….”
But they do talk about gravity:
3.1. Climate Implications of the Ideal Gas Law
“…c) on a planetary scale, pressure in the lower troposphere is effectively independent of other variables in Eq. (5) and is only a function of gravity (g), total atmospheric mass (Mat), and the planet surface area (As), i.e. Ps = g Mat/As. Hence, the near-surface atmospheric dynamics can safely be assumed to be governed (over non-geological time scales) by nearly isobaric processes on average, i.e. operating under constant pressure….” {Interesting. The subscripts don’t copy and paste correctly.}

JimF
December 31, 2011 8:29 pm

@hotrod (larry L) says:
December 29, 2011 at 3:00 pm “…we assume that atmospheric mass is more or less constant over geological time. But is that is a valid assumption?…”
Maybe not. Thanks to Crosspatch and others, read these two articles (short, entertaining, and really raising my geologist’s eyebrows):
Earth’s atmosphere before the age of dinosaurs
http://pubs.acs.org/subscribe/archive/ci/30/i12/html/12learn.html
Was the Atmospheric Pressure Different at the Time of Dinosaurs?
http://levenspiel.com/octave/dinosaurs.htm
The upshot of these two articles is that the Earth’s atmosphere was far more dense, mainly due to CO2, for much of its history. Much of the CO2 is now caught up in rocks and sediments and to a lesser degree, in coal and such. Much of the rest of it is dissolved in the ocean. The Earth’s atmospheric pressure has declined as the CO2 was removed from the gaseous state to its present 1 bar.
In fact, we are in a situation where CO2 is in short supply, close to the point where plants cannot live. When they die, we die. Too bad we don’t have a Carl Sagan or other who can get this notion across to people in general. A lot of the rampant global warming and environmental radicalism would come to a screeching halt.

cba
December 31, 2011 8:50 pm

“Joel Shore
I hate to be defending this silly paper…but they did not make the factor of two error that you seem to think they did. Their integration is correct for what they are trying to do, but what they are trying to do is silly: I.e., they are calculating what the average surface temperature of the Earth would have to be if radiative balance had to be satisfied locally, i.e., if the local temperature were determined by the local (and instantaneous) insolation. It is a pretty silly approximation but they have done the calculation correctly as near as I can tell and have obtained about the same answer that Gerlich and Tscheuschner obtained for the same calculation (except neglecting the 3 K background temperature). And, they have gotten an answer that obeys Holder’s Inequality, whereas your proposed result does not.
Their result is wrong because it is too naive conceptually, not because they made any calculation error.

Joel,
Look at this again! They integrate over a hemisphere (0 – 1 for u= cosTHETA and 0-2pi for phi and they divide by 4*pi – the area value for a unit radius sphere. Theta ranges from 0 to pi/2, not 0 to pi. The integration is not over the entire sphere. What is being integrated is the incoming power per unit area adjusted by the cosine of the angle wrt the zenith which gives the temperature of each incremental area of the surface times that surface area. You cannot divide an integration over a hemisphere by the area of a whole sphere and get the right answer.
I’m not sure what holder’s inequality is or why it might be relevent but the wiki article on it says it’s a <= proposition not a << proposition as they state. It also suggests that the lambertian disk approximation isn't even approximately right. Just look at the value for the very simple case of uniform albedo and emissivity just assume it with a high rotation rate on the equinox day. Except for the cosine curve pole to pole the power distribution would be totally uniform. It would also claim the average T would be 150 K for an airless body. Considering over half the surface T would be over 300k , even 0K for the rest of the sphere would not be sufficient for the average surface T to be only 150k.

Ned Nikolov
December 31, 2011 9:21 pm

Fellows,
I will explain the gray-body temperature calculation and the Holder’s inequality in my official reply next week. Theoretical results from Eq. 2 actually match pretty well recent spatial retrievals of Moon’s surface temperature by NASA’s Diviner Lunar Orbiter. The GH effect is indeed MUCH larger than 33K. …No need to waste your energy on this now.
Instead, think about how is it possible for Equation 8 in our paper to predict so accurately mean surface temperatures of planets over such a broad range of atmospheric and radiative environments by using ONLY 2 independent variables – average surface pressure and TOA solar irradiance?

wayne
December 31, 2011 9:42 pm

cba says:
December 31, 2011 at 8:50 pm
Except for the cosine curve pole to pole the power distribution would be totally uniform.
>>
Hate to say it but in this case Joel is right. By that statement from you it appears you are missing the cosine curve in the horizontal dimension. Remember, this is massless view and every point on the backside is logically zero, there is no incoming or outgoing radiation, the sphere has no mass and cannot store energy, it can be either stationary or rotating, same results. But by dropping the horizontal cosine you should be getting ~179K instead of ~154K. Hope that helps you to understand exactly what is being said, it is no more or less nonsensical than Kiehl and Trenberth’s conjecture and per most current climate scientists and by the IPCC. I agree, what the IPCC says in the reports on this matter of radiation is totally meaningless without mass.

wayne
January 1, 2012 12:43 am

@ dr.bill
Hi back. It has been a while, in fact all of the way back to a white planet and a black identical planet at the same distance from a sun and its temperature. Right? lol! Now try that example on Dr. Nikolov & Zeller’s new theory. I might have been correct after all! Back then, it was just a gut physics feeling.
On rotation. I think we are just passing each other in terms, and some missing terms. I re-read all of the topics on hyperphysics links again, have many times. I was speaking with my mind on a glass of water equalized at a temperature. Put it in the microwave, it causes rotational energies to be absorbed, and the temp goes up. Stop right there. Missing term, thermalization. Neither of us ever said that word and that seems the only misunderstanding. At the higher temperature is will have more both translational and rotational energy due to some rotational energy thermalizing by equipartition, the relative ratio keeping the same within a small temperature range (ratio is specific heat related). You seem to be saying that the additional rotational energy at the new equilibrium will not be seen in temperature rise and on that I agree. Doesn’t that clear the air?

Richard S Courtney
January 1, 2012 1:34 am

Joel Shore:
At December 31, 2011 at 2:32 pm you say to me:
“But, since you asked the question seemingly about their larger hypothesis: Convection does not explain how the Earth and its atmosphere could be absorbing 240 W/m^2 but emitting 390 W/m^2, unless you are suggesting that convection transfers heat from the colder atmosphere to the warmer Earth surface, which would of course really violate the Second Law of Thermodynamics.”
I asked no such question, and you are talking nonsense.
I can do no more than to quote my response to your spouting such drivel that I gave on the other thread. I quote it here to avoid others needing to find it.
Richard
Richard S Courtney says:
December 31, 2011 at 11:05 pm
Joel Shore:
It is reasonable for you to evangelise your faith in the AGW hypothesis.
And it is reasonable for you to dispute the Jelbring Hypothesis.
But it is NOT reasonable for you to defend your faith by misrepresenting anything – including the Jelbring Hypothesis – which challenges your faith. And that is what you are doing in this thread.
Firstly, at December 31, 2011 at 2:16 pm and again at December 31, 2011 at 2:16 pm, you attempt to refute the hypothesis by asking:
“Look, you have 240 W/m^2 of energy coming in and 390 W/m^2 going out. Where is the extra 150 W/m^2 coming from? Be specific.”
And you compound that at December 31, 2011 at 3:13 pm saying;
“Just to add a bit more specificity to my last comment: If you propose that energy is coming from the gravitational field, that means that the gravitational potential energy is decreasing (at some rate like 150 W/m^2 of earth’s surface). What is causing this large decrease in gravitational potential energy?”
Only you has mentioned the radiative energy imbalance of 150 W/m^2 suggested by Kiehl & Trenberth, and only you has suggested that the imballance “is coming from the gravitational field”.
The answer to your question, of course, is that the radiative energy imbalance of 150 W/m^2 results from back radiation. But so what?
I remind that I wrote at December 31, 2011 at 12:56 am
“The Jelbring Hypothesis (now also presented by Nikolov & Zeller) amounts to the following.
‘All the radiative, convective and evaporative effects in a planet’s atmosphere adjust such that the atmosphere obtains a temperature lapse rate close to that defined by –g/cp, and this lapse rate defines the planet’s average surface temperature. The average surface temperature is observed to agree with the Jelbring Hypothesis on each planet with a substantial atmosphere that has a mass which varies little through the year.’
Clearly, some atmospheric effects (e.g. convection) do adjust in response to gravity. At issue is whether the interaction of all the radiative, convective and evaporative effects provides the suggested adjustment.”
So, the hypothesis says that any effect of your asserted very, very disputable “extra 150 W/m^2” is to increase evaporation and conduction that cool the surface such that –g/cp is maintained.
So, what relevance of any kind does your question have to any consideration of the hypothesis? Be specific.
Secondly, at December 31, 2011 at 2:16 pm you assert:
“Apparently, violating the Law of Conservation of Energy is not a substantial enough flaw for you guys!”
Where have we guys who support and adhere to the scientific method violated the Law of Conservation of Energy? Be specific.
Richard
PS I again remind that I do not know if the Jelbring Hypothesis is right or wrong. I am writing to object to your behaviour that is very wrong.

January 1, 2012 4:19 am

Richard S Courtney said:
http://wattsupwiththat.com/2011/12/29/unified-theory-of-climate/#comment-848293
“But the IPCC AR4 said “committed warming” (i.e. warming certain to occur because it was already ‘in the pipeline’) would be 0.2 C per decade averaged over the first two decades of this century:”
No it didn’t; it said .1C/decade:
“Committed climate change (see Box TS.9) due to atmospheric composition in the year 2000 corresponds to a warming trend of about 0.1°C per decade over the next two decades, in the absence of large changes in volcanic or solar forcing. About twice as much warming (0.2°C per decade) would be expected if emissions were to fall within the range of the SRES marker scenarios.”
http://www.ipcc.ch/publications_and_data/ar4/wg1/en/tssts-5-1.html
“So, the warming you cite as having happened is less than a tenth of that which the IPCC said was certain to happen over that period”
He said that the warming was .18C/decade since Jan 1999 (according to UAH), you say it should be .2C/decade and therefore actual temps are less than a tenth of what should have happened. Huh? That’s a .02C/decade difference. That’s really close agreement.

D. Patterson
January 1, 2012 6:39 am

JimF says:
December 31, 2011 at 8:29 pm
@hotrod (larry L) says:
December 29, 2011 at 3:00 pm “…we assume that atmospheric mass is more or less constant over geological time. But is that is a valid assumption?…”
Maybe not. Thanks to Crosspatch and others, read these two articles (short, entertaining, and really raising my geologist’s eyebrows):

There ae no maybes about it, the Earth’s atmosphere was originally about 90 to 100 bar of pressure.

Earth’s atmosphere before the age of dinosaurs
http://pubs.acs.org/subscribe/archive/ci/30/i12/html/12learn.html

The Earth’s first atmosphere was dominated by hydrogen and helium from the nebula and vaporized rock. This first atmosphere was very quickly transformed into the second atmosphere overwhelmingly composed of carbon dioxide. The article’s conjecture saying “CO and CH4, combined with oxygen to form CO2”and “All this took about half of Earth’s lifetime, and it left the atmosphere depleted of oxygen” makes no sense at all. The BIF (banded iron formations) occur in varves and at various episodes between about 3.7Gya to ~2.2Gya and again about 1.8/1.9Gya.
The earliest forms of life were anaerobic, meaning they could not exist in the presence of an atmosphere having free oxygen as described in the article. Observation of these and other factors seem to clearly rule out the possibility that free oxygen was present in the second atmosphere in any other than trace amounts until cyanobacteria released it in the process of respirating carbon dioxide.
Also, the origin of the Earth’s water is still subject to considerable current research. The article comments on the synthesis of the Earth’s water in the early atmospheric chemistry. The article does not comment on the research underway in an effort to identify the isotopic origns of water from the nebula and cometary impacts versus chemical synthesis in the early environment of the Earth.
These are two of a number of possible examples why this article must be used with caution.
Carbon dioxide is indeed in short supply from the point of view of the biosphere.

Joel Shore
January 1, 2012 6:50 am

Richard S Courtney says:

‘All the radiative, convective and evaporative effects in a planet’s atmosphere adjust such that the atmosphere obtains a temperature lapse rate close to that defined by –g/cp, and this lapse rate defines the planet’s average surface temperature. The average surface temperature is observed to agree with the Jelbring Hypothesis on each planet with a substantial atmosphere that has a mass which varies little through the year.’

(1) How do radiative effects adjust themselves?
(2) It is already understood why the lapse rate in an atmosphere strongly heated from below and cooled from above is close to the adiabatic lapse rate: A lapse rate larger than the adiabatic lapse rate is unstable to convection and thus convection does indeed adjust itself to maintain such a lapse rate. That is well-understood science.
(3) The lapse rate does not uniquely determine the surface temperature for the same reason that the slope m of a line y=m*x + b does not allow you to uniquely determine the value of y for any given value of x: You need to know “b” (which you can get by knowing the value of y for one particular value of x). In practice, what the greenhouse gases in the atmosphere do is set the “effective radiating level” at which radiation can escape to space…and the average temperature at this particular level is determined trivially from the total amount of radiation absorbed by the planet and its atmosphere from the sun (assuming that there is not significant heat flow from the planetary interior or that the planet is not undergoing continual gravitational collapse, as these would represent additional sources of energy).

So, the hypothesis says that any effect of your asserted very, very disputable “extra 150 W/m^2” is to increase evaporation and conduction that cool the surface such that –g/cp is maintained.

As I have explained, you can’t evoke evaporation and conduction because the extra 150 W/m^2 that I am talking about means that the planet’s surface is already emitting more energy than it could if there were not a greenhouse effect. Evaporation and conduction only make the problem worse, unless you are proposing that evaporation and conduction transfer 150 W/m^2 FROM the atmosphere TO the surface!

Joel Shore
January 1, 2012 6:53 am

Ned Nikolov says:

Instead, think about how is it possible for Equation 8 in our paper to predict so accurately mean surface temperatures of planets over such a broad range of atmospheric and radiative environments by using ONLY 2 independent variables – average surface pressure and TOA solar irradiance?

One reason it is possible is because your N_TE contains 4 free parameters! Another reason it is possible is that your Equation (8) contains two more free parameters!

Joel Shore
January 1, 2012 7:04 am

cba says:

Joel,
Look at this again! They integrate over a hemisphere (0 – 1 for u= cosTHETA and 0-2pi for phi and they divide by 4*pi – the area value for a unit radius sphere. Theta ranges from 0 to pi/2, not 0 to pi.

Except for the cosine curve pole to pole the power distribution would be totally uniform.

Okay…I am beginning to believe that you are right. It looks like they have made multiple errors in their calculation:
(1) The factor of 2 error that you note.
(2) The error in not including the longitudinal obliquity.
They have also assumed that the albedo due to clouds changes…which is not the usual assumption made to calculate the greenhouse effect alone, although might be justified by their considering all atmospheric effects. By coincidence, they arrived at nearly the same answer as Gerlich and Tscheuschner got by doing a calculation that is “correct” (given some rather bizarre and unphysical assumptions, as I have discussed above), so I thought they had probably done this correctly…but it looks like I was wrong. This paper is worse than I thought.

January 1, 2012 7:22 am

Joel,
Your understanding is substantially flawed in many respects.
Please stop.

cba
January 1, 2012 7:48 am

It wouldn’t be the first time that multiple fatally disastrous errors have compounded to give an apparently correct result for something. However, the notion that there is no thermal storage means that where ever there is no incoming solar, the temperature must immediately drop to 0K (or 2.7K) on the unlit hemisphere will result in an overall average temperature that is essentially half that of the lit side average temperature.
There will be a fundamental difference between the Moon & Mercury compared to an airless Earth or Mars in that the rotation rates are drastically different. Earth and Mars are roughly 24hrs while the Moon is almost 30 times that and Mercury is even longer. This permits the Moon and Mercury lit sides to reach an equilibrium radiative temperature whereas even an airless Earth or Mars would provide a far more even distribution of absorbed power and hence a more uniformly distributed temperature. Once this happen, the T^4 values related to power can be approximated well by just using T due to the differences being small. (Series expansion of the funtion approximates a linear result for small variations as higher order terms become much smaller).

G. Karst
January 1, 2012 8:30 am

Wow! This has become the best thread I have read for some time. If this is an indication of discussions to come in 2012, it is going to be a great year. Even the trolls are behaving and contributing useful opinion. If nothing else, this discussion clearly illustrates, how unsettled the science really is. Happy New year everyone… Y’all deserve it. GK

gnarf
January 1, 2012 8:56 am

It is advised to make a spreadsheet in which you divide the earth surface along longitude and latitude circles, and calculate the temperature for each part of the surface using the black body.
This way you perform manually the integral shown in this document, and the result is very different, around 250K.
I do not find it anymore, but one or two years ago I found such a spreadsheet on a forum, following a heated discussion about the very same subject: estimating average temperature of earth without atmosphere to estimate the greenhouse effect.

davidmhoffer
January 1, 2012 9:15 am

Ned Nikolov says:
December 31, 2011 at 9:21 pm
I will explain the gray-body temperature calculation and the Holder’s inequality in my official reply next week. Theoretical results from Eq. 2 actually match pretty well recent spatial retrievals of Moon’s surface temperature by NASA’s Diviner Lunar Orbiter. The GH effect is indeed MUCH larger than 33K.>>>>
I’m looking forward to it! I’ve been harping on the issue of averaging T instead of the fourth root of T in regard to energy balance for a long time now. It wasn’t until this thread that I had my “duh!” moment and realised that the calculation of GH was subject to the exact same colossal math error.
We have been collectively making the precise same error in attempting to trend T in order to determine if earth’s energy balance is positive or negative. Just as one cannot calculate the GH effect based on the average of T, one cannot calculate the energy balance based on the average of T.
So invested have we become in trending T, that even the satellite records, which directly measure P, are then converted, on a point by point basis, to T, and then averaged. Since the goal was to determine what the over all energy balance is, why would we convert from P to T? Trend in P is what we are trying to determine! So instead of averaging P and trending it, we first convert to T, which is NOT directly proportional, average it, and then draw conclusions about average P!
I’ve provided examples in multiple threads that demonstrate that it is possible to have a positive trend in T accompanied by a negative trend in P, making the trending of average T to determine what is happening from an overall energy balance perspective absolutely ludicrous. I haven’t gotten much traction on that point, but am fervently hoping that you do.
Averaging T rather than the fourth root of T amounts to the biggest math error in human history, and the mess it makes of GH calculations should be blindingly obvious to anyone who understands SB Law. The exact same is true regarding the insanity of averaging T across the globe over time and believing that the resulting trend has any value at all in determining earth’s overall energy balance.
thankyou thankyou thankyou!

Richard S Courtney
January 1, 2012 9:50 am

Robert Murphy:
Your post at January 1, 2012 at 4:19 am said;
“Richard S Courtney said:
http://wattsupwiththat.com/2011/12/29/unified-theory-of-climate/#comment-848293
“But the IPCC AR4 said “committed warming” (i.e. warming certain to occur because it was already ‘in the pipeline’) would be 0.2 C per decade averaged over the first two decades of this century:”
No it didn’t; it said .1C/decade:
“Committed climate change (see Box TS.9) due to atmospheric composition in the year 2000 corresponds to a warming trend of about 0.1°C per decade over the next two decades, in the absence of large changes in volcanic or solar forcing. About twice as much warming (0.2°C per decade) would be expected if emissions were to fall within the range of the SRES marker scenarios.”
http://www.ipcc.ch/publications_and_data/ar4/wg1/en/tssts-5-1.html
“So, the warming you cite as having happened is less than a tenth of that which the IPCC said was certain to happen over that period”
He said that the warming was .18C/decade since Jan 1999 (according to UAH), you say it should be .2C/decade and therefore actual temps are less than a tenth of what should have happened. Huh? That’s a .02C/decade difference. That’s really close agreement.”
OK. Two points.
Firstly, the IPCC said, and you quote from the link I provided,
“About twice as much warming (0.2°C per decade) would be expected if emissions were to fall within the range of the SRES marker scenarios.”
The emissions HAVE fallen “within the range of the SRES marker scenarios” so my statement was and is correct.
Secondly, as you say, “he” did say “the warming was .18C/decade since Jan 1999 (according to UAH)”.
But – in fact – that warming was NEGATIVE and not positive. Even this warmist site shows about -0.18C or less.
http://chartsgraphs.wordpress.com/2010/08/19/enhanced-uah-channel-5-temperature-anomaly-trend-chart/
There is complete disagreement. The IPCC says it was certain to warm: it cooled.
Richard

Joel Shore
January 1, 2012 10:11 am

G. Karst says:

If nothing else, this discussion clearly illustrates, how unsettled the science really is.

No…What this thread illustrates is that there is almost no limit to the scientific nonsense that people will cling onto in order to believe what they want to believe! It does not illustrate that the science is unsettled any more than similar debate on a creationist website illustrates how unsettled the science of evolution is.

Richard S Courtney
January 1, 2012 10:35 am

Joel Shore:
At January 1, 2012 at 6:50 am you ask me:
“(1) How do radiative effects adjust themselves?”
I answer, they don’t. They adjust in response to various changes (i.e. temperature, humidity, etc.).
Then your post makes a platitude about lapse rates.
And you follow that with complete twaddle which proves you have completely failed to understand the hypothesis being discussed. We are attempting to evaluate that hypothesis with a view to determining whether it should be accepted, rejected or amended. Please try to understand the hypothesis because your twaddle disrupts sensible discussion.
An example of the twaddle is provided by your having written;
“As I have explained, you can’t evoke evaporation and conduction because the extra 150 W/m^2 that I am talking about means that the planet’s surface is already emitting more energy than it could if there were not a greenhouse effect. Evaporation and conduction only make the problem worse, unless you are proposing that evaporation and conduction transfer 150 W/m^2 FROM the atmosphere TO the surface!”
This is so wrong that one could write a book on its errors.
For simplicity, I will point out only one of them.
Evaporation occurs (if you dispute this then explain how rain happens).
And evaporation cools the surface.
Increase evaporation and the surface cooling increases.
Reduce evaporation and the surface cooling reduces.
Please explain how evaporation “makes the problem worse”. What problem? And how could either an increase or reduction to evaporation fail to provide a possible reduction to it?
Richard

Myrrh
January 1, 2012 10:57 am

I’ve notice I’ve made the same typing error here as I made in another discussion, making the Earth with atmosphere but without the water cycle read ‘minus’ 67°C
http://wattsupwiththat.com/2011/12/29/unified-theory-of-climate/#comment-846626
Myrrh says:
December 29, 2011 at 5:16 am
We’ve always had a unified theory of climate. The general figure as given as I’ve seen it is that the whole of the atmosphere is Earth’s greenhouse and therefore all the gases in it are greenhouse gases, predominantly nitrogen, oxygen and water.
Without the atmosphere the Earth would be -18°C, but, it only gets the +33°C warming to 15°C via the dynamics of all the greenhouse gases –
without any atmosphere -18%deg;C
with atmosphere but minus the Water Cycle – 67°C

The water cycle cools the Earth by 52°C
– to bring it down to the 15°C from the 67%deg;C it would be with the main greenhouse gaseous ocean of nitrogen and oxygen above us, pressing down on us a ton/square foot.
==========================================================
The bold should read:
without any atmosphere: -18°C
with atmosphere but minus the water cycle: 67°C
=============================
Sorry.

Joel Shore
January 1, 2012 11:12 am

Richard S Courtney says:

Please explain how evaporation “makes the problem worse”. What problem? And how could either an increase or reduction to evaporation fail to provide a possible reduction to it?

What it makes worse is the problem that you are faced with in achieving energy balance in the absence of the atmospheric greenhouse effect. And, the reason why a reduction in evaporation won’t help you is that it can’t be reduced to a negative value.

D. Patterson
January 1, 2012 11:53 am

Joel Shore says:
January 1, 2012 at 10:11 am
G. Karst says:
If nothing else, this discussion clearly illustrates, how unsettled the science really is.
No…What this thread illustrates is that there is almost no limit to the scientific nonsense that people will cling onto in order to believe what they want to believe! It does not illustrate that the science is unsettled any more than similar debate on a creationist website illustrates how unsettled the science of evolution is.

You appear to be describing your own comments.

Joel Shore says:
January 1, 2012 at 6:50 am
[….]
(2) It is already understood why the lapse rate in an atmosphere strongly heated from below and cooled from above is close to the adiabatic lapse rate: A lapse rate larger than the adiabatic lapse rate is unstable to convection and thus convection does indeed adjust itself to maintain such a lapse rate. That is well-understood science

Can you even recognize why your statement is not science at all?

Joel Shore
January 1, 2012 12:23 pm

D. Patterson: Your comment adds absolutely nothing to the scientific discussion and are thus not worthy of any response.

January 1, 2012 1:01 pm

@dr. bill
> The derivation is at the link I gave earlier, and it’s in every
> Physics book on the subject, going back forever.
> Click on the “Show” button to get the details. I’m done with it…
Ok, I think I see the problem here, dr. bill is referring to the simplified “ideal” (monatomic-like) gas model, where by definition only translational energy is considered. Indeed, there is a HyperPhysics disclaimer stating such below the “Show” button:

It is important to note that the average kinetic energy used here is limited to the translational kinetic energy of the molecules. That is, they are treated as point masses and no account is made of internal degrees of freedom such as molecular rotation and vibration.

And even when rotation and vibration are accounted for under the equipartition theorem, it is a common practice to simplify the model by treating all degrees of freedom as “uncoupled”, i.e. no interaction between the degrees of freedom. Under such simplifying assumptions rotation/vibration have no effect on translational energy. Physicists are entitled to make these simplifying, “first-order” approximations, in order to get their heads around complex concepts, such as radiation transfer and thermal equilibrium.
But in the the real world, translation, rotation and vibration can be coupled together, such that (for example) absorbed vibrational energy (e.g. IR photon absorbed by CO2 molecule) can be coupled (“equipartitioned”, “thermalized” etc) to translational energy and thus enhance the translational kinetic energy.
Here is Tom Vonk’s summary of these coupling options:

Translation-translation interaction . This is your usual billiard ball collision .
Translation-vibration interaction . Here energy is exchanged between the vibration modes and the translation modes .
Translation-rotation interaction . Here energy is is exchanged between the rotation modes and the translation modes .
Rotation-vibration interaction … etc .

http://wattsupwiththat.com/2010/08/05/co2-heats-the-atmosphere-a-counter-view/
Yes, it’s true that translational energy dominates the diffusion of thermal energy (“heat transfer”), in order to solve the heat equation with finite differentials in the x,y,z directions. But rotation and vibration still must be considered in calculating the heat capacity of real world gases. And they can transfer molecular momentum directly to the diffusion process. (Energy and momentum must always be preserved. “Vibrating molecules” are essentially collections of atoms in motion)
Yes, the effects of these DOF’s vary, according to temperature, density, polarization etc, so for certain configurations one or more DOF’s may be declared “frozen out” or ignored as “neglible”.
Or not.
Bottom line: it is incorrect to assert as an absolute principle (as dr. bill did) that “only translational energy is involved in determining temperature”

January 1, 2012 1:30 pm

It seems to me that contributors to this thread have jointly made progress on the issue of the significance of the conjecture of Drs. Nicolov and Zeller (N&Z). It has been shown that a violation of energy conservation does not lie in N&Z’s conjecture when it is made conjunction with the radiative emission of 390 W/m^2 at Earth’s surface and radiative emission of only 239 W/m^2 at the top of the atmosphere, contrary to the assertion of Joel Shore. In claiming violation of energy conservation, Shore seems to have overlooked the fact that 239 W/m^2 is the intensity of a heat flux while 390 W/m^2 is the intensity of a radiative energy flux making this an apples vs. oranges comparison. At Earth’s surface, the heat flux is the vector sum of an upward pointing vector with an intensity of 390 W/m^2 and a number of other vectors each representing a flux of energy. According to Trenberth et al(2008), over the period from March 2000 to May 2004 the vector sum of these vectors was 238 W/M^2. The difference between the intensities of the heat fluxes at the bottom and top of the atmosphere, – 1 W/M^2, was absorbed by the land and oceans with the result that energy was conserved.
Among the vectors representing fluxes of energy at Earth’s surface is the one which Trenberth et al label the “back radiation.” According to Trenberth et al, it had an intensity of – 333 W/m^2. This vector is expected to grow in its absolute intensity with increases in the concentrations of greenhouse gases in the atmosphere. If a rise in this intensity were the sole response from increases in these concentrations then the equilibrium temperature at Earth’s surface would have to rise in response for energy conservation. However, a rise in the absolute intensity of the back radiation is NOT the sole effect from increases in greenhouse gas concentrations.
According to Trenberth et al, in the period between March 2000 and May 2004 convective heat transfer with an average intensity of 17 W/m^2 was operative at Earth’s surface. Wherever there was convective heat transfer in the atmosphere with a positive intensity, this was sufficient to have set up a feedback loop that persistently forced the lapse rate toward the adiabatic lapse rate. In these regions, the rate of decline of the atmospheric temperature with altitude was set by the adiababic lapse rate and not by the the intensity of the back radiation. The adiabatic lapse rate was insensitive to the concentrations of greenhouse gases but sensitive to the pressure at Earth’s surface. Thus, contrary to the implied claim of Shore, the existence at Earth’s surface of back radiation of a positive intensity does nothing to invalidate the conjecture of N&Z.
Shore correctly observes that the lapse rate determines the surface temperature only to within an integration constant. If there is an effect of variations in the concentrations of the various greenhouse gases upon the surface temperature then it must be manifested in the numerical value of this constant.
Let this constant be represented by the function f(P, C1, C2…) where the variables C1, C2… designate the concentrations of the various greenhouse gases and let P designate the surface pressure. Let TL(P) designate the result from integrating the adiabatic lapse rate from the surface up into the atmosphere in regions in which convective heat transfer was operative. Let TS designate the surface temperature. The mechanism already described suggests it as approximately true that
TS(P, C1, C2…) = TL(P) + f(P, C1, C2…)
The above equation clarifies the significance of the work of N&Z by separating the surface temperature into the additive effects of a term that varies only with the surface pressure, TL(P), and the residual term, f(P, C1, C2…). The residual term is observable as scatter about the function TL(P).
The work of Nicolov and Zimmer advances global climatology by suggesting: a) a functional form for TL(P) and b) bounds on the magnitude of f(P, C1, C2…) under wide variations in the CO2 concentration. As Shore points out, the conclusions of N&Z are based upon a curve fit in which there were many free parameters but few observations of the values of the variables. When a model is generated in this manner it is especially prone to error. Thus, as always, the conclusions of N&Z should be tested with reference to sufficient observed events not employed in the construction of their model for the statistical significance of their finding (or lack of same) to be demonstrated.

Richard S Courtney
January 1, 2012 1:34 pm

Joel Shore:
Thankyou for your post at January 1, 2012 at 11:12 am that answers my questions; viz.
“Please explain how evaporation “makes the problem worse”. What problem? And how could either an increase or reduction to evaporation fail to provide a possible reduction to it?”
Your answer says;
“What it makes worse is the problem that you are faced with in achieving energy balance in the absence of the atmospheric greenhouse effect. And, the reason why a reduction in evaporation won’t help you is that it can’t be reduced to a negative value.”
Oh dear, NO!
The global energy balance is not relevant to present discussion. You are trying to pretend (to yourself?) that the discussed hypothesis denies the existence of back-radiation: it does not.
And the energy balance does reduce “to a negative value” at the poles.
The tropics are net absorbers of radiation and the poles are net emitters of radiation.
I do genuinely appreciate your answers to my questions because they confirm my suspicion that your “problem” only exists in your imagination.
But, importantly, and of course, none of this consideration of your imaginary problem adds anything to our ability to accept, reject or amend the hypothesis being assessed in this thread.
Richard

D. Patterson
January 1, 2012 2:14 pm

Joel Shore says:
January 1, 2012 at 12:23 pm
D. Patterson: Your comment adds absolutely nothing to the scientific discussion and are thus not worthy of any response.

Your inability and/or flat refusal to consider the errors in your assumptions demonstrates how you are engaging in a non-scientific discussion. The above quote of your statement focuses in part on your comment “the lapse rate in an atmosphere strongly heated from below and cooled from above is close to the adiabatic lapse rate[….].
The problem with your comments is the way in which you erroneously assume the unrealistic conditions of an idealized model exist in the real world. You talk about “lapse rate” and adiabatic lapse rate” as they exist in the idealized model without observing how the real world often does not exhibit such behavior. Now, if that isn’t scientific enough for you, it can only be concluded that you are living on the other side of the mirror in Alice’s Wonderland.

dr.bill
January 1, 2012 2:56 pm

John Day, January 1, 2012 at 1:01 pm :
OK, I guess I’m not done with it. 🙂
John: At the risk of sounding pedantic, let me reiterate that there is no approximation, first-order or otherwise, involved in the definition of temperature in terms of Translational Kinetic Energy. That’s simply what it is, and this has nothing to do with the fact that energy can be absorbed by molecules via their other energy modes, such as rotations and vibrations, which do contribute to the heat capacity, just as do the translational modes, but do not contribute to the temperature while the energy is still in one of those other modes.
None of that, however, precludes the possibility of having energy that is initially absorbed, for example, in a rotational mode, get “converted” to translational energy via various interactions. That is precisely what happens when you heat something in a microwave oven.
My original point, which seems to have been missed altogether, was that this kind of “conversion” might actually be more likely under higher pressure or density conditions in a gas, just as it is in a liquid (which you might think of as a super-dense gas), and could thus be an argument in support of the “enhancement” that Nikolov and Keller are proposing. I don’t know if that is really the case, but to me it points in that direction.
I think we’ve simply been in “violent agreement”. 🙂
/dr.bill

wayne
January 1, 2012 7:00 pm

dr.bill says: (and John Day)
January 1, 2012 at 2:56 pm
My original point, which seems to have been missed altogether, was that this kind of “conversion” might actually be more likely under higher pressure or density conditions in a gas, just as it is in a liquid (which you might think of as a super-dense gas), and could thus be an argument in support of the “enhancement” that Nikolov and Keller are proposing. I don’t know if that is really the case, but to me it points in that direction.
>>
Very good point dr.bill! I missed that somewhere above, and you are correct. This thermalization occurs due to collisions and the number of collisions has to do with density and N&Z are saying that the only free parameter is density in relation to temperature. Now that you, John Day and I are all well versed in this area and appear to be parallel, we should collective help to answer the very next question.
Look at Venus, the pressure is set by the mass, 92 times Earth’s. BUT, density is just 65 times and the molar mass ratio is ~44/29 which does calculate very close to Venus’s mean surface temperature. Why?
The closest I have come is that it seems to have a tie to the “mass extinction coefficient” of substances. The more mass, the more absorption, therefore, more internal energy which will, by equipartition, thermalize, not all as I said above dr.bill, but a ratio related to specific heat, and through density show as temperature for pressure can’t change, and lo and behold, the temperature does match. Why ‘o why does this pressure/density/molar mass end up correct on so many other bodies temperatures?
That tends to say the “mass extinction coefficient” supplies secondarily the energy to have the correct density at all times and every altitude level by first principles. But I can’t seem to tie that down so far.
Let me see if I can summarize in one long string: Assuming a planet’s atmosphere has a preference point any increase in temperature above that would increase the volume and lower the density giving a thinner atmosphere everywhere allowing radiation to escape easier which would lower the temperature. If temperature decreased the opposite would occur. An atmospheric thermostat or a radiative pressure relief vale so to speak. That kind of logic. Now to find if that does in fact occur by simple first principles. I tend to think it is that simple. Physics loves symmetry and simplicity.
Both of you, please help here if possible, we might be able to help others understand this very non-intuitive relation. (or we can just wait a week for N&Z’s more detailed description that might answer that)

G. Karst
January 1, 2012 10:40 pm

Joel Shore says:
January 1, 2012 at 10:11 am
No…What this thread illustrates is that there is almost no limit to the scientific nonsense that people will cling onto in order to believe what they want to believe!…

What I see is serious and knowledgeable people considering the implications implied in the hypothesis.
Consideration is not indicative of the faith required for belief. Faith and belief are internal processes that are best discussed with a therapist or as in your case, a high AGW priest.
Your arguments did sharpen the discussion as your every point was dismantled to it’s arm waving core. Scientific nonsense should be easily demonstrated as such. I am still waiting to read your demonstration (hopefully one that does not include evolution). GK

Steve Richards
January 2, 2012 4:13 am

The is no such thing as back radiation
Heat energy travels from hot to cold.
Lets assume the surface of the earth and the atmosphere close to it are at 288K and the top of the atmosphere is at 3K, There will (excluding temporary anomalies, density and gravity etc.) be a linear fall of measurable temperature from surface to space.
Molecules at 288K will be in contact with adjacent molecules of a lower temperature/energy level. The radiative energy from the 288K collection will transfer energy to these less energetic colleges, increasing their energy level, causing them to increase their radiation.
The newly energised molecules have “used up” so of the original energy.
the newly energised molecules will have a temperature/radiation output somewhat lower than 288K.
This process will continue until all/most of the energy has been used up transferring from molecule to molecule, cooling along the way until the edge of atmosphere is reach and the final, outer layer are at 3K.
Nowhere is energy returning to the surface of the earth!
In this scenario, the earth is warm, space is cold.
If we could, in some way, impede the flow of energy from earth to space, what would happen?
Perhaps by introducing a change in the gas mix, the insulation effect of the atmosphere was enhanced. We would see the temperature of the earth/atmosphere interface increase to a level that gives sufficient increase to the earth/space temperature gradient.
The larger the temperature gradient, the greater the energy/heat flow.
As an aside, as gases warm their thermal conductivity increases, increasing heat/energy flow, whic could help explain some of the self regulating aspects of our planet.
S.

Reply to  Steve Richards
January 2, 2012 12:23 pm

Steve Richards (Jan. 2, 2012 at 4:13 am):
The back radiation is real but is not a heat flux. A few years ago, terminologically sloppy climatologists set off a controversy over this matter by stating incorrectly that the back radiation was a heat flux, prompting critics to point out that for heat to flow from cold to hot matter in the absense of a heat pump would be to violate the second law of thermodynamics. For a while, I thought that the second law violation invalidated AGW as a concept but I then learned that the apparent second law violation resulted only from the mislabelling of a concept by these climatologists.
The confusion can be avoided by assigning the back radiation to the class of vectors to which it belongs. This is the class of vectors that are called “vector irradiances.” At a space point in a radiation field, the radiative heat flux is the vector difference of two vectors. The first belongs to the class of vector radiosities. The second belongs to the class of vector irradiances. In the literature that was left to us by the sloppy climatologists, each of the two vectors is called a “heat flux” while their vector difference (the radiative heat flux) is called the “net radiative heat flux.”
I’ve alerted a number of professional climatologists to the terminological error and the potential for misunderstanding that is created by it without finding a single one of them who is willing to lift a finger to clean up the error.

tallbloke
January 2, 2012 5:30 am

Joel Shore says:
January 1, 2012 at 10:11 am
It does not illustrate that the science is unsettled any more than similar debate on a creationist website illustrates how unsettled the science of evolution is.

The quality of your argumentation doesn’t seem to be evolving. In fact, you seem to be turning into a knuckle dragger. Hoist those hams and block your ears or cover your eyes with them Joel. Best of all, use them to cover your mouth and save yourself further embarrassment.

cba
January 2, 2012 6:14 am

“Steve Richards says:
January 2, 2012 at 4:13 am
The is no such thing as back radiation
Heat energy travels from hot to cold.

Think about it again!
2nd law (and common sense) is referring to net amount of heat flowing from cold to hot doesn’t happen. Net heat only flows from hot to cold. However, radiative transfer, the emission of radiation occurs all the time for objects above 0K and the object radiating has no knowledge of what the temperature is of what might be surrounding the object and absorbing that radiation so the object has no clue it shouldn’t radiate. Hence an object with emissivity = 1 at a temperature of 288k will emit 390 Watts from every meter^2 of its surface according to Stefan’s law. If the object is in a box whose sides have emissivity =1 that is at a temperature of 0k, then the object will radiate 390 W from each square meter of surface area and receive no power radiating back from the box. This will last until the object cools down to 0K and reach thermal equilibrium (which also means less radiation as it cools over time).
If the sides of the box are at 288k, the object will still radiate 390 W /m^2 but since the object is in thermal equilibrium with the sides of the box, there can be NO NET flow of heat in or out of the object as that would change its temperature and would cause the loss of thermal equilibrium. That means the object will receive 390 W/m^2 of power as well as radiating 390 W/m^2.
If the sides of the box are at 500k, the object will still radiate 390 W/m^2 but the sides of the box will be radiating about 1240 W/m^2 and there will be a net heat transfer from the sides of the box to the object until the object has warmed to 500k and it finally is radiating at 1240 W/m^2 and thermal equilibrium has been reached again.
No violation of 2nd law. No change of thermal emission rate for a fixed temperature. No NET flow of heat from cold to hot even though there is power flowing both directions.

tallbloke
January 2, 2012 6:25 am

Joel Shore says:
January 1, 2012 at 7:04 am
cba says:
Joel,
Look at this again! They integrate over a hemisphere (0 – 1 for u= cosTHETA and 0-2pi for phi and they divide by 4*pi – the area value for a unit radius sphere. Theta ranges from 0 to pi/2, not 0 to pi.
Okay…I am beginning to believe that you are right. It looks like they have made multiple errors in their calculation:
(1) The factor of 2 error that you note.

Why not have a go at refuting Hans Jelbring’s similar conclusions, proven using a model earth which does not have the spinning – illuminated from one side issue to deal with:
http://tallbloke.wordpress.com/2012/01/01/hans-jelbring-the-greenhouse-effect-as-a-function-of-atmospheric-mass/

Jim D
January 2, 2012 6:53 am

tallbloke, thanks for the link. Jelbring specifically says that climate change is not caused by mass change which is opposite to the UTC’s (junk) proposition. He also defines Greenhouse Effect as the temperature difference between the radiating level and surface (like Postma did). This is not a good way to remove the effect of greenhouse gases because the radiating level depends on their distribution. If there are none, the radiating level is the surface. If there are GHGs like for earth, it radiates from throughout the atmosphere, depending on wavelength but averaging 5 or more km globally. So, conversely to Jelbring, GHGs do determine temperature via affecting he radiating level, and a higher radiating level means a warmer surface because there is a lapse rate set by convection.

tallbloke
January 2, 2012 7:22 am

Jim, long term climate change would be affected mass changes under Jelbring’s scheme. Non-condensing GHG’s may affect the radiating level. Just not very much…

January 2, 2012 7:30 am

Nikolov and Zeller suggest that there is no back radiation, just the temperature of the air above the surface. I agree and this is why.
We do not need the non condensing GHGs at all in order to set the surface temperature of the atmosphere.
Atmospheric pressure dictates the energy value of the latent heat of vaporisation so it is atmospheric pressure that dictates the rate at which energy can leave the oceans. The more it costs in terms of energy to achieve evaporation the warmer the oceans must become before equilibrium is reached.
So the oceans will build up to whatever temperature is permitted by atmospheric pressure with or without any non condensing GHGs in the air at all.
Once that ocean temperature is achieved the energy for the baseline temperature of the air above the surface is then supplied to the air by energy leaving the oceans and NOT by energy coming in from the sun and especially NOT by energy flowing down from above as so called back radiation.
So the upshot is that the oceans accumulate solar energy until they radiate 390 at current atmospheric pressure, at that point 170 continues to be added by solar but to balance the budget the atmosphere by virtue of its density retains whatever energy is required to achieve balance.
A feature of non condensing GHGs is that they add to the temperature of the air proportionately more than other gases in the atmosphere but in the end it is surface pressure that controls the energy value of the latent heat of vaporisation which is the ultimate arbiter of what rate of energy transfer can be achieved from oceans to air.
So if non condensing GHGs add a surplus over and above that required by surface pressure for equilibrium then the system has to make an adjustment but what it cannot do is alter the energy value of the latent heat of vaporisation in the absence of any change in atmospheric mass or pressure. So instead it is the rate of evaporation that must change to balance the budget in the absence of a significant change in surface pressure. Thus a change in the size or speed of the water cycle removes in latent form any excess energy produced as a result of non condensing GHGs.
There is no back radiation, merely a temperature for the atmosphere just above the surface and it is wholly pressure dependent. That temperature is a consequence not of downward atmospheric scattering of outgoing longwave but simply a consequence of atmospheric density slowing down energy loss first from sea to air and then by separate mechanisms from the air above the sea surface to space.
So if one increases atmospheric pressure at the surface the amount of energy required to provoke evaporation at the sea surface rises and the equilibrium temperature of the whole system rises including the temperature of the air above the surface.
The opposite if one decreases atmospheric pressure at the surface.
We have been looking at back radiation from the wrong point of view. There is no such thing. What we see is simply the air temperature near the surface and it is pressure dependent and not non condensing GHG dependent.

Jim D
January 2, 2012 7:40 am

tallbloke, it may be allowed under his scheme, but he specifically rules it out as an explanation for earth’s climate change in the second sentence of his section 3.
“Climate change is not caused by changes in
atmospheric mass on Earth.”
With no GHGs the radiating level would be the surface and that temperature would need to be 255 K. This is not a small effect.

tallbloke
January 2, 2012 7:53 am

Jim, I suspect Jelbring has short term climate fluctuations in mind when he makes that statement. Hopefully he might clarify the position on my blog
http://tallbloke.wordpress.com/2012/01/01/hans-jelbring-the-greenhouse-effect-as-a-function-of-atmospheric-mass/

Joel Shore
January 2, 2012 7:56 am

G. Karst says:

Your arguments did sharpen the discussion as your every point was dismantled to it’s arm waving core.

Really? Here are the links to my most substantive criticisms of this paper:
http://wattsupwiththat.com/2011/12/29/unified-theory-of-climate/#comment-846787
http://wattsupwiththat.com/2011/12/29/unified-theory-of-climate/#comment-846836
http://wattsupwiththat.com/2011/12/29/unified-theory-of-climate/#comment-847131
I await your links to where these criticisms has been “dismantled”.

Alan D McIntire
January 2, 2012 8:41 am

Used: T = P/ ρ • M/R
Venus Earth Mars
——– ——– ——–
P – pressure 9220000 101325 605 N/m2 (Pa)
ρ – density 65 1.217 0.015 kg/m3
M – molar mass 0.0434 0.02897 0.04334 kg/mol
R – gas constant 8.31451 8.31451 8.31451 J/K/mol
——– ——– ——–
T – temperature 740.40 290.09 210.24 K
A couple of years ago I read a article stating that Earth had a rouglhy 10% denser atmosphere during the Permian Period than it does now , which helped make it warmer than it is now. From that article I thought I could apply the PV =nrT law to get the additional greenhouse effect from an Earth with a denser atmosphere, and Venus’s; surface temperature,. I quickly found a snag in my reasoning,
M, and R are determined constants, P is an input for an atmospher of a given density, but RHO is not determined by initial conditions,
If P is 10% larger and Rho is ALSO 10% larger, you get the same temperature as before.
Venus has a P of roughly 90. What determines Rho? If Rho was 130, Venus would have
a temperature of 370 K.
I suspect that Nikolov is messing around with an identity. .I don’t think his formula tells us how to calculate rho with a given atmospheric composition and density, and a given solar flux.
What would the surface temperature of a planet with a venus type atmosphere at 1/10 the Venus pressure, on a planet 10% more massive than earth, , at the distance of Mars, with a star having 90% the flux of our sun be? Once you have rho you can compute the other figures, and they will match the above calculations- but rho will depend on the stellar flux reaching the surface and on the geenhouse effect of the given atmpsphere.

Richard S Courtney
January 2, 2012 9:09 am

Alan D McIntire:
Please explain the relevance to the Jelbring and Nikolov hyptheses of your post at January 2, 2012 at 8:41 am.
Richard

tallbloke
January 2, 2012 9:20 am

Alan, As I understand it, Nokilov and Zeller use the TOA insolation, not that at the surface.

Ned Nikolov
January 2, 2012 9:44 am

Reply to Alan D McIntire’s comment from January 2, 2012 at 8:41 am
Alan,
You address an issue which we explain at length in our full paper, but it is not discussed in details in our poster. I will elaborate on this in my official reply later this week. Basically, the Gas Law equation ρT = P M / R (Eq. 6 in our poster) cannot be solved for T alone, because we have one equation with two unknown (T and ρ). Solving this requires a SECOND equation, which is provided by the planetary regression curve relating T to P in our Eq. (7) and more specifically by Eq. (8). Combining Equations (6) and (8) with a third equation that defines surface pressure as a function of atmospheric mass (Ma), planet surface area (A), and gravity (g), i.e. P = g Ma / A, results in a 3-equation model with the following chain of causality:
1) Pressure (P) is a function of total air mass, planet surface area, and average gravity;
2) Surface air temperature (T) is a function of TOA solar irradiance and average atmospheric surface pressure (Eq. 8);
3) Near-surface air density (ρ) depends on the mean temperature and atmospheric pressure (Eq. 6);
This implies that air density does NOT control surface temperature, atmospheric pressure does! Instead, density is a function of temperature, which is maintained by pressure and solar heating. This is a critical difference from the concept of Hans Jelbring! That is because the volume (density) of the atmosphere increases or decreases depending on the energy content of the system, while surface pressure is constant. On a planetary scale, the average thermodynamic process at the surface is ISOBARIC in nature (see Section 3.1 in our poster). Hence, temperature is INDEPENDENT of atmospheric composition, while air density is affected by that composition through the molecular mass of air (M)… It all fits together perfectly! Wouldn’t you agree? … 🙂

Joel Shore
January 2, 2012 10:44 am

Ned Nikolov says:

\It all fits together perfectly! Wouldn’t you agree? … 🙂

If you don’t let minor details like conservation of energy bother you!

davidmhoffer
January 2, 2012 10:51 am

Folks, as I watch this discussion I keep seeing people get lost in the details. Stand back and look at the big picture.
N&Z have provided a formula that appears to have predictive skill. One CANNOT falsify it by arguing the details! Sure radiative absorption and re-emission happens in a certain way. Sure convection happens in a certain way. Sure lapse rate works in a certain way.
So What?
If there is one thing we’ve learned over the last few years of the climate debate it is (or should be) that our understanding of the mechanisms and how they interact with one another is woefully incomplete. If we were anywhere near to understanding all the pieces of the puzzle and how they fit together, we’d have climate models with predictive skills coming out the yin yang. But the fact is we don’t.
I liken this discussion to being given a pail full of gravel and being asked to determine the weight of the gravel. I could thoroughly mix the gravel, extract a representative sample, weigh each rock, pebble and grain of sand, extrapolate the expected change in distribution of the rocks, pebbles, and sand from top of the bucket to the bottom of the bucket based on known paramaters for the settling of gravel over time, and from there arrive at an estimate of the weight of the gravel in the pail.
Or I could weigh the gravel and the pail, then pour the gravel out, and weigh the pail.
What N&Z are purporting to do is the latter. One cannot falsify their results by arguing about what the proper distribution of grains of sand is or how gravel does or does not settle when poured into a pail. The only way to determine if they are on to something is to weigh the gravel.
What they have said is that for a given TOA radiance, and a given mean surface atmospheric pressure, they can calculate the average surface temperature of a planet. They’ve even published their predictions for no less than EIGHT planetary bodies!
The only question we should be interested in at this point (it seems to me) is this:
Did they get the surface temps of those planetary bodies right or not?
If no, then their formulas are wrong.
If yes, then it seems to me there are only two possibilities.
1. Their formulas are correct, we just don’t know exactly WHY they are correct.
or
2. They successfully predicted the surface temps of 8 celestial bodies by coincidence.
If the latter, that’s one awfull big coincidence!
So, would it not make sense to dispense with the arguments about the life time of a photon in earth atmosphere, how convection changes with pressure, what absorption bands various gases have and just answer the question:
Did they nail the temps of those planetary bodies? Or not?

Richard S Courtney
January 2, 2012 11:19 am

Ned Nikolov:
At January 2, 2012 at 9:44 am you say:
“This implies that air density does NOT control surface temperature, atmospheric pressure does! Instead, density is a function of temperature, which is maintained by pressure and solar heating. This is a critical difference from the concept of Hans Jelbring!”
Yes, it is a “critical difference”. But it is not obvious and I (for one) missed it. Thankyou for pointing it out.
I write to ensure that your point is emphasised.
Richard

Richard S Courtney
January 2, 2012 11:27 am

Joel Shore:
Your post at January 2, 2012 at 10:44 am says;
“Ned Nikolov says:
\It all fits together perfectly! Wouldn’t you agree? … 🙂
If you don’t let minor details like conservation of energy bother you!”
Please explain in what way you think anything Ned Nikolov wrote contradicts conservation of energy?
Your failure to provide this explanation would show you to be making an even bigger ass of yourself than you have already repeatedly achieved in this thread.
Richard

Arthur Rörsch, The Netherlands
January 2, 2012 11:49 am

Joel
I understand your worry about the conservation of energy. What is coming in as energy from the sun, has to go out at the top of the atmosphere.(TOA) But if the atmosphere as an intermediate between TOA and surface, had gained over time a particular temperature, with a specific density- and temperature- gradient, from surface to TOA, then the law of conservation of energy would not necessarily be violated, if incoming energy from the sun still equals what is going out at the TOA. Could you explain where you see the law violated?
Give another thought the fact that the entropy of the incoming quanta (visible light) is different from the out going (IR). The system as a whole is not in at thermodynamic equilibrium (maximum entropy status). The consequence is that some ‘order’ is maintained. (Like in a living cell). What kind of order would you expect? I guess the maintenance of a gradient. With a higher temperature at the surface than at the TOA.

David Ball
January 2, 2012 11:53 am

Joel Shore, I noticed your arguments were completely dismantled on Judith Curry’s blog. Still singing the same old song though. Hope you are not a representative sample of academic thinking.

Arthur Rörsch, The Netherlands
January 2, 2012 12:33 pm

A side line
IPCC produces a report WG1, named the ‘scientific base’. In the current draft for AR5 it is claimed that there is increasing evidence for CO2 increasing global warming. However, I do not see evidence for that presented in this draft. And no arguments, as presented on this blog, are (of course) mentioned on the disputed ‘greenhouse effect’. Rather then comment on the thousands of issues in the draft, I have chosen for the preparation of a manuscript entitled:
‘A warning, there is insufficient scientific evidence in draft WG1 AR5’ with emphasis on some science philosophical issues which underlie modern natural sciences. (E.g. complexity theory)
I would like to subject this manuscript for peer-review among them who are reading the draft. Just to check whether I made mistakes in my analysis. Who is prepared to referee?
I am not waiting for emotional comments, nor that much ‘facts from figures’ The latter will be produced by critical expert reviewers, which are allowed by IPCC to do so. I want the basic principle of the ‘scientific base’ be discussed .
Arthur@keykey.nl

January 2, 2012 1:00 pm

Terry Oldberg,
Thanks for helping to tidy up terminological issues.
I took your earlier point that both the radiative greenhouse effect and the gravitationally induced greenhouse effect can both be regarded as thermodynamic processes. I picked up a bad example from someone else in attaching the term ‘thermodynamic’ only to the latter.
Your suggested refinement of terminology for the so called back radiation is also very helpful. I had been struggling with a way of saying that it does not exist without being accused of some sort of denialism. Your suggestion now means that I can say that there is no NET downward heat flux (because the net flux is always upward) so that the term ‘back radiation’ is misleading even though there is a downward flow from atmospheric molecules radiating in all directions.
I think N & Z are correct if what they are saying is that what is termed ‘back radiation’ is actually just the temperature of the near surface air molecules AFTER the GHGs higher up have had their effect in reducing the upward flow of energy.

January 2, 2012 1:01 pm

Arthur Rörsch, The Netherlands (January 2, 2012 at 11:49 am):
Shore seems to be claiming that if the intensity of the back radiation increases by Delta F then the intensity of the heat flux into Earth’s surface must increase by Delta F else energy conservation is violated. However, Shore’s claim is falsified if the increase in the intensity of the back radiation is offset by a numerically equal increase in the intensity of the convective heat transfer. A mechanism that produces this result is operative in the atmosphere. It operates because any increase in the intensity of the back radiation increases the local lapse rate. If the local lapse rate grows to exceed the local adiabatic lapse rate, the state of the atmosphere is destabilized by the buoyancy of the local air column with respect to its surroundings. This column floats upward and as it does so, it
restores the local lapse rate to the adiabatic lapse rate. Thus, outside of inversion layers, the lapse rate is set by a natural feedback control mechanism to the adiabatic lapse rate. The adiabatic
lapse rate, though, is insensitive to the composition of the atmosphere. The finding of Nikolov and Zeller suggests the possibility that this feedback control mechanism dominates over fluctuations in
the intensity of the back radiation in setting the surface temperatures of planets with atmospheres.

January 2, 2012 1:08 pm

“This implies that air density does NOT control surface temperature, atmospheric pressure does! Instead, density is a function of temperature, which is maintained by pressure and solar heating. This is a critical difference from the concept of Hans Jelbring!”
Is it a critical difference ?
Higher atmospheric pressure induced by gravity increases density at the surface so that the heat content then rises more than it would have done at a lower pressure when a given amount of solar irradiation is added. It is true that in the first instance the cause is indeed atmospheric pressure but it isn’t far out to say that the temperature rise is caused by density if it is taken as a given that the density was caused by atmospheric pressure.

January 2, 2012 1:14 pm

“Terry Oldberg says:
January 2, 2012 at 1:01 pm”
“The finding of Nikolov and Zeller suggests the possibility that this feedback control mechanism dominates over fluctuations in the intensity of the back radiation in setting the surface temperatures of planets with atmospheres.”
Agreed, just as I have been pointing out for the past 4 years.
However there is a miniscule climate effect because the process of negating the effect of non condensing GHGs involves a change in the speed or size of the water cycle which alters the surface air pressure distribution a fraction.
It couldn’t be measured as against natural variations induced by sun and oceans though.

Richard S Courtney
January 2, 2012 1:40 pm

Terry Oldberg:
At January 2, 2012 at 1:01 pm you say to the venerable and erudite Arthur Rörsch (whose presence here we should all welcome):
“ Shore seems to be claiming that if the intensity of the back radiation increases by Delta F then the intensity of the heat flux into Earth’s surface must increase by Delta F else energy conservation is violated. However, Shore’s claim is falsified if the increase in the intensity of the back radiation is offset by a numerically equal increase in the intensity of the convective heat transfer.”
Etc.
And you then give a detailed explanation of how the falsification occurs.
However, I do not think that can be what Joel Shore is claiming because I completely refuted that to him in my post at January 1, 2012 at 1:34 pm.
So, either
Joel Shore means something other than you think he means
or
Joel Shore is repeating what he knows to be a falsehood.
Therefore, I think we need to pressure him to clarify what he is claiming before trying to answer it. This is why at January 2, 2012 at 11:27 am I asked him;
“Please explain in what way you think anything Ned Nikolov wrote contradicts conservation of energy?”
Richard

Phil.
January 2, 2012 2:44 pm

Myrrh says:
December 31, 2011 at 3:51 pm
Robert Brown says:
December 30, 2011 at 7:19 am
Myrrh said something like “Water is transparent to visible light, therefore, visible light does not heat the oceans, for example.” What this indicates is that Myrrh should not participate in discussions with “paid scientists” unless/until he learns some basic physics.
FWIW, let’s correct this. Water is not transparent to visible light. It is simply more transparent than, say, rock.
Hmm, I prefer to go with the paid physicists who taught me..

Thermal infrared, heat energy, thermal energy on the move direct from the Sun by radiation does have the oomph to move whole molecules into vibration, to raise the temperature , to heat them up.
Those physicists must be spinning in their graves! According to Myrrh less energetic IR has enough ‘oomph’ to move whole molecules but the more energetic visible can only move electrons!

Arthur Rörsch, The Netherlands
January 2, 2012 2:52 pm

It is obvious that Richard C and I are asking for an explanation of Joel why he thinks that the ‘theory ’is violating the first law.

tallbloke
January 2, 2012 3:02 pm
Joel Shore
January 2, 2012 3:03 pm

Richard S Courtney says:

Please explain in what way you think anything Ned Nikolov wrote contradicts conservation of energy?

I’ve explained it here: http://wattsupwiththat.com/2011/12/29/unified-theory-of-climate/#comment-846787
The idea is very simple: The Earth’s surface can’t be at a temperature where it radiates about 390 W/m^2 when when the Earth’ surface plus atmosphere are only absorbing 240 W/m^2 from the sun unless one or more of the following is true:
(1) It is cooling down extremely rapidly, which it is not.
(2) There is some gargantuan energy source that is supplying 150 W/m^2 of power, which nobody has identified could possibly be present.
(3) Some of the radiation emitted by the Earth’s surface is absorbed by the atmosphere.
Hence (3) is the only thing that can explain the Earth’s elevated surface temperature. And, we know in fact that (3) is what is happening because we have satellite measurements, radiative transfer calculations, and measurements of gas absorption lines that all confirm that this is the case.

Your failure to provide this explanation would show you to be making an even bigger ass of yourself than you have already repeatedly achieved in this thread.

The people who are making a**es out of themselves are those skeptics who are not running as fast and furiously away from this nonsense as they can. Young Earth creationists have absolutely nothing on such folks!

tallbloke
January 2, 2012 3:31 pm

“Young Earth creationists have absolutely nothing on such folks!”
Making a donkey of yourself now Joel?
you have missed the point of this theory. I can tell, because you didn’t list alternatives.
4)The temperature boost at the surface due to the presence of an atmosphere constrained by a gravity well.
What you don’t seem to get is that your calcs, which you think all add up nicely to give a radiative answer actually already contain the pressure effect as well as the radiative element. The problem is you don’t understand that the presure element is dominant, and the radiative effect small.
Never mind, given the self conditioning you’ve put yourself through for these many years, you’ll probably ‘get it’ long after everybody else.

Steve Richards
January 2, 2012 3:45 pm

Terry Oldberg says:
“January 2, 2012 at 12:23 pm
Steve Richards (Jan. 2, 2012 at 4:13 am):
The back radiation is real but is not a heat flux. A few years ago, terminologically sloppy climatologists set off a controversy over this matter by stating incorrectly that the back radiation was a heat flux, prompting critics to point out that for heat to flow from cold to hot matter in the absense of a heat pump would be to violate the second law of thermodynamics. For a while, I thought that the second law violation invalidated AGW as a concept but I then learned that the apparent second law violation resulted only from the mislabelling of a concept by these climatologists.
The confusion can be avoided by assigning the back radiation to the class of vectors to which it belongs. This is the class of vectors that are called “vector irradiances.” At a space point in a radiation field, the radiative heat flux is the vector difference of two vectors. The first belongs to the class of vector radiosities. The second belongs to the class of vector irradiances. In the literature that was left to us by the sloppy climatologists, each of the two vectors is called a “heat flux” while their vector difference (the radiative heat flux) is called the “net radiative heat flux.”
I’ve alerted a number of professional climatologists to the terminological error and the potential for misunderstanding that is created by it without finding a single one of them who is willing to lift a finger to clean up the error.”

Thank you for your answer. I am happy that someone agrees that ‘back radiation’ in not heat producing.
However, a quick scan on google on this new term (to me) of ‘vector irradiances’ showed that climate researchers use Cosine Collectors to measure ‘vector irradiances’ which is named ‘back radiation’.
This would appear to me to be a way of squeezing more power into the GHG simulations without violating any fundamental rules of thermodynamics.

Richard S Courtney
January 2, 2012 3:49 pm

Joel Shore:
Your recent post at January 2, 2012 at 3:03 pm proves to all observers that you are so blinded by prejudice that you are incapable of admitting when you are wrong: you keep muttering your error in the hope that repetition will shout down the explanations of your error.
Your recent post says;
“The idea is very simple: The Earth’s surface can’t be at a temperature where it radiates about 390 W/m^2 when when the Earth’ surface plus atmosphere are only absorbing 240 W/m^2 from the sun unless one or more of the following is true:
(1) It is cooling down extremely rapidly, which it is not.
(2) There is some gargantuan energy source that is supplying 150 W/m^2 of power, which nobody has identified could possibly be present.
(3) Some of the radiation emitted by the Earth’s surface is absorbed by the atmosphere.”
Your point (2) was addressed by me at January 1, 2012 at 1:34 pm where I wrote e.g.
“The global energy balance is not relevant to present discussion. You are trying to pretend (to yourself?) that the discussed hypothesis denies the existence of back-radiation: it does not.”
And your point (3) was refuted by me
at December 31, 2011 at 9:39 am and
at January 1, 2012 at 1:34 am and
most directly at January 1, 2012 at 1:34 pm.
Subsequently, Terry Oldberg refuted it at January 2, 2012 at 1:01 pm where he provided a detailed and explicit explanation of why your point (3) is plain wrong.
Both Terry Oldberg and I told you that additional radiation to the atmosphere is not required for energy balance if the additional thermal transport to the atmosphere were provided by enhanced evaporation and conduction from the surface.
Indeed, I responded to Terry Oldberg’s refutation of your point (3) by saying to him:
“I do not think that can be what Joel Shore is claiming because I completely refuted that to him in my post at January 1, 2012 at 1:34 pm.
So, either
Joel Shore means something other than you think he means
or
Joel Shore is repeating what he knows to be a falsehood.”
I now make a public apology to Terry Oldberg. I was wrong to have suggested that Terry Oldberg was mistaken in his implication that you were repeating what you know to be a falsehood: your recent post states that in fact you were repeating what you know to be a falsehood.
And your recent post that I am replying does it again.
Richard

January 2, 2012 3:56 pm

Actually, 3) Some of the radiation emitted by the Earth’s surface is absorbed by the atmosphere.
is just an incomplete version of 4) The temperature boost at the surface due to the presence of an atmosphere constrained by a gravity well.
Joel’s 3) assumes that the energy ‘absorbed’ by the atmosphere is there in the atmosphere because of the radiative characteristics of GHGs.
In fact all or nearly all of it may be there because of gravity compressing the atmosphere to create the lapse rate as per 4)
Joel previously accepted the pressure driven lapse rate as I recall.
What I want to know from a warmist proponent is:
How much of the 150 Wm2 is due to radiative effects of GHGs alone and how much due to gravitational compression of ALL the molecules in the air including Oxygen and Nitrogen ?
Then, how much of the residue of Wm2 is due to CO2 alone ?
Then, how much is due to man’s emissions alone ?
Have warmists done those calculations in the models or have they not ?
Joel says nothing about gravitational pressure here:
“we have satellite measurements, radiative transfer calculations, and measurements of gas absorption lines that all confirm that this is the case.”

Joel Shore
January 2, 2012 5:55 pm

Richard S Courtney says:

Your point (2) was addressed by me at January 1, 2012 at 1:34 pm where I wrote e.g.
“The global energy balance is not relevant to present discussion. You are trying to pretend (to yourself?) that the discussed hypothesis denies the existence of back-radiation: it does not.”

What it claims is that one can calculate the surface temperature knowing only the surface pressure and the solar irradiance. Since it is possible in principle to have an atmosphere that is essentially transparent to IR radiation, this means that an Earth with an atmosphere transparent to IR radiation, having the same surface pressure would have the same surface temperature. And, this notion indeed violates energy conservation.

Both Terry Oldberg and I told you that additional radiation to the atmosphere is not required for energy balance if the additional thermal transport to the atmosphere were provided by enhanced evaporation and conduction from the surface.

As I have explained to you again and again, an earth with a hypothetical IR-transparent atmosphere would be emitting back out into space more radiation than it absorbs. You cannot remedy this by transferring additional energy away from the Earth. It only makes the “deficit” worse.

Joel Shore
January 2, 2012 6:14 pm

Stephen Wilde says:

Joel’s 3) assumes that the energy ‘absorbed’ by the atmosphere is there in the atmosphere because of the radiative characteristics of GHGs.
In fact all or nearly all of it may be there because of gravity compressing the atmosphere to create the lapse rate as per 4)

Stephen: This makes no sense. How is gravity supplying 150 W/m^2 of power? Gravity cannot supply energy unless the gravitational potential energy of the Earth and its atmosphere is decreasing.

How much of the 150 Wm2 is due to radiative effects of GHGs alone and how much due to gravitational compression of ALL the molecules in the air including Oxygen and Nitrogen ?

It is all due to radiative effects. Gravitational compression does not supply energy unless the Earth and its atmosphere are undergoing gravitational collapse so that gravitational potential energy is being converted into other forms of energy. That is not happening.

Tim Folkerts
January 2, 2012 7:35 pm

Let me take a shot at clarification — using a variety of gedanken experiments. I’ll give my conclusions (which have actually sharpened considerably as I wrote this). .
In each case we will start with a barren world with no water (and consequently no clouds), somehow “painted” so that the albedo is 0.3 (absorption = 0.7 for incoming solar radiation) and the IR emissivity is 1.0. I will also assume the rotation rate and thermal conductivity are large enough that temperatures are fairly constant over the surface. This simplifies the discussion with becoming completely unphysical, (At least in my opinion. In any case, this correction could be added later when analyzing in more detail.)
Also, let me define two components that contribute to the temperature of the atmosphere and the surface (ie these combine to produce “the greenhouse effect”).
* RTE (radiative thermal effect) of the atmosphere as the direct effect of the IR absorption/emission in the atmosphere. This effect attempts to bring different parts of the atmosphere to the same temperature.
* GTE (gravitational thermal effect) of the atmosphere, due to changes in potential energy with altitude. This effect tries to create a temperature gradient of ~ 10 K/km as altitude increases (ie the “dry adiabatic lapse rate” = DALR).
1) No atmosphere. I conclude the “average surface temperature” would be ~ 255 K, as required by Stephan-Boltzmann calculations.
2) A pure N2 atmosphere similar in mass to earth’s atmosphere, which we will assume is perfectly transparent. There is no RTE. The GTE will cause the temperature to DROP by 10 K/km STARTING FROM 255 K at the surface.
There is no enhancement of the surface temperature.
3) A thin glass shell at an altitude of 1 km (but no atmosphere). This shell is perfectly transparent to sunlight (wavelength 4 um). A total of 240 W/m^2 is still absorbed, and 240 W/m^2 must radiate. This 240 W/m^2 will all radiate from the glass shell (and thermal radiation from the ground will get blocked by the the shell on the way up). RTE will try
There is STILL no enhancement of the surface temperature.
4) A glass shell AND an N2 atmosphere. Again, the shell will have to be at 255 K. The RTE will try to keep the ground at 255K, but the GTE will try to keep the ground at 265 K (due to the 10 K/km adiabatic lapse rate). The true average temperature must be somewhere in between, and the true lapse rate (TLR) must be 0 < (TLR) < DALR. (This agrees with Joel's intuition and my intuition.) For the sake of argument, assume the TLR is 5 K/km (although I suspect it would be higher); the surface temperature would be 260 K.
An enhancement of the surface temperature requires BOTH RTE and GTE!
4B) Raise the shell to an altitude of of 2km. The shell at 2 km would be at 255 K to radiate enough energy, and the ground would be T = 255 K + TLR*2 = 265 K.
4C) Shells at both 1 km and 2 km. The top shell will still be 255 K (as in 4B). The middle shell will be 255 K + TLR = 260 K. The surface will be 260 K + TLR = 265 K.
Intermediate shells make no difference! Any “saturation of IR absorption” between the ground and the TOA (top of atmosphere) is of little/no importance. Only the radiation from the TOA sets the temperature at the ground in this gedanken experiment.
5) A shell at 2 km, but it only blocks 1/2 of outgoing “earthshine” (more like real GHGs). The RTE will be reduced because there is less IR flying around to equilibrate the land and the TOA, So the TLR will be larger than in (4); let’s call it 8 K/km. Some outgoing IR will come from the ground & some will come from the TOA.
If my numbers are right, then the TOA will be 246.5 K and the surface will be 262.6 K. This is less than before in (4B), which makes sense because we have a reduced RTE ie a reduced “greenhouse effect” from less IR absorption.

—————————————–
NOTE: this is not a “new theory” of climate. It is taking bits of N&Z’s approach and (hopefully) more properly accounting for the need for IR absorption in the atmosphere to obtain a true “GHE”.
Of course, there are endless refinements that could be made, but I think this shows why there is a need for both the “RTE” and “GTE” in the “GHE”.

Joel Shore
January 2, 2012 8:13 pm

Tim Folkerts says:

3) A thin glass shell at an altitude of 1 km (but no atmosphere). This shell is perfectly transparent to sunlight (wavelength 4 um). A total of 240 W/m^2 is still absorbed, and 240 W/m^2 must radiate. This 240 W/m^2 will all radiate from the glass shell (and thermal radiation from the ground will get blocked by the the shell on the way up). RTE will try
There is STILL no enhancement of the surface temperature.

Tim: Actually, I think you start to go astray here. The glass shell is essentially a blackbody shell for terrestrial radiation (and transparent to solar radiation). Hence, what you have is one of the very simplest models of the greenhouse effect, in fact the one shown in Fig. 1 above. The glass shell will radiate 240 W/m^2 up and 240 W/m^2 down. The Earth’s surface will radiate 480 W/m^2 up and hence will be at an elevated temperature of 255K * (fourth root of 2) = 303 K. [Note that because of the lack of an atmosphere, there is no convection to worry about…It is purely a radiative problem.]

Joel Shore
January 2, 2012 8:37 pm

Tim,
I’ll also go on to say what I think happens in 4). In this case, radiation alone will be trying to create a temperature gradient larger than the adiabatic lapse rate…This will actually spark convection which will bring the temperature difference down to the adiabatic lapse rate. The final temperatures will then be obtained from the following three equations for the surface temperature T_s, glass shell temperature T_a, and convective intensity C:
Energy balance at the surface: 240 W/m^2 + sigma*(T_a)^4 = sigma*(T_s)^4 + C
Energy balance at the shell: sigma*(T_s)^4 + C = 2*sigma*(T_a)^4
Enforcement of lapse rate condition: T_a – T_s = 10 K.
The solution to these three equations is: T_a = 255 K, T_s = 265 K, and C = 200 W/m^2. (We could have guessed T_a had to be 255 K because that’s obviously the only way we have radiative balance for the whole system.)
I would note that 4B) no longer is very interesting…You just get somewhat different numerical values than 4A). However, if we raised the shell to, say, 10 km, then we would get the same temperatures that we got in 3) because the lapse rate of ~4.8 K per km would be less than the adiabatic lapse rate and hence would be stable to convection. (Some heat transfer would occur due to conduction through the N_2 but it would be negligible.)

Joel Shore
January 2, 2012 9:02 pm

Tim: Just to note something that is implicit in my discussion above, but is worth stating explicitly: There is no GTE that tries to keep the lapse rate at the adiabatic lapse rate (as should be clear from the parts of the atmosphere that are nowhere near the adiabatic lapse rate, such as the stratosphere). Rather, the adiabatic lapse rate represents a stability boundary for the actual lapse rate. That is to say, the atmosphere is perfectly happy with a lapse rate less than the adiabatic lapse rate. However, if the lapse rate exceeds the adiabatic lapse rate, the atmosphere becomes unstable to convection, which transfers heat up through the atmosphere until the lapse rate is lowered back down to the adiabatic lapse rate.

Tim Folkerts
January 2, 2012 9:19 pm

Dang, you are right Joel. And I had seen that argument before (even made it myself) but seem to have had a mental lapse.
That then begs the question what happens with an atmosphere. The shell by itself wants to be ~ 255 K. The surface wants to be ~ 60 K warmer. The lapse rate still wants to be ~ 10 K/km for the pure N2 atmosphere. So if the shell is at an altitude of 6 km, everything is fine — the ground is ~ 305 K radiating 480 W/m^2 and the shell is ~ 255 K radiating 240 W/m^2 both up and down, and energy is balanced and the lapse rate is where is wants to be.
If the shell is LOWER, then there will be a large temperature gradient, and strong convection set up. If the convection is carrying 100 W/m^2 from the surface to the TOA, then the surface only needs to radiate 480-100 = 380 W/m^2 for an effective surface temperature of 286 K.
If the shell is HIGHER, then there will be a small temperature gradient. The air will be stable and not convect. Conduction will carry a very small amount of energy downward. This would warm the surface a very small amount compared to the shell at the optimal altitude.
If the shell is back at 6 km, but a second shell is added at 12 km, then top shell radiates 240 up and 240 down. The lower shell radiates 240 * 2 = 480 up and 480 down. The ground radiates 240 * 3 = 720 up. This should balance with no serious convection, and a temperature of ~ 336 K at the surface.
OK — now it gets to the point where you need to calculate the IR absorption at each location (altitude, latitude, longitude) along with the convection at each location, which depend in complicated ways on pressure, temperature, chemical composition, humidity, ….
In other words, either you live with overly simple models that gives the gist, or you go for intense computer models. You can’t hope to get a “good” answer with a “simple” model.
My revised conclusions are:
*RTE is real and very important. The “number of layers” is of primary importance, and adding more GHGs will raise the temperature even if the absorption is already “saturated”.
*GTE is real, but a much smaller effect (and fundamentally different that the GTE that many people are discussing), primarily important when the true lapse rate exceeds the adiabatic lapse rate.

Joel Shore
January 2, 2012 9:25 pm

By the way: There is one thing that is puzzling me that I will throw out to Tim and others for their thoughts. I found above that the surface temperature in the case of “no atmosphere” and the surface temperature in the case of an N_2 atmosphere are different (at least if the glass shell is close enough to the surface that convection occurs). A natural question that arises is this: How do things transition between these two cases, i.e., what happens if we have a really tenuous (low density) N_2 atmosphere?
My guess is that one of the two following things must be true:
(1) There is some limit on the rate of heat transfer due to convection, and this limit is lower as one makes the atmosphere more tenuous. This seems reasonable intuitively, but I’d be curious to know what the limit is.
(2) The adiabatic assumption breaks down, i.e., it becomes impossible to satisfy the adiabatic condition if the atmosphere becomes tenuous enough; hence the adiabatic lapse rate is no longer the appropriate stability limit on the lapse rate. This seems like a less likely explanation to me, but I can’t rule it out.

Tim Folkerts
January 2, 2012 9:39 pm

Joel,
It looks like you discussed many of the same points while I was composing my post.
Let me just address two quick points:
“There is no GTE that tries to keep the lapse rate at the adiabatic lapse rate ” (ALR)
I would now say that the “non-RTE” tries to keep the lapse rate near the adiabatic lapse rate. If observed lapse rate is larger than the ARL, then convection will carry energy upward. If the observed lapse rate is smaller than the ALR, then there will be conduction carrying energy downward. (But since the thermal conductivity of the air is so small, this will almost certainly be insignificant (as I now see you discussed — you beat me to it again!) and we can pretty accurately conclude that “the GTE tries to keep the lapse rate from going above the ALR”. )
“However, if the lapse rate exceeds the adiabatic lapse rate, the atmosphere becomes unstable to convection, which transfers heat up through the atmosphere until the lapse rate is lowered back down to the adiabatic lapse rate.”
I would say it is lowered back CLOSE to the ALR. In a steady state situation that would lead to a steep observed lapse rate, if convection could get you back to the lapse rate by cooling the surface sufficiently, then the air would again start to heat near the surface and convection would resume. The stable solution would be a combination of convection and a slightly larger lapse rate than the ALR.
But these are both minor points.

January 2, 2012 11:01 pm

Joel Shore (Jan. 2. 2012 at 3:03 pm):
It seems to me that I refuted your contention that there is necessarily a violation of energy conservation in my post of Jan. 1, 2012 at 1:30 p). In particular and in reference to the estimated energy budget of Trenberth et al (2008), energy is conserved by the various upward and downward energy fluxes that are described by Trenberth and his colleagues. That in this budget there is, at the top of the atmosphere, an upward facing heat flux vector of 239 W/m^2 and that at the bottom of the atmosphere there is a radiative energy flux vector with an intensity of 390 W/m^2 is irrelevant to the issue of the conservation of energy or lack of same..

tallbloke
January 3, 2012 12:16 am

Joel Shore says:
January 2, 2012 at 6:14 pm
Stephen: This makes no sense. How is gravity supplying 150 W/m^2 of power? Gravity cannot supply energy unless the gravitational potential energy of the Earth and its atmosphere is decreasing.

Stephen, allow me to try again.
Joel,
Keep re-reading this until you grasp it:
“Pressure by itself is not a source of energy! Instead, it enhances (amplifies) the energy supplied by an external source such as the Sun through density-dependent rates of molecular collision. This relative enhancement only manifests as an actual energy in the presence of external heating. Thus, Earth and Titan have similar NTE values, yet their absolute surface temperatures are very different due to vastly dissimilar solar insolation. While pressure (P) controls the magnitude of the enhancement factor, solar heating determines the average atmospheric volume (V), and the product P×V defines the total kinetic energy and temperature of the atmosphere. Therefore, for particular solar insolation, the NTE factor gives rise to extra kinetic energy in the lower atmosphere beyond the [expected] amount supplied by the Sun. This additional energy is responsible for keeping the Earth surface warmer than it would be in the absence of atmosphere, and is the source for the observed 44% extra down-welling LW flux in the lower troposphere”
[My italics and I’ve taken out the given quantity so we can concentrate on the concept without getting sidetracked]
Pressure does not have to supply energy ex nihilo to make this work: it is merely responsible for the way it is distributed. Perhaps you’d find it easier to understand if the Authors had used the word ‘more’ instead of ‘extra’, and added the redundant (but in your case seemingly necessary) statement: and less than expected in the upper atmosphere.
Energy is conserved, as it must be. The authors are not stupid people, much as you would like to paint them to be so.
Please treat people with respect.

January 3, 2012 2:11 am

Tim Folkerts said:
“My revised conclusions are:
*RTE is real and very important. The “number of layers” is of primary importance, and adding more GHGs will raise the temperature even if the absorption is already “saturated”.
*GTE is real, but a much smaller effect (and fundamentally different that the GTE that many people are discussing), primarily important when the true lapse rate exceeds the adiabatic lapse rate.”
Here we come to the nub of the issue namely the relative significance of the radiative and gravitational thermal effects.Joel currently denies ANY gravitational contribution even though he previously accepted the contribution of gravity to creating the adiabatic lapse rate. That is an inconsistency on its own but doesn’t matter for this post.
I agree that atmospheric layering is very important and also the GHG contributions to the thermal behaviour of each layer.
We can see that in the effects of changes in ozone amounts in the stratosphere and mesosphere as a result of variations in the mix of particles and wavelengths from the sun wnen the level of solar activity changes.
Apparently the sign of the atmospheric ozone response is the opposite between stratosphere and mesosphere when the level of solar activity changes.
That has an effect on the entire vertical temperature profile of the atmosphere thereby altering ALL the heights and in particulat the height of the tropopause.
I aver that the top down solar effect has a far, far greater effect on the global energy budget than changes in man made GHGs.We can see that in the changes between MWP and LIA and LIA to date.
Likewise there are cyclical variations in the rate of energy release from the oceans which again alter atmospheric heights between surface and tropopause from the bottom up. Again, a far far greater effect than can be aschieved by human GHGs as we can see from the rapid effects of SST changes on troposheric temperatures.
So let us then consider what happens at the surface when the heights change for whatever reason.
Change the height of the tropopause or the gradient in the height of the tropopause between equator and pole and you then change the entire tropospheric pressure distribution and the relative sizes, intensities and positions of ALL the permanent climate zones.
THAT is what adjusts the system back to or nearly back to the lapse rate set by gravitational pressure.It precisely controls the rate of energy flow through the system from surface to space.
The same response occurs WHATEVER the forcing process that tries to push the lapse rate away from the gravitationally induced temperature gradient.
The ENTIRE troposphere simply slides poleward or equatorward in each hemisphere below the tropopause in order to maintain the gravitationally set lapse rate WHATEVER the forces trying to disturb it.
And as I have pointed out the effect of human emissions is unmeasurable compared to the effect of natural variability from sun and oceans.
Climate change on the surface is nothing more and nothing less than the process in action as the system response seeks always to move bacxk towards the gravitationally induced lapse rate.
In that light the significance of the N & Z equations is that they show the system to be highly effective. They show that whatever effect GHGs have is negated by the system response.Indeed the effects of sun and oceans are also negated by the system response but they produce much larger climate zone shifts.
I have been saying that for 4 years now and N & Z provide the quantitative underpinning for my qualitative climate description.

Richard S Courtney
January 3, 2012 2:20 am

Joel Shore:
At January 2, 2012 at 5:55 pm you say to me:
“As I have explained to you again and again, an earth with a hypothetical IR-transparent atmosphere would be emitting back out into space more radiation than it absorbs. You cannot remedy this by transferring additional energy away from the Earth. It only makes the “deficit” worse.”
THE EARTH DOES NOT HAVE “AN IR TRANSPARENT ATMOSPHERE”!
Nobody is claiming it does (except, perhaps, you?)
And
WE ARE DISCUSSING THE REAL EARTH THAT HAS A PARTIALLY IR-OPAQUE ATMOSPHERE.
Please read the post by Terry Oldberg at January 2, 2012 at 11:01 pm. It provides a complete explanation of your gross misunderrstanding.
When you grasp your misunderstanding then you will be able to recognise why you are very, very wrong to repeatedly assert the falsehood that the Nikolov and Jelbring hypotheses contradicts conservation of energy.
Please try to learn.
It is obvious to almost everybody that you have great difficulty learning anything so you search for security by clinging to your prejudices. But, in this case, you really do need to try to learn.
Richard

Richard S Courtney
January 3, 2012 5:12 am

Joel Shore:
I have been trying to think of ways to explain your error to you in another way in hope that I can help you to see your error. I think this may do it.
At January 2, 2012 at 5:55 pm you say to me:
“As I have explained to you again and again, an earth with a hypothetical IR-transparent atmosphere would be emitting back out into space more radiation than it absorbs. You cannot remedy this by transferring additional energy away from the Earth. It only makes the “deficit” worse.”
OK. So, for the sake of argument, I will consider your “earth with a hypothetical IR-transparent atmosphere”.
In this hypothetical case the atmosphere would not absorb any IR and it would not emit any IR.
Therefore, the only heating of the atmosphere would be by conduction from the planet’s surface so the atmosphere and surface would obtain a thermal equilibrium with no net energy flow between them.
Importantly, radiative absorbtion and emission would be to and from the planet’s surface alone.
The absorbtion and emission would be equal because there could not be any radiative heating of the surface from the atmosphere which does not absorb or emit IR.
This is somewhat simplistic because IR is not the only radiation, but for this explanation only IR needs to be considered because the same argument applies to all other electromagnetic wavelengths.
THERE IS NO “DEFICIT”.
Richard

Richard S Courtney
January 3, 2012 5:41 am

Joel Shore:
This is a second part to my attempt to help you understand your error. I did not include it in my first part because this is a clarifying addendum which may have introduced confusion if not kept separate from the basic explanation.
IR is emitted as an energy flux proportional to the fourth power of the temperature of the emitting surface (i.e. the flux is proportional to T^4). And a planet has a wide range of surface temperatures.
A small change to hot planetary surface (e.g. in a tropical region) provides a large change to emitted IR (because the flux is proportional to T^4). But a large change to cold planetary surface (e.g. in a polar region) provides a small change to emitted IR (because the flux is proportional to T^4).
Atmospheric convection transfers heat from the tropics and day-time surfaces to the polar and night-time surfaces.
An average surface temperature of a planet can be obtained by an infinite number of temperature distributions over the surface. Therefore, the average surface temperature can change while the emitted flux of IR energy remains constant (and vice versa). And, thus, the equilibrium average surface temperature of a planet with an IR-transparent atmosphere is governed by atmospheric convection.
Richard

Joel Shore
January 3, 2012 5:45 am

Tim Folkerts says:

I would say it is lowered back CLOSE to the ALR. In a steady state situation that would lead to a steep observed lapse rate, if convection could get you back to the lapse rate by cooling the surface sufficiently, then the air would again start to heat near the surface and convection would resume. The stable solution would be a combination of convection and a slightly larger lapse rate than the ALR.

Yeah…I think you are right. And, this would provide a nice resolution of the issue that is puzzling me ( http://wattsupwiththat.com/2011/12/29/unified-theory-of-climate/#comment-851299 ) because I imagine that the rate of convection would be proportional to the amount by which the lapse rate is exceeded but the proportionality constant would also involve the density of the atmosphere in some way, so as you made the atmosphere more tenuous, the deviation from the ALR would become larger and larger.

Joel Shore
January 3, 2012 5:53 am

Terry Oldberg says:

In particular and in reference to the estimated energy budget of Trenberth et al (2008), energy is conserved by the various upward and downward energy fluxes that are described by Trenberth and his colleagues. That in this budget there is, at the top of the atmosphere, an upward facing heat flux vector of 239 W/m^2 and that at the bottom of the atmosphere there is a radiative energy flux vector with an intensity of 390 W/m^2 is irrelevant to the issue of the conservation of energy or lack of same..

Terry: Yes, the budget balances in Trenberth et al.’s case but that is only because we have an atmosphere that absorbs some of the 390 W/m^2 emitted by the Earth’s surface. If we still had 390 W/m^2 being emitted from the surface and no absorption by the atmosphere, it would be impossible for it to balance. If you think it is possible, then show me.
tallbloke says:

Pressure does not have to supply energy ex nihilo to make this work: it is merely responsible for the way it is distributed. Perhaps you’d find it easier to understand if the Authors had used the word ‘more’ instead of ‘extra’, and added the redundant (but in your case seemingly necessary) statement: and less than expected in the upper atmosphere.
Energy is conserved, as it must be. The authors are not stupid people, much as you would like to paint them to be so.

Sorry, this doesn’t work. The problem is not an issue of energy distribution in the atmosphere. The problem is that a certain surface temperature necessarily leads to a certain amount of radiative emission and, if there is nothing to prevent that radiative emission from escaping to space, then there is no way to prevent the Earth-atmosphere system from emitting energy at a rate higher than it is receiving energy from the sun.

Paul Bahlin
January 3, 2012 6:03 am

There are many lapse rates in the atmosphere, temperature being the ‘default’ one when the term is commonly used. But, there’s a density lapse rate. There’s a pressure lapse rate. And, I would submit there is an energy lapse rate, a portion of which is derivative of the pressure lapse rate.
It baffles me why anyone would think that the paper is advocating the creation of energy from pressure. To me the energy lapse rate creates the temperature lapse rate since temperature is nothing more than a proxy for energy content. No magic has to be invoked to see this.
Each meter in a column of air contains less and less energy as you move away from the surface. If we were talking about a column of steel heated at one end and cooled at the other no one would question a temperature gradient along its length.
Wouldn’t it be true that the pressure gradient sets up an ‘idealized’ energy storage profile and EVERYTHING else that is going on in the real world simply serves as perturbation to this ideal? In fact it could even be that the paper is simply saying that all the various feedbacks combine to seek this ideal.
You can’t invoke the static logic for a can of air on my column of air because it always has an energy differential across its height. You can’t claim an IR transparent atmosphere would equilibrate to no temp lapse rate because it will not ever equilibrate with the warm earth on one end and outer space on the other.

Joel Shore
January 3, 2012 6:05 am

Stephen Wilde says:

Here we come to the nub of the issue namely the relative significance of the radiative and gravitational thermal effects.Joel currently denies ANY gravitational contribution even though he previously accepted the contribution of gravity to creating the adiabatic lapse rate. That is an inconsistency on its own but doesn’t matter for this post.

Actually, it is not an inconsistency at all and your statement above has been helpful to me in realizing exactly what it is that I am saying. It is this — The adiabatic lapse rate matters but rather in sort of the opposite way as people are contending: The radiative effects are what provide the greenhouse effect and the adiabatic lapse rate is what limits the extent to which the radiative greenhouse effect can be offset by convection. So, in other words, if the adiabatic lapse rate were zero, i.e., any temperature decrease with height spurred convection, then the greenhouse effect would basically be canceled out by convection. However, the fact that the adiabatic lapse rate is non-zero is what allows the atmosphere to maintain a temperature profile which decreases with height and hence insures that the radiative greenhouse effect is not canceled out by convective effects (although its magnitude is reduced somewhat).
This also explains where Nikolov has screwed up in Section 2.1B) where he discusses convection: His Equation (4) has put in convection in such a way that it tries to equalize the temperatures T_a and T_s even if they are such that the lapse rate is less than the adiabatic lapse rate. This is WRONG, WRONG, WRONG.

Joel Shore
January 3, 2012 6:16 am

Richard S Courtney says:

In this hypothetical case the atmosphere would not absorb any IR and it would not emit any IR.
Therefore, the only heating of the atmosphere would be by conduction from the planet’s surface so the atmosphere and surface would obtain a thermal equilibrium with no net energy flow between them.
Importantly, radiative absorbtion and emission would be to and from the planet’s surface alone.
The absorbtion and emission would be equal because there could not be any radiative heating of the surface from the atmosphere which does not absorb or emit IR.

You have just proven my point: In a planet without an IR-absorbing atmosphere, you would not get the temperature enhancement at the surface that we call the greenhouse effect. Ergo, this effect is due to the IR-absorbing properties of the atmosphere, not to any “pressure effect”

An average surface temperature of a planet can be obtained by an infinite number of temperature distributions over the surface. Therefore, the average surface temperature can change while the emitted flux of IR energy remains constant (and vice versa). And, thus, the equilibrium average surface temperature of a planet with an IR-transparent atmosphere is governed by atmospheric convection.

But, you have forgotten about Holder’s Inequality, which even Gerlich and Tscheuschner know about. You are correct that the average temperature is not uniquely determined by the amount of power emitted. However, there is a bound on the average temperature…and that bound is that the highest average temperature that leads to the emission of a certain amount of radiative power is that which occurs when the temperature distribution is uniform. From this, it follows that the highest average temperature for a planet with an IR-transparent atmosphere that absorbs 240 W/m^2 (and is essentially a blackbody emitter over the wavelengths of its emission) is 255 K. Any non-uniform temperature distribution emitting this amount of power will have a lower average temperature.

Joel Shore
January 3, 2012 7:21 am

I said:

However, the fact that the adiabatic lapse rate is non-zero is what allows the atmosphere to maintain a temperature profile which decreases with height and hence insures that the radiative greenhouse effect is not canceled out by convective effects (although its magnitude is reduced somewhat).

By the way, since the dry adiabatic lapse rate is proportional to the gravitational acceleration g, this means that people who want to believe that gravity is somehow responsible for the surface being warmer, while wrong, do have a small germ of truth in their idea. The actual fact is that it is the radiative greenhouse effect that is responsible for the surface being warmer; however, gravity (and, in particular, the nonzero adiabatic lapse rate in a gravitational field) is what prevents convection from canceling out the greenhouse effect.

January 3, 2012 7:31 am

Richard S Courtney says:
January 3, 2012 at 2:20 am
To: Joel Shore:
When you grasp your misunderstanding then you will be able to recognise why you are very, very wrong to repeatedly assert the falsehood that the Nikolov and Jelbring hypotheses contradicts conservation of energy.
A simple first law is Q=U+W. sign depends on work done by or work on the system
PV = W
So Joel where is the violation you speak of?

D. Patterson
January 3, 2012 7:43 am

Joel Shore says:
January 3, 2012 at 6:05 am
[….]

You are repeating the same fallacy I referred to before. You are using the unreal conditions of a conjectural model and erroneously attempting to say they represent the real world conditions. There is a real difference between taking the lapse rate of a particular air mass whose temperature decreases with height within an atmosphere versus making the unreal assumption that the temperature profile of a column of air through the Earth’s atmosphere always “decreases with height,” which of course it most certainly does not.

Joel Shore
January 3, 2012 8:12 am

mkelly says:

A simple first law is Q=U+W. sign depends on work done by or work on the system
PV = W

Actually, that should be Delta_U (i.e., change in internal energy), not U. The point is that if you consider the Earth-atmosphere system as a whole and you assume that the Earth has an atmosphere that is transparent to the radiation emitted by its surface, then Q /A*(delta_t) = 240 W/m^2 – 390 W/m^2 = -150 W/m^2, delta_U has to be at least approximately zero on any reasonable timescale…and W is zero. Hence, Q = U + (Delta_U) is not satisfied. [Here, delta_t is the time over which you consider the energy accumulation and A is the surface area of the Earth.]

Joel Shore
January 3, 2012 8:14 am

D. Patterson: I’ve read your comment over 3 times now and I haven’t a clue what you are trying to say.

Ned Nikolov
January 3, 2012 8:18 am

Fellows,
The whole concept of a ‘greenhouse gas’ is somewhat distorted in the mind of the average person and even the average scientist. Most people (including Roy Spencer) seem to think that what makes a GH gas is the molecular structure of the gas. This is only partially true! The other big component is pressure. There is a phenomenon in gas spectroscopy called ‘pressure broadening of absorption lines’. Higher pressure makes any gas absorb more IR due to broadening of its absorption spectrum by reducing the gaps between absorption lines. So, any gas can become a significant GH gas under high enough pressure! This physical fact is not widely known, and rarely emphasized in undergraduate school, which is why most people have this ‘black & white’ image in their minds about what constitutes a GH gas … 🙂
The reality is that N2 and O2 (the major gases in our atmosphere) are not at all 100% transparent to IR radiation. From what I know, the IR opacity of an atmosphere is closely related to (correlated with) total surface pressure (and the vertical pressure gradient), so that there is no such thing as a 100% IR-transparent atmosphere. The IR opacity grows in parallel with pressure, meaning that anytime you have a gas in a gravitational field (i.e. under some pressure), its IR emissivity/absorptivity will always be greater than ZERO! … For example, Mars’ atmosphere is 95% CO2, yet radiative physicists tell us that it’s very ‘leaky’ with respect to IR radiation with a rather weak ‘Greenhouse effect’ due to low overall pressure. In other words, the IR radiative transfer within an atmosphere is regulated by the vertical pressure gradient as much as (or even more than) by composition. Since atmos. pressure is independent of the energy balance (or radiative transfer), it must be considered as a controlling factor of the latter.

January 3, 2012 8:20 am

“Wouldn’t it be true that the pressure gradient sets up an ‘idealized’ energy storage profile and EVERYTHING else that is going on in the real world simply serves as perturbation to this ideal? In fact it could even be that the paper is simply saying that all the various feedbacks combine to seek this ideal.”
Nicely put.
And the N & Z equations (together with real world observations) confirm that negative feedbacks are extremely effective at attaining that ideal.
Hence the constant latitudinal shifting of the global climate zones to continually adjust the lapse rate between surface and tropopause.

davidmhoffer
January 3, 2012 9:01 am

Joel Shore;
You’re losing it bud!
Please allow me to summarize this dogs breakfast of a discussion at the big picture level.
The theory of AGW is founded upon the notion that absorption and re-radiation of earth radiance by CO2 and other GHG’s increases the temperature of the earth. The theory further rests upon the notion that the effect of the GHG’s has an additional “positive feedback” that increases temperatures further by some amount. The debate has condensed into two seminal issues:
1. What is the magnitude of the direct effect of CO2 (and other GHG’s)?
2. What is the magnitude) and sign (+/-) of the feedback effects of CO2?
What we know from direct observation:
1. The combined direct and feedback effects of CO2 have been substantively less than expected which further implies that;
2. The feedbacks are not only less positive than expected, they may actually be negative.
What, at day’s end, are N&Z saying? They are saying that:
1. The combined direct and feedback effects of CO2 are insignificant.
2. The feedback effects are most likely negative, cancelling or nearly cancelling the direct effects of CO2.
3. Given that the net direct and feedback effects of CO2 appear to approach zero, the governing factors remaining in regard to temperature of earth surface are mean insolation at TOA and mean surface pressure.
They have developed forumulas dependant upon mean insolation at TOA and mean surface pressure in order to predict surface temperatures, and appear to have done so accurately. The fact that they can do so suggests that the GHG effects of CO2 etc are in fact negated nearly 100% by feedbacks, and real world observations strongly suggest that this is the case as well.
There is nothing in what they have presented that violates the laws of thermodynamics ad you have claimed. Your refutation of their position by focusing on what happens from the perspective of radiative physics is immaterial unless you can present with certainty what the combined direct and feedbadck effects of GHG’s are and that they are a) significant and b) don’t cancel each other out. You can spout theory all you want, but real world data compared to theory suggests that the net including feedbacks is, in fact, insignificant. Your accusation that they are curve fitting is equally spurious. Yes, in fact, they ARE curve fitting! How else does one derive constants? If you have a problem with this, then might I suggest you review the work of Stefan and Boltzmann in arriving at SB Law.
Curve fitting that shows predictive skill is, in fact, science. Curve fitting that shows no predictive skill but is nonetheless presented as being accurate requires “faith” to merit any consideration and as such lies outside the field of science. You’ve accused others of drifting into the realm of faith based acceptance of facts, but at days end the truth is:
1. The data shows that feedbacks are most likely negative, making the combined direct and feedback effects of GHG’s such as CO2 negtligible and;
2. The formulas arrived at by N&Z to quantify surface temperature show predictive skill.

Joel Shore
January 3, 2012 9:38 am

@Ned Nikolov:
In your latest comment ( http://wattsupwiththat.com/2011/12/29/unified-theory-of-climate/#comment-851749 ), you have introduced two new issues:
(1) Pressure-broadening of the absorption lines of greenhouse gases.
(2) The idea that all gases, even those like N_2 and O_2 whose molecules cannot absorb radiation are in fact able to absorb at higher pressures due to collisional effects.
You are correct about (1): The total atmospheric pressure is relevant for the width of the absorption lines of the greenhouse gases in the atmosphere.
You are technically correct about (2), although the amount of absorption present at Earth pressures makes it basically irrelevant.
So, yes, there are reasons to believe that there ought to be some positive correlation between pressure and “surface temperature enhancement” due to the greenhouse effect. (The third, and probably most important, reason for a correlation to exist is that planets with low atmospheric pressure don’t have much atmosphere at all and hence necessarily don’t have much greenhouse gases. And, planets with high surface pressures have a considerable atmosphere and, unless it is made up exclusively…or nearly exclusively…of non-greenhouse gases, they will have enough greenhouse gases to have a considerable greenhouse effect.)
However, none of this says that the pressure alone quantitatively determines the enhancement. And, in particular, none of it demonstrates that the greenhouse effect is not responsible for the enhancement. In fact, it clearly is, as the effect is necessary to have conservation of energy, and abundant empirical evidence in the case of the Earth (e.g., the emission spectrum observed from space) show that it is indeed the greenhouse effect that is responsible for the temperature enhancement.
In addition, we have pointed out fatal errors in your calculations: (1) Your estimate of the surface temperature in the absence of the enhancement suffers from a number of calculational and conceptual errors. (2) Your calculation that convection cancels out most of the greenhouse effect is wrong because you have put convection into the equation in such a way that it drives the system toward an isothermal profile with height, whereas we know that in reality convection drives the system toward having the profile given by the appropriate (dry or saturated) adiabatic lapse rate. The cancellation of most of the greenhouse effect in your Equation (4) is due to this error.

Joel Shore
January 3, 2012 9:55 am

davidmhoffer: Your latest comment is a hopeless confusion:
(1) We are not discussing the issue of feedbacks. In particular, we are discussing what is responsible for the OBSERVED surface temperature enhancment on various planetary bodies.
(2) Your claims that most of the empirical data supports your notion that the net effect of CO2 (once feedbacks are included) is small not correct, but this is not the appropriate thread to discuss them. Let’s stick to one topic at a time. Don’t try to bring other things into the discussion in a desperate attempt to derail the topic and save a doomed “theory”.
(3) Yes, the Laws of Thermodynamics are violated. Feedbacks are irrelevant: It is simply not possible to have the Earth emitting 390 W/m^2 from its surface when it is only absorbing 240 W/m^2 from the sun unless the atmosphere is absorbing the difference. [And, in fact, we know that the Earth as seen from space is not emitting 390 W/m^2 but only about 240 W/m^2, confirming the fact that the atmospheric absorption is what prevents a violation of conservation of energy…and the spectrum of that emission even agrees with what radiative transfer models predict it to look like.]
(4) A curve fitting exercise only shows predictive skill if it actually PREDICTS, i.e., if it actually can predict the value for the surface temperature of planetary bodies that were not used in deriving the empirical formula. (And, for the reasons I noted above, one would expect a generally positive correlation between pressure and surface temperature enhancement.)
David: I think you are intelligent enough not to be taken in by this nonsense. I would strongly suggest you think a little more before associating yourself in any way with this “theory”.

January 3, 2012 10:29 am

@Ned Nikolov
> There is a phenomenon in gas spectroscopy called ‘pressure broadening of absorption lines’.
> Higher pressure makes any gas absorb more IR due to broadening of its absorption
> spectrum by reducing the gaps between absorption lines. So, any gas can become a
> significant GH gas under high enough pressure!
Yes, higher pressures causes the apparent thickness of spectral lines to increase, and heat absorption increases too. But I believe that this ‘spectral broadening’ is just a macroscopic artifact induced by the increased Doppler shift spread across the distribution of velocities.
At the microscopic level, this is just equipartitioning or “thermalization” caused by increased collisions and more interactions between translational, vibrational and rotational degrees of freedom, ultimately a very non-linear process.
In other words, individual molecules don’t carry barometers in their shirt pockets, and thus have no concept of ‘pressure’. They merely transfer momentum and energy when bumped into by other molecules.

D. Patterson
January 3, 2012 10:30 am

Joel Shore says:
January 3, 2012 at 6:05 am
However, the fact that the adiabatic lapse rate is non-zero is what allows the atmosphere to maintain a temperature profile which decreases with height and hence insures that the radiative greenhouse effect is not canceled out by convective effects (although its magnitude is reduced somewhat).

Your statement,“However, the fact that the adiabatic lapse rate is non-zero is what allows the atmosphere to maintain a temperature profile which decreases with height[….],” is a false statement. The atmosphere most certainly does not “maintain a temperature profile which decreases with height.” A particular lapse rate is not applicable to the Earth’s atmosphere as a whole, so any effort to construct a model based upon such a false assumption can only produce an unreal model and and a false conclusion.

Steve Richards
January 3, 2012 11:38 am

Joel Shore says:
January 2, 2012 at 8:13 pm
The glass shell will radiate 240 W/m^2 up and 240 W/m^2 down. The Earth’s surface will radiate 480 W/m^2 up and hence will be at an elevated temperature of 255K * (fourth root of 2) = 303 K. [Note that because of the lack of an atmosphere, there is no convection to worry about…It is purely a radiative problem.]

If the original energy source provides 240 W/m^2 then any energy level measured within your system will be 240 W/m^2 or less.
How you can imagine 480 W/m^2 appearing is a mystery to many.

Joel Shore
January 3, 2012 12:04 pm

Steve Richards says:

If the original energy source provides 240 W/m^2 then any energy level measured within your system will be 240 W/m^2 or less.
How you can imagine 480 W/m^2 appearing is a mystery to many.

That statement is simply and demonstrably wrong. The equations of radiative transfer for this case are trivial to write down and solve (and they explicitly obey conservation of energy). If you are incapable of doing this, then you are incapable of making pronouncements about it. You are just speaking from ignorance.
If you want to understand via analogy why your claim is nonsense, see here: http://wattsupwiththat.com/2011/12/28/sense-and-sensitivity-2/#comment-848185

Tim Folkerts
January 3, 2012 12:05 pm

I am noting two very interesting and very different approaches to a very complex problem.
One approach (typical of Joel & me in these discussions) follows these lines:

Let’s start from basic science that we know & trust – like the laws of thermodynamics & newtonian physics. Let’s invent some admittedly overly-simplified models that show the key issues – like the idea that molecules in the atmosphere that absorb IR photons do theoretically and experimentally produce warming of the surface. Then add more and more details to the model to try to get closer and closer to the observed conditions on earth — conditions that vary in position and time on large scales and small scales.

The other approach is rather the opposite.

We know that conditions on Earth vary in position and time on large scales and small scales. Accurately predicting this chaotic system from basic physics is a hopeless task, so let’s look instead at observations and look for patterns. For example, ice cores show that CO2 lags the temperature, so elevated CO2 levels are an effect of global warming. Experiment always trumps theory in science, so actual observations — like the flattening of the temperatures for the last decade — shows the basic theory is wrong.

Both are useful approaches. Both provide insights. The challenge is to find a way to meet in the middle. Yes, GHGs and IR radiation affect the temperatures in a very basic and important way. Yes, GHG’s and IR radiation are only a part of the overall equations governing climate.
When people get caught-up focusing only at one end or the other, then understanding suffers.

Paul Bahlin
January 3, 2012 12:05 pm

Here’s a thought experiment for you all…
If you had a column of IR transparent gas (pick a mass if it floats your boat) over a completely dry surface with sunlight pouring in at the top (only) and you were able to isolate it from any horizontal energy influences, what would its temperature profile be? I’m assuming no evaporation and nothing to start convection. Leave it that way for 1000 years. It will obtain some steady state won’t it? What does it look like?
Wouldn’t one end be heated to some temperature by the constant temperature hot surface while the other was constant at 0K? And doesn’t there have to be a temperature lapse rate, else you’ve got to incur some discontinuous temperature change somewhere in the column? Nature hates discontinuities so molecular collisions alone would erode any discontinuity into a nice non-linear lapse rate (column is gaining volume per unit height as it goes up).
Wouldn’t this define the natural state that the atmosphere wants to be in? You still have (once the bottom reaches constant temp) shortwave watts in equal to longwave watts out but you have to have a temperature gradient. The longwave watts out would be a combination of surface radiation and radiation originating throughout the column.

Joel Shore
January 3, 2012 12:08 pm

D. Patterson:

A particular lapse rate is not applicable to the Earth’s atmosphere as a whole, so any effort to construct a model based upon such a false assumption can only produce an unreal model and and a false conclusion.

For heaven’s sake, stop picking nits! As a general rule, the temperature in the troposphere is a decreasing function of height and an average lapse rate cited is 7.5 K per km. If you want to run a full-fledged climate model that computes the temperature at various points on the earth’s surface and heights in the atmosphere, then go ahead. It is intellectually immature and dishonest to play the sort of game you are playing…You are not making any sort of substantive argument; you are just coming up with reasons to believe what you want to believe.

Richard S Courtney
January 3, 2012 12:17 pm

Joel Shore:
In response to my explanation provided for your edification at January 3, 2012 at 5:41 am you have replied at January 3, 2012 at 6:16 am by saying to me:
“But, you have forgotten about Holder’s Inequality, which even Gerlich and Tscheuschner know about. You are correct that the average temperature is not uniquely determined by the amount of power emitted. However, there is a bound on the average temperature…and that bound is that the highest average temperature that leads to the emission of a certain amount of radiative power is that which occurs when the temperature distribution is uniform. From this, it follows that the highest average temperature for a planet with an IR-transparent atmosphere that absorbs 240 W/m^2 (and is essentially a blackbody emitter over the wavelengths of its emission) is 255 K. Any non-uniform temperature distribution emitting this amount of power will have a lower average temperature.”
I am truly astonished that you have the gall to write such twaddle.
I have NOT “forgotten about Holder’s Inequality”. IT IS NOT RELEVANT.
As I pointed out, the planet is heated on its day-side and not on its night-side.
Therefore, THE PLANET IS NEVER AT THE LIMIT set by Holder’s Inequality.
If that is the best excuse you can provide for your sticking to your error then it is certain that you know you are plain wrong.
Richard

Tim Folkerts
January 3, 2012 12:23 pm

Steve Richards says: January 3, 2012 at 11:38 am
If the original energy source provides 240 W/m^2 then any energy level measured within your system will be 240 W/m^2 or less.
How you can imagine 480 W/m^2 appearing is a mystery to many.

How you misunderstand this is a mystery to me.. First of all, these are RATES, not QUANTITIES, so there is no “energy appearing”. But the main point is that there is a balance in all the rates.
I find analogies with money (or water) to be rather effective here, so lets do a very simple analogy. Consider four people named Mr. Sun, Mr GHG, Mr Ground and Mr Space. Every second, Mr Sun pays $240 to Mister Ground (Mr Sun has infinite cash, so he will never run out). Every second, Mr GHG pays $240 to Mister Space and $240 to Mr Ground. Every second, Mister Ground pays Mr GHG $480.
It is very easy to work out that
* Mr Sun loses $240 every second
* Mr GHG holds even
* Mr Ground holds even
* Mr Space gains $240 every second.
There is no “mystery money” that is being created or destroyed. No CPA would have trouble balancing the books.
Of course, these are averages for a whole day.over the whole earth (with no convection and a complete blocking of IR by the GHGs, and a few other simplifying assumptions). A particular square meter of ground will absorb more than 240 W/m^2 during the day, sorting the extra energy in the form of elevated temperatures. At night, the extra energy gets siphoned away as the ground cools. Same thing for the atmosphere.
It is really no mystery at all.

Arthur Rörsch, The Netherlands
January 3, 2012 12:26 pm

The hot ball gedanken experiment.
Versus an ice ball theory.
Why start with a gedanken experiment of a solid planet with a very low surface temperature, warmed solely by radiation from a sun? How did a solid Si, Fe planet came into being in the very cold gaseous cosmos?
Why is the nucleus of the earth still very hot?
Some speculative early cosmology
Probably the earth originated from an other very hot body. Or from a single very large body (at a big bang) that split in a number of other ones, of which one large one became the gravitational centre of a system, (a ‘sun’) with smaller ones (the ‘planets’) rotating around the centre one. The bodies are hot, because they have a particular density in which nuclear fusion occurs, and heaver elements come in to being. The bodies will cool their surface by radiation to space. Whereas the warming from within continues by nuclear reactions. In a ‘sun’ the gaseous state continues to exist and warming from inside will match the radiative cooling. In a ‘planet’ we can assume that the inside nuclear power decreased so that at the surface a solid crust is formed. (Thanks to the radiative cooling to space).
This is of course speculative but let’s assume we have a the end of the early evolution process a solid ball with a surface temperature of a 1000 C, with gravity and consequently a gaseous atmosphere with a temperature lapse rate.. We have no idea where the water originated from, but it would have been at this temperature all in the vapour phase. But the system as a whole (solid planet + gaseous atmosphere) will continue to cool at its top (TOA) producing liquid water which, thanks to the gravity, will not escape to space but return to the surface. The solid planet becomes partly a water planet at its surface. Then continued evaporation at the surface will cool the surface further and enhance the transport of heat from surface to space, through the atmosphere. But it continues to keep the atmosphere warm, without violating laws of conservation of energy. The initial coming into being of a warm atmosphere, with a temperature lapse rate, is just due to the assumption that we start the gedanken experiment with an originally hot planet, instead of a solid ice ball, of which it is not easy to imagine how it could have come into being, consisting of many heavy elements like Fe and Si. in a cold and empty cosmos. It is obvious that if there was water vapour from the beginning, this must have been coming to help to reduce the initial temperature to a value which allows the water to exist at the surface as a liquid.
And apparently an equilibrium state was reached over billions of years, with temperatures between – 40 C and + 40 C, depending on latitude and local insolation..
Why is not the cooling continuing? Here may be brought in the radiative temperature effect (RTE) , causes by the vapour phase and the clouds. But it should be kept in mind that this is the secondary effect of the water cycle that came into being by an evolution process from an initially hot planet with an already warm atmosphere. Then we can see the RTE as a stabiliser for a particular surface temperature and atmospheric temperature lapse rate. The gravitational temperature effect (GTE) is still the cause.
Then two questions arises (1) what would become the equilibrium temperature state at the surface withhout an IR active gas in the troposphere? (2) And what would it be if there is an additional IR active molecule (CO2) present.?
Ad (1) NZ&J theories starts reasoning from the existence of a certain specific surface temperature and claim that it can be argued to arise from the GTE by applying physical laws such as PV=kRT . If so, I see a difficulty if there is no contribution from radiative processes in the troposphere. The surface is, in my opinion, at first sight, than expected, to continue radiation at its emission temperature and will continue to cool. We see it demonstrated strongly (over land) during the night. The surface temperature may fall 10 C, causing an inversion in the lapse rate. The cooling is, however intercepted by the rise of the sun. If the sun does not rise, e.g. during the polar night, the surface temperature falls to – 40 C. During the tropical night (12 h) the troposphere temperature at some altitude above 3 km, does, however not show much temperature change. Probably because the radiative capacity which leads to heat loss, of the gasses is much less than then of solid surface emitting at a broad spectrum. This looks like a support for the NZ&J theories: If the troposphere was holding some heat from the beginning, it will continue to keep the surface warm, despite a low contribution from continued insolation over the surface as a whole.
Or is there still a flaw if we consider the long run over centuries? Where is the radiation emission to space coming from in the system as a whole (surface + troposphere) if there are no IR active molecules in the air? There would not be an TOA with an emissive capacity at a particular emission temperature, different from the surface.. All radiation to space should originate from the surface, and it would continue to cool.
In all these considerations are neglected the processes above the tropopause, in the stratosphere, where the temperature rises again, even to above the surface temperature. We may have to pay more attention to the influences of the stratospheric processes on the troposphere below it, before we can come to a better understanding.
Ad (2) What is the expected effect of an additional active IR molecule like CO2? If we can agree that the surface temperature is mainly established by a combined RTE and GTE effect, cause by the water cycle, and by the important redistribution of heat entering the system through very uneven distribution of insolation, then we need to try to understand these processes on a global scale first, before we can come to an “unified theory of climate”. And come to on conclusion what CO2’s role is in it.
Lastly, I find it surprising how rapidly discussants on this blog are capable to respond to an
argument of somebody else. It takes me an hour to try to understand the way of reasoning of a particular person and then how to digest it in my own view.

January 3, 2012 12:39 pm

Joel Shore says:
January 3, 2012 at 8:12 am
mkelly says:
A simple first law is Q=U+W. sign depends on work done by or work on the system
PV = W
Actually, that should be Delta_U (i.e., change in internal energy), not U. The point is that if you consider the Earth-atmosphere system as a whole and you assume that the Earth has an atmosphere that is transparent to the radiation emitted by its surface, then Q /A*(delta_t) = 240 W/m^2 – 390 W/m^2 = -150 W/m^2, delta_U has to be at least approximately zero on any reasonable timescale…and W is zero. Hence, Q = U + (Delta_U) is not satisfied. [Here, delta_t is the time over which you consider the energy accumulation and A is the surface area of the Earth.]
Joel, I said simple on purpose as there are folks here that read the comments trying to learn. They have limited knowledge on some issues (as most of us do) but everyone can understand equality formula. So, please cut down on your little snipes.
You content that no work is done in the atmosphere by anything ie gravity , DWLWIR, convection?
Also if the atmosphere per you statement “transparent to the radiation emitted by its surface” where did the 390 W/m^2 come from?

Joel Shore
January 3, 2012 12:42 pm

Richard S. Courtney says:

I am truly astonished that you have the gall to write such twaddle.
I have NOT “forgotten about Holder’s Inequality”. IT IS NOT RELEVANT.
As I pointed out, the planet is heated on its day-side and not on its night-side.
Therefore, THE PLANET IS NEVER AT THE LIMIT set by Holder’s Inequality.
If that is the best excuse you can provide for your sticking to your error then it is certain that you know you are plain wrong.

Only the hoplelessly lost or naive will be fooled by what you are saying here. Whether or not the planet is at the limit set by Holder’s Inequality is not relevant. What is relevant is that if you have an INEQUALITY, hence my statement: “From this, it follows that the highest average temperature for a planet with an IR-transparent atmosphere that absorbs 240 W/m^2 (and is essentially a blackbody emitter over the wavelengths of its emission) is 255 K. Any non-uniform temperature distribution emitting this amount of power will have a lower average temperature.”
[By the way, in practice the Earth actually comes pretty close to the limit set by Holder’s Inequality, that is to say, the temperature variation on an absolute scale is such that the difference between averaging T^4 and taking the fourth root OR just averaging T is not that great, as Willis has pointed out by direct calculation using HadCRUT3 data: http://wattsupwiththat.com/2011/12/29/unified-climate-theory-may-confuse-cause-and-effect/#comment-850001 The day-side, night-side issue is a red herring for a planet for which there is enough thermal inertia and advection of heat that the temperature remains fairly uniform. But, again, this is not relevant because all we need from Holder’s Inequality is the inequality, not how close it is to the limit of that inequality.]

davidmhoffer
January 3, 2012 12:56 pm

Joel Shore says:
January 3, 2012 at 9:55 am
(3) Yes, the Laws of Thermodynamics are violated. Feedbacks are irrelevant: It is simply not possible to have the Earth emitting 390 W/m^2 from its surface when it is only absorbing 240 W/m^2 from the sun unless the atmosphere is absorbing the difference. [And, in fact, we know that the Earth as seen from space is not emitting 390 W/m^2 but only about 240 W/m^2, confirming the fact that the atmospheric absorption is what prevents a violation of conservation of energy…and the spectrum of that emission even agrees with what radiative transfer models predict it to look like.]>>>>
Where did N&Z say that the atmosphere doesn’t absorb the difference? In fact, they said the opposite! and yes, as seen from space, in going and out going balance at 240 w/m2. What does the spectrum have to do with it? If the composition of the atmosphere was different, then the spectrum of emission would also be different. And in coming and out going would still equal each other. Don’t you get it? Composition influences the emission spectrum, but NOT the total amount emitted!
Joel Shore says:
January 3, 2012 at 9:55 am
(4) A curve fitting exercise only shows predictive skill if it actually PREDICTS, i.e., if it actually can predict the value for the surface temperature of planetary bodies that were not used in deriving the empirical formula. (And, for the reasons I noted above, one would expect a generally positive correlation between pressure and surface temperature enhancement.)>>>>
Exactly. They DID predict the value of other celestial bodies using only a single equation with two variables. That’s the whole point! As you agree that one would expect correlation between pressure and surface temperature, the fact that their two variable equation has predictive skill without considering composition ought to sound that giant gong inside your head that rings when you have an epiffany regarding the falsity of your world view.
Joel Shore says:
January 3, 2012 at 9:55 am
David: I think you are intelligent enough not to be taken in by this nonsense. I would strongly suggest you think a little more before associating yourself in any way with this “theory”.>>>>
I dumped all over this theory when I first read it. Then I realized that what I have been saying for some time now about calculating energy balance from averaged surface temps instead of averaged fourth root of surface temps applies 100% to what N&Z are saying about how to determine surface temps. I was dumping on them for using in their calculations exactly the same approach I was advocating for a related purpose. In other words, that giant gong in my head went off, I had an epiffany! I started by admitting that my first impression of what they were saying was wrong. Yes, I admitted I was wrong about something!
You could do well to admit the same.

Joel Shore
January 3, 2012 1:12 pm

mkelly says:

Joel, I said simple on purpose as there are folks here that read the comments trying to learn. They have limited knowledge on some issues (as most of us do) but everyone can understand equality formula. So, please cut down on your little snipes.

I don’t see where there were any snipes in my previous response to you. In fact, I thought it was remarkably restrained especially given that our previous interactions have hardly led me to believe that you are someone who can be convinced on even the most basic scientific issues…But perhaps you’ve turned over a new leaf for the New Year.

You content that no work is done in the atmosphere by anything ie gravity , DWLWIR, convection?
Also if the atmosphere per you statement “transparent to the radiation emitted by its surface” where did the 390 W/m^2 come from?

The things you mention do not do work on the Earth-atmosphere system as a whole. In order to explain where an additional source of energy is for the earth-atmosphere system, you have to identify an actual source of additional energy, not just energy being moved around. For example, a ball of gas undergoing gravitational collapse has a source of energy: Gravitational potential energy is being lost by getting converted to other forms of energy. However, the Earth and its atmosphere are not undergoing gravitational collapse.
Besides which, the emissions seen from the Earth by satellites confirm that the Earth + atmosphere as seen from space is not emitting more than ~240 W/m^2. And, its emissions, as one would expect, look like the blackbody emission of an object of ~288 K (that would emit ~390 W/m^2) but with “bites” taken out of the emission spectrum, just where they are expected to be based on our knowledge of the various absorption bands of the greenhouse elements in the atmosphere.
[Also, I am not sure what you mean by “DWLWIR” as my whole point is that one can’t satisfy conservation of energy without invoking the fact that the atmosphere is not transparent to radiation from the surface, but absorbs and emits, some.]
The 390 W/m^2 is the amount of radiation emitted by the surface of the Earth, which emits very nearly as a blackbody of temperature ~288 K.

Phil.
January 3, 2012 1:15 pm

Joel Shore says:
January 3, 2012 at 12:04 pm
Steve Richards says:
If the original energy source provides 240 W/m^2 then any energy level measured within your system will be 240 W/m^2 or less.
How you can imagine 480 W/m^2 appearing is a mystery to many.
That statement is simply and demonstrably wrong. The equations of radiative transfer for this case are trivial to write down and solve (and they explicitly obey conservation of energy). If you are incapable of doing this, then you are incapable of making pronouncements about it. You are just speaking from ignorance.

Indeed, the equation (2) in the OP is in fact derived from doing such an energy balance for a 4 layer model (that’s where the 2/5 comes from). For some reason N & Z fail to point that out.

tallbloke
January 3, 2012 1:34 pm

“The radiative effects are what provide the greenhouse effect and the adiabatic lapse rate is what limits the extent to which the radiative greenhouse effect can be offset by convection.”
In your bad dream Joel.

Joel Shore
January 3, 2012 1:35 pm

davidmhoffer says:

Composition influences the emission spectrum, but NOT the total amount emitted!

Is that supposed to be some new exciting result? Of course the total amount emitted doesn’t change; it is determined by radiative balance. However, the temperature of the surface necessary to have this emission does change.

Exactly. They DID predict the value of other celestial bodies using only a single equation with two variables. That’s the whole point!

No…They used Equation (7) to fit to the data for the celestrial bodies. That equation contains 4 free parameters. Then going from N_TE to T_E seems to involve at least one more free parameter. That makes 5 free parameters fit to 8 pieces of data (and some of the data points, for the planets with very tenuous atmospheres are kind of boring). Hardly impressive that they get a good fit.

I dumped all over this theory when I first read it. Then I realized that what I have been saying for some time now about calculating energy balance from averaged surface temps instead of averaged fourth root of surface temps applies 100% to what N&Z are saying about how to determine surface temps.

They didn’t do this correctly whatsoever. They made a huge conceptual error in assuming that the temperature is determined by the local insolation and then they made some calculational errors in implementing that. Our Earth is actually much closer to the limit of a uniform temperature distribution than the temperature distribution you obtain by considering the local insolation…Willis has showed you this using actual temperature data.

In other words, that giant gong in my head went off, I had an epiffany! I started by admitting that my first impression of what they were saying was wrong. Yes, I admitted I was wrong about something!
You could do well to admit the same.

Your first impression was correct.
And, I plainly admit when I am wrong about something. For example, in this very thread, when cba claimed that Nikolov et al had made a calculational error in determining their T_sb, I said that he was wrong and that they had made only a poor assumption but had implemented their poor assumption for the surface temperature distribution correctly ( http://wattsupwiththat.com/2011/12/29/unified-theory-of-climate/#comment-849077 ). I believed this because Gerlich and Tscheuschner had gotten almost the same numerical answer making the same poor assumption. However, cba persisted that they had in fact made calculational errors and then, after looked more closely, I realized that he seemed to be correct…and I told him so: http://wattsupwiththat.com/2011/12/29/unified-theory-of-climate/#comment-849744

Phil.
January 3, 2012 1:42 pm

Ned Nikolov says:
January 3, 2012 at 8:18 am
Fellows,
The whole concept of a ‘greenhouse gas’ is somewhat distorted in the mind of the average person and even the average scientist. Most people (including Roy Spencer) seem to think that what makes a GH gas is the molecular structure of the gas. This is only partially true! The other big component is pressure. There is a phenomenon in gas spectroscopy called ‘pressure broadening of absorption lines’. Higher pressure makes any gas absorb more IR due to broadening of its absorption spectrum by reducing the gaps between absorption lines.

Not ‘any gas’ only those that have absorption lines in the frequency range of interest, N2, O2 and Ar do not have such lines in the emission range of the Earth’s surface so there’s nothing to broaden. CO2 does have many such lines which are substancially broadened, this is the reason why the Martian atmosphere doesn’t have much of a GH effect, since its absorption lines are very narrow due to the lack of broadening. Below is an illustration of the difference between Mars and Earth for a portion of the CO2 spectrum.
http://i302.photobucket.com/albums/nn107/Sprintstar400/Mars-Earth.gif
The reality is that N2 and O2 (the major gases in our atmosphere) are not at all 100% transparent to IR radiation. From what I know, the IR opacity of an atmosphere is closely related to (correlated with) total surface pressure (and the vertical pressure gradient), so that there is no such thing as a 100% IR-transparent atmosphere.
No that’s why I use N2 gas as the diluent in my FTIR spectrometer, because it’s transparent in the IR!
The IR opacity grows in parallel with pressure, meaning that anytime you have a gas in a gravitational field (i.e. under some pressure), its IR emissivity/absorptivity will always be greater than ZERO!
Only if it absorbs in the first place!
… For example, Mars’ atmosphere is 95% CO2, yet radiative physicists tell us that it’s very ‘leaky’ with respect to IR radiation with a rather weak ‘Greenhouse effect’ due to low overall pressure. In other words, the IR radiative transfer within an atmosphere is regulated by the vertical pressure gradient as much as (or even more than) by composition. Since atmos. pressure is independent of the energy balance (or radiative transfer), it must be considered as a controlling factor of the latter.
The pressure is a secondary effect.

Phil.
January 3, 2012 1:44 pm

tallbloke says:
January 3, 2012 at 1:34 pm
“The radiative effects are what provide the greenhouse effect and the adiabatic lapse rate is what limits the extent to which the radiative greenhouse effect can be offset by convection.”
In your bad dream Joel.

No Joel’s right, it’s your bad dream.

Joel Shore
January 3, 2012 2:23 pm

I said to Ned Nikolov here( http://wattsupwiththat.com/2011/12/29/unified-theory-of-climate/#comment-851821 ):

(1) Your estimate of the surface temperature in the absence of the enhancement suffers from a number of calculational and conceptual errors.

Correction: I no longer believe that there are calculational errors in T_sb. It seems only to be a conceptual issue, what I would call a poor approximation, to assume that the local (in space and time) insolation alone determines the local temperature.

Joel Shore
January 3, 2012 2:31 pm

tallbloke says:

“The radiative effects are what provide the greenhouse effect and the adiabatic lapse rate is what limits the extent to which the radiative greenhouse effect can be offset by convection.”
In your bad dream Joel.

It would be helpful if you would actually explain WHY you believe this is wrong using correct physics that actually obeys laws like Conservation of Energy. So far, we haven’t seen that from you…The only semi-coherent statement we got from you was easily shown to be nonsense that didn’t address the point: http://wattsupwiththat.com/2011/12/29/unified-theory-of-climate/#comment-851644

January 3, 2012 2:33 pm

Tim Folkerts says:
January 3, 2012 at 12:23 pm

It is very easy to work out that
* Mr Sun loses $240 every second
* Mr GHG holds even
* Mr Ground holds even
* Mr Space gains $240 every second.
There is no “mystery money” that is being created or destroyed. No CPA would have trouble balancing the books

I wonder if you would do me a favour by extending your analogy a little.
Add a 2nd Mr GHG to stand alongside the 1st Mr GHG
Start passing the money around.

January 3, 2012 2:48 pm

.
> CO2 does have many such lines which are substancially broadened,
> this is the reason why the Martian atmosphere doesn’t have much
> of a GH effect, since its absorption lines are very narrow due to
> the lack of broadening.
You are implying that these broadened spectral lines somehow enhance the ability of CO2 molecules to capture more IR energy, as if the increased line width corresponded with some kind of “cross section” property, which is somehow a function of pressure.
But ‘pressure’ per se doesn’t exist at the microscopic molecular level. Pressure is a macroscopic property of an atmosphere, which appears to be a scalar field: a unique real-number value at each x,y,z location, with a Markovian flavor, such that nearby locations tend to have the same pressure.
But microscopically, such a scalar field doesn’t exist. You might say ‘speed corresponds to pressure’, but that’s not the same because at that level ‘speed’ is a distribution, not a scalar field. Some molecules move fast, some are slow. A collision can transfer all of the momentum from a fast molecule to a stationary molecule. So reversing their “pressures”?
Pressure corresponds to the average of these speeds at the macro level. Individual molecules know nothing about this ‘average speed’.
So, in reality, the line spreading is caused by a Doppler shift at the receiving instrument, due the molecules traveling to and from the instrument. This shift is not detectable by the molecules themselves, in the same sense that you can’t perceive the Doppler shift of a train whistle if you’re riding on the train.
Then why does Mars, which has 30 times more CO2 per unit of surface area than Earth, have a tiny GHE?
You are correct that it’s because of reduced pressure. But it has nothing to do with “line widths”. It’s because the reduced density (1% of Earth) reduces the opportunities for collisions with other molecules, thus preventing the vibrational energy from the absorbed IR photon diffusing as heat.
I think I’m reading here that the N&Z theory doesn’t account for this absorption, but I will wait for the expanded paper before making any further remarks about that.

davidmhoffer
January 3, 2012 3:23 pm

Joel Shore;
Willis has showed you this using actual temperature data.>>>
No he did not.
He calculated P using T averaged and raised to the power of four and then compared to P calculated by T raised to power of 4 and then averaged.
As the two numbers were close to one another (they differed by about 6 w/m2 which is actually huge in the context of the climate debate, he concluded that the two methods were roughly comparable. In brief, he made less of a mistake, but he still made a mistake, as I pointed out to him in my reply.
In order to arrive at a number of any value, one must not only take the average of T raised to the power of 4, one must also do it over time. The data Willis used was (I believe, I have to admit I did not check for certain) annual.
Annual data is the average of monthly T. Monthly T is the average of daily T. Daily T is the average of hourly T. Given that T can vary by 20 degrees in a single day, and in the spring and fall months, daily average can swing by 30 or 40 degrees in temperate zones, taking the average of annual data raised to the power of 4 is nothing more than taking the proper mathematical approach to annual data which is already wrong in the first place!
Really Joel, you are fooling yourself to no end on this issue. There are legitimate criticisms to be made of N&Z, but I’m not going to shout about them until I see the full explanation in the follow on article they have promised. That said, if you truly think there are problems, you haven’t managed to put your finger on any of significance, and your rebuttal amounts to no more than criticizing them for things you claim they said which they did not.

January 3, 2012 4:00 pm

Joel Shore (January 3, 2012 at 5:53 am):
To understand what I’m saying, you need to think about the change in the energy budget of Trenberth et al (2008) in which, at a point in Earth’s surface, the intensity of the back radiation increases by Delta F. If this is the only change, then the intensity of the absorption of IR by the Earth must increase by Delta F for energy conservation. However, if in addition the intensity of the convective heat transfer increases by Delta F, energy conservation ensures that the intensity of the absorption of IR at Earth’s surface does not change.
There is a naturally occurring mechanism which, in theory at least, automatically increases the intensity of of the convective heat transfer by Delta F. This mechanism is the feedback control mechanism in which the lapse rate serves as the variable that is under control and the adiabatic lapse rate serves as the set point.

Joel Shore
January 3, 2012 4:16 pm

Terry Oldberg says:

To understand what I’m saying, you need to think about the change in the energy budget of Trenberth et al (2008) in which, at a point in Earth’s surface, the intensity of the back radiation increases by Delta F. If this is the only change, then the intensity of the absorption of IR by the Earth must increase by Delta F for energy conservation. However, if in addition the intensity of the convective heat transfer increases by Delta F, energy conservation ensures that the intensity of the absorption of IR at Earth’s surface does not change.

Terry:
(1) This does not in any way get around the issue regarding conservation of energy that I have identified.
(2) This sort of thing is well-known, although you may have misconceptions about how it operates. For example, if you read “Global Warming: The Hard Sciecne” by L.D. Danny Harvey (Box 3.1 on pp. 41-42) he does a little calculation comparing the effect of changing the radiative transfer between the atmosphere and the surface by an certain amount vs changing the radiative emission back out into space. And, you are right that the former is compensated almost completely by a change in convection. This occurs to the extent that a 10 W/m^2 change in radiative transfer between the atmosphere and surface only warms the surface by about 0.1 C. Unfortunately, however, the same is not true of a 10 W/m^2 change in the radiative forcing at the top of the atmosphere, as would occur via a change in greenhouse gas concentrations…That produces a much larger temperature change because it cannot for the most part be compensated for by a change in convection.

Joel Shore
January 3, 2012 4:28 pm

davidmhoffer says:

In order to arrive at a number of any value, one must not only take the average of T raised to the power of 4, one must also do it over time. The data Willis used was (I believe, I have to admit I did not check for certain) annual.

David: Fine, so make other realistic estimates for what the temperature distribution is and you will still see it is a small different and basically irrelevant to what we are talking about here. Really, you are making a mountain out of a molehill. By contrast, you are giving Nikolov a free pass in making the truly horrific approximation that the global average temperature can be calculated using a local temperature calculated from the local (in space and time!) insolation. You want to talk about unrealistic approximations!
And, the fact is that this is the least of the problems with Nikolov’s “theory”. The bigger problems include
(1) The fact that he showed the greenhouse effect mainly goes away under convection by putting in convection in a completely incorrect way…i.e., he assumed the atmosphere is being driven to an isothermal state with height. Under that assumption, nobody who understands the greenhouse effect would expect otherwise! The fact that convection can never even nearly get rid of the lapse rate is the sole reason why it can’t largely cancel the greenhouse effect.
(2) Any version of his theory that claims that the surface temperature enhancement is not due to the radiative greenhouse effect violates conservation of energy AND a mountain of empirical evidence and a half century of radiative transfer theory that has been successfully used to study the empirical data…basically most of the field of remote sensing! (He, and some of his proponents, seem to be trying to back-pedal on this, so I no longer understand what the theory actually says.)

Richard S Courtney
January 3, 2012 4:38 pm

Joel Shore:
Your behaviour seems to be wilfully misleading. You would need to be an idiot to fail to see the direct contradiction of yourself by yourself in your posts to me. I shall spell it out so it is plain for all to see.
You suggested consideration of an imaginary planet with an atmosphere containing no GHGs.
At January 3, 2012 at 5:41 am I explained how that imaginary planet would lose heat from its hottest surface regions and gain heat at its coolest surface regions because its atmosphere would transport heat by conduction and convection. And I explained why this would result in a change to the planet’s average surface temperature.
At January 3, 2012 at 6:16 am you gave a ridiculous reply saying to me:
“But, you have forgotten about Holder’s Inequality, which even Gerlich and Tscheuschner know about. You are correct that the average temperature is not uniquely determined by the amount of power emitted. However, there is a bound on the average temperature…and that bound is that the highest average temperature that leads to the emission of a certain amount of radiative power is that which occurs when the temperature distribution is uniform.”
And at January 3, 2012 at 12:17 pm I replied to that saying;
“I am truly astonished that you have the gall to write such twaddle.
I have NOT “forgotten about Holder’s Inequality”. IT IS NOT RELEVANT.
As I pointed out, the planet is heated on its day-side and not on its night-side.
Therefore, THE PLANET IS NEVER AT THE LIMIT set by Holder’s Inequality.”
At that point any reasonable person would have agreed you had written nonsense but not you.
At January 3, 2012 at 12:42 pm you have the brass-neck to write:
“Only the hoplelessly lost or naive will be fooled by what you are saying here. Whether or not the planet is at the limit set by Holder’s Inequality is not relevant. What is relevant is that if you have an INEQUALITY, hence my statement: “From this, it follows that the highest average temperature for a planet with an IR-transparent atmosphere that absorbs 240 W/m^2 (and is essentially a blackbody emitter over the wavelengths of its emission) is 255 K. Any non-uniform temperature distribution emitting this amount of power will have a lower average temperature.”
Say what!!
The transfer of heat from hottest to coldest regions will REDUCE temperature differences across the planet.
So, at January 3, 2012 at 6:16 am you rightly said;
“the highest average temperature that leads to the emission of a certain amount of radiative power is that which occurs when the temperature distribution is uniform.”
I pointed out that convection would make the temperature distribution more uniform.
But later on the same day, at January 3, 2012 at 12:42 pm, you say;
“Any non-uniform temperature distribution emitting this amount of power will have a lower average temperature.”
And you bolster that contradiction of yourself by saying to me;
“Only the hoplelessly lost or naive will be fooled by what you are saying here.”
The Moderators would snip my only appropriate response to that.
Then you try to obfuscate the matter further by mention of the real Earth.
YOU WERE THE ONE WHO INSISTED ON CONSIDERATION OF A PLANETARY ATMOSPHERE WITH NO GHGs.
It seems you are incorrigible.
Richard

Joel Shore
January 3, 2012 4:56 pm

Richard S Courtney:
My posts are there for people to read without your willful misinterpretation of them. Willis Eschenbach is trying to explain the same thing to people in the other thread on the subject ( http://wattsupwiththat.com/2011/12/29/unified-climate-theory-may-confuse-cause-and-effect ) If you want to cement your position in the neanderthal wing of the “AGW skeptic” movement, there is not anything I can do to stop you.

January 3, 2012 5:11 pm

Joel Shore (January 3, 2012 at 4:16 pm):
Your claim that “This does not in any way get around the issue regarding conservation of energy” is falsified by the fact that energy is conserved. To prove that your claim is falsified is an exercise in 3rd grade math, as it requires only an ability to add and subtract. For a person like yourself who has a PhD in physics, this exercise ought to be duck soup.

Joel Shore
January 3, 2012 5:22 pm

Terry,
I am interested in science…Not pseudo-logistical semantic word games.

Reply to  Joel Shore
January 3, 2012 9:30 pm

Joel Shore (January 3, 2012 at 5:22 pm):

You’ve switched the issue under debate from violations of energy conservation to me; allegedly, I am guilty of “pseudo logistical word games.” This is an example of an “ad hominem argument.” Like all ad hominem arguments, this one is logically fallacious.
Before you switched the topic to me, the issue under debate was your claim that “This does not in any way get around the issue regarding conservation of energy.” I say that your claim is falsified by the law of of energy conservation. I’ve proved the falsification of your claim mathematically. Are you able to respond to this issue or do you need to change the topic in order to avoid defeat?

dr.bill
January 3, 2012 5:32 pm

I ordinarily stick to Physics and Math in my comments here, but I cannot help but notice that Joel Shore appears to have given up his entire life, seemingly 24 hours per day, to attempt a refutation of the Nikolov and Zeller work. He’s everywhere, all the time, on every blog where there’s a thread discussing the topic.
It seems to me that if the work of Nikolov and Zeller were truly such a piece of crap, it would not be necessary to attack it like a dog foaming at the mouth. It would die a normal death in due course after the actual papers have been presented and reasoned rebuttals had been presented. So why is that?
Why the rush? Why the vehemence? Why the foaming?
/dr.bill

Ned Nikolov
January 3, 2012 10:35 pm

Bill,
I think I can explain why the rush may be to discredit our work:
1) The radiative GH theory was the ‘only game in town’ until now, and although there is virtually no empirical support for it (i.e. evidence outside computer models), it was still a game that everyone, warmist and skeptic alike, was forced to play. Most skeptics currently are only arguing the degree of CO2-induced warming and hardly anyone dares to question its principle existence or physical reality .. One of our papers analyzes the CO2-temperature relationship using several global paleo-climate data sets over a number of time scales going back 65.5M years (the entire Cenozoic era). We found no empirical evidence that CO2 has ever affected Earth’s climate throughout history! The data analysis showed that CO2 changes have always lagged (followed) temperature changes and this lag increases exponentially with the time scale of the data set reaching 12.2M years at the scale of ocean sediment records covering the past 65M years… So, from the standpoint of empirical evidence, the radiative GH theory is on very shaky grounds. Maybe that’s why the mainstream science measures the GH effect only by the amount of absorbed outgoing IR radiation with little reference to its temperature effect. That’s because IR absorption is easy to measure and argue for, but it provides no proof for the physical cause of the GH effect. AND as shown by our study, that absorption is irrelevant …
2) We now have an alternative theory of the GH effect based on an irrefutable 160-year old Gas Law. This new theory has quite powerful predictive skills, so much so that it can accurately estimate the average temperature on hard-surfaced planets throughout the entire solar system, something that even 3D climate models have a challenge with. AND it does it by using only two parameters – TOA solar irradiance and mean surface pressure! …
I think you’d agree that those two sets of facts present a deadly combination by putting the current GH theory in a true chess-mat situation … 🙂

dr.bill
January 3, 2012 10:58 pm

Ned Nikolov, January 3, 2012 at 10:35 pm :
That’s pretty much what I was implying in my “questions”, Ned, but thanks for the summary. The alarmists should be happy. Now they finally have a real reason for wetting their pants. ☺
All the best with your work,
/dr.bill

davidmhoffer
January 4, 2012 12:44 am

Ned Nikolov;
This new theory has quite powerful predictive skills, so much so that it can accurately estimate the average temperature on hard-surfaced planets throughout the entire solar system, something that even 3D climate models have a challenge with. AND it does it by using only two parameters – TOA solar irradiance and mean surface pressure!>>>
Nicely summarized.
It took me a while to get my head wrapped around your theory because I did exactly what Joel shore and your other detractors in the various threads did. I made the mistake of trying to add up all the tiny details into a cohesive whole. Then I had my aha! moment. We don’t need no stinkin’ details!
I suggested an analogy in another thread that I think bears repeating. We’ve been asked to determine the weight of gravel in a bucket. You’ve taken the obvious approach, weighed the bucket with the gravel in it, then poured the gravel out, and weighed the bucket. Simple arithmetic gives us the weight of the gravel from those two numbers. What are we hearing from the nay sayers?
o well, you’ve got a formula that substracts the weight of the pail from the weight of the gravel plus the pail. But gravel is made up of sand, pebbles and rocks. Those three categories can be broken down into small, medium, and large for each, giving us at least 9 variables governing the weight of the gravel. There’s no way one can arrive at a model with any predictive skill with 9 variables.
o well, I’ve taken a sample of the gravel, I’ve calculated the weight distribution of gravel sorted into particle size and added them up and your result is impossible.
o well, you have no way of proving that the weight of the pail before you poured the gravel out and after you poured the gravel out are the same.
o well, I’ve determined from the lack of documentation in your paper that the method you used to pour the gravel out is not definitive, and you may be measuring residual gravel that failed to leave the pail during the emptying procedure.
o well, I repeated your experiment with a pail of gravel but with the results adjusted for gravity on various planets and got a different answer every time, proving that weight is a function of gravity, a factor which your formula doesn’t take into account. you’ve obviously confused mass and weight and your results are thus suspect.
Seriously nay sayers, you’ve wandered off into some sort of land of magical excuses. You cannot argue that N&Z are wrong based on how you THINK radiative, conductive, and convective processes work. Why? Because we’ve been trying to apply how we THINK those things work to climate models for years and years and ALL the climate models have proven useless!
The climate models are useless because they are based on how we think the minutia works. If we actually knew how the minutia worked, the climate models would be correct. But the climate models are not correct, proving that we DO NOT understand the minutia. Given that we DO NOT understand the minutia, arguing that N&Z are wrong because their method is falsified by the minutia that we DO NOT UNDERSTAND is ludicrous.

Richard S Courtney
January 4, 2012 2:06 am

Joel Shoire:
Your resort to name-calling in your post at January 3, 2012 at 4:56 pm is a demonstration that you know I have shown you are wrong.
I ask all observers to read my post at January 3, 2012 at 4:38 pm and your immediately following post at January 3, 2012 at 4:56 pm then to decide for themselves which of us is guilty of “willful misinterpretation”.
Richard

January 4, 2012 4:31 am

Richard S Courtney said:
http://wattsupwiththat.com/2011/12/29/unified-theory-of-climate/#comment-849873
“Firstly, the IPCC said, and you quote from the link I provided,
“About twice as much warming (0.2°C per decade) would be expected if emissions were to fall within the range of the SRES marker scenarios.”
The emissions HAVE fallen “within the range of the SRES marker scenarios” so my statement was and is correct.”
No, it isn’t and wasn’t. You said “But the IPCC AR4 said “committed warming” (i.e. warming certain to occur because it was already ‘in the pipeline’) would be 0.2 C per decade”. In fact the IPCC said “Committed climate change (see Box TS.9) due to atmospheric composition in the year 2000 corresponds to a warming trend of about 0.1°C per decade over the next two decades, in the absence of large changes in volcanic or solar forcing.”, as I said. You were wrong. Why can’t you admit you were wrong? It’s not exactly ambiguous.
“Secondly, as you say, “he” did say “the warming was .18C/decade since Jan 1999 (according to UAH)”.
But – in fact – that warming was NEGATIVE and not positive. Even this warmist site shows about -0.18C or less.”
No, it wasn’t. He said specifically that from January 1999 there has been .18C/decade of warming according to UAH, and that is exactly what the record shows.
http://www.woodfortrees.org/data/uah/from:1999/to:2012/trend
“#Least squares trend line; slope = 0.0175143 per year
1999 0.0274169
2011.92 0.253643”
It did not show a cooling at all over the last 13 years, and your link doesn’t support that claim either. One wonders why you thought posting it would help your case.
So nothing you said was correct.

Keith Gordon
January 4, 2012 4:35 am

Keith Gordon says.
It seems to me this new theory has great potential. The current AGW theory has some consensus but, this only condemns it to just a hypothesis, If it were universally accepted there would be no need for a consensus, that is why there is no consensus for E=mc2, because it has been tested and universally accepted. Both sides of the “climate change” debate have some sort of consensus, both remain as a hypothesis in the eyes of the other. Only enough indisputable proof can change each others position,
Why then is there so much potential in this new theory, firstly it is simple and beautiful (most good theories are) it is also testable on other rocky planetary bodies in the Solar System with atmospheres, it purports to explain past and future climates on earth, I think it will explain the faint sun paradox, it seems to explain the shortcoming’s of AGW and Sceptical theory, it explains why Co2 levels didn’t drive past climate, It explains why the Sun is the main driver of climate, as we all knew, It could be the basis of everything we don’t know and everything we do know about climate, and wouldn’t that be an amazing and wonderful thing. Give this theory a chance, nothing I have heard so far in 550+ comments makes me think it wrong, if it is, it will get found out, and if there is only a reasonable consensus for it, it remains just a theory. Thank you to authors and good luck with your work. I await your next post with extreme interest.
Keith Gordon

Joel Shore
January 4, 2012 6:25 am

Terry Oldberg says:

Before you switched the topic to me, the issue under debate was your claim that “This does not in any way get around the issue regarding conservation of energy.” I say that your claim is falsified by the law of of energy conservation. I’ve proved the falsification of your claim mathematically. Are you able to respond to this issue or do you need to change the topic in order to avoid defeat?

Let me explain why it is a semantic word game: You seem to be claiming that there is no way not to have conservation of energy because we have the Law of Conservation of Energy. However, this is a word game because I am not saying that energy is not conserved in the real world. What I am saying is that it would be impossible to have conservation of energy in a world where you had a planet like Earth but with an atmosphere transparent to terrestrial radiation and still had the elevated average surface temperature of 288 K.
The reason that conservation of energy would in fact be obeyed as it must is that such a planet would not have such an elevated surface temperature, but would rather have an average surface temperature of 255 K or less (the less because it depends on the temperature distribution with a uniform distribution yielding the 255 K average temperature). Ergo, the elevated surface temperature is due to the fact that, in our actual Earth, we have elements in the atmosphere that absorb terrestrial radiation. Although this follows from conservation of energy, there is also abundant empirical evidence confirming it, including the fact that the Earth as seen from space only radiates ~240 W/m^2 rather than the ~390 W/m^2 being radiated by its surface, and the fact that the spectral distribution of this radiation is in good agreement with radiative transfer calculations based upon the known absorption spectra of the atmospheric constituents.

Joel Shore
January 4, 2012 6:43 am

dr bill says:

I ordinarily stick to Physics and Math in my comments here, but I cannot help but notice that Joel Shore appears to have given up his entire life, seemingly 24 hours per day, to attempt a refutation of the Nikolov and Zeller work. He’s everywhere, all the time, on every blog where there’s a thread discussing the topic.
It seems to me that if the work of Nikolov and Zeller were truly such a piece of crap, it would not be necessary to attack it like a dog foaming at the mouth. It would die a normal death in due course after the actual papers have been presented and reasoned rebuttals had been presented. So why is that?

Bill, this is a good question (which others…my girlfriend, and physics colleagues) have often asked when Ithey learn what I do on the web, but your presumptive answer is way off the mark! The reason I have dedicated so much time to this is most likely a combination of the following:
(1) I have a lot of free time at the moment because we are on break…and this is a good distraction from whatever non-pressing work I could be doing.
(2) A certain obsessiveness, best summarized by what I think is perhaps the best cartoon ever: http://xkcd.com/386/ Arguing against nonsense on the internet really does have an addictive nature to it. And, as a physicist, I hate seeing physics abused, which is why I have spent a lot of time of these “theories” such as Nikolov’s and various “the greenhouse effect doesn’t exist theories” like Postma’s. A more rational part of me thinks that I should just let the AGW skeptics who endorse such nonsense make complete fools of themselves and probably do more to discredit the movement than I can with reasoned scientific debunkings, but it is hard to go with that part.
By the way, if you think that your and Ned Nikolov’s explanation of why I am spending so much time debunking this “theory” is correct, how do you explain why the most scientifically-literate skeptics like Roy Spencer, Willis Eschenbach, and even (although I hesitate to put him in this category) Monckton are similarly trying to point out its obvious fatal flaws? As “AGW skeptics”, surely they should be embracing it if they think it is anything but nonsense.

G. Karst
January 4, 2012 8:36 am

Joel Shore says:
January 4, 2012 at 6:43 am
…surely they should be embracing it if they think it is anything but nonsense.

Repetition does not strengthen your arguments. Throwing the word “nonsense” into a group of people trying to critically analyze a hypothesis is deliberately insulting those people’s intellectual labor. You should realize – that is inflammatory and useless argumentation. This is a falsifiable hypothesis (unlike most AGW hypothesis/conjecture), and therefore can pass through normal scientific process. It doesn’t rely on post-normal, pseudo science. Your proper skeptical viewpoint has been duly noted. Now why not apply the same skepticism to the post-normal – “CO2 is evil and will kill us all meme.”
It has been clearly shown, that this hypothesis is NOT nonsense. It has NOT been shown that the hypothesis is correct. Validity and correctness are two different parameters. The hypothesis is valid, it remains, to be determined, whether it is correct or not.
Citing people who disagree is a very weak argument, and may be indicative, of misunderstanding the assertions of the hypothesis. Give it some more critical thought and time. If the hypothesis violates the conservation of energy (I disagree), then the hypothesis will die a sudden and complete death soon. Vindication and nose rubbing rights, will be yours. Happy days. GK

dr.bill
January 4, 2012 9:08 am

Joel Shore, January 4, 2012 at 6:43 am :
Hi Joel,
Well, at least you haven’t responed with “I AM NOT FOAMING !!!”, but xkcd notwithstanding, if you’re that obsessive, I’m surprised you still have a girlfriend. I also don’t believe you, but that’s just me. Pay no attention… 🙂
To answer your question briefly, I am also a physicist (Theoretical Condensed Matter), and after reading (several times) the pdf of the Nikolov and Keller poster that I got at tallbloke’s place, I think that their ideas have merit, and should be examined and explored more fully. Nothing more complicated than that. They might be in error, but their errors, if any, have not been identified yet, and are certainly not contained in any of the things I’ve seen you write in response to it.
On the other hand, you’ve written so much, and in so many places, in so few days, that I doubt that anyone has read even half of it. At some point, so much activity starts to look foolish and desperate, even if it were to contain something of merit.
Happy foaming,
/dr.bill

Joel Shore
January 4, 2012 9:11 am

Richard S Courtney says:

I ask all observers to read my post at January 3, 2012 at 4:38 pm and your immediately following post at January 3, 2012 at 4:56 pm then to decide for themselves which of us is guilty of “willful misinterpretation”.

Of course, Richard wants you to read only his sophistry and where I finally got fed up with his repeated sophistry and refused to continue to engage him in his nonsense. He does not want you to read what I wrote in previous posts responding to him, like here:
http://wattsupwiththat.com/2011/12/29/unified-theory-of-climate/#comment-852024
and here:
http://wattsupwiththat.com/2011/12/29/unified-theory-of-climate/#comment-852129

Ned Nikolov
January 4, 2012 9:26 am

Joel,
The professional (PhD) skeptics are very similar in their thinking to professional AGW supporters in one respect – they are quite knowledgeable about the details involving radiative transfer and various aspects of atmospheric dynamics. However, that knowledge of details overwhelms their holistic perception, so that they oftentimes cannot see the forest for the trees. This phenomenon is observed not only in atmospheric/climate science, but in all fields of science! It’s part of human psychology rooted in the workings of our minds… For example, if you read carefully Roy Spencer’s response to our paper, you’ll note that he does not address any of the central points/findings of our study such as the amazing predictive skill of Eq. 8, the 3 major problems of the current GH theory outlined in Section 2.1, or the implications of the Gas Law for planetary climates discussed in Section 3.1. Instead he spends all his time discussing energy balance issues and putting forward theoretical arguments about how pressure could not have a thermal enhancement effect, while the Gas Law and analysis of actual planetary data clearly show that it can! … Knowledge is power, but it can also be an obstacle in seeing new paradigms! This is why major breakthroughs in science (especially new theories) have mostly been developed by ‘outsiders’, i.e. people who have a broader overall knowledge, but are not part of the ‘establishment’ in a given discipline, and are not ‘handicapped’ by technical details … With respect to our particular topic, I was surprised to see that prominent researchers had difficulty with one of the most fundamental premises in thermodynamics that higher pressure would always result in a higher equilibrium temperature for the same amount of absorbed energy … I’m addressing this issue in more detail my official reply …

Ned Nikolov
January 4, 2012 9:40 am

Fellows,
If I may, I would like to invite everyone participating in this discussion to restrain from resorting to personal attacks. What we are discussing here should have nothing to do with our personalities or private lives. The discussion is strictly about the physical truth, and no one should take any remarks made here personally … I believe that one of the highest virtues in science is the ability to stay ‘detached’ from (not fully identified with) one’s own views and theoretical concepts, so that when new evidence comes along, one could easily transition to a new concept. .. After all, no theory is ‘final’ … 🙂

January 4, 2012 9:45 am

Joel Shore says:
January 3, 2012 at 1:12 pm
I forgot to say thanks for the response.
However, since PdV is W (work) and work is done on/at the boundary I believe you are incorrect in your W is zero statement. As work is a path function and not a property the work done by gravity may not be offset by the work done by convection. Further, if IR can do no work then I find it hard to accept that it can increase the T of the earth’s surface.

Joel Shore
January 4, 2012 9:51 am

Ned Nikolov says:

Most skeptics currently are only arguing the degree of CO2-induced warming and hardly anyone dares to question its principle existence or physical reality.

Lots of skeptics have dared to question this and your paper in fact bears a lot of resemblance to the previous arguments that have been made, e.g., by Gerlich & Tscheuschner, the “Slaying the SkyDragon” book, and Joseph Postma. Yet, no serious climate scientist has endorsed these “theories” (including skeptics like Roy Spencer, Richard Lindzen, and Willis Eschenbach) because it is obvious that they are incorrect, just as yours is (and, indeed, both Spencer and Eschenbach seem to think this too).

We found no empirical evidence that CO2 has ever affected Earth’s climate throughout history! The data analysis showed that CO2 changes have always lagged (followed) temperature changes and this lag increases exponentially with the time scale of the data set reaching 12.2M years at the scale of ocean sediment records covering the past 65M years

Personally, I doubt you can resolve the temperature and CO2 data this accurately as you go back further in time…and given the elementary errors you have made in your “theory”, there is not reason to believe you have done any better with paleoclimate data. That said, it is true that our current situation is somewhat unique: In the past, only natural processes could increase CO2 concentrations in the atmosphere; In the current situation, we have an entirely new phenomenon where sentient beings can take carbon that has been locked away from the atmosphere for millions of years and very rapidly (on geologic timescales) release it into the atmosphere.

So, from the standpoint of empirical evidence, the radiative GH theory is on very shaky grounds. Maybe that’s why the mainstream science measures the GH effect only by the amount of absorbed outgoing IR radiation with little reference to its temperature effect. That’s because IR absorption is easy to measure and argue for, but it provides no proof for the physical cause of the GH effect.

Yes, it does provide “proof” (to the extent you can ever have proof in science, which is inductive, not deductive): The only way that one can satisfy energy conservation when the Earth’s surface emits ~390 W/m^2 while the Earth and atmosphere absorb only ~240 W/m^2 from the sun is to have ~150 W/m^2 of that ~390 W/m^2 be absorbed by the atmosphere. And, this is exactly what the satellite data shows is happening and in exactly the way that radiative transfer models using the empirically-measured absorption lines of the atmospheric constituents predict it happens.

AND as shown by our study, that absorption is irrelevant …

How have you done this? You certainly haven’t done it by incorporating convection correctly into a model of the greenhouse effect. Instead, you’ve incorporated it in a way that drives T_a and T_s to be equal. In reality, convection only drives the temperatures to the point where the lapse rate is equal to the appropriate adiabatic lapse rate. And, any elementary book on climate science (such as Ray Pierrehumbert’s) would emphasize that the greenhouse effect relies vitally on there being a lapse rate so that the level of the atmosphere where radiation successfully escapes to space is colder than the surface. For example, on p. 148 of Ray’s book, he says, “The key insight taken from this discussion is that the greenhouse effect only works to the extent that the atmosphere is colder at the radiating level than it is at the ground.” You’ve eliminated that possibility by allowing convection to drive the two temperatures to equality, in contrast to what elementary thermodynamics tells us will happen!
And, you certainly haven’t done this by fitting data for 8 celestial bodies (several of which exhibit almost no surface enhancement) to a model with 4 or 5 free parameters.

We now have an alternative theory of the GH effect based on an irrefutable 160-year old Gas Law.

Do you honestly think you are the first climate scientists to apply the Ideal Gas Law to the atmosphere? Read a textbook on climate science, for heaven’s sake!

This new theory has quite powerful predictive skills, so much so that it can accurately estimate the average temperature on hard-surfaced planets throughout the entire solar system, something that even 3D climate models have a challenge with. AND it does it by using only two parameters – TOA solar irradiance and mean surface pressure! …

And, your model has, by my count, 5 free parameters. (I am sure that 4 of them are free, since you state as much.) You haven’t predicted anything…You have just fit some data. And, for a reason that you noted (absorption line-broadening) and a reason I noted (celestrial bodies need an atmosphere with some significant pressure to have any gases, and thus a significant amount of greenhouse gases), one expects there to be a generally positive correlation between surface temperature enhancement and surface pressure. So, once you have several adjustable parameters to play with, it is not surprising that you can get a good fit to the data.
My prediction of two things that we will not see in your final paper:
(1) We will not see any detailed physical explanation of how one could possibly get a surface temperature enhancement (above the value predicted by the Stefan-Boltzmann equation for a UNIFORM temperature distribution) without the greenhouse effect and still obey conservation of energy. We will not see a detailed explanation of this because it is impossible to do so.
(2) We will not see any correction of the way that you incorporated convection into the simple greenhouse effect model in Section 2.1 above. This is because a correct incorporation of convection would not give the desired result of essentially eliminating the greenhouse effect. (We also won’t see any acknowledgement of the fact that correctly-formulated radiative-convective models are already used to actually do quantitative calculations of the greenhouse effect.)

Reply to  Joel Shore
January 4, 2012 11:51 pm

Joel Shore (Jan. 4, 2012 at 9:51 am):
You assert that “The only way that one can satisfy energy conservation when the Earth’s surface emits ~390 W/m^2 while the Earth and atmosphere absorb only ~240 W/m^2 from the sun is to have ~150 W/m^2 of that ~390 W/m^2 be absorbed by the atmosphere.” As Richard Courtney and I believe we have proved on numerous occasions in the past couple of days, this assertion is false. Rather than endlessly repeating your counter claim, why don’t you attempt a proof that it is true?
Hint
While the vector of intensity 390 W/m^2 is a flux of energy, it is not a flux of heat. On the other hand, th vector of intensity 240 W/m^2 is a flux of heat. By its definition in thermodynamics, the “heat” is the energy that crosses the referenced boundary .At Earth’s surface the heat flux is the vector sum of a number of different vectors, only one of which has an intensity of 390 W/m^2. Once the additional vectors are taken into account, the apparent violation of energy conservation is falsified. Climatologists have adopted an abberant definition of the word “heat” under which it it not necessarily the energy that crosses the referenced boundary. In doing so, they have quite possibly created the AGW scare.

wayne
January 4, 2012 10:09 am

davidmhoffer says:
January 4, 2012 at 12:44 am
It took me a while to get my head wrapped around your theory because I did exactly what Joel shore and your other detractors in the various threads did. I made the mistake of trying to add up all the tiny details into a cohesive whole. Then I had my aha! moment. We don’t need no stinkin’ details! >>>
There you go David, seems it is sinking in.
The only thing I had still in question was on the matter of density, it being the only dependent variable when you look at a temperature at a given layer and point within an atmosphere, by ρT = constant. But I have since come across a comment made by Ned that relieves me at bit, that this explanation in more detail will be forthcoming in his explanations this week.
You’re right, standing back at the planetary scale there are few details necessary. Any details will be more intra-atmospheric variances or weather type event since our planet is never in local (portions of the planet) equilibrium though it is at a planetary scale. The later, weather event variances in the atmosphere we largely already know. It is the intra-atmospheric variances, that is the specific energy at each layer and altitude which is pretty much still wide open and Dr. Miskolczi’s papers are the first I ever saw that is touching on those empirically. That is why I do like his papers even though there might be a question on the correctness here and there. You have heard me talk of that before.
You are good at the talking, spread it. It will fall down there.

Joel Shore
January 4, 2012 10:28 am

Ned,
I see that our posts have crossed, so let me briefly address the scientific part of your latest post:

For example, if you read carefully Roy Spencer’s response to our paper, you’ll note that he does not address any of the central points/findings of our study such as the amazing predictive skill of Eq. 8, the 3 major problems of the current GH theory outlined in Section 2.1, or the implications of the Gas Law for planetary climates discussed in Section 3.1.

Predictive skill: I’ve discussed that in my most recent post.
Section 2.1(b): I’ve discussed that in my most recent post.
Section 2.1(a): I’ve discussed that but just to re-iterate, your calculation of surface enhancement convolves two different things. One is the true enhancement of the power that the surface emits above that which the body (including its atmosphere) absorb. The other is the fact that for any given emitted power, there are an infinite number of temperature distributions that will give that emitted power. Extremely broad distributions, such as the one that you have proposed by assuming that at each point on the surface emits at the same rate as it is receiving radiation from the sun (i.e., no heat flow & no heat storage), will give lower average temperatures because of the upward curvature of T^4 vs T (or Holder’s Inequality, if you want). Thus, it is probably better to talk about the enhancement in emitted power than the enhancement in average temperature. (The 33 K number quoted for the magnitude of the greenhouse effect is derived by assuming a uniform temperature distribution…and, indeed, the current earth’s actual temperature distribution is close enough to uniform that you get close to the same result for emitted power as you get using a uniform distribution.)
Section 2.1(c): Frankly, this argument is just confused. For one thing, while the heat capacity of the atmosphere may be small in comparison to the ocean, it is not THAT small. (E.g., you can calculate the amount of time it would take the atmosphere to cool down by a certain amount if it were not absorbing any energy and it takes some significant time.) For another, there is simply no reason whatsoever why the flux of radiation from the atmosphere has to be less than that received from the sun. (I have made an analogy with aluminum recycling whereas Tim Folkerts has made one with money here: http://wattsupwiththat.com/2011/12/29/unified-theory-of-climate/#comment-852001 )
Section 3.1: This is again just very confused. Yes, there are implications that follow from the ideal gas law and other thermodynamic considerations, such as the adiabatic lapse rate. However, these do not result in an increase in surface pressure on a planet where there is no atmospheric absorption of IR emissions from the surface. On such a planet, the average temperature (or, more precisely, the average of T^4) is set by radiative balance considerations and you’d get the temperature of the atmosphere by working up from the surfacxe. On a planet with atmospheric absorption of IR emissions from it’s surface, the average temperature at the “effective radiating level” in the atmosphere is set by radiative balance considerations and then the surface temperature follows from the lapse rate and the altitude of this effective radiating level. The effective radiating level (i.e., the level at which radiation can successfully escape to space without being absorbed) depends, of course, on the composition of the atmosphere.
The reason that Roy Spencer has chosen not to address the confusions above is probably because it is a bit painful to wade through the errors you have made, and your “theory” can be dismissed on the much simpler grounds that he has discussed.

Phil.
January 4, 2012 12:04 pm

Ned Nikolov says:
January 3, 2012 at 10:35 pm
Bill,
I think I can explain why the rush may be to discredit our work:

No-one’s trying to discredit your work just pointing out your mistakes, better to get them sorted out now than by the reviewer if you submit a paper.
2) We now have an alternative theory of the GH effect based on an irrefutable 160-year old Gas Law. This new theory has quite powerful predictive skills, so much so that it can accurately estimate the average temperature on hard-surfaced planets throughout the entire solar system, something that even 3D climate models have a challenge with. AND it does it by using only two parameters – TOA solar irradiance and mean surface pressure! …
You have done no such thing, you’ve fitted an equation with multiple constants and showed that you can recover the numbers you started with, you’ve demonstrated no predictive ability at all.
While you’re here perhaps you’ll address a few points:
Your equation 2 shows the limits of 0 to 1 for the integration wrt μ, it should obviously be from -1 to 1 which by symmetry is double the value of the integral you show. Perhaps you could explain where this factor of 2 has gone in your derivation (this point has been raised by others here but you haven’t addressed it).
Mysteriously you claim a perfect fit to the observed surface temperature of the Moon at 153K, which is far from the observed value, indeed it’s fairly close to the minimum temperature reached on the night side! Where did you get that value? The fact that it is exactly the same as the value obtained from your calculation is surprising!
The claimed similarity between Figs 5 and 6, is only striking if you ignore the axes! Especially since the exponent in the Poisson relationship is only appropriate for atmospheres which are composed of air, those which consist of mainly CO2 will have quite different exponents.
“Hölder’s inequality Tgb ≪ Te” This should be ‘less than or equal to’.
There are other problems which have been brought up by others but some answers to these questions would be appreciated.

Richard S Courtney
January 4, 2012 12:11 pm

Joel Shore:
I take extreme umbrage at your statement at January 4, 2012 at 9:11 am which says of me:
“Of course, Richard wants you to read only his sophistry”
I have posted nio “sophistry” and your only reason for suggesting I have is that I showed you were plain wrong and that you contradicted yourself.
Apologise then go away.
Richard

Ned Nikolov
January 4, 2012 12:19 pm

Joel,
You are experiencing the cognitive “blocking effect” of a wrong mental paradigm. You need to take off your ‘radiative transfer’-coated glasses and look at the GH phenomenon from an entirely different perspective. In my reply (currently in progress), I try to ‘force’ the reader to do just that. Without a shift in perception, one could never understand our new theory …
For starters, consider the possibility that the 390 W m-2 emitted by the surface is THE RESULT of a higher surface temperature (compared to that of an airless gray body) caused by solar heating and air pressure. In other words, what’s the chance that near-surface IR exchange is caused by the temperature rather than causing the temperature?

G. Karst
January 4, 2012 12:54 pm

I am not sure why, but this debate reminds me of the early arguments on electrical flow. Of course, as humans, we get everything ass backwards, at first. We just are not very good at assigning + or – to our narrow calculations. Check any battery markings for verification. GK

Ned Nikolov
January 4, 2012 1:04 pm

Phil,
It’s not the perfect fit of a curve to some points in itself, which is significant, but the IMPLICATIONS that this fit carries, and that requires a change of glasses as I explained in my reply to Joel above …
To your question – I’m addressing the issue about Moon’s true temperature at length in my official reply. I will only say here that the IR temperature mapping of the Moon surface conducted by NASA’s Lunar Reconnaissance Orbiter Diviner from 2009 to 2011 show that the Moon is MUCH colder than 255K on overage, and the mean diurnal temperature of the surface is very close to our theoretical estimate of 154K! … What’s even more amazing is that NASA’s astrophysicists had already a model predicting accurately Moon’s temperature back in 1999! This model has now been verified by Diviner’s IR measurements… This raises the question, where have the climate scientists been for the past 12 years not realizing that our moon is much much colder that their simple (and mathematically incorrect) application of the S-B equation suggests? … In fact, the Diviner data show that there is NO latitude on the Moon surface, where the average diurnal temperature is even close to 255K. Even on the lunar equator, the mean temperature does not exceed 210K and it drops to about about 95K at the poles … You do the math!

Joel Shore
January 4, 2012 1:20 pm

Phil. says:

While you’re here perhaps you’ll address a few points:
Your equation 2 shows the limits of 0 to 1 for the integration wrt μ, it should obviously be from -1 to 1 which by symmetry is double the value of the integral you show. Perhaps you could explain where this factor of 2 has gone in your derivation (this point has been raised by others here but you haven’t addressed it).

That particular issue is not a problem. See my post here, and Arthur’s nice paper in particular, for details: http://wattsupwiththat.com/2011/12/29/unified-climate-theory-may-confuse-cause-and-effect/#comment-852111
There is an issue, however, with defining an average surface temperature T_sb by assuming a temperature distribution to be given by requiring local energy balance at each point in space and time (i.e., ignoring heat storage or heat flow). It creates an artificially large “surface enhancement” to be explained, most of which can be explained away by having a uniform temperature distribution. That is why the scientific community attributes a rise of the surface temperature 33 K to the natural greenhouse effect on earth, not 133 K. The other 100 K can be obtained by making the surface temperature uniform in such a way that the outgoing radiation from the surface is still 240 W/m^2.

Ned Nikolov
January 4, 2012 1:22 pm

Phil,
One more comment regarding the integral of our Eq. 2 – Have you considered the fact that μ is the cosine of the solar zenith angle, and that a negative cosine implies that there is NO radiation reaching the surface (which is on the shaded part of the hemisphere), thus making the temperature ZERO in those areas and eliminating the need for an explicit integration? What Eq. 2 does is integrating the non-zero temperatures over the sunlit part ONLY and then dividing that number by the TOTAL surface area of the sphere to get the true average temperature … Do you understand it now?

Joel Shore
January 4, 2012 1:25 pm

Ned Nikolov says:

For starters, consider the possibility that the 390 W m-2 emitted by the surface is THE RESULT of a higher surface temperature (compared to that of an airless gray body) caused by solar heating and air pressure. In other words, what’s the chance that near-surface IR exchange is caused by the temperature rather than causing the temperature?

Ned,
Anything is possible if we don’t have to satisfy energy conservation! The hard part is explaining how pressure can cause a rise in surface temperature in a way that satisfies energy conservation.
And, sorry, but pressure can’t directly affect “near-surface IR exchange” unless there are elements in the atmosphere that absorb IR radiation, i.e., there is a greenhouse effect. Pressure can modulate the greenhouse effect in certain ways (e.g., by pressure-broadening of the absorption lines), but it can’t create something from nothing.

Joel Shore
January 4, 2012 1:32 pm

Ned Nikolov says:

This raises the question, where have the climate scientists been for the past 12 years not realizing that our moon is much much colder that their simple (and mathematically incorrect) application of the S-B equation suggests? … In fact, the Diviner data show that there is NO latitude on the Moon surface, where the average diurnal temperature is even close to 255K. Even on the lunar equator, the mean temperature does not exceed 210K and it drops to about about 95K at the poles … You do the math!

Read this: http://arxiv.org/abs/0802.4324 And, note that Arthur is not claiming to have discovered any new physics. He is just explaining what is already known in light of Gerlich & Tscheuschner’s paper obfuscating the issue.
In other words, it is well-known that the convention calculation using the S-B Equation sets a maximum value on the average temperature that occurs in the absence of a greenhouse effect. That maximum is realized for a planet with a uniform (***in space and time***) surface temperature. For a planet with little or no atmosphere (and no liquid water), the temperature can be very non-uniform and the resulting average temperature considerably lower.

Joel Shore
January 4, 2012 1:37 pm

G. Karst says:

I am not sure why, but this debate reminds me of the early arguments on electrical flow.

Perhaps because you are not qualified to judge the scientific merits of the arguments? Otherwise, it would remind you more of various screwball hypotheses by people who think they’re the next Galileo and ain’t.

January 4, 2012 1:43 pm

http://wattsupwiththat.com/2011/12/29/unified-theory-of-climate/#comment-853213
“In fact, the Diviner data show that there is NO latitude on the Moon surface, where the average diurnal temperature is even close to 255K.”
That’s funny, the Diviner website says the following:
“Mean Surface Temperatures: Day: 380K; Night: 120K”
http://diviner.ucla.edu/science.shtml
Want to try again? It’s not like we haven’t had people on the surface of the Moon; it’s temperature isn’t a big secret.

kzeller
January 4, 2012 2:55 pm

Something to do while waiting for Friday’s further enlightenment (? a second coming?).
I really liked Dr. Bill’s analogy (somewhere above) of weighing the whole gravel bucket rather than propose weights of individual pieces. A better analogy since Dr. Nikolov & I typically deal with separating trees from the forest and haven’t figured out how to weight the forest.
Taking Dr. Bill’s suggestion, I would offer a simple grammar school (ok maybe high school) experiment for you folks who love science but are having problems (wasting time?) dealing with the pebbles. Here it is:
Given that: Nikolov & Zeller are obviously from outer space hence couldn’t have a clue about our planet’s surface temperature let alone GHG physics.
Hypothesis: The Nikolov/Zeller outer-space Equation 8 that they propose will predict the average surface temperature on any planet or moon in the entire universe with an atmosphere (including Earth!) is a joke and as such will not work. Furthermore clearly it won’t work because radiative transfer and greenhouse gases are not addressed in their equation.
Materials: Nikolov-Zeller Equations 2 & 7 (used in Eq. 8, above this blog), and measured data from either Nikolov-Zeller Table 1 (above this blog) or better obtain the same values as needed from Wikipedia and other sources.
Procedure: 1. Remove all thoughts and beliefs of GHG theories, etc. from your mind (may require serious religious council). 2. Use Equation 2 to calculate the grey-body temperature(s) for each planet or moon to be tested. 3. Use Equation 7 to calculate surface temperatures as a function of surface pressure and grey-body temperature calculated from Eq. 2. (clue: first using algebra, you will have to solve Eq. 7 for the surface temperature by multiplying both sides by the grey-body temperature)
Results: Compare the calculated average surface temperatures with independent temperatures that have been measured as available in the literature remembering that it is the global average(s) over time that is/are being addressed.
Discussion: talk about your findings and speculate what they mean. How does radiative transfer affect the results? How do the chemical constituents of the various atmosphere(s) impact the surface temperature calculations? Bonus: Given that Venus’s atmosphere is mostly CO2 and an obvious runaway greenhouse gas example, does the Nikolov-Zeller approach to calculate Venus’s surface temperature using Venus pressure & Venus grey-body temperature differ at all from the approach used to make the same calculation on Earth?
Conclusion(s): Draw conclusion(s) and then report to WUWT so that once and for all the truth about these 2 outer-space alien-cadets can be known.

Ned Nikolov
January 4, 2012 3:05 pm

Robert,
I saw that very statement on their website, but after reading some of the papers on the same website discussing data and models (look at the the publications section http://www.diviner.ucla.edu/publications.shtml ), it became clear to me that the 380K ‘mean daily’ temperature quoted on that page actually refers to the mean daily MAXIMUM temperature, NOT the temperature averaged over the light portion of a lunar day. The 380K temperature is only reached for couple hours in the middle of the lunar day and only at latitudes below 60 degree …The same goes for the reported 120K at night. This is the mean MAXIMUM nighttime temperature! Temperatures during a lunar night drop to about 80K at lower altitudes and to 40-50K at the poles! This is published in NASA’s research papers! … Please investigate further.
Also, direct temperature measurements taken at the Apollo 15 landing site (26 deg N) over a period of about 5 years in the 1970s, suggest as well that the 380K is the MAXIMUM daily temperature, see this peer-reviewed paper by Huang (2008):
http://www.geo.lsa.umich.edu/~shaopeng/Huang07ASR.pdf
One must always look beyond the first set of evidence to find the truth! That is what we have been doing over the past 2 years while working on our theory …

davidmhoffer
January 4, 2012 3:08 pm

reposted from the Cause and Affect thread by Ira Glickstein>>>>
The more I watch this debate the more blindingly obvious it becomes that N&Z are right.
Willis et al, you’ve asked for a physical mechanism and Richard S Courtney gave it to you. If I may, here is his explanation in slightly different terms.
Assumptions
Rocky planet.
atmosphere that neither absorbs nor emitts
average insolation of 240 w/m2
Goal
Mechanism by which the presence of the atmopshere raises average T without raising average P.
Explanation
By SB Law, we expect an average T of -17.9 C
However, we CANNOT calculate average of T for a given average P since P doesn’t vary with T but with T^4!
FURTHER, the atmosphere, while it cannot absorb or emitt, must still participate in energy transfer via conductance and convection. This has no possible result but to remove energy from the “tropics” (which receive higher than average insolation and are at a higher than average T) to the “arctic zones” (which receive lower than average insolation and so are at a lower than average T)
Example calculation
Assumptions
Without presence of atmosphere:
Tropics are subject to an average P of 2 x 240 = 480.
Vis SB Law, T = 30 C
Arctic Zones are subject to an average P of 0
Via SB Law, T = -273 C
Assumptions
WITH presence of atmosphere
Some portion of P at the tropics is transferred to the atmosphere via conduction. This in turn causes convection. The convection in turn moves warm air from the tropics to the arctic zones, and cool air the other way. This cools the tropics and warms the arctic. For the purposes of simplicity, let us assume equal areas between the tropics and the arctics. Let us further assume that the net effect is to cool the tropics by 50 w/m2 and warm an equal area of the arctics by 50 w/m2. Let us apply SB LAw to see affect on temperatures.
Tropics = 480 – 50 = 430
Via SB Law, T = 22 C
Change = -8 degrees.
Arctics = 0 + 50 = 50
Via SB Law, T = -100 C
Change = + 173 degrees
And there you have your mechanism. By removing 50 w/m2 from the tropics, that area sees a reduction in temperature of 8 degrees. The 50 w/m2 added to an equal area in the artic sees an increase in temperature of 173 degrees. Since the areas are equal in size, and one decreased in P by the same amount as the other increased in P, the laws of thermodynamics are intact.

davidmhoffer
January 4, 2012 3:15 pm

…continued, snipped off the bottom part by accident
Average T however is higher. MUCH higher.
Average T^4 HAS NOT CHANGED!
Average P HAS NOT CHANGED!
Of course you’d never actually get 0 degrees K at the poles even in a planet with no atmosphere because the planet ITSELF will conduct heat from the tropics to the poles to some extent. There are other minutia I can think of that would throw the numbers off, but this should serve to illustrate a mechanism by which the presence of an atmosphere increases average T simply by redistributing energy from the “hot” zones to the “cold” zones via conduction and convection.
And that is why relying on the average of T instead of the average of T^4 has bolloxed up this entire conversation for the last several decades. average of T means diddly squat. average of T^4 is the ONLY way to determine if the planet is in an energy imbalance and by how much.

Ned Nikolov
January 4, 2012 3:30 pm

Reply to Robert Murphy (January 4, 2012 at 1:43 pm):
Robert,
One additional comment regarding the ‘mean temperatures’ listed on the Diviner’s webpage.
http://www.diviner.ucla.edu/science.shtml
Note that they compare Moon’s ‘Mean Daily Temperature’ of 380K with Earth’s ‘Mean Daily Temperature’ of 295K … What’s the Earth actual average surface temperature? 287.6K! Where do we have 295K average temperature on Earth? On the equator! … This gives one also a clue how accurate the listed mean temperatures on that page really are. Apparently someone at NASA was not paying attention, when populating that page with content … 🙂

Phil.
January 4, 2012 3:34 pm

Ned Nikolov says:
January 4, 2012 at 1:22 pm
Phil,
One more comment regarding the integral of our Eq. 2 – Have you considered the fact that μ is the cosine of the solar zenith angle, and that a negative cosine implies that there is NO radiation reaching the surface (which is on the shaded part of the hemisphere), thus making the temperature ZERO in those areas and eliminating the need for an explicit integration? What Eq. 2 does is integrating the non-zero temperatures over the sunlit part ONLY and then dividing that number by the TOTAL surface area of the sphere to get the true average temperature … Do you understand it now?

Yes I always did, it appears that you do not. In order to integrate cosine over a hemisphere you would normally integrate from define the angle relative to the zenith and integrate from cos(-pi) to cos(pi) or (more easily) integrate from 0 to 1 and multiply by 2, integrating only from 0 to 1 only integrates over quarter of the sphere not a hemisphere hence the need for the x2. You appear to be unaware of this judging by your answer, so you are only integrating over half the sunlit part.
Do you understand the point now?

Phil.
January 4, 2012 3:45 pm

Joel Shore says:
January 4, 2012 at 1:20 pm
Phil. says:
“While you’re here perhaps you’ll address a few points:
Your equation 2 shows the limits of 0 to 1 for the integration wrt μ, it should obviously be from -1 to 1 which by symmetry is double the value of the integral you show. Perhaps you could explain where this factor of 2 has gone in your derivation (this point has been raised by others here but you haven’t addressed it).”
That particular issue is not a problem. See my post here, and Arthur’s nice paper in particular, for details:

Reproduced here:
Joel Shore says:
January 3, 2012 at 2:16 pm
However, the integral is more easily carried out if you make theta = 0 correspond to the point on the earth where the sun is directly overhead because you have symmetry about such an axis. And, in fact, the integral is then what Nikolov et al have written down.

Exactly my point, but if you do that the integral is twice what Nikolov has used!

January 4, 2012 4:03 pm

“it became clear to me that the 380K ‘mean daily’ temperature quoted on that page actually refers to the mean daily MAXIMUM temperature, NOT the temperature averaged over the light portion of a lunar day.”
It’s still nowhere near 154K.
“Note that they compare Moon’s ‘Mean Daily Temperature’ of 380K with Earth’s ‘Mean Daily Temperature’ of 295K … What’s the Earth actual average surface temperature? 287.6K!”
It’s a helluva lot closer than your imaginary number for the Moon, 154K. It’s astounding how some people think they can invent numbers for well known values and revolutionize fields they have no training in. You can’t even *conceive* of the possibility that you might be wrong and all those PhD atmospheric scientists might have a better grasp of the subject than you. You’re too busy comparing yourself to Copernicus.

Joel Shore
January 4, 2012 4:23 pm

davidm hoffer: Since you apparently believe your post is so important that you need to repeat it here, I will just repeat the answer I gave over in the other thread:
davidmhoffer says:

Average T however is higher. MUCH higher.
Average T^4 HAS NOT CHANGED!
Average P HAS NOT CHANGED!

Read this: http://wattsupwiththat.com/2011/12/29/unified-climate-theory-may-confuse-cause-and-effect/#comment-853148 , many times if you have to.
What you have just shown is why the vast majority of climate scientists talk about the “surface temperature enhancement” being 33 K and not 133 K (as Nikolov et al say it is): Those scientists understand that you can get different values by having a different temperature distribution. Hence, if you have a surface emitting 240 W/m^2 and having the distribution that Nikolov et al. talk about, the temperature will be ~155 K. If you move the heat around, you can indeed raise the average temperature up further. In fact, you can get it up to 255 K by having a perfectly uniform temperature distribtuion. But, no matter how hard you try, you are not going to get a surface emitting 240 W/m^2 to have a higher average temperature than that (for a surface with emissivity approximately equal to 1).
I think you are learning the hard way that climate scientists are a little smarter than you think and you and Nikolov might be just a little bit less smart than you think.

Joel Shore
January 4, 2012 4:24 pm

Phil. says:

Exactly my point, but if you do that the integral is twice what Nikolov has used!

No…Phi. 0 to 90 deg encompasses the entire sunlit hemisphere for that geometry.

Phil.
January 4, 2012 4:53 pm

Joel Shore says:
January 4, 2012 at 4:24 pm
Phil. says:
Exactly my point, but if you do that the integral is twice what Nikolov has used!
No…Phi. 0 to 90 deg encompasses the entire sunlit hemisphere for that geometry.

Afraid not Joel, a hemisphere is from 0-180º.

Joel Shore
January 4, 2012 5:35 pm

Phil. says:

Afraid not Joel, a hemisphere is from 0-180º.

The Northern hemisphere goes from 0 to 90 deg north latitude and zero to 360 degrees longitude. He is integrating over the “Northern hemisphere” except that the sphere is tipped by 90 deg so it is facing toward the sun. He needs to integrate from 0 to 360 deg in longitude and 0 to 90 deg in latitude on such a sphere. (Latitude is the polar angle. longitude is the azimuthal angle.)

Ned Nikolov
January 4, 2012 5:41 pm

Reply to Robert Murphy (January 4, 2012 at 4:03 pm)
Robert,
I did not expect that I have to spell out everything for you. Apparently, you do not know what a diurnally average temperature is, nor do you understand the meaning of a MAXIMUM mean temperature. So, let me try to clarify this for you one more time:
1) A diurnal mean temperature is an AVERAGE between the daytime mean temperature and the nighttime mean temperatures. The 380K quoted on the Diviner webpage is VERY close to the ABSOLUTE MAXIMUM temperatures measured on the moon, see these papers
http://diviner.ucla.edu/docs/fulltext.pdf
http://www.geo.lsa.umich.edu/~shaopeng/Huang07ASR.pdf
and
http://diviner.ucla.edu/docs/paige_2010.pdf
The diurnal temperature amplitude (i.e. the difference between daily max and daily min temp) on the Moon is VERY large. On the lunar equator, this amplitude is about 290K and declines to about 70-80K at the poles. Also, the daily max temperature on the Moon is only maintained (lasts) over a short period of the lunar day (which, by the way, equals 27.3 Earth days). At any given location on the Moon, the temperature drops quickly as one moves away from the a noon point. By sunset, the temperature is already around 120K for most latitudes (outside the polar regions), and continues to drop during the lunar night reaching about 75-80K just before sunrise. In addition, the polar regions have temperature about 150K during daytime and 40-50K during nighttime. Taking into account this pattern of temperature change and considering the fact that a lunar night nearly equals in length a lunar day, one can calculate the AVERAGE DIURNAL temperature at the lunar equator and at the poles using the information presented in the above papers. The result is about 210K mean diurnal temperature at the lunar equator, and 90-100K mean diurnal temperature at the poles. The actual mean temperature for the entire Moon surface falls between those two limits (210K and 95K), and given the purely radiative nature of the heating on the Moon, the final mean is very likely close to the average of the arithmetic extremes, i.e. (210 + 95)/2 = 152.2. So, there you have it! … I encourage you to study carefully the above references.

Ned Nikolov
January 4, 2012 6:10 pm

Phil,
What kind of a science background do you have? Apparently your calculus skills can use some improvement.
The cos(theta) gets integrated between 0 and 1, because on the sunlit hemisphere, the solar ZENITH angle (theta) varies from 0 to 90 deg. What’s a solar zenith angle? It’s the angle between the solar beam and the axis perpendicular to the surface. So, theta varies between 0 deg at the equator and 90 deg at the poles. Since we have 2 poles (2 quarter spheres), we use a second integral from 0 to 2*pi, which essentially doubles the estimate obtained from the first integration of cos(theta). Note that 2*pi is used to account for the surface of a hemisphere. This completes the integration of the temperature field over the SUNLIT hemisphere. Since the temperature of the shaded hemisphere is 0, to estimate the overall mean temperature for the entire sphere, we now have to divide the above integral by 4*pi … and voila! Do you get it now? …

wayne
January 4, 2012 6:56 pm

Phil. says:
January 4, 2012 at 4:53 pm

Joel Shore says:
January 4, 2012 at 4:24 pm

Phil. says:
Exactly my point, but if you do that the integral is twice what Nikolov has used!

No…Phi. 0 to 90 deg encompasses the entire sunlit hemisphere for that geometry.

Afraid not Joel, a hemisphere is from 0-180º.
>>>
ROTFLMAO!
Sorry, I thought both of you had at least Calculus for Dummies. If you did then I thought you two knew in science how to consider all possible paths. (and that at last explains the deficiencies we all have seen!)
OK guys, try considering the integral 0 to 1 is from the center of the hemisphere to the edge, OK?
Now do either of you know how far it is around the circumference or should I explain that too?

Ned Nikolov
January 4, 2012 7:36 pm

Reply to davidmhoffer (January 4, 2012 at 3:08 pm)
David,
At first glance, your thought experiment appears to provide a good insight about how a redistribution of energy by an atmosphere from the equator to the poles might increase the average global temperature and thus explain the thermal enhancement of the atmosphere. However, you have made one critical conceptual : Basically, your calculations show this effect due to the fact that radiation intensity changes very fast with temperature at low temperature values versus high temperatures. In other words, the partial derivative of P with respect to T is much larger at low temperatures compared to high temperatures. This is due to the 4th power of T. However, the problem is that the real atmosphere does NOT transport (redistribute) radiative fluxes but air masses of certain temperatures. In a sense, you can say that the atmosphere transports ‘temperatures’. This means that high temperatures must first be created in the tropics and then transported poleward, where they radiate and conduct/convect heat to air masses and land surfaces at high latitudes. This essentially nullifies the radiative effect you are talking about …
One cannot escape the physical fact that pressure enhances the energy content of a system compared to zero-pressure conditions thanks to increased rates of molecular collision, and this premise comes straight from the Gas Law! This fact was widely known and undisputed 30-40 years ago, but thanks to our modern ‘radiatively skewed’ physical education, it has been totally downplayed to the point that PhD scientists now believe that pressure brings NOTHING to a thermodynamic system…. I mean, it’s really embarrassing! This used to be a high-school level knowledge, at least during my high-school days in Europe anyway … 🙂

dr.bill
January 4, 2012 7:42 pm

kzeller, January 4, 2012 at 2:55 pm :

Dr. Bill’s analogy… of weighing the whole gravel bucket…

That analogy (a good one, in my opinion) was made by davidmhoffer.
/dr.bill

davidmhoffer
January 4, 2012 8:01 pm

davidmhoffer says:
January 4, 2012 at 7:41 pm
Tim Folkerts;
Agreed!
I used an extreme example to illustrate the issue, and I made a couple of mistakes along the way. Better would have been (using round numbers here just to simplify things) if I’d supposed insolation to be 1000 w/m2 after albedo, the scenario would look more like this:
“Average” insolation = 250 w/m2
“Average” insolation at tropics = 500 w/m2
“Peak” insolation at tropics = 1,000 w/m2
“Average” insolation at poles = 0
(because my pretend rock has no inclination to the orbital plain. Since it is MY make believe rock I can give it any inclination I want 😉 )
“Peak” insolation at poles = 0
This would provide for a power curve of “average” insolation of 250 w/m2 at the tropics that declines with altitude until you get to the poles where it is zero. It wouldn’t be a linear decline since my rock has a curved surface, but the exact power curve isn’t important in terms of understanding the concept.
However, “peak” insolation would be 1,000 w/m2 at the tropics, and decline to zero at the poles. This brings up a series of concepts. While the poles would obviously trend toward an equilibrium temperature of zero, calculating the equilibrium temperature of the tropics is not so straight forward. At night, insolation is zero, and so the tropics would cool toward zero at night. Assuming a 24 hour day, they’d never get anywhere near zero of course, the sun would rise long before that could happen. So what equilbrium temperature do the tropics tend toward in the day?
If we assume that P starts at zero, peaks at 1000 at noon, and then falls back to zero in evening, we could justify a rough estimate of 500 w/m2 average which would yield via SB Law an “average” temperature of 33.4 C. But for over half the daylight hours, insolation would actually be more than that. Briefly at noon, insolation would be at 1,000 w/m2 which would translate to 91.4 C. Of course it would never get to 91.4 C anymore than it would get to zero at night. The point being that the 250 w/m2 = -15.3 C is no more accurate than 500/33.4C or 1000/91.4 C. You’d have to integrate across the daylight curve to arrive at the equilibrium temperature for the tropics, and it would NOT be the same as what one would get by simply calculating from the “average” insolation.
With that in mind, let’s throw in an non absorbing non emitting atmosphere. What happens?
In my illustrative comment above, I invented some equilibrium temperatures at poles and tropics and tried to show what would happen if the processes of conduction and convection moved 50 w/m2 away from the tropics and to the poles. That’s not realistic of course. A better model would be that considerably more than 50 w/m2 would be removed from the tropics.
Again, just to illustrate the concept, I’m picking numbers here. Since the temperate zones and arctic zones are far cooler than the tropics simply due to angle of inclination to the sun, the atmosphere has little choice but to warm by conduction and convect. The warm air must rise, pulling cold air beneath it from the poles. The hot air rises to some height, then spill toward the poles.
Now we have a whole bunch of things happening at the same time. For starters, we can now suggest that the “average” temperature of the tropics themselves would be higher than without an atmosphere, and that not one single extra watt is required to accomplish that.
Since the atmosphere is warmed by conduction during the day, at night, it must give back by conduction. But because P varies with T^4, the number of degrees that the atmosphere “gives back” is greater than what what it took in the first place. The tropics have no choice but to warm the atmosphere, which then redistributes the energy absorbed in two ways. One is via convection which drives energy toward the poles, and the other is conduction BACK to the tropics as soon as night time temperatures fall below the temperature of the atmosphere.
So again, I’m making up rough numbers to illustrate the point that there is in fact a mechanism for a non absorbing, non emitting atmosphere to raise surface temps without violating the laws of thermodynamics.
Suppose for argument’s sake that the atmosphere removes an average of 100 w/m2 from the tropics during the day. It must distribute that energy absorbed in two ways. Convection forces some energy to be distributed toward the poles, and the night/day cycle means that some gets sent back to the surface at night time. For illustrative purposes, let’s suppose that 25 w/m2 of the 100 w/m2 get redistributed via conduction back to the tropics when they cool off at night.
So… the target equilibrium P during the day for the tropics with no atmosphere might have been 500 w/mw, but with an atmosphere it is only 400 w/mw. From SB Law:
500 w/m2 = 33.4 C
400 w/m2 = 16.8 C
The atmosphere reduces the “target” equilibrium temperature by 16.6 degrees. But what happens at night when the atmosphere “gives back” via conduction, 25 w/m2?
0 w/m2 = -273 C
25 w/m2 = -128.1 C
See what happened? Moving 100 w/m2 away from the tropics during the day reduces the equilibrium target temperature by 16.6 degrees, but taking just 25% of those watts/m2 back at night increases the night time equilibrium temperature by a whopping 144.9 degrees! All we need do is average the tropics night time and day time equilibrium temperatures to see that a miniscule 25 watts moved by conduction via the atmosphere to the night time results in a massive temperature increase without any change in the energy balance at all. But I said the atmosphere would in this example move 100 w/m2 away from the tropics in day time, and I only sent 25 w/m2 to the night time, let’s figure in them other 75 watts.
Convection ensures that warm air moves toward the poles, heating the earth below via conductance. The night time “temperature boost” applies across all latitudes. The day time temperature boost is lowest in the south temperate zones, higher in the north temperate zones and highest (on a degrees per watt/m2 basis at least) in the arctic zones. Surface areas being different, angle of incidence varying, and so on, we can actually say that our remaining watts/m2 are going to be distributed in a fashion that adds up to 75w/m2. Unless of course my invented rock has surface area irregularities that result in this being the case. Since it is my rock that I invented, and I’m way too lazy to do the math properly, I’m going to go with that.
Suppose that our south temperate, north temperate, and arctic zones each get a 25 w/m2 boost from conduction from the atmosphere, and that the areas equal out such that the square meters match that of the tropics (I know, rather addly shaped rock, but just ignore that and stick with the arithmetic).
The night time temperature boost of all three zones (the target equilibrium temperature they would now tend toward, though they would be unlikely to actually get there before sunrise) would be plus 144.9 degrees. The 100 watts/m2 we liberated via conduction from the tropics in the first place only dropped the temperature of the tropics by 16.6 degrees!
The day time equilibrium temperature of the arctic (previously with zero insolation) would now also be higher by 144.9 degrees. The temperature boost in the temperate zones would depend on what their “average” temperature without an atmosphere would be. Let’s guestimate that the “average” P of the south temperate zones was 300 w/m2 and the north temperate zone 200 watts/m2, and that each get a boost of 25 w/m2 from conduction from the atmosphere.
300 w/m2 = -3.3 C
325 w/m2 = 2.2 C
“warming” = 5.5 degrees
200 w/m2 = -29.3 C
225 w/m2 = -22.0 C
“warming” = 7.3 degrees
So we now have a planet that fluctuates between day time warming and night time cooling. Each day the warming tends toward the equilibrium high, but never makes it before cooling sets in. Each night, the cooling tends toward the equilibrium low, but never makes it before sunrise and warming starts again. In return for reducing the day time equilibrium high of the tropics by just a few degrees, the night time low equilibrium point of the entire planet increases by over 100 degrees. The equilibrium day time highs of the south temperate, north temperate and arctic zones also increase, collectively “on average” more than what the tropics lost, even though the tropics lost 4 times as much in watts/m2 as the other zones gained.
I’m getting bleary eyed and I’m pretty certain I’ve got some messed up math in there on all sorts of issues, but I think by now the mechanism that everyone is screaming cannot exist without breaking the laws of thermodynamics, does in fact exist, and need not add or substract a single joule of energy from the system to still arrive at higher surface temperatures, and without emitting or absorbing a single photon.
Which has what to do with surface pressure? Glad someone asked. I’ve made up the numbers to illustrate how surface temperatures can be increased without adding or subtracting energy from the system. But how much conductance actually occurs for any given atmosphere.
Well, that’s dependant upon one thing. The density of the gas. Which is dependant upon…
mean surface pressure. Exactly how N&Z have said it.

Joel Shore
January 4, 2012 8:09 pm

Ned Nikolov says:

One cannot escape the physical fact that pressure enhances the energy content of a system compared to zero-pressure conditions thanks to increased rates of molecular collision, and this premise comes straight from the Gas Law!

Yes, to PRESSURIZE a system, you have to perform work, i.e., you have to input energy, and as a result the pressurized system has higher energy. However, the atmosphere of the Earth isn’t being pressurized. Yes, there is air that is descending and that air is being pressurized and gaining internal energy. However, there is also air that is ascending and that air is expanding and decreasing its internal energy (i.e., it is doing work on the rest of the atmosphere when it expands).
But…this doesn’t solve our energy balance problem: To explain how the temperature of the surface can be such that it emits 390 W/m^2 whereas the entire Earth-atmosphere system is absorbing only 240 W/m^2, you have to explain what SOURCE OF ENERGY is supplying the additional 150 W/m^2 to the earth-atmosphere system. In other words, you have to identify an external source (like the sun) or an internal source (like heat from the core of the Earth, with the ultimate source of that being radioactive decay, as I recall).
You won’t be able to do that, of course, because we in fact know that the Earth is NOT emitting 390 W/m^2 as seen from space; it is only emitting 240 W/m^2…And, the reason is that is that the atmosphere is absorbing some of the emission from the surface. This is what is called “the greenhouse effect” and it is the only thing that allows for the possibility that the surface can be at a temperature where it emits 390 W/m^2.

davidmhoffer
January 4, 2012 8:16 pm

Ned Nikolov;
However, the problem is that the real atmosphere does NOT transport (redistribute) radiative fluxes but air masses of certain temperatures. >>>
My point was that the air mass, via simply conduction and convection, can redistribute energy. One doesn’t need radiative processes for that to happen. Willis Eschenbach and others were asking the question as to how temperature could be increased without violating the laws of thermodynamics. What I was trying to illustrate is that any redistribution of energy, including conductance and convection, must increase average T without increasing average P. In fact, it matters not what the mechanism is, as long as their is a mechanism. In this case, Willis had asked how surface temps could increase if there were no radiative processes at play, and I was attempting to answer that question.

kzeller
January 4, 2012 8:24 pm

Whoops, sorry about that davidmhoffer & thank you for the analogy & thank you dr.bill for the correcting the credit. Both of you are excused from having to do the experiment in any case! 🙂

Joel Shore
January 4, 2012 8:29 pm

Ned: The reason why radiation is considered is because that is the only way that the Earth+atmosphere can communicate energy (in any significant way) with the rest of the Universe. And, what is getting you (and a lot of others) hung up is that you are thinking about lots of things that move energy around in our atmosphere, which climate scientists know about and include in their models…But it doesn’t help you deal with the fundamental issue here, which is the “top of the atmosphere” energy budget.
Just stepping back for a second, what is it that makes you think that you are that much smarter than the entire climate science community? Can you even entertain the notion that the reason why you are getting a different answer than them is because of some boneheaded mistakes on your part? Is it within your abilities of self-awareness to understand this possibility?

I mean, it’s really embarrassing!

On that we can definitely agree; we just disagree on what “it” is.

Joel Shore
January 4, 2012 9:05 pm

davidmhoffer says:

My point was that the air mass, via simply conduction and convection, can redistribute energy. One doesn’t need radiative processes for that to happen. Willis Eschenbach and others were asking the question as to how temperature could be increased without violating the laws of thermodynamics.

You have created a complete strawman. WIllis did not ask that. Willis asked how the Earth could be emitting about 390 W/m^2 when the Earth and atmosphere are only absorbing about 240 W/m^2 from the sun. For everybody but you, the fact that one can redistribute energy and get a different average temperature is not a brilliant new observation. We’ve known it for a long time.

davidmhoffer
January 4, 2012 9:47 pm

Joel Shore says:
January 4, 2012 at 4:23 pm
davidm hoffer: Since you apparently believe your post is so important that you need to repeat it here, I will just repeat the answer I gave over in the other thread>>>
I find it unfortunate that you feel compelled to dispute me to your own detriment in two threads rather than one.

January 4, 2012 11:09 pm

Joel Shore (Jan. 4, 2012 at 6:26 am):
Toward the end of progress in our scientific understanding of Earth’s climate, I request your stipulation to the proposition that if and when the previously described lapse rate control mechanism is operative and the intensity of the back radiation at Earth’s surface increases by Delta F, the intensity of the convective heat transfer at Earth’s surface increases by Delta F. It follows from energy conservation that the increase in the intensity of the electromagnetic radiation that is absorbed at Earth’s surface is nil.

January 5, 2012 1:30 am

Look Ned, if you really think that you, a forest ecologist, are going to come up with a better number for the Moon’s average surface temperature, a figure that has been well known for quite a while; that you, a forest ecologist, are going to overthrow all of atmospheric physics with an online manifesto; that you, a forest ecologist, are the next Copernicus about to usher in a new scientific revolution, you really need help. I hope you seek it, especially after nobody takes your claims seriously and the world of science passes you and your failed hypothesis by. That kind of rejection and failure can lead to all kinds of resentment and conspiracy theorizing about scientific censorship and intolerance. You already have enough issues (Dunning-Kruger Effect for one).

Brian H
January 5, 2012 1:54 am

As is commonly done in figgering black-body transactions, put a continuous shell around the planet at the highest altitude that “receives” the 240 W/m^2. It is not possible, no matter the ingenuity of the vector bookkeeping, to have 390 W/m^2 outbound from that same altitude. Not for long, anyway.

Reply to  Brian H
January 5, 2012 8:58 am

Brian H (Jan. 5 at 1:54 am):
As the vector with intensity of 390 W/m^2 lies at the bottom of the atmosphere while the vector with intensity of 240 W/m^2 lies at the top of the atmosphere, the rules of vector addition do not apply to the situation you describe. At the bottom of the atmosphere, the heat flux is the vector sum of the upward pointing vector of intensity 390 W/m^2 and a downward pointing vector. If the latter vector is of intensity 390 W/m^2 then the intensity of the heat flux is nil. Similarly, at the top of the atmosphere, the heat flux is the vector sum of the downward pointing vector of intensity 240 W/m^2 and an upward pointing vector. If the intensity of the latter vector is 240 W/m^2 then the intensity of the heat flux is nil. In my example, the heat fluxes at both boundaries of the atmosphere are nil with the consequence that the atmosphere is not heating. Neither is the Earth heating. Heating is not happening despite the disparity in the intensities of the two vectors that you reference.

January 5, 2012 5:44 am

Wayne:
Please have patience with us lesser lights who still have difficulty with Ned’s Equation 2 (although you may remember that I verified that it comes out with essentially the right answer if you ignore its imposing anisotropy on background radiation).
Here’s why I think many of us have difficulty. Thinking of the earth, we tend naturally to integrate with respect to latitude and longitude. Testimony to that is that you and I both used that approach in computing Equation 2 numerically. But Ned didn’t, so he lost me right off.
What he did if I understand it is move the sun to Polaris’s position so that it shines directly on the North Pole. Then he translated the measure of latitude so that the North Pole’s latitude is zero and the South Pole’s is 180, and mu is the cosine of the resultant latitude. Another way of looking at it is that, for a point on the surface of a unity-radius sphere, mu is the distance along the axis in the direction of the sun from the sphere’s center to the plane perpendicular thereto that contains that point. Theta is longitude, or angular position around the circle in which the plane intersects the sphere. So Ned integrates along the axis and around the circle of intersection. mu < 0 implies nighttime, so integration limits for mu can be 0 and 1 if we want to ignore (de minimis) night-time background radiation. Fair enough, although it took me awhile to recognize what his coordinate system was.
But my problem with Equation 2 did not end there. Equation 2's radical is the expression for radiation power density at (mu, theta), so what's left in the integral, i.e., d mu d theta, should be differential area. But I don't see how d mu d theta is differential area.Yes, I recognize that simply integrating d mu d theta with his limits does indeed yield 2 pi, as it should. But that looks to me as though it's computing the area of a cylinder, not a sphere. I would have thought a sphere's differential area would be (again, for unity radius) cos(phi) d phi d theta, where phi is (Nikolov-translated) latitude. Since d phi = -d mu//sqrt(1 – mu^2), I would have expected to see -mu d mu d theta/sqrt(1 – mu^2) as the differential area. But I don't. Yet I've I verified Ned's numerical result, so he somehow got it right, whereas I would have expected too high a value: I've no doubt done the trig or calculus wrong.
Since you've had no trouble with that equation, might I impose upon you to help with my math?

Joel Shore
January 5, 2012 5:47 am

davidmhoffer says:

I find it unfortunate that you feel compelled to dispute me to your own detriment in two threads rather than one.

The reason I am doing that is because you felt that your result that you can have the same emission with two different temperature distributions having two different average temperatures (something nearly everyone but you knew already) was so important that it warranted copying your post explaining this to both threads. I have continued the discussion in the other thread.

January 5, 2012 5:48 am

Wayne:
Actually, I guess I didn’t verify the equation; I just verified the result Ned reported he got when he applied it. Like you, I actually used a different equation, as any lurker can see from my (erroneously commented) R code in the Feedback about Feedbacks thread.

Joel Shore
January 5, 2012 6:11 am

Terry Oldberg says:

Hint
While the vector of intensity 390 W/m^2 is a flux of energy, it is not a flux of heat. On the other hand, th vector of intensity 240 W/m^2 is a flux of heat. By its definition in thermodynamics, the “heat” is the energy that crosses the referenced boundary .At Earth’s surface the heat flux is the vector sum of a number of different vectors, only one of which has an intensity of 390 W/m^2. Once the additional vectors are taken into account, the apparent violation of energy conservation is falsified.

Frankly, this is gobbly-gook. The point is that the Earth is emitting a flux of radiative energy that is 390 W/m^2 at the surface of the Earth. If none of that radiation is absorbed by the atmosphere, then the flux of radiation that escapes to space is 390 W/m^2. It is irrelevant what the net heat flow is.

Toward the end of progress in our scientific understanding of Earth’s climate, I request your stipulation to the proposition that if and when the previously described lapse rate control mechanism is operative and the intensity of the back radiation at Earth’s surface increases by Delta F, the intensity of the convective heat transfer at Earth’s surface increases by Delta F. It follows from energy conservation that the increase in the intensity of the electromagnetic radiation that is absorbed at Earth’s surface is nil.

No…That is not correct. It is correct if the increase in radiative transfer from the atmosphere to the Earth occurs without a reduction in the intensity of radiation emitted to space. However, in the case of adding greenhouse gases, where the amount of radiation emitted to space is reduced, this compensation does not occur.
The reason that the maintenance of the lapse rate does not imply that the convective heat transfer balances things is found from going through the argument about what happens: When you add more greenhouse gases, the Earth system is now emitting less energy to space then it receives from the sun because the emission is occurring from higher levels in the troposphere where it is colder. In other words, the “effective radiating level” was at 255 K but it has now moved up in the atmosphere and is at a colder temperature.
As a result, energy accumulates and the Earth system warms. Once the system has warmed and radiative balance is again restored, this new effective radiating level will have warmed to 255 K…And so, given the same lapse rate, you will now have a higher surface temperature (because the surface temperature is determined by taking the altitude at which the temperature is 255 K and using the lapse rate to extrapolate down to the surface and that altitude is now higher).

kwik
January 5, 2012 8:28 am

Robert Murphy says:
January 5, 2012 at 1:30 am
“Look Ned, if you really think that you, a forest ecologist….”
Aha, ad hominem attacks. The first sign that someone wants to discuss who’s authority is correct, instead of discussing the science.

Ned Nikolov
January 5, 2012 8:37 am

Reply to Robert Murphy (January 5, 2012 at 1:30 am):
Robert to your comment above:
Look Ned, if you really think that you, a forest ecologist, are going to come up with a better number for the Moon’s average surface temperature, a figure that has been well known for quite a while; that you, a forest ecologist, are going to overthrow all of atmospheric physics with an online manifesto; that you, a forest ecologist, are the next Copernicus about to usher in a new scientific revolution, you really need help. I hope you seek it, …
First of all, thank you for your honesty! I would like to respond by pointing out two facts: 1) The number for the average Moon temperature I presented to you is not mine invention, but a product of NASA’s own research as documented in the peer-reviewed publications I referred you to; 2) I suppose you also have a problem with the Theory of Relativity, since it was proposed by a low-level clerk working for a Swiss Patent Office … 🙂

January 5, 2012 8:52 am

“The number for the average Moon temperature I presented to you is not mine invention…”
Sure it is Ned. That’s why the number NASA (and everybody else) gives is nowhere near the number you invented.
“I suppose you also have a problem with the Theory of Relativity, since it was proposed by a low-level clerk working for a Swiss Patent Office…”
So now you’re Einstein too? Copernicus no longer good enough for you?. Are you aware he was a doctoral student in physics working on his thesis while he was working at the patent office? Apparently your knowledge of science history is as bad as your knowledge of atmospheric physics. Look up the Dunning–Kruger effect and take an honest look in the mirror.

kzeller
January 5, 2012 9:04 am

Hey Ned,
we had an agreement! You only got to be Copernicus because I got to be Einstein! You can’t be both it’s not fair. I’m going to tell my Mommy!

Ned Nikolov
January 5, 2012 9:17 am

Robert, to your comment:
“So now you’re Einstein too? Copernicus no longer good enough for you?. Are you aware he was a doctoral student in physics working on his thesis while he was working at the patent office?”
Yes, I’m aware that Einstein was a doctoral student at that time … And I already got my PhD in biophysics 15 years ago! How about you? What kind of advanced degree do you have?
Again, read the papers I sent you earlier, and use you brain to figure out the Moon’s temperature from the official peer-reviewed research. And, please, stop ridiculing yourself with personal remarks .. Thank you!

Joel Shore
January 5, 2012 9:20 am

As the vector with intensity of 390 W/m^2 lies at the bottom of the atmosphere while the vector with intensity of 240 W/m^2 lies at the top of the atmosphere, the rules of vector addition do not apply to the situation you describe. At the bottom of the atmosphere, the heat flux is the vector sum of the upward pointing vector of intensity 390 W/m^2 and a downward pointing vector. If the latter vector is of intensity 390 W/m^2 then the intensity of the heat flux is nil.

We are not talking about heat flux. We are talking about radiative energy. If you want you could count photons and add up their energies. Can’t you guys have some self-awareness about when you are flailing around horribly in order to believe what you want to believe. This debate is not about science…It is about people desperate to believe what their ideology dictates they have to believe to be true.

Reply to  Joel Shore
January 5, 2012 12:15 pm

Joel Shore (Jan. 5, 2012 at 10:47 am):
Your response evades the issue of the energy conservation.

Joel Shore
January 5, 2012 9:24 am

By the way, I posted something in the other thread that I am re-posting here, because it is very important. There is a very important factor that I was leaving out that explains the strong positive correlation between surface pressure and “surface temperature enhancement” in the way that Nikolov has defined it:
They [Nikolov’s fit to the celestial data] are a 5-parameter fit and there is good reason to believe that in general the surface pressure enhancement will be positively correlated with surface pressure for the following three reasons:
(1) Pressure causes a broadening of the absorption lines of the greenhouse gases, increasing the greenhouse effect.
(2) Atmospheres with higher surface pressure have more of all substances that they consist of…and hence will tend to have a larger amount of greenhouse gases if any significant proportion of the atmospheric constituents are greenhouse gases.
(3) Since Nikolov has chosen to measure surface temperature enhancement by considering T_sb to be determined by the approximation that the local insolation determines the local temperature (no heat flow or storage), most of the “surface temperature enhancement” will occur due to the fact that larger atmospheric pressures means that the atmosphere will have more heat flow and heat storage in it. For example, Nikolov says the Earth’s temperature is enhanced by 133 K but we in fact know that the greenhouse effect only is responsible for 33 K. The other 100 K comes from the more uniform temperature distribution that results when one has a significant atmosphere and is most certainly correlated with pressure. In fact, of the 8 bodies considered, I think that the “surface temperature enhancement” only has any significant component due to the greenhouse effect in 3 of them (Earth, Triton, and Venus) and probably only in Venus is it the dominant component of the so-called “surface temperature enhancement”.
I hadn’t mentioned (3) before but in fact it is probably the most important reason why there is a strong correlation between surface pressure and “surface temperature enhancement” defined as Nikolov has defined it.

January 5, 2012 9:34 am

Joel Shore (Jan. 5, 2012 at 6:11 am):
I get frustrated when you refuse to stipulate to facts, change the topic when you’re about to be pinned down or evade the issue by issuing an insulting ad hominem argument. I suggest to you that Richard Courtney and I have pinned you down on a fact but you lack the decency to stipulate to this fact so we can move the conversation along. The fact to which you refuse to stipulate is that the feedback control mechanism which pins the lapse rate to the adiabatic lapse rate does not violate energy conservation.

Joel Shore
January 5, 2012 9:53 am

At this point, it’s really just a “mopping up” operation. We now clearly understand in gory detail why Nikolov and Zeller’s work is fatally flawed. There is essentially no aspect of their paper that survives once one considers all of the serious errors that have been found and they and others have no intelligible answer to these errors that have been pointed out. Anyone who continues to believe their work at this point is simply discrediting themselves.
It is time to jump from the sinking ship, folks!

January 5, 2012 9:58 am

“Yes, I’m aware that Einstein was a doctoral student at that time”
Then you know how silly it is to speak of him as just a lowly patent clerk as if he had no other relevant background.
“And I already got my PhD in biophysics 15 years ago!”
Yes, I read your biographical information and how you received your PhD in Forest Ecology in 1997. Your research has been in weather and forest fires mostly. You are stepping well outside of your field in your attempt to overthrow atmospheric science.
“Again, read the papers I sent you earlier, and use you brain to figure out the Moon’s temperature from the official peer-reviewed research.”
Use your own and ask yourself why all of those people who actually spend their time studying the Moon for a living come up with a completely different number than you. Did you even bother to ask a relevant scientist why their number is so different? Or would that have been beneath you?
BTW, in your online manifesto with Zeller, you claim that over the last 50 million years or so the atmosphere has lost 53% of its mass, and this is the main reason that temps have fallen. Do you have anything besides your equations to back this up? Are you aware of the implications such a change in atmospheric mass (and pressure) would have on the biosphere? Where is your physical evidence this happened?

Joel Shore
January 5, 2012 9:58 am

Terry Oldberg says:

The fact to which you refuse to stipulate is that the feedback control mechanism which pins the lapse rate to the adiabatic lapse rate does not violate energy conservation.

It is you who are misrepresenting things and evading the facts. I never said that such a mechanism violates energy conservation. However, the lapse rate does not alone determine the surface temperature. If the height at which the temperature is equal to 255 K goes up, the surface temperature goes up even while the lapse rate remains the same. Read this again until you understand it: http://wattsupwiththat.com/2011/12/29/unified-theory-of-climate/#comment-853955 It is the mechanism of the greenhouse effect that has been worked out by people with far more knowledge in the field than you or I and for which there is clear evidence for.

davidmhoffer
January 5, 2012 10:08 am

JOEL SHORE
1. Do not draw the conversation into snarky witticisms. You are poorly equipped to compete with me in that regard. My mother taught me not to fight an unarmed man, but I think she will make an exception in this case. I suggest you stick to science.
2. There is no need to demonstrate a mechanism by which the average surface temperature of earth could exceed the black body temperature of earth. All that is required to support N&Z’s hypothesis is to demonstrate that there is a mechanism by which the average surface temperature of a planet could be increased by the presence of a non radiative atmosphere WITHOUT changing the EFFECTIVE black body temperature odf the planet. Not only I have I done so, but I have done so in language that should be comprehensible to anyone who passed high school algebra.
3. Your suggestion to Ned Nikolov that he should give his head a shake because the consensus amongst climate scientists is in disagreement with him is spurious. The fact that his writings have sparked spirited and intense debate in this and other forums amongst highly qualified scientists with PhD’s in a variety of directly and indirectly related fields while all we hear from the climate “scientists” is the sweetness and harmony of consensus should give you pause.
4. Your contention that Ned Nikolov’s background in forestry somehow undermines his credibility is egregious. If you cannot discredit him through science alone, then it matters not if his education ended with a hundred PhD’s or if he quit school in grade 2.
5. If you continue to contend that Ned Nikolov’s background in forestry discredits his opinion on climate issues, then I ask that you stick to your position and publicly ask Mssrs Briffa, Mann and Jones to withdraw their work on global temparature reconstructions from tree ring data on the basis that they have no background in forestry.
Regards,
dmh

Joel Shore
January 5, 2012 10:37 am

Terry Oldberg: To be more specific about where I am disagreeing with you, I think it is here:

I request your stipulation to the proposition that if and when the previously described lapse rate control mechanism is operative and the intensity of the back radiation at Earth’s surface increases by Delta F, the intensity of the convective heat transfer at Earth’s surface increases by Delta F.

I don’t see any reason why the convection increases in such a way that it exactly cancels out the increase in the back radiation when that increase in back radiation comes about from the process of an increase in greenhouse gases. To be clear, it may be true that if you look immediately after a delta-function increase in greenhouse gas concentrations, then this is true. (It is not obvious to me that it definitely so at the moment…but it is at least conceivable.) However, the important thing about such an increase in greenhouse gases is not the increase in back-radiation but the change in the top-of-the-atmosphere energy budget, i.e., the fact that the earth-atmosphere system is now absorbing more energy from the sun as it is emitting. The ONLY way this can be remedied is by the system to heat up until that radiative balance is restored and the way it heats up is, as your assumption notes, by keeping the lapse rate approximately constant. And, as I discussed above, this results in a higher surface temperature in the final state (balance restored), because the altitude at which the average temperature is 255 K will be higher up in the atmosphere than it was before.
The most important thing for anyone to understand about energy balance is that it is the top-of-the-atmosphere energy balance that is fundamentally the most important. It is much easier to figure out what happens with that and then work down to the surface than it is to figure out the surface on its own. This is because the surface energy balance is complicated, since it involves conduction and convection (including evaporation) as well as radiation. The top-of-the-atmosphere balance involves radiation only.

January 5, 2012 10:38 am

My nomination for this week’s textbook example of psychological projection, written by Joel Shore:

Can’t you guys have some self-awareness about when you are flailing around horribly in order to believe what you want to believe. This debate is not about science…It is about people desperate to believe what their ideology dictates they have to believe to be true.

Joel Shore’s own ‘ideology’ is CO2=CAGW, and he’s still ‘flailing around horribly’ trying to find that mysterious heat hidden somewhere in Trenberth’s pipeline.☺
Despite what Joel Shore claims, this debate is about science, not ‘ideology’.

Joel Shore
January 5, 2012 11:04 am

davidmhoffer says:

2. There is no need to demonstrate a mechanism by which the average surface temperature of earth could exceed the black body temperature of earth. All that is required to support N&Z’s hypothesis is to demonstrate that there is a mechanism by which the average surface temperature of a planet could be increased by the presence of a non radiative atmosphere WITHOUT changing the EFFECTIVE black body temperature odf the planet..

This statement is really egregious! N&Z have added 100 K to what every actual climate scientist agrees is the greenhouse effect of 33 K (and have called it by the name “surface temperature enhancement”). Now, you want us to be impressed because they can subtract that 100 K away without invoking the greenhouse effect!?! That truly has to be one of the most absurb statements that I have seen on this subject…and that is saying something!
As for the rest of his post: I am not the one who brought up (or even mentioned, as far as I remember) what field he is working in. And, I am not saying that he is wrong on the basis of what his background is; I am saying he is wrong because I have laid out in gory detail repeatedly exactly what the issues are that utterly destroy his “theory”.
However, I am also appealing in general here for a little more humility: We could spend a lot less time having to explain to people why their misconceptions are wrong if people would actually accept the basic premise that they are unlikely to be so totally freakin’ brilliant that they can march into a field, having apparently not even read or absorbed information in the most basic elementary textbooks, and instantly come up with a “theory” that displaces a century of work in the field! For heaven’s sake, every second person on this site seems to think he is the next incarnation of Einstein or Galileo or something! (I exaggerate, but only slightly!) If everyone had a little more humility and understanding that they (and those who happen to agree with them) are not the only intelligent lifeforms in the universe, we wouldn’t have to put up with a constant barrage of nonsense.

Joel Shore
January 5, 2012 11:09 am

By the way, David, I should add that the question is what you mean by “support their hypothesis”. You have reduced their hypothesis to a form that nobody would disagree with. If the question is whether thicker atmospheres can lead to a higher average surface temperature than thinner atmospheres by virtue of an evening out of the temperature distribution, then the answer is YES and this is not in any way a new idea. (It is in fact completely bizarre that you keep trumpeting the fact that you have shown this as if there was anybody here arguing against N&Z who did not already know that!)
However, if the question is whether their idea can explain any of the 33 K radiative greenhouse effect that conventional climate scientist say exists on Earth, the answer is NO.

January 5, 2012 12:06 pm

Joel,
You just self destructed. See a doctor 🙂

Phil.
January 5, 2012 12:13 pm

Joe Born says:
January 5, 2012 at 5:44 am
Here’s why I think many of us have difficulty. Thinking of the earth, we tend naturally to integrate with respect to latitude and longitude. Testimony to that is that you and I both used that approach in computing Equation 2 numerically. But Ned didn’t, so he lost me right off.
What he did if I understand it is move the sun to Polaris’s position so that it shines directly on the North Pole. Then he translated the measure of latitude so that the North Pole’s latitude is zero and the South Pole’s is 180, and mu is the cosine of the resultant latitude. Another way of looking at it is that, for a point on the surface of a unity-radius sphere, mu is the distance along the axis in the direction of the sun from the sphere’s center to the plane perpendicular thereto that contains that point. Theta is longitude, or angular position around the circle in which the plane intersects the sphere. So Ned integrates along the axis and around the circle of intersection. mu < 0 implies nighttime, so integration limits for mu can be 0 and 1 if we want to ignore (de minimis) night-time background radiation. Fair enough, although it took me awhile to recognize what his coordinate system was.

Indeed, that’s why I asked about the missing factor of two, but when I first asked I got an incorrect answer that said that a negative zenith angle meant that that was the dark side which of course it doesn’t. When I persisted I got a more accurate description of what Ned is doing and of course it isn’t “calculating the temperature at every point on the Earth sphere first by taking the 4th root from the S-B relationship and then averaging the resulting temperature field across the planet surface”. What Ned does is integrate and average the sunlit side temperature assuming a Lambertian profile (not a bad approximation for an atmosphere-less planet as shown by the Moon) to obtain the Grey-body temperature of the sunlit side and then divide it by 2 (more on this later). The Tgb for the sunlit side is thus 308.6K, which seems a reasonable value for the Moon, (assuming a uniform temperature gives ~269K by my back of the envelope calculation). This is not the global grey body temperature though because no account has been made of the dark side of the planet. Ned’s way of dealing with this is to model the dark side as instantly dropping to absolute zero at sunset and remaining there until sunrise (thus introducing the spurious division by 2)! As the Moon data indicates this is a very poor choice. So Ned’s “According to Eq. (2), our atmosphere boosts Earth’s surface temperature not by 18K—33K as currently assumed, but by 133K!” is based on this non-physical model. By his own statements regarding the Moon, which has a 27.5 Earth-day night, the average temperature on the dark side must be about 90K which combined with this integration for the sunlit side would give a global value of ~200K, not 154K.
So the 133K warming is clearly nonsense because of this unrealistic model since it’s obviously conceptually incorrect, and from Hölder’s inequality you’d expect it to be worse value since it introduces a larger inhomogeneity in the temperature!
As to his use of the Gas Laws to determine the near surface temperature is nothing new, it’s a standard problem in High school classes. However, you have to have measurements of the average gas density near the surface which is no easier to do than to determine the average temperature near the surface. For example at the NP-39 Arctic research base the Temperature is 240K and Pressure is 1013 whereas in Boa Vista, Brazil it’s about 310K at the same pressure. The issues of the use of curve fitting have been addressed by others, there’s no predictive value to it.

January 5, 2012 12:24 pm

Ned, your “observed” number for Mercury’s mean surface temp is also way, way off as well. You state that the mean surface temperature for Mercury is 248K, which is 40 degrees colder than the Earth’s. The actual observed number (as opposed to your invention) is 440K. You have an “observed” mean surface temperature for Mars (182K) that is lower than the *minimum* temperature in its diurnal range of 184K-222K. The actual mean surface temperature for Mars is about 210K.
Between that and your claim that the Earth lost over 50% of it’s atmosphere during the last 50 million years, is there any surprise that the scientific world isn’t paying much attention to your manifesto?

January 5, 2012 12:30 pm

Joel Shore (Jan. 5, 2012 at 9:58 am):
I gather that you stipulate to the fact of energy conservation under maintenance of the lapse rate at the adiabatic lapse rate via convection. Thank you.

Joel Shore
January 5, 2012 12:34 pm

Terry: I have explained the physics to you as best I can; What part are you having a hard time understanding?

Ned Nikolov
January 5, 2012 12:55 pm

Fellows,
About the Moon temperature issue – I would like to put this discussion to rest by sharing with you the following information I just got from two key people on the NASA’s Diviner project.
I talked to the Co-Principle Investigator, Dr. Carlton Allen (at NASA’s Johnson Space Center in Houston TX) and the person in charge of the moon temperature data, Dr. Matthew Siegler at UCLA. Dr. Siegler told me that the Diviner webpage containing the temperature table which Robert Murphy refers to (see http://www.diviner.ucla.edu/science.shtml) has actually been put together by undergraduate students, and not by him. When I pointed out the discrepancy between the numbers shown in the table and the actual data reported in the papers, he acknowledged that the values in the tables are inaccurate for what they claim to be. He also agrees that both modeled and observed moon temperature series indicate that the mean diurnal temperature is about 210K at the lunar equator and about 110K at the lunar poles. He apologized for the misleading information on the Webpage and promised to ask the people maintaining the page to correct it. We will probably see an an update in a few days …
Now, please, lets continue with the science discussion and drop the personal attacks and insinuations. Thank you!

davidmhoffer
January 5, 2012 1:01 pm

Joel Shore says:
January 5, 2012 at 11:09 am
By the way, David, I should add that the question is what you mean by “support their hypothesis”. You have reduced their hypothesis to a form that nobody would disagree with.>>>
Fantastic! We have a consensus then!
Joel Shore says:
January 5, 2012 at 11:09 am
If the question is whether thicker atmospheres can lead to a higher average surface temperature than thinner atmospheres by virtue of an evening out of the temperature distribution, then the answer is YES and this is not in any way a new idea.>>>>
Glad to see you admitting that despite having contradicted it at every turn when the discussion intitially started. Good to see that you recognize that your original position was false, though that also makes your statement that it isn’t a new idea false since it seems to be one for you.
Joel Shore says:
January 5, 2012 at 11:09 am
(It is in fact completely bizarre that you keep trumpeting the fact that you have shown this as if there was anybody here arguing against N&Z who did not already know that!)
Again, glad to see you stipulating to my position after denigrating it ad naseum earlier.
Joel Shore says:
January 5, 2012 at 11:09 am
However, if the question is whether their idea can explain any of the 33 K radiative greenhouse effect that conventional climate scientist say exists on Earth, the answer is NO.>>>
Aw, Joel, you ruined it. You’ve agreed to my position, agreed to my explanation, and my explanation makes it pretty clear that 133K is far more accurate than 33K. If you cannot dispute the algebra, and in fact endorse it, then you must also endorse the 133K! You cannot say that the algebra I’ve presented is RIGHT but that the number calculated by the algebra is WRONG!
Well, I guess you can, you just did.

Joel Shore
January 5, 2012 1:44 pm

Terry Oldberg says:

I gather that you stipulate to the fact of energy conservation under maintenance of the lapse rate at the adiabatic lapse rate via convection. Thank you.

I don’t know what this even means. Your jargon is frankly incomprehensible. Read what I wrote to you in detail explaining the physics, e.g. here and earlier: http://wattsupwiththat.com/2011/12/29/unified-theory-of-climate/#comment-854225 It is not useful to try to summarize everything in a single jargon-filled sentence that could be interpreted about 10 different ways.

Joel Shore
January 5, 2012 1:53 pm

davidmhoffer says:

You’ve agreed to my position, agreed to my explanation, and my explanation makes it pretty clear that 133K is far more accurate than 33K. If you cannot dispute the algebra, and in fact endorse it, then you must also endorse the 133K!

I missed the part where you demonstrated that 133 K is far more accurate than 33 K. I don’t even know what such a sentence means. More accurate for what? As an estimate of the radiative greenhouse effect, 33 K is certainly more accurate. As an estimate of the total effect in going from an airless planet to our current one, then 133 K is probably more accurate, but it also convolves the radiative greenhouse effect with the effect of making the surface temperature more uniform (because of the presence of significant heat storage and horizontal heat transport).
So, what exactly do you think N&Z have shown and how does it relate to their purported claim “that the physical nature of the so-called GH effect is a Pressure-induced Thermal Enhancement (PTE), which is independent of the atmospheric chemical composition”? In fact, the entire 33 K effect that most climate scientists talk about is due to atmospheric chemical composition because it is due to the radiative greenhouse effect. Yes, the other 100 K that N&Z talk about is not due to this, but nobody claimed that the radiative greenhouse effect raises the temperature by 133 K; we claimed it raises the temperature by 33 K.

Ned Nikolov
January 5, 2012 2:09 pm

Reply to Robert Murphy (January 5, 2012 at 12:24 pm)
Robert,
I address the Mercury and Mars temperature issues in my official reply. But here is the situation in a nutshell. The Mercury temperature of 440K reported by NASA is an estimate using the S-B equation, and not based on actual observations. As such, it suffers from the same problem as the 250K estimate for the Moon temperature – an incorrect application of the S-B law! So we used our Eq. 2 to estimate Mercury’s termperature.
The derivation of Mars temperature is briefly discussed in Section 3.2 of our paper (have you read it?). We used spatial data for surface temperature and pressure derived by researchers at Stanford University using satellite observations by the Mars Global Surveyor (MGS) spacecraft from 1999 to 2005. The 210K value for Mars mean global temperature is erroneous, since it is based on measurements made at only 2 locations at LOW latitudes by the Viking probes in the 1970s.
About past atmospheric pressures – yes, there is a very good indirect evidence that, 50-60M years ago, pressure was much higher than today. That evidence is in the equable climate conditions that existed in that era. ‘Equable climate’ means that the temperature gradient between equator and the poles was nearly non-existent. This is inferred from the fossil record, which shows that tropical ecosystems thrived around and beyond the polar circle. So, not only was the mean global temperature 12-16K higher than today, but the Earth was also uniformly warm. That’s why they call it ‘hothouse climate’. Current climate models cannot reproduce these equable conditions using present atmos. pressure, and this is one of the main conundrums in paleo-climatology today. The ONLY way that such an uniform climate could have existed is if the atmospheric mass and pressure were much higher than today. Higher pressure means higher air density, which means more efficient meridional (poleward) heat transport. Paleo-climatologists all agree that these ‘hothouse’ equable climates require a VERY efficient meridional heat transport, but they have not found a mechanism for it yet, because no one has thought to look at pressure! Since the poleward heat transport is due 80% to atmospheric currents and 20% to ocean currents, it’s obvious that a higher atmospheric density will significantly increase this transport … Also, Venus is a present-day example of how high atmospheric pressure brings about high average temperature and a uniform climate. The surface of Venus is at 738K throughout the whole planet including the nighttime hemisphere despite the extremely slow axial rotation (one Venusian day lasts some 227 Earth days). The Moon surface cools 250K shortly as the Sun goes down, while Venus’ nighttime hemisphere stays led-melting hot for many months without any sunlight. That’s because high pressure (and the resulting high air density) transports heat from the sunlit to the shaded portion of the planet… I hope this helps evolve your thinking …

dr.bill
January 5, 2012 2:24 pm

Shore, passim:
Scattered throughout your comments, you keep repeating the statement that the tropopause rises and the lapse rate then determines the temperature of the surface of the Earth. Why do you do that? In fact, it is the heating from below, combined with the lapse rate, that causes the temperature in the troposphere to (ordinarily) decrease with altitude. Cause and effect…
By the way, in your “item (3)” earlier, was that just a “major blink”, or the preparatory stage of incipient back-pedalling? ☺
/dr.bill

davidmhoffer
January 5, 2012 2:42 pm

Joel Shore,
C’mon buddy, think it through!
If you agree that 133K is about right for combined GH effect due to GHG’s + conductance +convection, then it ought to be obvious to you what happens when you alter the ratio of GHG’s to the rest of the atmosphere!
The whole system works in a giant feedback loop because it has no choice but to act in any other manner.
Start with a zero GHG atmosphere in equilibrium.
Add some GHG’s.
They absorb some amount of upward bound LW that would otherwise have escaped.
Consequence => atmosphere is now warmer than it was before. Warmer atmosphere = reduced temperature differential compared to the surface = LESS CONDUCTION!
Atmosphere re-radiates a portion of the absorbed LW back toward earth where it is absorbed by earth surface. This raises earth surface temperature = MORE CONDUCTION,
but since the atmosphere is in general warmer than it was before = LESS CONVECTION!
…and round round we go.
Intercepting outbound LW by GHG’s alters the amount of energy that is moved to the atmosphere through conductive processes and also alters the manner in which energy is distributed via conduction to night side and higher latitudes. The more GHG feedback there is, the less conduction/convection there is.
Do they exactly cancel out?
I don’t have a clue!
But given the accuracy of N&Z’s results, it is pretty clear that if the GHG number currently is 33K and it adds to the 100K from conduction/convection for a total of 133K, then there is every reason to expect that without GHG’s, there would be more conduction/convection arriving at about 133K.

January 5, 2012 2:44 pm

Ned,
Excellent replies to Mr. Murphy. Thank you.

Joel Shore
January 5, 2012 3:00 pm

davidmhoffer says:

But given the accuracy of N&Z’s results, it is pretty clear that if the GHG number currently is 33K and it adds to the 100K from conduction/convection for a total of 133K, then there is every reason to expect that without GHG’s, there would be more conduction/convection arriving at about 133K.

No, David. You are still missing the important point: It is impossible for the Earth to be at a temperature of 288 K and be emitting 240 W/m^2 unless there are GHGs, so, no, there is absolutely no way to go up from N&Z’s T_sb to our actual Earth without GHGs. This rise of 100 K that you are talking about is really sort of a figment of the imagination. It just doesn’t make much sense to talk about an average temperature when the temperature distribution is that extreme. As you have noted, it is much better to average T^4 over the Earth. If you average T^4 over the earth and then take the 4th root, you get a temperature of 255 K, independent of the temperature distribution.
You have everything exactly backwards: You have correctly concluded that averaging T^4 is better than averaging T and yet you are going with N&Z, who have defined the temperature by averaging T…and doing so for a temperature distribution that is extremely broad and is only a good approximation to reality in the case of a planet with no atmosphere.

Joel Shore
January 5, 2012 3:07 pm

dr bill says:

By the way, in your “item (3)” earlier, was that just a “major blink”, or the preparatory stage of incipient back-pedalling? ☺

It is not back-pedalling at all. I have always maintained that the fact that N&Z get a good fit to the data arises for various reasons but in no way advances their bogus claim that somehow the atmospheric greenhouse effect is not responsible for the 33 K warming that we attribute to it on Earth. Item (3) represented a breakthrough in understanding another one of the reasons why they get a decent fit (although with 5 parameters, it is not clear how much explaining needs to be done) despite the fact that they are completely off-base in their conclusion that they can explain any of the 33 K radiative greenhouse effect by other means!

January 5, 2012 3:10 pm

“I address the Mercury and Mars temperature issues in my official reply. But here is the situation in a nutshell.”
Look, Mercury is not cooler than the Earth. The Moon is not cooler than Mars. You made up numbers, called them “observed” numbers in your manifesto, and then patted yourself on the back saying how strong your claims are because your “observed” numbers match your equations. Well, duh! They came from your equations, not from observations.
“About past atmospheric pressures – yes, there is a very good indirect evidence that, 50-60M years ago, pressure was much higher than today. That evidence is in the equable climate conditions that existed in that era.”
That’s nuts. What do you think would happen to life on Earth as a result of this reduction by 53% of the Earth’s atmosphere? Did you think of that? You don’t just get to arbitrarily claim a 53% reduction of the Earth’s atmosphere because people are having trouble modelling the Eocene equable climate. What’s next? Why not change the gravitational constant of the universe while you’re at it.

Joel Shore
January 5, 2012 3:24 pm

I said:

No, David. You are still missing the important point: It is impossible for the Earth to be at a temperature of 288 K and be emitting 240 W/m^2 unless there are GHGs, so, no, there is absolutely no way to go up from N&Z’s T_sb to our actual Earth without GHGs.

Sorry…This is a little unclear. What I mean is that it is impossible for the Earth’s surface to be at a temperature of 288 K and for the Earth + atmosphere to be emitting only 240 W/m^2 as seen from space unless there are GHGs.
The basic point is this: You have assumed that because you can raise the average temperature by 100 K without GHGs, why can’t you raise it another 33 K without GHGs? And, the answer is that you raised this average temperature will keeping the surface emission constant at 240 W/m^2. However, there is no way to go beyond 255 K surface temperature and still emit 240 W/m^2.

philincalifornia
January 5, 2012 3:41 pm

Robert Murphy says:
January 5, 2012 at 3:10 pm
That’s nuts. What do you think would happen to life on Earth as a result of this reduction by 53% of the Earth’s atmosphere? Did you think of that?
====================================
It evolved.

dr.bill
January 5, 2012 3:44 pm

Joel Shore, January 5, 2012 at 3:07 pm :
So then, your multi-day tour de force has produced the following:
You started with: “This is all nonsense.”
And now you say: “OK, &frac34; of it is obvious, but the other &frac14; is still nonsense.”
Hmm…
/dr.bill

dr.bill
January 5, 2012 3:46 pm

Joel Shore, January 5, 2012 at 3:07 pm :
So then, your multi-day tour de force has produced the following:
You started with: “This is all nonsense.”
And now you say: “OK, ¾ of it is obvious, but the other ¼ is still nonsense.”
Hmm…
/dr.bill

davidmhoffer
January 5, 2012 3:47 pm

Joel Shore;
You have correctly concluded that averaging T^4 is better than averaging T and yet you are going with N&Z, who have defined the temperature by averaging T>>>
But they didn’t.
They averaged the fourth root of T which for the way they went about their calcs is the right way to do it. I used average of T^4 in my explanation, but averaging 4th root of T works just as well.
It is right there in their description just prior to equation 2, followed by equation 2 itself. You can tell Joel by that big box in the first line with the hook shape on the left hand side and the number FOUR written beside it. That would be high school math, would it not?
You’ve now managed to dispute their results by claiming they used a mathematical construxt which they most certainly did not.
I’m going to assume that it was a mistake on your part rather than a deliberate attempt at misdirection.
I know you’re probably speechless right now, having a PhD in physics and all and losing an argument to a dim wit computer salesman who learned calculus so long ago that slide rules were high tech, so let me help you out. I think the words you are looking for are:
Sorry. I was wrong.

Kevin Kilty
January 5, 2012 3:55 pm

OzWizard says:
December 29, 2011 at 7:20 pm
Kevin Kilty says:
December 29, 2011 at 12:23 pm
“The surface temperature results from 1) irradiance that is absorbed predominantly high in the atmosphere leading to a high temperature there, and 2) then a lot of work input by gravity as convection takes parcels from up high to the surface. There is also a little irradiance absorbed at the surface which is what drives the convection.”
OK, Kevin. Engineering degrees at 10 paces. Ready?
I may be misreading what you are trying to say but, your proposition that the irradiance on Venus is “absorbed predominantly high in the atmosphere”, flies in the face of the following ‘facts’:
1. The density there is lowest.
2. The atmosphere is transparent to incoming irradiance.
So how does convection get the high, hot, ‘thin’ stuff down to the ‘relatively cool’ surface? Won’t convection tend to keep the hot stuff high up. Hot CO2 rises, cool CO2 falls, n’est-ce pas?
You do mention the surface having a high temperature, but ascribe this to energy absorbed at TOA. I don’t follow your thinking about ‘little irradiance absorbed at the surface’. ( I make no comment about your ‘parcels’ bringing it down.)
So, I’m with Ned and Karl at present and you are not swaying my thinking one little bit, yet.

OzWizard,
I apologize for not coming back to look at this thread for so many days. I wasn’t ignoring you anyway–the duel at 10 paces worried me, though!
The Venusian atmosphere is not structured cool at the surface and hot higher up. It is a gradual decrease from surface to about 60km above the surface. What I am saying is that on Venus the temperature of the atmosphere at 60km elevation is very high (where it is around 250-260K )compared to what it is on Earth at the tropopause (230K). Incoming irradiance is scattered and absorbed quite high in the atmosphere, not much reaches the surface. The atmosphere is not transparent to incoming irradiance, the atmosphere on Venus is like 90 times the mass of that on Earth, and even at 60km it is not especially “thin” (pressure is about 300mb at this height). But enough irradiance does reach the surface to drive convection. The adiabatic lapse rate resulting from this convection, projected from the temperature 60km above the surface (where the convection must end) leads to a very high surface temperature. Let me make my point in another way.
If, somehow, the Earth’s atmosphere were to absorb more radiation at 10km (top of troposphere) and reach a temperature that is, say 10K higher than at present, and the current lapse rate to be maintained (the result of a variety of processes including convection) then the surface would be 10K warmer also. This additional absorbed radiation could have come from LWIR welling up from ground surface (a greenhouse effect), or from Solar irradiance if there were gases that could absorb such.
Joel Shore would maintain that this is all “greenhouse effect” on Venus, that IR absorption by CO2 determines the high temperature at 60km height, but I am not so certain it is entirely. Venus has 90 times as much atmosphere, the atmosphere is not transparent to SW, and it is composed of not only CO2, but also sulfur compounds. I’m rambling a bit here, so I’ll stop, but I just wanted to make my point more clear.

davidmhoffer
January 5, 2012 3:58 pm

Joel Shore;
Sorry…This is a little unclear. What I mean is that it is impossible for the Earth’s surface to be at a temperature of 288 K and for the Earth + atmosphere to be emitting only 240 W/m^2 as seen from space unless there are GHGs>>>
Wake up Joel. 288K is the average of T which you’ve already admitted is a math error. If there are no GHG’s the temperature of the earth is 255K plus the “ehancement” due to conduction/convection. If there are GHG’s, then the temperature of the earth surface is 255K plus the “enhancement” due to conduction/convection plus the enhancement due to GHG back radiation MINUS the reduction in conduction/convection due to REDUCED conduction/convection as a by-product of back radiation.
Catching on yet?
words…sorry….wrong….

Joel Shore
January 5, 2012 6:26 pm

davidmhoffer says:

They averaged the fourth root of T which for the way they went about their calcs is the right way to do it. I used average of T^4 in my explanation, but averaging 4th root of T works just as well.

It is hard to know how one can pack so many incorrect statements into two sentences. Let’s deconstruct this, shall we:
(1) It is not the 4th root of T; it is the fourth root of a something having the units power per unit area divided by the appropriate constants. Taking the fourth root gives you a temperature (since the power is proportional to T^4); that is what they averaged.
(2) Why would that work just as well as averaging something directly proportional to the power? You were complaining before that the should have been averaging the appropriate power of the temperature (T^4) so that they are averaging P instead of averaging the temperature. Now, all of a sudden, it simply doesn’t matter; apparently, averaging temperature to the power of pi would be fine and dandy if that is what they did. [In fact, it is interesting, because what you have claimed is that averaging T^(1/4) power is just as good as averaging T^4 power. Now that I have shown you that they are averaging T, and assuming you are willing to admit the obvious, I imagine it will turn out that averaging T “works just as well”.]
Let me explain to you what they did, since you seem to have no actual conception:
(a) They computed the local insolation at each point on the Earth. This is an incoming intensity in W/m^2.
(b) They then derived a temperature from this by dividing by epsilon*sigma and taking the 4th root. This means that they are simply assuming that the temperature at that point is determined by the local insolation at that particular point, at that particular time, i.e., that there is absolutely no storage or horizontal flow of energy.
(c) They averaged the resulting temperature distribution over the earth’s surface.
David, I am not losing this argument amongst anybody who actually has the ability to follow this….And, I think you know that.

Joel Shore
January 5, 2012 6:32 pm

davidmhoffer says:

Wake up Joel. 288K is the average of T which you’ve already admitted is a math error.

What I and Willis have very patiently tried to explain to you is that for an Earth-like temperature distribution, the difference you would get for the temperature between averaging T and averaging T^4 and then taking the 4th root is very small. If you want to argue that we should really be saying 289 K or 287 K, I won’t dispute you.

If there are no GHG’s the temperature of the earth is 255K plus the “ehancement” due to conduction/convection.

There is no enhancement due to conduction / convection. 255 K is the absolute maximum average temperature that the Earth can have and be emitting 240 W/m^2. It can have considerably lower temperatures if the temperature distribution is quite uneven but it cannot have any higher temperatures.

Kevin Kilty
January 5, 2012 7:12 pm

Joel Shore says:
January 5, 2012 at 6:32 pm
davidmhoffer says:

“Wake up Joel. 288K is the average of T which you’ve already admitted is a math error.
What I and Willis have very patiently tried to explain to you is that for an Earth-like temperature distribution, the difference you would get for the temperature between averaging T and averaging T^4 and then taking the 4th root is very small. If you want to argue that we should really be saying 289 K or 287 K, I won’t dispute you. …”

I have only been paying attention to this conversation in a peripheral way, but, with all due regard to you, Joel and Willis, I think you are missing a point that I tried to make very early in this thread. I think the difference between taking the fourth root of the average of T^4 and taking the mean value is a significant issue. You have already admitted that the more extreme the variations in temperature the lower is the mean temperature at which the Earth can radiate away all it absorbs. The reason is, of course, that a small hot radiator is quite effective when the emitted power is proportional to T^X, when X is greater than 1. As you point out, on a black radiator with no greenhouse blanket this would be to the fourth power. In an atmosphere with a water-vapor window at a surface temperature near 280K it will be more like a 4.6 power, but in any case it is a power greater than 1.0
For sake of argument let’s say that the Earth can radiate in equilibrium (i.e. maintain a constant temperature) if one hemisphere is 255K and the other is 303K. The radiating temperature using the 4.6th root of the mean of the 4.6 power is about 283K. The linear mean is 279K. That is nearly 4K of difference. You say this is insignificant. Yet, what this suggests to me is that there is a variation of 4K that could come about at a single value of absorbed irradiance–but at different surface temperature distributions. This is about 4 times the temperature increase that we are arguing about due to greenhouse warming. My point is that different configurations of radiator could account for this without any enhanced driver of climate.

January 5, 2012 8:00 pm

“Joel Shore says:
January 5, 2012 at 10:37 am
The most important thing for anyone to understand about energy balance is that it is the top-of-the-atmosphere energy balance that is fundamentally the most important. It is much easier to figure out what happens with that and then work down to the surface than it is to figure out the surface on its own. This is because the surface energy balance is complicated, since it involves conduction and convection (including evaporation) as well as radiation. The top-of-the-atmosphere balance involves radiation only.”
A fundamental energy balance based on conservation of energy does not exist. Energy balances are the result of the Second Law not the First. The Earth could stop radiating and start accumulating energy until it gets as hot as the Sun, the energy will be conserved. There is no direction for an energy flux in a Universe without a Second Law.
There is no balance at TOA, but rather one throughout the system mostly developing near Earth surface. Radiation from the Sun enters and gets absorbed at the surface (or reflects). Emission will follow only because of the Second Law when energy can get dispersed (entropy), and this is a fast process.
But a large part of the energy dissipates (entropy); flows into the ocean, vaporizes water, heats the atmosphere – , is used in chemical processes/photosyn., doing work, maintains hydro-/carboncycle etc. often followed by other processes and interconnected. All these have various speeds (up to years, or hundreds in the ocean) and are governed or balanced by the Second Law.
So there is a multi-faceted time varying energy balance by Second Law at work, mainly from the ground up that eventually works it’s way up and leaves as downgraded radiation. So there is no surprise that over a certain time frame (but how long?) radiation in = radiation out, and real time differences tell you nothing about what’s all happening inside. You can’t derive temperatures from it, certainly not for the small mass of the atmosphere.
Earth system does not care about a radiative balance at TOA, all processes one by one including weather follow the path of dissipation of energy and have no knowledge of an outside TOA balance and do not respond to such a thing.
The energy fluxes respond to gradients of temperature, pressure and concentration. The Second Law tells which way to go but even then it is not clear at which speed the processes will take place. Nature has all these speed bumps otherwise all energy would dissipate in a jiffy. So we have activation energy for the chemical processes, and the speed of the atmospheric transport from equator to poles seems not to be simply correlated to dT or dP but happens under the Maximum Entropy Production Principle.

Richard M
January 6, 2012 4:46 am

I thought I should lay out my idea for a maximum greenhouse effect (GHE) here on the UCT thread. My idea is simple. Rather than the findings of Nikolov and Jelbring being independent of the GHE they describe a method to discover the maximum possible GHE.
This way of thinking about the UCT eliminates the problems that would occur when one tries to discuss an atmosphere without GHGs. It also accepts that the work of climate scientists for many years is completely valid up to a point. Finally, it provides what many skeptics have seen in the data. Temperatures do not follow CO2 variations. The currently defined GHE just isn’t showing up in the data.
The idea is pretty simple. The GHE starts off strong when a few GHGs are added to an atmosphere. However, as the concentration grows the added effect becomes smaller and approaches the maximum GHE (maxGHE) asymptotically. Eventually, the gain becomes inconsequential.
It turns out the maxGHE is defined by the atmospheric mass and surface pressure for any given value of solar input. The reason the UCT initially thought the heating was independent of GHG concentration was due to the fact that different mixes of gases on different planets all led to the same predictable temperature profile. However, Willis supplied the thought process that demonstrated a planet surface could not warm without GHGs. Hence, there had to be some kind of relationship to GHGs. The concept of a maxGHE allows many different atmospheres to give the same temperature profile as long as they have sufficient GHGs to reach the maxGHE.

Joel Shore
January 6, 2012 5:27 am

Kilty (January 5, 2012 at 7:12 pm): I’ll address your question in two parts –
(1) Is the issue you talk about relevant to the current discussion?
The answer here is clearly NO. We have explained that the highest average temperature that you could possibly have for any distribution that radiates 240 W/m^2 is 255 K. We know that the average temperature of the Earth is somewhere around 288 K. Whether it is actually 287 or 289 K won’t make a difference.
(2) Is the issue that you talk about relevant to the issue of temperature trends under global warming?
Basically, the issue that you raise is closely related to one that was raised by Essex et al.: http://ruby.fgcu.edu/courses/twimberley/EnviroPhilo/Exist.pdf The only real substance of their paper applied to real world data was when they looked at temperature trends for 12 different stations and purported to show that different ways of averaging produced different trends. In order to demonstrate this, they averaged T^r where r is a positive or negative integer and then took the rth root. When they made r really large in magnitude (either positive or negative), they did find a substantially different trend, but by really big or really small, you have to understand that they used a range of r going from -125 to +125. If you look at their data and just compare r=1 and r=4 (which is what we are talking about here), the difference in the trend is not very large.
Furthermore, it is easy to diagnose why the trend did change when they made r really big in magnitude: When r was a large positive value, their average essentially just put all of the weight on the highest temperature data point for each month in their temperature series. When r was a large negative value, their average essentially just put all of the weight on the lowest temperature data point for each month. Clearly, this is not going to be a good average and when you have a relatively sparse set of 12 temperature records, this can produce a large effect…but when you have hundreds of temperature records it will be more difficult to weight just one temperature each month and there is very good reason to believe that the effect will be largely washed out. [And, at any rate, as I noted, even for 12 records, the effect was only very dramatic if you started choosing ridiculous values of r, not if you chose 4 instead of 1.]
And, it is not hard to understand why this is not going to make a large difference: Yes, you might be able to get a difference of a degree or two for an Earth-like temperature distribution depending on how you do the average. However, your temperature distribution is not going to radically change as the Earth warms…To the first approximation, the distribution will just shift to warmer temperatures while not changing shape. To a better approximation, there might be a small change in the shape of the distribution (e.g., because the arctic warms faster than the tropics) but this is change in shape is going to be very small (after all, we’re not expecting the arctic not to be cold and the tropics not to be hot).
A good analogy might be if I tried to use a yardstick that was off by 1 inch over its 36 inch length to measure how much a bar of length 30 inches expanded when it was heated. If the bar expanded by only, say, 1 inch, you might worry that this ruler was not up to the task since it is off by 1 inch! However, with a little thought, I think you can convince yourself that in fact this ruler will be able to correctly measure how much the bar expands to within less than 3%.

Phil.
January 6, 2012 8:14 am

Ned Nikolov says:
January 5, 2012 at 2:09 pm
Reply to Robert Murphy (January 5, 2012 at 12:24 pm)
Robert,
I address the Mercury and Mars temperature issues in my official reply. But here is the situation in a nutshell. The Mercury temperature of 440K reported by NASA is an estimate using the S-B equation, and not based on actual observations. As such, it suffers from the same problem as the 250K estimate for the Moon temperature – an incorrect application of the S-B law! So we used our Eq. 2 to estimate Mercury’s termperature.
Which has a blatantly false model for the temperature distribution which leads to a large underestimation, a far larger error than the assumption of a uniform temperature. The assumption of a Lambertian profile for the sunlit hemisphere is a good one for an atmosphere-less planet as is demonstrated by the data from the Moon. Your assumption of the dark side being at absolute zero is a huge underestimate as is demonstrated by the data you quoted for the Moon (which would be a close candidate with its long night). It is this assumption which leads to an error of ~100K in your estimate for the Earth.

Joel Shore
January 6, 2012 8:51 am

Phil. says:

The assumption of a Lambertian profile for the sunlit hemisphere is a good one for an atmosphere-less planet as is demonstrated by the data from the Moon. Your assumption of the dark side being at absolute zero is a huge underestimate as is demonstrated by the data you quoted for the Moon (which would be a close candidate with its long night). It is this assumption which leads to an error of ~100K in your estimate for the Earth.

To be technically precise, for the Earth, I don’t think what your call the “Lambertian profile” (and I would call just approximating the local temperature by requiring radiative balance with the local insolation…hence neglecting any energy storage or movement) is very good either. So, the error that you speak of in the estimate for the Earth is not only due to the poor assumption for the dark side (although it may well be the majority of it)….It is due to the temperature distribution for both sides being a poor assumption.

Phil.
January 6, 2012 10:37 am

Joel, the Lambertian profile is basically how you described it, a cos^0.25 profile, for the Moon it’s a good approximation and for a planet without an atmosphere it should be good. The real problem with N & Z is the assumption of absolute zero for the dark side which is absolutely terrible however!

Ned Nikolov
January 6, 2012 10:41 am

Phil and Joel,
About the temperature of an airless planet:
Apparently you are confusing (unable to make distinction between) a temperature DISTRIBUTION over the surface of a body and the MEAN temperature of that body. The mean temperature depends on the shape of the celestial object (e.g. sphere vs. flat disk) and the distance of the object from the Sun. Those two factors along with the albedo are responsible for the amount of solar energy absorbed by the object. The DISTRIBUTION of the temperature across the surface, on the other hand, depends on thermal properties of the substrate of the object such as thermal capacity and thermal conductivity as well as on the speed of axial rotation (faster rotation will produce a more uniform temperature field, while a slow or absent rotation will result in a highly non-uniform ‘contrast’ temp environment).
Our gray-body model in Eq. 2 assumes a non-rotating planet that has reached an equilibrium with the incoming solar radiation. This assumption is ‘bad’ only if one is concerned with the surface temperature distribution, but has no consequence whatsoever if one is interested in the MEAN planetary temperature. Think about it …
Any real rotating planet with no atmosphere will have a temperature on the dark side greater than 0.0K. In fact, the lowest temperature in Deep Space is about 2.72K, and not zero. This is accounted in our model by the small constant added to So. If So = 0, Eq. 2 produces 2.725K.
After talking to Dr. Siegler yesterday (the person in charge of the Diviner lunar temperature data), he agreed that the mean diurnal temperature on the lunar equator is around 210K (in fact his own estimate was 206K), and that the mean diurnal temp. at the lunar poles is about 100K. So, the moon average diurnal temperatures span the range 206K – 100K, which gives a mean of (206 + 100)/2 = 153K. Our Eq. 2 produces 154K … In reality, the true MEAN temperature of the Moon may be shifted a bit towards the warmer side (206K) due to the spherical distribution of the temp. field, and it may deviate somewhat above the arithmetic mean of 153K, but it will still be MUCH closer to our theoretical estimate than to the 250K currently believed by climatologists … What’s amazing about the 250K is that this temperature as a diurnal mean does NOT occur ANYWHERE on the surface of the Moon!
So, please drop your arguments and embrace the reality.
Thank you!

Joel Shore
January 6, 2012 11:26 am

Ned Nikolov says:

Our gray-body model in Eq. 2 assumes a non-rotating planet that has reached an equilibrium with the incoming solar radiation. This assumption is ‘bad’ only if one is concerned with the surface temperature distribution, but has no consequence whatsoever if one is interested in the MEAN planetary temperature. Think about it …

I have thought about it and it is a manifestly silly statement. In fact, there are plenty of different temperature distributions that result in the same power output from the surface (which is what is constrained by the solar input). In general, these will not have the same mean temperature. In fact, two that clearly do not are your distribution and a uniform temperature distribution (the former giving the mean temperature that you got and the latter giving the standardly-assumed blackbody temperature of the Earth of 255 K)…They differ by 100 K! [I am a little confused about what you used for the albedo and epsilon in your model…but this doesn’t really seem to matter since others who have used the conventional values of alpha = 0.3 and epsilon = 1 report about the same mean temperature as you.]
The reason this is so, of course, is because the 4th root of the average of T^4 is not the same as the average of T for any distribution T other than a uniform distribution.

Phil.
January 6, 2012 11:28 am

Ned
You need to get a grip on reality yourself, a strange way to average for the surface of a sphere, what do you suppose the relative areas of the poles and the tropics are? Your idea that your model produces a true mean for a rotating planet is a fantasy as demonstrated by the Moon.

Joel Shore
January 6, 2012 4:13 pm

Ned: While you are stopping in here, would you mind telling me what values of alpha_gb and epsilon you used for the planets in order to calculate your T_gb? I find that I get pretty close to your T_gb if I choose and alpha_gb = 0.125 (which you mention in the context of earth) and epsilon = 0.949 for all of the celestial bodies. Is this what you used and where did the values for these two parameters come from?
Thanks,
Joel

Joel Shore
January 6, 2012 5:34 pm

…Never mind my last question…I see now where you say what epsilon and alpha_gb you chose.

Kevin Kilty
January 6, 2012 6:24 pm

Joel Shore says:
January 6, 2012 at 5:27 am
Kilty (January 5, 2012 at 7:12 pm): I’ll address your question in two parts –
(1) Is the issue you talk about relevant to the current discussion?
The answer here is clearly NO. We have explained that the highest average temperature that you could possibly have for any distribution that radiates 240 W/m^2 is 255 K. We know that the average temperature of the Earth is somewhere around 288 K. Whether it is actually 287 or 289 K won’t make a difference….

Mr. Shore, the highest temperature one can have is not 255K on an earth without atmosphere, but depends on other factors. It could be well over 325K if the region most directly illuminated radiates away its absorbed energy. It is a maximum of 255K only if you demand that the earth radiate uniformly, yet transfer absorbed heat perfectly; but it does not, wouldn’t even if the rest of your assumptions held true. It radiates much more strongly beneath the sun, and much less so at night. The moon does so as well. I have attempted to show that there is a range of mean temperature that is compatible with equilibrium at any given irradiance/albedo/etc. If one drifts from one such configuration to another mean temperature will rise or fall–perhaps by enough to be significant in terms of our ongoing AGW debate.
I say the places that actually act as radiators, be they polar regions, deserts, equatorial ocean, or cloud tops, will affect the mean temperature even if driving does not change one iota. I say the actual distribution of temperature matters–you say it hardly does. We have reached the end of this discussion, I suppose.

Phil.
January 7, 2012 5:16 am

Kevin Kilty says:
January 6, 2012 at 6:24 pm
Joel Shore says:
January 6, 2012 at 5:27 am
Kilty (January 5, 2012 at 7:12 pm): I’ll address your question in two parts –
Mr. Shore, the highest temperature one can have is not 255K on an earth without atmosphere, but depends on other factors. It could be well over 325K if the region most directly illuminated radiates away its absorbed energy. It is a maximum of 255K only if you demand that the earth radiate uniformly, yet transfer absorbed heat perfectly; but it does not, wouldn’t even if the rest of your assumptions held true.

Joel is talking about the average temperature for which the maximum value corresponds to a uniform distribution because of the non-linearity of the response.
It radiates much more strongly beneath the sun, and much less so at night. The moon does so as well. I have attempted to show that there is a range of mean temperature that is compatible with equilibrium at any given irradiance/albedo/etc. If one drifts from one such configuration to another mean temperature will rise or fall–perhaps by enough to be significant in terms of our ongoing AGW debate.
I say the places that actually act as radiators, be they polar regions, deserts, equatorial ocean, or cloud tops, will affect the mean temperature even if driving does not change one iota. I say the actual distribution of temperature matters–you say it hardly does. We have reached the end of this discussion, I suppose.

Joel doesn’t say that the distribution doesn’t matter, on the contrary he says that it does, what he does say is that the distribution which gives the highest mean temperature is a uniform distribution.

Joel Shore
January 7, 2012 5:19 am

Keven Kelty says:

Mr. Shore, the highest temperature one can have is not 255K on an earth without atmosphere, but depends on other factors. It could be well over 325K if the region most directly illuminated radiates away its absorbed energy. It is a maximum of 255K only if you demand that the earth radiate uniformly, yet transfer absorbed heat perfectly; but it does not, wouldn’t even if the rest of your assumptions held true.

For a surface emitting an average of 240 W/m^2, I am not saying the highest temperature that exists on the planet can’t exceed 255 K. Sure it can. I am saying the ****AVERAGE**** temperature can’t exceed 255 K. If the planet is at a uniform temperature, then that uniform temperature is 255 K; if the temperature is non-uniform then the average temperature will be less than 255 K. This is due to a rigorous mathematical theorem called Holder’s Inequality that, applied to this problem, tells us that the fourth root of the average of T^4 is always greater or equal to the average of T for any distribution T.
If you want to see how this works, try some simple distributions. For example, look at a planet where the dark half of it is at 0 K and figure out what the other half has to be if this other half is at uniform temperature in order to get an average emission of 240 W/m^2. What you will find is that it has to be at 303 K. This gives an average temperature of the planet of 152 K, which is much less than 255 K. This example also shows you why it works out this way: Because power emitted depends so strongly on temperature, the fact that you have to emit twice as much power (480 W/m^2) on the bright side of the planet in order to have the average over the planet work out to be 240 W/m^2 does not necessitate going to a temperature of twice 255 K…You only have to go up from 255 K by a factor of the fourth root of 2.

I say the actual distribution of temperature matters–you say it hardly does.

Yes…The distribution does indeed matter. A planet emitting 240 W/m^2 will only have an average temperature of 255 K if the distribution is uniform. If it is non-uniform, the average temperature will necessarily be LOWER.

Joel Shore
January 7, 2012 10:18 am

Sorry, Kevin. I appeared to have banished “i”‘s from my keyboard when I tried to type your name.

A. C. Osborn
January 9, 2012 4:45 am

Love it, “So, please drop your arguments and embrace the reality.”

John Brookes
January 10, 2012 4:44 am

OK. So the post is wrong for one obvious reason, and that is that there is already a perfectly good way of describing this, well known to those who construct computer models of the atmosphere. And the method described in this post differs from that method. In an area of such great public interest, the chance that a major part of it is just plain wrong is very small.
What I don’t get is how someone smart enough to have written the post is not smart enough to see that its wrong.

Ned Nikolov
January 11, 2012 9:12 am

To All:
Just want to let you know that Karl Zeller and I are working on our official reply to the blog comments. Due to unexpected work load last week, we could not finish it as planned. The article is now coming along pretty well, and we’ll be able to share it with you soon.
Thank you for your patience!
-Ned

kwik
January 13, 2012 12:36 pm

Ned and Karl;
There are lots of comments here that you can use to improve your paper in a positive way, I am sure.
Personally I am looking forward to your next move ! It will be very interesting.